Nclex Test Review

  • April 2020
  • PDF

This document was uploaded by user and they confirmed that they have the permission to share it. If you are author or own the copyright of this book, please report to us by using this DMCA report form. Report DMCA


Overview

Download & View Nclex Test Review as PDF for free.

More details

  • Words: 118,222
  • Pages: 222
The NCLEX tests are required to receive professional licensure in the field of nursing and are created by the National Council of State Boards of Nursing (NCSBN). The NCLEX means National Council Licensure Examination. Both the NCLEX-RN and NCLEX-PN cover a lot of overlapping material; however, the scoring and number of questions vary between the exams.

NCLEX Test Information

You should answer NCLEX questions using “book” knowledge and not practical experience. On the NCLEX hospitals operate on massive budgets and no expense is spared to provide proper care. NCLEX test writers are covering all their bases and focus on patient care that is sometimes unrealistic in today’s healthcare world. Generally, The correct answer is the anThe NCLEX tests are designed to be one swer that identifies the safest approach. It may of the final hurdles in your nursing career. not be the fastest or the best, but it is the safest. Consequently, the questions focus on your The old medical slogan of “Do No Harm” applies ability to make decisions in various patient to NCLEX test takers. NCLEX test writers are care scenarios under critical conditions. The trying to make sure that you are competent and NCLEX test requires that you understand the recognize that safety is the key. basic principles of nursing and apply this to different elements of patient safety manage- NCLEX Test Information ment.

NCLEX Introduction Many stare at limited funding and the overwhelming task of studying to pass the NCLEX. This website has been created to help students overcome the challenge of the NCLEX test. The key NCLEX testing tips are stated as follows: 1. Assess, Assess, Assess: In almost all cases something can be done before contacting the MD. 2. Prioritize: Delegate to the appropriate support personal and prioritize your tasks. 3. Review Medical Terminology: Understand the definition of all medical abbreviations and terminology used in the NCLEX questions. The hours of studying and the class work are finally worth it when you are free to practice nursing after passing the NCLEX, and you have highly sought after job skills. Please take your time to review all of the course notes and links put together on this site about the NCLEX test and the pitfalls that some students fall into with the NCLEX test. Hopefully you can avoid mistakes others have made when preparing for the NCLEX and will find the following information to be helpful and informative on dealing with the NCLEX test.

Another key point on reviewing for the NCLEX, is know your normal laboratory data ranges. Lab test results on the NCLEX will not be flagged with an asterisk if the number is outside of normal ranges. NCLEX test takers must memorize the basic lab values. Focus on the blood gas values. These values in particular can be complicated. If you do not remember the normal ranges you will have no chance of answering lab value questions on the NCLEX. Many times these abnormal values will require further assessment. It is also important to note that notifying the physician or contacting other health care workers is not The correct answer in many cases on the NCLEX. Remember the guidelines: Assess, Assess, Assess. Choose answers that require further assessment before contacting someone else on the NCLEX questions. Basically, collect more data and factual information before calling in other healthcare professionals. Finally, think safety with all types of patient care on the NCLEX. If equipment breaks down and the patient is in trouble, work on solving the patient’s problems before getting someone else to fix the equipment. Let maintenance deal with the equipment mess and focus on getting the patient in a safe environment. The NCLEX is attempting to determine competencies related to all of the above situations.

Collected by :DeepaRajesh [ 1 ] [email protected] Kuwait

NCLEX Format

Question3: What is the maximum amount of time that I could spend on each Questionon the NCLEX? Answer: If you take the maximum number of questions a safe time would be around 1 minute per Questionon The NCLEX CAT testing format stands the NCLEX. for a computer adaptive testing format. The computer during the NCLEX test Question4: What study aides have you will give you harder questions if you found that help you understand the answer a Questioncorrectly or easier format better? questions if you answer a Question- Answer: The link on the right of this page offers valuable help with the NCLEX format. incorrectly. The first Questionon the NCLEX will be below the baseline re- Question5: What if the CAT format ofquired passing score. Consequently, fers questions beyond the minimum a graph could be constructed using number required to pass? questions as points on the graph. The Answer: Keep taking the test and don’t get nervpoints above and below the passing ous. You still have the opportunity to do well and baseline contribute to your overall test- pass. ing score. The NCLEX test attempts to match you with questions that are at Learn How to Quickly Solve Difficult your level of nursing knowledge and NCLEX Test Questions NCLEX Flashcard Study System Free Online understanding. Sample Practice Test Questions Note: The NCLEX Exam is offered by Pearson VUE. The National Council of State Boards of Nursing has partnered with Pearson VUE to deliver the NCLEX exam. Pearson VUE offers both the NCLEX-RN and NCLEX-PN exams.

The NCLEX does not time each Questionpresented in the CAT format. You are allowed to answer each Questionwithout time constraints. However, the NCLEX does have a test taking time of 5 hours.

Click here to see free sample flashcards. Dear Friend, Here’s a little “secret” about the NCLEX Examination: the NCLEX is what we in the test preparation field call a “content driven” test.

Question1: What is the maximum While some tests are looking to see what you are amount of time allowed to take the ABLE to learn, the purpose of the National Council Licensure Examination for Registered Nurses NCLEX?

(NCLEX-RN) and National Council Licensure Examination for Practical Nurses (NCLEX-PN), Question2: How do I prepare for the offered by the National Council of State Boards of Nursing (NCSBN), is to test your understandCAT format on the NCLEX? ing of what you have already learned. The goal Answer: Allow each Questiona reason- of the NCLEX is to make sure you have a miniable amount of time and thought. Treat mum competency level to protect the integrity of each Questionwith the same level of the testing process. Answer: 5 hours

difficulty. Don’t be scared if questions are getting “easier,” and you think that you are falling below the passing baseline of difficulty.

In other words, it’s more about what you know than your ability to solve clever puzzles. This is good news for those who are serious about being prepared, because it boils down to a very Collected by :DeepaRajesh [ 2 ] [email protected] Kuwait

simple strategy: You can succeed on the NCLEX and become a Registered Nurse (RN) or Practical Nurse (PN) by learning critical concepts on the test so that you are prepared for as many questions as possible. Repetition and thorough preparation is a process that rewards those who are serious about being prepared, which means that succeeding on the NCLEX is within the reach of virtually anyone interested in learning the material. This is great news! It means that if you’ve been worried about your upcoming NCLEX, you can rest easy IF you have a good strategy for knowing what to study and how to effectively use repetition to your advantage. But it also creates another set of problems. If you tried to memorize every single possible thing you can for the NCLEX, the field of possible things to review would be so huge that you could not hope to cover everything in a reasonable time. That’s why I created the NCLEX Flashcard Study System: I have taken all of the possible topics and reduced them down to the hundreds of concepts you must know and provided an easy-touse learning method to guarantee success on the NCLEX. I wanted this system to be simple, effective, and fast so that you can succeed on your NCLEX with a minimum amount of time spent preparing for it. Note: This product will also work for the HESI exit exam and help you graduate from your nursing program. Here Are Some of the Features of Our NCLEX Flashcard Study System * Study after study has shown that repetition is the most effective form of learning- and nothing beats flashcards when it comes to making repetitive learning fun and fast * Flashcards engage more of your senses in the learning process- you “compete” with yourself to see if you know the answers to the

questions, and the flipping action gets you actively involved in the learning process * Our cards are printed on heavy, bright white 67 lb. cover stock, and are laser printed at 1200 dpi on our industrial printers- these are professional-quality cards that will not smear or wear out with heavy usage * We cover all of the major categories of the NCLEX test (see the list below) * Our flashcards include an edge that is microperforated, which means that you are much less likely to have a painful papercut on your fingers when moving quickly through the cards * Our cards are portable, making it easy for you to grab a few and study while waiting for the bus or the doctor, or anywhere where you have a spare moment that would otherwise be wasted * Our cards are written in an easy to understand, straightforward style - we don’t include any more technical jargon than what you need to pass the test * The cards are a generous size- 3.67 x 4.25 inches- they fit perfectly in your hands and they aren’t so small that you have to use a magnifying glass to read tiny type- all questions and answers are in a normal-size print for easy studying * Our cards include in-depth explanationsyou won’t see any “one word” answers on our cards that require you to go get a textbook to understand why your answer was wrong- all of our cards include generous, thorough explanations so you not only get it right or wrongbut you also know why! * We use a font created by Microsoft to make reading easier- this will enable you to absorb more information painlessly during late night study sessions * Our system enables you to study in small, digestible bits of information- unlike using boring textbooks, flashcards turn learning into a “game” you can play until you’ve mastered the material * It’s easy for a friend to help you study- they don’t even have to know anything about the NCLEX- if they can read, then they can quiz you with our flashcards! Now, let me explain what the NCLEX Flashcard Study System is not. It is not a comprehensive review of your education. There’s no way we Collected by :DeepaRajesh [ 3 ] [email protected] Kuwait

could fit that onto flashcards- if we claimed to, it would be an insult to what you know. Don’t get us wrong: we’re not saying that memorization alone will automatically result in a passing NCLEX score- you have to have the ability to apply it as well. However, without the foundation of the core concepts, you cannot possibly hope to apply the information. After all, you can’t apply what you don’t know. NCLEX Flashcard Study System is a compilation of the 615+ critical concepts you must understand to pass the NCLEX. Nothing more, nothing less. Here’s Exactly What You Get With the NCLEX Flashcard Study System When you order the NCLEX Flashcard Study System, you’ll get our set of over 615 flashcards specially selected to give you the most NCLEX performance improvement for the least time. This is just a small sampling of the topics covered:

NCLEX Exam Topics:

* Types of Nosocomial Infections * Principles of Surgical Asepsis * Medical Testing and Labs * TURP Procedure * Romberg’s Test * Lithotripsy Procedure * Levels of Consciousness * Mental Exam Basics * Grading of Deep Tendon Reflexes * Glascow Coma Scale * Normative Values * Methods of Oxygen Delivery * Dementia and Delirium * Types of Injections * Ethical Duties of Nurses * Patient Rights * Bioethical Principles * Changes Associated with Aging * Drip Rate Calculations * Barriers to Communication * Nutrition and TPN * Attributes of Nutrients * Methods of Absorption * Metabolism and Nutrition * Medical Nutrition Therapy * Cultural Aspects of Diets * Placenta Previa * Stages of Labor

* Assessing Fetal Lung Maturity * Pathology of Eclampsia * PMS and Menopause * Attributes of Battered Women * Apgar Scores * Types of Cardiomyopathies * Opportunistic Infections * Classifications of Cancer * Medical Nutritional Therapy * Staging of Pressure Ulcers * Disease Pathology * Types of Shock * Lipid Profile Labs * Coagulation Studies * CBC Components * Acne Treatment Medications * Phases of Adolescence * Three Types of Jaundice * Pain Assessment * Lymphoma Characteristics * Sexually Transmitted Diseases * Tanner Staging * Vaccinations and Immunizations * Symptoms of Child Abuse * Performing Newborn Assessments * Motor Development * Development of Language * Pharmacology * Types of Adrenergic Receptors * Properties of Decongestants * Classifications of Drugs * Antipsychotic Classifications * Drug Interactions * Major Injection Sites * Calcium Channel Blockers * Phases of Burn Management * Types of Burns * Wound Healing Phases Click here to see 3 free NCLEX Flashcard Study System sample cards. Remember, this is just a small sampling of the topics covered in our system. Overall, you get over 615 premium-quality flashcards covering everything you’ll need to succeed on the NCLEX. The price for this package is only $39.95. Receive the Following Bonus Since I know it’s 100% to your benefit to use our flashcards, I want to sweeten the pot and give you every possible reason to say YES! With your order, you’ll also receive the following: Collected by :DeepaRajesh [ 4 ] [email protected] Kuwait

Special Report- The “Leitner Method” for Maximizing Flashcard Learning- in the 1970’s a German psychologist developed a learning system that turned the humble flashcard into an advanced learning technology. His method teaches you to learn faster by playing a simple game with your flashcards, with the help of a few items you probably have around your home. Simple to learn, but incredibly effective- our free report shows you exactly how to use his method in plain, easy-to-understand language. Note: we cannot guarantee that this free report will be available indefinitely, so act now to lock in your copy. By the way, this bonus is yours to keep even in the unlikely event you decide to take advantage of our ironclad money-back guarantee: You Cannot Lose With My No-Questions-Asked 1-Year Money-Back Guarantee We stand behind our offer with a no-questionsasked 1-year guarantee on our products. So go ahead and order your copy of the NCLEX Flashcard Study System today. Read them, study them, and profit from them. If you don’t think they are helping you prepare for your NCLEX exam - you can return them for an immediate 100% refund of your purchase price, no questions asked.

antee. So you have nothing to lose and everything to gain. My belief is simple: either this product helps you or you don’t pay. Period. No gimmicks, no asterisks.

NCLEX Medical Terminology Review Understanding the medical terminology used on the NCLEX should be a top priority when preparing for the NCLEX. Medical terms can sometimes be confusing due to the use of medical abbreviations. If you are unable to understand the medical terminology used on the NCLEX then you will have poor chance of picking The correct answer. Depending on your clinical rotations you may also be more familiar with certain medical terms in a specific area of nursing. Generally, nurses that have the broadest experience with medical terminology will have a better understanding to answer questions that contain complex medical terminology on the NCLEX test. Take time to review the following abbreviations on the NCLEX test as well as a more thorough list as found in the NCLEX study guide linked to the right hand side of this page.

ADH antidieuretic hormone AML acute myelogenous leukemia I think that speaks volumes about our confidence APC atrial premature contraction in our products. We are also members of the Bet- ASD atrial septal defect ter Business Bureau of Southeast Texas. BPH benign prostatic hypertrophy BUN blood, urea, nitrogen If you think there’s even the smallest chance that Ca calcium these flashcards will help you, you owe it to your- CA cancer self to try them out. Don’t let fear or doubt stand CAPD continuous ambulatory peritoneal dialysis in the way of what could be your best opportunity CC chief complaint to achieve the test score you need to fulfill the CPK creatine phosphokinase dream you deserve. CRP C-reactive protein DIFF differential blood count What I’m saying is, don’t decide now if these DOE dyspnea on exertion flashcards are for you. Just get them and try D/W dextrose in water them out. If they don’t do everything I say and ECT electroconvulsive therapy more, if you don’t save money, time and frustra- ESRD end stage renal disease tion, if they aren’t what you thought they were, FUO fever of undetermined origin if they don’t work for you, you have nothing to GH growth hormone worry about because you can get every dime of GSC glascow coma scale your money back under our no-loopholes guar- Hg mercury Collected by :DeepaRajesh [ 5 ] [email protected] Kuwait

HLA Hz ICS IPG JRA

human leukocyte antigen hertz intercostal space impedance plethysmogram juvenile rheumatoid arthritis

Practicing nurses have the luxury of being able to look up medial abbreviations and definitions before making patient care decisions. However, the NCLEX test does not allow that option. If you are confused by the medical terminology on the NCLEX, you will not be able to use a medical dictionary for reference purposes.

NCLEX-RN and NCLEX-PN is the different number of questions. Please review the following chart:



NCLEX-RN

NCLEX-PN

Minimum Number of Questions 75 85 Maximum Number of Questions 265 205

Both of these tests require the same basic understanding of nursing practice and knowledge. The NCLEX-RN questions and the NCLEX-PN questions are presented with four multiple choice answer scenarios. In some cases, the NCLEX Preparation NCLEX is using a more difficult Questionformat that requires multiple right The most important thing that you can do preanswers to be selected. However, the paring for the NCLEX is not stress out. A score in the 90th percentile is not required to pass the material that is covered is the same. NCLEX test. You only have to show a minimum level of competency in the field of nursing.

NCLEX-RN vs. NCLEX-PN

On the NCLEX-RN nursing students are required to concentrate for a longer period of time due to the higher number of questions. If the computer doesn’t turn off at the minimum number of questions, continue to answer each Questionin a reasonable amount of time. Do not begin to rush through the questions, because you may have to answer the maximum number of ques**The testing center will require at least 2 forms tions on the exam. Anticipate going the of identification to allow you to take the NCLEX distance and concentrating on each test. question. 1. Begin your preparation by sending in your application to the board of licensure. 2. Then schedule with the Chauncey Group for the exam. Finally, you will be sent authorization to test (ATT). 3. Next set-up a time that works for you and show up with all the required documents at the testing center for the NCLEX. If you are recently married with a name change, bring your marriage license. You may not need it, but if you did it could cause you to miss your testing time.

Don’t make the mistake of altering hair color or facial hair prior to the exam. Your picture has to match the application picture. You will also have to be thumb printed to take the NCLEX. In addition, bring a drink and some snack food for your testing break and wear layered clothing. Students that take the NCLEX in shorts and a tee shirt may find the testing center unbearable cold and be unable to concentrate.

Obviously, some of the questions related to delegation of responsibility are different between these exams. Registered nurses will be asked to assign tasks to practical nurses and nursing assistants while prioritizing their patients. Likewise practical nurses will be asked questions that require assigning tasks to nursing assistants and requesting more assistance from registered nurses.

NCLEX-RN vs. NCLEX-PN

Many of the study guides on the market have questions that help understand the delegation of responsibility task with nursing.

One of the primary differences between the Collected by :DeepaRajesh [ 6 ] [email protected] Kuwait

NCLEX Testing Recommendations 1. Watch out for the words: except, always and not in all NCLEX questions. 2. Answer each question. You are not penalized for taking an educated guess. 3. Get a good night’s sleep before the NCLEX. 4. Wear layered clothing to the exam. 5. Practice with a watch and bring a watch to the test. 6. Study for each material section of the NCLEX individually. 7. If you extremely weak in one area of content focus on that area. 8. Don’t cram for the NCLEX. Read over a good practice study guide at least one week in advance. 9. Stay away from negative talk about the NCLEX with other students. 10. Know the route to the testing center, 11. Double check that you have the appropriate ID prior to the NCLEX test. 12. Work through several practice tests prior to the exam. 1. A nurse is reviewing a patient’s medication during shift change. Which of the following medication would be contraindicated if the patient were pregnant? Note: More than one answer may be correct. A: Coumadin B: Finasteride C: Celebrex D: Catapress E: Habitrol F: Clofazimine 2. A nurse is reviewing a patient’s PMH. The history indicates photosensitive reactions to medications. Which of the following drugs has not been associated with photosensitive reactions? Note: More than one answer may be correct. A: Cipro B: Sulfonamide C: Noroxin D: Bactrim E: Accutane F: Nitrodur

look discolored. If you believe this change is due to medication, which of the following patient’s medication does not cause urine discoloration? A: Sulfasalazine B: Levodopa C: Phenolphthalein D: Aspirin 4. You are responsible for reviewing the nursing unit’s refrigerator. If you found the following drug in the refrigerator it should be removed from the refrigerator’s contents? A: Corgard B: Humulin (injection) C: Urokinase D: Epogen (injection) 5. A 34 year old female has recently been diagnosed with an autoimmune disease. She has also recently discovered that she is pregnant. Which of the following is the only immunoglobulin that will provide protection to the fetus in the womb? A: IgA B: IgD C: IgE D: IgG 6. A second year nursing student has just suffered a needlestick while working with a patient that is positive for AIDS. Which of the following is the most important action that nursing student should take? A: Immediately see a social worker B: Start prophylactic AZT treatment C: Start prophylactic Pentamide treatment D: Seek counseling 7. A thirty five year old male has been an insulindependent diabetic for five years and now is unable to urinate. Which of the following would you most likely suspect? A: Atherosclerosis B: Diabetic nephropathy C: Autonomic neuropathy D: Somatic neuropathy

3. A patient tells you that her urine is starting to 8. You are taking the history of a 14 year old girl Collected by :DeepaRajesh [ 7 ] [email protected] Kuwait

who has a (BMI) of 18. The girl reports inability to eat, induced vomiting and severe constipation. Which of the following would you most likely suspect? A: Multiple sclerosis B: Anorexia nervosa C: Bulimia D: Systemic sclerosis 9. A 24 year old female is admitted to the ER for confusion. This patient has a history of a myeloma diagnosis, constipation, intense abdominal pain, and polyuria. Which of the following would you most likely suspect? A: Diverticulosis B: Hypercalcaemia C: Hypocalcaemia D: Irritable bowel syndrome 10. Rho gam is most often used to treat____ mothers that have a ____ infant. A: RH positive, RH positive B: RH positive, RH negative C: RH negative, RH positive D: RH negative, RH negative

Answer Key 1. (A) and (B) are both contraindicated with pregnancy. 2. (F) All of the others have can cause photosensitivity reactions. 3. (D) All of the others can cause urine discoloration. 4. (A) Corgard could be removed from the refigerator. 5. (D) IgG is the only immunoglobulin that can cross the placental barrier. 6. (B) AZT treatment is the most critical innervention. 7. (C) Autonomic neuropathy can cause inability to urinate. 8. (B) All of the clinical signs and systems point to a condition of anorexia nervosa. 9. (B) Hypercalcaemia can cause polyuria, severe abdominal pain, and confusion. 10. (C) Rho gam prevents the production of antiRH antibodies in the mother that has a Rh positive fetus.

11. A new mother has some questions about (PKU). Which of the following statements made by a nurse is not correct regarding PKU? A: A Guthrie test can check the necessary lab values. B: The urine has a high concentration of phenylpyruvic acid C: Mental deficits are often present with PKU. D: The effects of PKU are reversible. 12. A patient has taken an overdose of aspirin. Which of the following should a nurse most closely monitor for during acute management of this patient? A: Onset of pulmonary edema B: Metabolic alkalosis C: Respiratory alkalosis D: Parkinson’s disease type symptoms 13. A fifty-year-old blind and deaf patient has been admitted to your floor. As the charge nurse your primary responsibility for this patient is? A: Let others know about the patient’s deficits. B: Communicate with your supervisor your patient safety concerns. C: Continuously update the patient on the social environment. D: Provide a secure environment for the patient. 14. A patient is getting discharged from a SNF facility. The patient has a history of severe COPD and PVD. The patient is primarily concerned about their ability to breath easily. Which of the following would be the best instruction for this patient? A: Deep breathing techniques to increase O2 levels. B: Cough regularly and deeply to clear airway passages. C: Cough following bronchodilator utilization D: Decrease CO2 levels by increase oxygen take output during meals. 15. A nurse is caring for an infant that has recently been diagnosed with a congenital heart defect. Which of the following clinical signs would most likely be present? Collected by :DeepaRajesh [ 8 ] [email protected] Kuwait

A: Slow pulse rate B: Weight gain C: Decreased systolic pressure D: Irregular WBC lab values 16. A mother has recently been informed that her child has Down’s syndrome. You will be assigned to care for the child at shift change. Which of the following characteristics is not associated with Down’s syndrome? A: Simian crease B: Brachycephaly C: Oily skin D: Hypotonicity 17. A patient has recently experienced a (MI) within the last 4 hours. Which of the following medications would most like be administered? A: Streptokinase B: Atropine C: Acetaminophen D: Coumadin

D: The life span of RBC is 120 days.

Answer Key 11-20. 11. (D) The effects of PKU stay with the infant throughout their life. 12. (D) Aspirin overdose can lead to metabolic acidosis and cause pulmonary edema development. 13. (D) This patient’s safety is your primary concern. 14. (C) The bronchodilator will allow a more productive cough. 15. (B) Weight gain is associated with CHF and congenital heart deficits. 16. (C) The skin would be dry and not oily. 17. (A) Streptokinase is a clot busting drug and the best choice in this situation. 18. (A) Green vegetables and liver are a great source of folic acid. 19. (D) Cl. difficile has not been linked to meningitis. 20. (D) RBC’s last for 120 days in the body.

18. A patient asks a nurse, “My doctor recommended I increase my intake of folic acid. What type of foods contain the highest concentration of folic acids?”

21. A 65 year old man has been admitted to the hospital for spinal stenosis surgery. When does the discharge training and planning begin for this patient?

A: Green vegetables and liver B: Yellow vegetables and red meat C: Carrots D: Milk

A: Following surgery B: Upon admit C: Within 48 hours of discharge D: Preoperative discussion

19. A nurse is putting together a presentation on meningitis. Which of the following microorganisms has noted been linked to meningitis in humans?

22. A child is 5 years old and has been recently admitted into the hospital. According to Erickson which of the following stages is the child in?

A: S. pneumonia B: H. influenza C: N. meningitis D: Cl. difficile 20. A nurse is administering blood to a patient who has a low hemoglobin count. The patient asks how long to RBC’s last in my body? The correct response is. A: The life span of RBC is 45 days. B: The life span of RBC is 60 days. C: The life span of RBC is 90 days.

A: Trust vs. mistrust B: Initiative vs. guilt C: Autonomy vs. shame D: Intimacy vs. isolation 23. A toddler is 16 months old and has been recently admitted into the hospital. According to Erickson which of the following stages is the toddler in? A: Trust vs. mistrust B: Initiative vs. guilt C: Autonomy vs. shame D: Intimacy vs. isolation Collected by :DeepaRajesh [ 9 ] [email protected] Kuwait

toacidosis. Which of the following would you not 24. A young adult is 20 years old and has been expect to see with this patient if this condition recently admitted into the hospital. According were acute? to Erickson which of the following stages is the adult in? A: Vomiting B: Extreme Thirst A: Trust vs. mistrust C: Weight gain B: Initiative vs. guilt D: Acetone breath smell C: Autonomy vs. shame D: Intimacy vs. isolation 30. A patient’s chart indicates a history of meningitis. Which of the following would you not ex25. A nurse is making rounds taking vital signs. pect to see with this patient if this condition were Which of the following vital signs is abnormal? acute? A: 11 year old male – 90 b.p.m, 22 resp/min. , 100/70 mm Hg B: 13 year old female – 105 b.p.m., 22 resp/min., 105/60 mm Hg C: 5 year old male- 102 b.p.m, 24 resp/min., 90/65 mm Hg D: 6 year old female- 100 b.p.m., 26 resp/min., 90/70mm Hg 26. When you are taking a patient’s history, she tells you she has been depressed and is dealing with an anxiety disorder. Which of the following medications would the patient most likely be taking? A: Elavil B: Calcitonin C: Pergolide D: Verapamil 27. Which of the following conditions would a nurse not administer erythromycin? A: Campylobacterial infection B: Legionnaire’s disease C: Pneumonia D: Multiple Sclerosis

A: Increased appetite B: Vomiting C: Fever D: Poor tolerance of light

Answer Key 21-30. 21. (B) Discharge education begins upon admit. 22. (B) Initiative vs. guilt- 3-6 years old 23. (A) Trust vs. Mistrust- 12-18 months old 24. (D) Intimacy vs. isolation- 18-35 years old 25. (B) HR and Respirations are slightly increased. BP is down. 26. (A) Elavil is a tricyclic antidepressant. 27. (D) Erythromycin is used to treat conditions A-C. 28. (D) Answer choices A-C were symptoms of acute hyperkalemia. 29. (C) Weight loss would be expected. 30. (A) Loss of appetite would be expected. 31. A nurse if reviewing a patient’s chart and notices that the patient suffers from conjunctivitis. Which of the following microorganisms is related to this condition?

A: Yersinia pestis B: Helicobacter pyroli 28. A patient’s chart indicates a history of hyper- C: Vibrio cholera kalemia. Which of the following would you not D: Hemophilus aegyptius expect to see with this patient if this condition were acute? 32. A nurse if reviewing a patient’s chart and notices that the patient suffers from Lyme disease. A: Decreased HR Which of the following microorganisms is related B: Paresthesias to this condition? C: Muscle weakness of the extremities D: Migranes A: Borrelia burgdorferi B: Streptococcus pyrogens 29. A patient’s chart indicates a history of ke- C: Bacilus anthracis Collected by :DeepaRajesh [ 10 ] [email protected] Kuwait

D: Enterococcus faecalis 33. A fragile 87 year-old female has recently been admitted to the hospital with increased confusion and falls over last 2 weeks. She is also noted to have a mild left hemiparesis. Which of the following tests is most likely to be performed? A: FBC (full blood count) B: ECG (electrocardiogram) C: Thyroid function tests D: CT scan 34. A 84 year-old male has been loosing mobility and gaining weight over the last 2 months. The patient also has the heater running in his house 24 hours a day, even on warm days. Which of the following tests is most likely to be performed? A: FBC (full blood count) B: ECG (electrocardiogram) C: Thyroid function tests D: CT scan

D: Frequent attempts with positive reinforcement. 38. A parent calls the pediatric clinic and is frantic about the bottle of cleaning fluid her child drank 20 minutes. Which of the following is the most important instruction the nurse can give the parent? A: This too shall pass. B: Take the child immediately to the ER C: Contact the Poison Control Center quickly D: Give the child syrup of ipecac 39. A nurse is administering a shot of Vitamin K to a 30 day-old infant. Which of the following target areas is the most appropriate? A: Gluteus maximus B: Gluteus minimus C: Vastus lateralis D: Vastus medialis

40. A nurse has just started her rounds delivering 35. A 20 year-old female attending college is medication. A new patient on her rounds is a 4 found unconscious in her dorm room. She has a year-old boy who is non-verbal. This child does fever and a noticeable rash. She has just been not have on any identification. What should the admitted to the hospital. Which of the following nurse do? tests is most likely to be performed first? A: Contact the provider A: Blood sugar check B: Ask the child to write their name on paper. B: CT scan C: Ask a co-worker about the identification of the C: Blood cultures child. D: Arterial blood gases D: Ask the father who is in the room the child’s name. 36. A 28 year old male has been found wandering around in a confused pattern. The male is Answer Key 31-40. sweaty and pale. Which of the following tests is most likely to be performed first? 31. (D) Choice A is linked to Plague, Choice B is linked to peptic ulcers, Choice C is linked to A: Blood sugar check Cholera. B: CT scan 32. (A) Choice B is linked to Rheumatic fever, C: Blood cultures Choice C is linked to Anthrax, Choice D is linked D: Arterial blood gases to Endocarditis. 33. (D) A CT scan would be performed for further 37. A mother is inquiring about her child’s ability investigation of the hemiparesis. to potty train. Which of the following factors is the 34. (C) Weight gain and poor temperature tolermost important aspect of toilet training? ance indicate something may be wrong with the thyroid function. A: The age of the child 35. (C) Blood cultures would be performed to inB: The child ability to understand instruction. vestigate the fever and rash symptoms. C: The overall mental and physical abilities of the 36. (A) With a history of diabetes, the first rechild. sponse should be to check blood sugar levels. Collected by :DeepaRajesh [ 11 ] [email protected] Kuwait

37. (C) Age is not the greatest factor in potty training. The overall mental and physical abilities of the child is the most important factor. 38. (C) The poison control center will have an exact plan of action for this child. 39. (C) Vastus lateralis is the most appropriate location. 40. (D) In this case you are able to determine the name of the child by the father’s statement. You should not withhold the medication from the child following identification.

NCLEX Skeletal Muscle Review

2. Nervous Tissue: Nervous tissue is composed of cells (neurons) that respond to external and internal stimuli and have the capability to transmit a message (impulse) from one area of the body to another. This tissue thus induces a response of distant muscles or glands, as well as regulating body processes such as respiration, circulation, and digestion. 3. Epithelial Tissue: Epithelial tissue covers the external surfaces of the body and lines the internal tubes and cavities. It also forms the glands of the body. Characteristics of epithelial tissue (epithelium) are that it

1. has compactly aggregated cells; In order for the human being to carry out the many 2. has limited intercellular spaces and subintricate movements that must be performed, approximately 650 skeletal muscles of various stance; 3. is avascular (no blood vessels); lengths, shapes, and strength play a part. Each 4. lies on a connective tissue layer—the muscle consists of many muscle cells or fibers held together and surrounded by connective tis- basal lamina; 5. has cells that form sheets and are polarsue that gives functional integrity to the system. ized; Three definite units are commonly referred to: 6. is derived from all three germ layers. 1. endomysium—connective tissue layer enMicrovilli—fingerlike projections of plasma veloping a single fiber; membranes. 2. perimysium—connective tissue layer enveloping a bundle of fibers; Cilia—motile organelles extending into the 3. epimysium—connective tissue layer envelluman consisting of specifically arranged microoping the entire muscle tubules.

Muscle Attachment and Function

Flagella—similar to cilia. Primary examples are human spermatozoa.

For coordinated movement to take place, the muscle must attach to either bone or cartilage Stereocilia—are actually very elongated Mior, as in the case of the muscles of facial exprescrovilli. sion, to skin. The portion of a muscle attaching 4. Connective Tissue: Connective tissue is the to bone is the tendon. A muscle has two extremipacking and supporting material of the body tisties, its origin and its insertion. sues and organs. It develops from mesoderm (mesenchyme). All connective tissues consist of NCLEX Four Basic Tissues three distinct components: ground substance, cells and fibers. 1. Muscle Tissue: Muscle tissue is contractile in nature and functions to move the skeletal system * Ground substance. Ground substance and body viscera. is located between the cells and fibers, both of which are embedded in it. It forms an amorphous Type Characteristics Location intercellar material. In the fresh state, it appears Skeletal Striated, voluntary Skeletal musas a transparent and homogenous gel. It acts as cles of the body a route for the passage of nutrients and wastes Smooth Non-striated, involuntary W a l l s to and from the cells within or adjacent to the of digestive tract and blood vessels, uterus, uriconnective tissue. nary bladder * Fibers. The fiber components of conCardiac Striated, involuntary Heart Collected by :DeepaRajesh [ 12 ] [email protected] Kuwait

nective tissue add support and strength. Three Thalamus Contralateral thalamus pain, contypes of fibers are present: collagenous, elastic tralateral hemisensory loss and reticular. Pineal gland Early hydrocephalus, papillary abnormalities, Parinaud’s syndrome Internal capsule Hemisensory loss, homonyNCLEX Cardiac Review mous hemianopsia, contralateral hemiplegia Contralateral dystonia, ConThe heart is a highly specialized blood vessel Basal ganglia which pumps 72 times per minute and propels tralateral choreoathetosis about 4,000 gallons (about 15,000 liters) of blood Pons Diplopia, internal strabismus, VI and VII involvement, contralateral hemisensory and daily to the tissues. It is composed of: hemiparesis loss, issilateral cerebellar ataxia Broca’s area Motor dysphasia Endocardium (lining coat; epithelium) Precentral gyrus Jacksonian seizures, generalized seizures, hemiparesis Myocardium (middle coat; cardiac muscle) Superficial parietal lobe Receptive dysphasia Epicardium (external coat or visceral layer of pericardium; epithelium and mostly connective tissue)

Impulse conducting system Cardiac Nerves: Modification of the intrinsic rhythmicity of the heart muscle is produced by cardiac nerves of the sympathetic and parasympathetic nervous system. Stimulation of the sympathetic system increases the rate and force of the heartbeat and dilates the coronary arteries. Stimulation of the parasympathetic (vagus nerve) reduces the rate and force of the heartbeat and constricts the coronary circulation. Visceral afferent (sensory) fibers from the heart end almost wholly in the first four segments of the thoracic spinal cord. Cardiac Cycle: Alternating contraction and relaxation is repeated about 75 times per minute; the duration of one cycle is about 0.8 second. Three phases succeed one another during the cycle: a) atrial systole: 0.1 second, b) ventricular systole: 0.3 second, c) diastole: 0.4 second The actual period of rest for each chamber is 0.7 second for the atria and 0.5 second for the ventricles, so in spite of its activity, the heart is at rest longer than at work.

NCLEX Lesion Review

Occipital Lobe Homonymous hemianopsia, partial seizures with limited visual phenomena

NCLEX Tumor Review Primary Tumors

* Neuromas- 80-90% of brain tumors, named for what part of nerve cell affected. * Meningiomas- outside of arachnoidal tissue, usually benign and slow growing * Glioblastoma Multiform-50% of all primary tumors, linked to specific genetic mutations Secondary Tumors * Metastatic carcinomas Scale –degree of anaplasia: differentiation of mature (good) vs. immature cells (bad) Grade I: up to 25% anaplasia Grade II: 26-50% anaplasia Grade III: 51-75% anaplasia Grade IV: 76-100% anaplasia Primary Tumor Effect: 1. Headaches 2. Vomiting

Secondary Tumor Effect: 1. Direct compression/necrosis 2. Herniation of brain tissue Collected by :DeepaRajesh [ 13 ] [email protected] Kuwait

3. Increase ICP Noteworthy Tumor Markers 1. AFP 2. Alkaline phosphatase 3. b-hCG 4. CA-125 5. PSA

NCLEX Movement Terms Flexion is bending, most often ventrally to decrease the angle between two parts of the body; it is usually an action at an articulation or joint. Extension is straightening, or increasing the angle between two parts of the body; a stretching out or making the flexed part straight. Abduction is a movement away from the midsagittal plane (midline); to adduct is to move medially and bring a part back to the mid-axis.

Opposition is one of the most critical movements in humans; it allows us to have pulp-to-pulp opposition, which gives us the great dexterity of our hands. In this movement the thumb pad is brought to a finger pad. A median nerve injury negates this action.

NCLEX Cell Structure Review Endoplasmic Reticulum ( ER) This cellular organelle was first described using phase microscopy by Porter, Claude and Fallam in 1945. It is an extensive network of interconnecting channels. The endoplasmic reticular membranes are unit membranes (triminar). When ribosomes line the outer surface it is designated as rough endoplasmic reticulum ( RER). The primary form of this organelle is the rough variety. The smooth is derived from the rough due to loss of ribosomes. The amount of each depends on the cell type and the cellular activity. The RER is the synthetic machinery of the cell. It is mainly concerned with protein synthesis.

Circumduction is a circular movement at a ball and socket (shoulder or hip) joint, utilizing the movements of flexion, extension, abduction, and The Golgi Complex adduction. This structure was discovered by Camillo Golgi Rotation is a movement of a part of the body in 1898. All eukaryotic cells, except for the red around its long axis. blood cell, possess a Golgi apparatus. Generally speaking the Golgi complex is prominent in glanSupination refers only to the movement of the dular cells and is thought to function in the proradius around the ulna. In supination the palm of duction, concentration packaging, and transporthe hand is oriented anteriorly; turning the palm tation of secretory material. IN summary one can dorsally puts it into pronation. The body on its link the Golgi complex to: secretion, membrane back is in the supine position. biogenesis, lysosome formation, membrane recycling, hormone modulation. Pronation refers to the palm of the hand being oriented posteriorly. The body on its belly is the Lysosome prone position. Inversion refers only to the lower extremity, specifically the ankle joint. When the foot (plantar surface) is turned inward, so that the sole is pointing and directed toward the midline of the body and is parallel with the median plane, we speak of inversion. Its opposite is eversion. Eversion refers to the foot (plantar surface) being turned outward so that the sole is pointing laterally.

Lysosomes are described as containing proteolytic enzymes (hydrolases).Lysosomes contain acid phosphatase and other hydrolytic enzymes.. These enzymes are enclosed by a membrane and are released when needed into the cell or into phagocytic vesicles. Lysosomal enzymes have the capacity to hydrolyze all classes of macromolecules. A generalized list of substrates acted upon by respective enzymes is given below: Collected by :DeepaRajesh [ 14 ] [email protected] Kuwait

Lipids by lipases and phospholipases; Proteins by proteases or peptidases; Polysaccharides by glycosidases;

NCLEX Cranial Nerve Review I-Olfactory-Smell

Nucleic acids by nucleases;

II-Optic-Vision acuity

Phosphates ( organic-linked) by phosphatases;

III-Oculomotor – Eye function

Sulphates (organic-linked) by sulfatases.

IV-Trochlear – Eye function

NCLEX Definitions

V-Trigeminal – Sensory of the face, chewing

Pachyderma-increased thickness of the skin

VI-Abducens – Eye function

Paroxysm-sudden attack

VII-Facial – Facial expression, wrinkle forehead, taste anterior tongue

Pathogenic-disease causing Pathologist-individual who studies pathology Pediculosis-condition of lice Percutaneous- penetrating the skin Pleuritis-inflammation of the pleura Pneumonia-disease of the lung related to infection Pneumoconiosis-dust in the lung’s condition Pneumothorax-air in the chest resulting in the collapse of a lung Pneumatocele-hernia associated with the lung Posterior-related to the rear/back position Prognosis-opinion of an individual about outcomes Pruritus-uncontrollable itching Pyelolithotomy-incision to remove a stone from the renal pelvis Pyeloplasty-repair of the renal pelvis Pyosalpinx-pus in the fallopian tube Pyuria-pus in the urine

VIII-Vestibulocochlear – Auditory acuity, balance and postural responses IX-Glossopharyngeal – Taste on posterior 33% of the scale X-Vagus – Cardiac, respiratory reflexes XI-Spinal Accessory - Strength of trapezius and Sternocleidomastoid muscles XII-Hypoglossal – Motor function of the tongue

NCLEX Cholinomimetrics 1. Muscarinic Agonists A. Bethanecol (URECHOLINE) – increase GI motility B. Carbachol (ISOPTO, MIOSTAT, CARBACHOL) – various types of glaucoma C. Methacholine (PROVOCHOLINE) – test hyperactivity of airways D. Pilocarpine – used for glaucoma 2. Anticholinesterases A. Pysostigmine (ANTILIRIUM) – treat glaucoma, crosses BBB, reverse anticholinergic toxicity. B. Neostigmine (PROSTIGMIN) – synthetic form of Pysostigmine (Anticholinesterases) – used for Myasthenia gravis, glaucoma, and to increase tone in bladCollected by :DeepaRajesh [ 15 ] [email protected] Kuwait

der

Symptoms of Anticholinesterase toxicity: 1. Miosis 2. Rhinitis 3. Bradycardia 4. GI spasms 5. brochoconstriction 6. involuntary voiding of urine

NCLEX Drug Distribution Review Bioavailability dependant on several things: 1. Route of administration 2. The drug’s ability to cross membranes 3. The drug’s binding to plasma proteins and intracellular component

Membrane Review: 1. Membranes separate the body in components 2. The ability of membranes to act as barriers is related to its structure 3. Lipid Soluable compounds (many drugs) pass through by becoming dissolved in the lipid bylayer. 4. Glucose, H20, electrolytes can’t pass on their own. They use pores. 5. In excitable tissues, the pores open and close. 1. Movement occurs by: 2. passive diffusion 3. active transport 4. facilitated diffusion 5. endocytosis

Passive Diffusion Review: 1. No energy expended. 2. Weak acids and bases need to be in nonionized form (no net charge). 3. Drugs can also move between cell junctions. BBB is exception. 4. Must be lipid soluable to pass through pores. 5. Osmosis is a special case of diffusion 1. A drug dissolved in H2O will move with

the water by “bulk flow” 2. Usually limited to movement through gap junctions because size too large for pores. Active Transport Review: 1. Requires energy and requires a transport protein 2. Drugs must be similar to some endogenous substance. 3. Can carry substances against a gradient 4. Some drugs may exert their effect by increasing or decreasing transport proteins.

Facilitated Diffusion Review: 1. Requires transport protein 2. Does not require energy 3. Very few drugs move this way

Endocytosis: 1. Drug gets engulfed by cell via invagination 2. Very few drugs move this way and only in certain cells. Regulation of distribution determined by: 1. Lipid permeability 2. Blood flow 3. Binding to plasma proteins 4. Binding to subcellular components Volume of Distribution (Vd) - is a calculation of where the drug is distributed. Vd = amount of drug given (mg) concentration in plasma (mg/ml) Calculate the Vd and compare to the total amount of body H20 in a person. -if Vd = total amount of body (approx. 42) is uniformly distributed -if Vd is less than 42 – retained in plasma and probably bound to plasma proteins -if Vd is more than 42 – concentrated in tissues This is not a “real value” but tells you where the drug is being distributed. Collected by :DeepaRajesh [ 16 ] [email protected] Kuwait

Placental Transfer of Drugs 1. Some drugs cause congenital anomalies 2. Cross placenta by simple diffusion 3. Must be polar or lipid-insoluable Not to Enter 4. Must assume the fetus is subjected to all drugs taken by the mother to some extent.

NCLEX Score Review Usually, NCLEX scores are sent out 2-6 weeks after the NCLEX test. Each state board is different with score reporting. Don’t call the state boards for a score report.

NCLEX Score Review

NCLEX Study Guide Recommendation The amount of effort required to pass the NCLEX varies between students. There is no magic formula that students can get plugged into to pass the NCLEX test . Consequently, some students may be able pass without reviewing any reference materials, and other will buy 5-7 reference guides and attend NCLEX review courses that are extremely expensive. Obviously, it is foolish to take the NCLEX without any review of potential questions and NCLEX content. The application fee and a failing score simply add up to frustration.

NCLEX Study Guide Recommendation

The NCLEX test takers should be aware of If you do not pass the NCLEX don’t lose hope. the fact that sometimes too much information Thousands of good nurses have missed a pass- on the NCLEX can lead to “overkill”. The right ing score by a few questions. Any given day a study guide offers brevity, precision, and no fluff. good student can make a few mistakes. You will NCLEX test takers should be aware of the many be required to wait 3 months to retest for the pitfalls of NCLEX preparation. The NCLEX will be the most important test nursing students take NCLEX. in the process of becoming a nurse. If you don’t pass the NCLEX ask yourself the folIf you memorize facts piled upon facts about palowing questions. tient care in a cram course before the NCLEX, you will most likely be distracted by the details 1. Did I know the material on the NCLEX? and not focus on the concepts. A concise review 2. Did I feel comfortable with testing format? 3. Was there something else going on in my life of the NCLEX that helps you recall details without giving you all the minute details will work the that was a distracter? 4. Did I feel like I was guessing on the NCLEX best, because you will be able to think on test day and not be attempting to regurgitate data. test?

5. Did I give each Questionmy best effort?

6. Did I focus on the practical and not book knowledge regarding patient care? 7. Did I prepare for the minimum number of questions and not the maximum on the NCLEX test? Use the above questions to improve your focus for your NCLEX preparation. Hundreds if not thousands of nursing students have never gone back to take the NCLEX test the second time. The NCLEX is just another hurdle in a series of qualifying requirements to become a nurse. Don’t lose hope and become a better nurse by pursing knowledge that will help you pass the NCLEX. Collected by :DeepaRajesh [ 17 ] [email protected] Kuwait

Free NCLEX-RN Sample Test Questions

Collected by :DeepaRajesh [ 18 ] [email protected] Kuwait

1. The correct answer is B.

Question: What are the needs of the patient with Needed Info: Mask, eye protection, face shield acute lymphocytic leukemia and thrombocytope- protect mucous membrane exposure; used if activities are likely to generate splash or sprays. nia? Gowns used if activities are likely to generate Needed Info: Lymphocytic leukemia, disease splashes or sprays. characterized by proliferation of immature WBCs. Immature cells unable to fight infection as com- (A) Gloves, gown, goggles, and surgical cap — petently as mature white cells. Treatment: chem- surgical caps offer protection to hair but aren>t otherapy, antibiotics, blood transfusions, bone required. marrow transplantation. Nursing responsibilities: (B) Sterile gloves, mask, plastic bags, and gown private room, no raw fruits or vegs, small fre- — plastic bags provide no direct protection and aren>t part of universal precautions quent meals, O2, good skin care. (C) Gloves, gown, mask, and goggles — COR(A) to a private room so she will not infect other RECT: must use universal precautions on ALL patients and health care workers — poses little patients; prevent skin and mucous membrane exposure when contact with blood or other body or no threat (B) to a private room so she will not be infected by fluids is anticipated other patients and health care workers — COR- (D) Double gloves, goggles, mask, and surgical RECT: protects patient from exogenous bacteria, cap — surgical cap not required; unnecessary to risk for developing infection from others due to double glove depressed WBC count, alters ability to fight in- 4. The correct answer is B. fection Question: What is the best position after tonsillec(C) to a semiprivate room so she will have stim- tomy to help with drainage of oral secretions? ulation during her hospitalization — should be placed in a room alone Strategy: Picture the patient as described. (D) to a semiprivate room so she will have the opportunity to express her feelings about her ill- (A) Sims> — on side with top knee flexed and ness — ensure that patient is provided with op- thigh drawn up to chest and lower knee less portunities to express feelings about illness sharply flexed: used for vaginal or rectal examination (B) Side-lying — CORRECT: most effective to 2. The correct answer is A. Question: What is the BEST way to prevent ac- facilitate drainage of secretions from the mouth and pharynx; reduces possibility of airway obcidental poisoning in children? struction. (C) Supine — increased risk for aspiration, would Strategy: Picture toddlers at play. not facilitate drainage of oral secretions (A) Lock all medications in a cabinet — COR- (D) Prone — risk for airway obstruction and asRECT: improper storage most common cause of piration, unable to observe the child for signs of bleeding such as increased swallowing poisoning; highest incidence in two-year-olds (B) Child proof all the caps to medication bottles 5. The correct answer is A. — children can open Question: Which patient is an appropriate as(C) Store medications on the highest shelf in a signment for the LPN/LVN? cupboard — toddlers climb (D) Place medications in different containers — Strategy: Think about the skill level involved in keep in original container each patient>s care.

3. The correct answer is C.

Question: What is the correct universal precaution? Strategy: Think about each answer choice. How is each measure protecting the nurse?

Needed Info: LPN/LVN: assists with implementation of care; performs procedures; differentiates normal from abnormal; cares for stable patients with predictable conditions; has knowledge of asepsis and dressing changes; administers medications (varies with educational background and Collected by :DeepaRajesh [ 19 ] [email protected] Kuwait

state nurse practice act). (A) A 72-year-old patient with diabetes who requires a dressing change for a stasis ulcer — CORRECT: stable patient with an expected outcome (B) A 42-year-old patient with cancer of the bone complaining of pain — requires assessment; RN is the appropriate caregiver (C) A 55-year-old patient with terminal cancer being transferred to hospice home care — requires nursing judgement; RN is the appropriate caregiver (D) A 23-year-old patient with a fracture of the right leg who asks to use the urinal — standard unchanging procedure; assign to the nursing assistant

Health Promotion and Maintenance 1. The answer is B.

Question: The fetus is ROA. Where should the nurse listen for the FHT? Strategy: Picture the situation described. It may be helpful for you to draw this out so that you can imagine where the heartbeat would be found. Needed Info: Describing fetal position: practice of defining position of baby relative to mother>s pelvis. The point of maximum intensity (PMI) of the fetus: point on the mother>s abdomen where the FHT is the loudest, usually over the fetal back. Divide the mother>s pelvis into 4 parts or quadrants: right and left anterior, which is the front, and right and left posterior, which is the back. Abbreviated: R and L for right and left, and A and P for anterior and posterior. The head, particularly the occiput, is the most common presenting part, and is abbreviated O. LOA is most common fetal position and FHT heard on left side. In a vertex presentation, FHT is heard below the umbilicus. In a breech presentation, FHT is heard above the umbilicus.

(C) Above the umbilicus, on the mother>s left side — found in breech presentation (D) Above the umbilicus, on the mother>s right side — found in breech presentation

2. The correct answer is B.

Question: What is a contraindication to receiving flu vaccine? Strategy: Think about what each answer choice means. Needed Info: Influenza vaccine: given yearly, preferably Oct.-Nov.; recommended for people age 65 or older; people under 65 with heart disease, lung disease, diabetes, immuno-suppression, chronic care facility residents. (A) A 45-year-old male who is allergic to shellfish — allergy to eggs is a contraindication (B) A 60-year-old female who says she has a sore throat — CORRECT: vaccine deferred in presence of acute respiratory disease (C) A 66-year-old female who lives in a group home — vaccine deferred only if patient has an active immunization (D) A 70-year-old female with congestive heart failure — no contraindication

3. The correct answer is D.

Question: What is the treatment for thrush? Strategy: Determine the outcome of each answer choice. Needed Info: Thrush (oral candidiasis): white plaque on oral mucous membranes, gums, or tongue; treatment includes good handwashing, nystatin (Mycostatin).

(A) Determine the baby>s blood glucose level — thrush in newborns caused by poor handwashing or exposure to an infected vagina during birth (B) Suggest that the newborn>s formula be changed — not related to thrush (C) Remind the caretaker not to let the infant sleep with the bottle — not related to thrush (D) Explain that the newborn will need to receive some medication — CORRECT: thrush most of(A) Below the umbilicus, on the mother>s left ten treated with nystatin (Mycostatin) side — found on right not left side 4. The correct answer is C. (B) Below the umbilicus, on the mother>s right Question: What will you see with congenital hip side — CORRECT: occiput and back are press- dislocation? ing against right side of mother>s abdomen; FHT would be heard below umbilicus on right side Strategy: Form a mental image of the deformity. Collected by :DeepaRajesh [ 20 ] [email protected] Kuwait

Needed Info: Subluxation: most common type of congenital hip dislocation. Head of femur remains in contact with acetabulum but is partially displaced. Diagnosed in infant less than 4 weeks old S/S: unlevel gluteal folds, limited abduction of hip, shortened femur affected side, Ortolani>s sign (click). Treatment: abduction splint, hip spica cast, Bryant>s traction, open reduction. (A) lengthening of the limb on the affected side — inaccurate (B) deformities of the foot and ankle — inaccurate (C) asymmetry of the gluteal and thigh folds — CORRECT: restricted movement on affected side (D) plantar flexion of the foot — seen with clubfoot

5. The correct answer is D.

Question: How do you determine the frequency of uterine contractions? Needed Info: There must be at least 3 contractions to establish frequency. (A) from the beginning of one contraction to the end of the next contraction — not accurate (B) from the beginning of one contraction to the end of the same contraction — defines duration (C) by the strength of the contraction at its peak — describes intensity (D) by the number of contractions that occur within a given period of time — CORRECT

2. The correct answer is B.

Question: What is your responsibility concerning informed consent? Needed Info: Physician>s responsibility to obtain informed consent. (A) The nurse should explain the procedure to the patient and ask her to sign the consent form — Physician should get patient to sign consent (B) The nurse should verify that the consent form has been signed by the patient and that it is attached to her chart — CORRECT (C) The nurse should tell the physician that the patient agrees to have the examination — Physician should explain procedure and get consent form signed (D) The nurse should verify that the patient or a family member has signed the consent form — must be signed by patient unless unable to do

3. The correct answer is C.

Question: What should you do to communicate with a person with a moderate hearing loss? Needed Info: Presbycusis: age-related hearing loss due to inner ear changes. Decreased ability to hear high sounds.

(A) Raise your voice until the patient is able to hear you — would result in high tones patient unable to hear (B) Face the patient and speak quickly using a high voice — usually unable to hear high tones (C) Face the patient and speak slowly using a slightly lowered voice — CORRECT: also decrease background noise; speak at a slow pace, 1. The correct answer is C. use nonverbal cues (D) Use facial expressions and speak as you Question: What is the goal of family therapy? would normally — nonverbal cues help, but need Needed Info: Symptoms of depression: a low low tones self-esteem, obsessive thoughts, regressive be- 4. The correct answer is C. havior, unkempt appearance, a lack of energy, weight loss, decreased concentration, withdrawn Question: What is the reason for the wife>s bebehavior. havior? (A) trust the nurse who will solve his problem — not realistic (B) learn to live with anxiety and tension — minimizes concerns (C) accept responsibility for his actions and choices — CORRECT (D) use the members of the therapeutic milieu to solve his problems — must do it himself

Needed Info: Stages of grief: 1) shock and disbelief, 2) awareness of pain and loss, 3) restitution. Acute period: 4-8 weeks, usual resolution: 1 year. (A) She has already moved through the stages of the grieving process — takes one year (B) She is repressing anger related to her Collected by :DeepaRajesh [ 21 ] [email protected] Kuwait

husband>s death — not accurate; second stage: crying, regression (C) She is experiencing shock and disbelief related to her husband>s death — CORRECT: denial first stage; inability to comprehend reality of situation (D) She is demonstrating resolution of her husband>s death — too soon

acute lymphocytic leukemia and thromocytopenia?

Needed Info: Thromocytopenia: decreased platelet count increases the patient>s risk for injury, normal count: 200,000-400,000 per mm3. Leukemia: group of malignant disorders involving overproduction of immature leukocytes in bone marrow. This shuts down normal bone mar5. The correct answer is C. row production of erythrocytes, platelets, normal Question: Is the depression normal, or some- leukocytes. Causes anemia, leukopenia, and thrombocytopenia leading to infection and hemthing to be concerned about? orrhage. Symptoms: pallor of nail beds and con(A) The treatment plan is not effective; the patient junctiva, petechiae (small hemorrhagic spot on requires a larger dose of lithium — not accurate skin), tachycardia, dyspnea, weight loss, fatigue. (B) This is a normal response to lithium therapy; Treatment: chemotherapy, antibiotics, blood the patient should continue with the current treat- transfusions, bone marrow transplantation. Nursing responsibilities: private room, no raw fruits or ment plan — does not address safety needs (C) This is a normal response to lithium therapy; vegs, small frequent meals, O2, good skin care. the patient should be monitored for suicidal behavior — CORRECT: delay of 1-3 weeks before (A) Potential for injury — CORRECT: low platelet increases risk of bleeding from even minor med benefits seen (D) The treatment plan is not effective; the patient injuries. Safety measures: shave with an elecrequires an antidepressant — normal response tric razor, use soft tooth brush, avoid SQ or IM meds and invasive procedures (urinary drainage catheter or a nasogastric tube), side-rails up, remove sharp objects, frequently assess for signs Physiological Integrity of bleeding, bruising, hemorrhage. 1. The correct answer is B. (B) Self-care deficit — may feel weak, doesn>t Question: Which lab values should you monitor address condition for a patient receiving Gentamicin? (C) Potential for self-harm — implies risk for purposeful self-injury, not given any info, assumpNeeded Info: Gentamicin: broad spectrum an- tion tibiotic. Side effects: neuromuscular blockage, (D) Alteration in comfort — patient is not comototoxic to eighth cranial nerve (tinnitus, vertigo, fortable, and comfort measures would address ataxia, nystagmus, hearing loss), nephrotoxic. problem Nursing responsibilities: monitor renal function, 3. The correct answer is C. force fluids, monitor hearing acuity. Draw blood Question: What is the best site for nitroglycerine for peak levels 1 hr. after IM and 30 min - 1 hr. afointment? ter IV infusion, draw blood for trough just before next dose. Strategy: Think about each site. (A) Hemoglobin and hematocrit — can cause anemia; less common (B) BUN and creatinine — CORRECT: nephrotoxic; will see proteinuria, oliguria, hematuria, thirst, increased BUN, decreased creatine clearance (C) Platelet count and clotting time — do not usually change (D) Sodium and potassium — hypokalemia infrequent problem

2. The correct answer is A.

Question: What nursing diagnosis is seen with

Needed Info: Nitroglycerine: used in treatment of angina pectoris to reduce ischemia and relieve pain by decreasing myocardial oxygen consumption; dilates veins and arteries. Side effects: throbbing headache, flushing, hypotension, tachycardia. Nursing responsibilities: teach appropriate administration, storage, expected pain relief, side effects. Ointment applied to skin; sites rotated to avoid skin irritaion. Prolonged effect up to 24 hours. Collected by :DeepaRajesh [ 22 ] [email protected] Kuwait

(A) muscular — not most important (B) near the heart — not most important (C) non-hairy — CORRECT: skin site free of hair will increase absorption; avoid distal part of extremities due to less than maximal absorption (D) over a bony prominence — most important is that the site be non-hairy

4. The correct answer is B.

Question: Why is a patient defibrillated? Strategy: Think about each answer choice. Needed Info: Defibrillation: produces asystole of heart to provide opportunity for natural pacemaker (SA node) to resume as pacer of heart activity. (A) increase cardiac contractility and cardiac output — inaccurate (B) cause asystole so the normal pacemaker can recapture — CORRECT: allows SA node to resume as pacer of heart activity (C) reduce cardiac ischemia and acidosis — inaccurate (D) provide energy for depleted myocardial cells — inaccurate

5. The correct answer is C.

Question: How should you regulate the IV flow rate? Strategy: Use formula and avoid making math errors. Needed Info: total volume x the drop factor divided by the total time in minutes. (A) 21 — inaccurate (B) 28 — inaccurate (C) 31 — CORRECT: 3,000 x 15 divided by 24 x 60 (D) 42 — inaccurate

Question1

A client has been hospitalized after an automobile accident. A full leg cast was applied in the emergency room. The most important reason for the nurse to elevate the casted leg is to A) Promote the client>s comfort B) Reduce the drying time C) Decrease irritation to the skin D) Improve venous return

Review Information: The correct answer is D: Improve venous return. Elevating the leg both improves venous return and reduces swelling. Client comfort will be improved as well.

Question2

The nurse is reviewing with a client how to collect a clean catch urine specimen. What is the appropriate sequence to teach the client? A) Clean the meatus, begin voiding, then catch urine stream B) Void a little, clean the meatus, then collect specimen C) Clean the meatus, then urinate into container D) Void continuously and catch some of the urine Review Information: The correct answer is A: Clean the meatus, begin voiding, then catch urine stream. A clean catch urine is difficult to obtain and requires clear directions. Instructing the client to carefully clean the meatus, then void naturally with a steady stream prevents surface bacteria from contaminating the urine specimen. As starting and stopping flow can be difficult, once the client begins voiding it>>s best to just slip the container into the stream. Other responses do not reflect correct technique.

Question3

Following change-of-shift report on an orthopedic unit, which client should the nurse see first? A) 16 year-old who had an open reduction of a fractured wrist 10 hours ago B) 20 year-old in skeletal traction for 2 weeks since a motor cycle accident C) 72 year-old recovering from surgery after a hip replacement 2 hours ago D) 75 year-old who is in skin traction prior to planned hip pinning surgery. Review Information: The correct answer is C: 72 year-old recovering from surgery after a hip replacement 2 hours ago. Look for the client who has the most imminent risks and acute vulnerability. The client who returned from surgery 2 hours ago is at risk for life threatening hemorrhage and should be seen first. The 16 year-old should be seen next because it is still the first post-op day. The 75 year-old is potentially vulnerable to agerelated physical and cognitive consequences in Collected by :DeepaRajesh [ 23 ] [email protected] Kuwait

skin traction should be seen next. The client who can safely be seen last is the 20 year-old who is 2 weeks post-injury.

D) Repeat the peak flow reading in 30 minutes

Question6

Review Information: The correct answer is C: the appearance of eyeballs that appear to «pop» out of the client>>s eye sockets. Exophthalmos or protruding eyeballs is a distinctive characteristic of Graves>> Disease. It can result in corneal abrasions with severe eye pain or damage when the eyelid is unable to blink down over the protruding eyeball. Eye drops or ointment may be needed.

Review Information: The correct answer is B: Administer the prn dose of albuterol. Peak Question4 A client with Guillain Barre is in a nonresponsive flow monitoring during exacerbations of asthma state, yet vital signs are stable and breathing is is recommended for clients with moderate-toindependent. What should the nurse document severe persistent asthma to determine the seto most accurately describe the client>s condi- verity of the exacerbation and to guide the treatment. A peak flow reading of less than 50% of tion? the client>>s baseline reading is a medical alert A) Comatose, breathing unlabored condition and a short-acting beta-agonist must B) Glascow Coma Scale 8, respirations regular be taken immediately. C) Appears to be sleeping, vital signs stable D) Glascow Coma Scale 13, no ventilator re- Question7 quired A client had 20 mg of Lasix (furosemide) PO at 10 AM. Which would be essential for the nurse to Review Information: The correct answer is include at the change of shift report? B: Glascow Coma Scale 8, respirations regular. A) The client lost 2 pounds in 24 hours The Glascow Coma Scale provides a standard B) The client’s potassium level is 4 mEq/liter. reference for assessing or monitoring level of C) The client’s urine output was 1500 cc in 5 consciousness. Any score less than 13 indicates hours a neurological impairment. Using the term coma- D) The client is to receive another dose of Lasix tose provides too much room for interpretation at 10 PM and is not very precise. Review Information: The correct answer is C: Question5 When caring for a client receiving warfarin so- The client’s urine output was 1500 cc in 5 hours. dium (Coumadin), which lab test would the nurse Although all of these may be correct information monitor to determine therapeutic response to the to include in report, the essential piece would be the urine output. drug? A) Bleeding time Question8 B) Coagulation time A client has been tentatively diagnosed with C) Prothrombin time Graves> disease (hyperthyroidism). Which of D) Partial thromboplastin time these findings noted on the initial nursing assessment requires quick intervention by the nurse? Review Information: The correct answer is A) a report of 10 pounds weight loss in the last C: Prothrombin time. Coumadin is ordered daily, month based on the client>>s prothrombin time (PT). B) a comment by the client «I just can>t sit This test evaluates the adequacy of the extrinsic still.» system and common pathway in the clotting cas- C) the appearance of eyeballs that appear to cade; Coumadin affects the Vitamin K depend- «pop» out of the client>s eye sockets ent clotting factors. D) a report of the sudden onset of irritability in the past 2 weeks A client with moderate persistent asthma is admitted for a minor surgical procedure. On admission the peak flow meter is measured at 480 liters/minute. Post-operatively the client is complaining of chest tightness. The peak flow has dropped to 200 liters/minute. What should the nurse do first? A) Notify both the surgeon and provider B) Administer the prn dose of albuterol C) Apply oxygen at 2 liters per nasal cannula

Question9

The nurse has performed the initial assessments Collected by :DeepaRajesh [ 24 ] [email protected] Kuwait

of 4 clients admitted with an acute episode of asthma. Which assessment finding would cause the nurse to call the provider immediately? A) prolonged inspiration with each breath B) expiratory wheezes that are suddenly absent in 1 lobe C) expectoration of large amounts of purulent mucous D) appearance of the use of abdominal muscles for breathing Review Information: The correct answer is B: expiratory wheezes that are suddenly absent in 1 lobe. Acute asthma is characterized by expiratory wheezes caused by obstruction of the airways. Wheezes are a high pitched musical sounds produced by air moving through narrowed airways. Clients often associate wheezes with the feeling of tightness in the chest. However, sudden cessation of wheezing is an ominous or bad sign that indicates an emergency -- the small airways are now collapsed.

Question10

During the initial home visit, a nurse is discussing the care of a client newly diagnosed with Alzheimer>s disease with family members. Which of these interventions would be most helpful at this time? A) leave a book about relaxation techniques B) write out a daily exercise routine for them to assist the client to do C) list actions to improve the client>s daily nutritional intake D) suggest communication strategies Review Information: The correct answer is D: suggest communication strategies. Alzheimer>>s disease, a progressive chronic illness, greatly challenges caregivers. The nurse can be of greatest assistance in helping the family to use communication strategies to enhance their ability to relate to the client. By use of select verbal and nonverbal communication strategies the family can best support the client’s strengths and cope with any aberrant behavior.

Question11

An 80 year-old client admitted with a diagnosis of possible cerebral vascular accident has had a blood pressure from 160/100 to 180/110 over the past 2 hours. The nurse has also noted increased lethargy. Which assessment finding should the nurse report immediately to the provider? A) Slurred speech

B) Incontinence C) Muscle weakness D) Rapid pulse Review Information: The correct answer is A: Slurred speech. Changes in speech patterns and level of conscious can be indicators of continued intracranial bleeding or extension of the stroke. Further diagnostic testing may be indicated.

Question12

A school-aged child has had a long leg (hip to ankle) synthetic cast applied 4 hours ago. Which statement from the parent indicates that teaching has been inadequate? A) «I will keep the cast uncovered for the next day to prevent burning of the skin.» B) «I can apply an ice pack over the area to relieve itching inside the cast.» C) «The cast should be propped on at least 2 pillows when my child is lying down.» D) «I think I remember that my child should not stand until after 72 hours.» Review Information: The correct answer is D: «I think I remember that my child should not stand until after 72 hours.». Synthetic casts will typically set up in 30 minutes and dry in a few hours. Thus, the client may stand within the initial 24 hours. With plaster casts, the set up and drying time, especially in a long leg cast which is thicker than an arm cast, can take up to 72 hours. Both types of casts give off a lot of heat when drying and it is preferable to keep the cast uncovered for the first 24 hours. Clients may complain of a chill from the wet cast and therefore can simply be covered lightly with a sheet or blanket. Applying ice is a safe method of relieving the itching.

Question13

Which blood serum finding in a client with diabetic ketoacidosis alerts the nurse that immediate action is required? A) pH below 7.3 B) Potassium of 5.0 C) HCT of 60 D) Pa O2 of 79% Review Information: The correct answer is C: HCT of 60. This high hematocrit is indicative of severe dehydration which requires priority attention in diabetic ketoacidosis. Without sufficient hydration, all systems of the body are at risk Collected by :DeepaRajesh [ 25 ] [email protected] Kuwait

for hypoxia from a lack of or sluggish circulation. In the absence of insulin, which facilitates the transport of glucose into the cell, the body breaks down fats and proteins to supply energy ketones, a by-product of fat metabolism. These accumulate causing metabolic acidosis (pH < 7.3), which would be the second concern for this client. The potassium and PaO2 levels are near normal.

Question14

The nurse is preparing a client with a deep vein thrombosis (DVT) for a Venous Doppler evaluation. Which of the following would be necessary for preparing the client for this test? A) Client should be NPO after midnight B) Client should receive a sedative medication prior to the test C) Discontinue anti-coagulant therapy prior to the test D) No special preparation is necessary Review Information: The correct answer is D: No special preparation is necessary. This is a non-invasive procedure and does not require preparation other than client education.

Question15

A client is admitted with infective endocarditis (IE). Which finding would alert the nurse to a complication of this condition? A) dyspnea B) heart murmur C) macular rash D) hemorrhage Review Information: The correct answer is B: heart murmur. Large, soft, rapidly developing vegetations attach to the heart valves. They have a tendency to break off, causing emboli and leaving ulcerations on the valve leaflets. These emboli produce findings of cardiac murmur, fever, anorexia, malaise and neurologic sequelae of emboli. Furthermore, the vegetations may travel to various organs such as spleen, kidney, coronary artery, brain and lungs, and obstruct blood flow.

Question16

The nurse explains an autograft to a client scheduled for excision of a skin tumor. The nurse knows the client understands the procedure when the client says, «I will receive tissue from A) a tissue bank.» B) a pig.» C) my thigh.»

D) synthetic skin.» Review Information: The correct answer is C: my thigh.». Autografts are done with tissue transplanted from the client>>s own skin.

Question17

A client is admitted to the emergency room following an acute asthma attack. Which of the following assessments would be expected by the nurse? A) Diffuse expiratory wheezing B) Loose, productive cough C) No relief from inhalant D) Fever and chills Review Information: The correct answer is A: Diffuse expiratory wheezing. In asthma, the airways are narrowed, creating difficulty getting air in. A wheezing sound results.

Question18

A client has been admitted with a fractured femur and has been placed in skeletal traction. Which of the following nursing interventions should receive priority? A) Maintaining proper body alignment B) Frequent neurovascular assessments of the affected leg C) Inspection of pin sites for evidence of drainage or inflammation D) Applying an over-bed trapeze to assist the client with movement in bed Review Information: The correct answer is B: Frequent neurovascular assessments of the affected leg. The most important activity for the nurse is to assess neurovascular status. Compartment syndrome is a serious complication of fractures. Prompt recognition of this neurovascular problem and early intervention may prevent permanent limb damage.

Question19

The nurse is assigned to care for a client who had a myocardial infarction (MI) 2 days ago. The client has many questions about this condition. What area is a priority for the nurse to discuss at this time? A) Daily needs and concerns B) The overview cardiac rehabilitation C) Medication and diet guideline D) Activity and rest guidelines Review Information: The correct answer is A: Collected by :DeepaRajesh [ 26 ] [email protected] Kuwait

Daily needs and concerns. At 2 days post-MI, the es, milk client’s education should be focused on the immediate needs and concerns for the day. Review Information: The correct answer is B: Ground beef patty, lima beans, wheat roll, raisins, Question20 A 3 year-old child is brought to the clinic by his milk. Iron rich foods include red meat, fish, egg grandmother to be seen for «scratching his bot- yolks, green leafy vegetables, legumes, whole tom and wetting the bed at night.» Based on grains, and dried fruits such as raisins. This dinthese complaints, the nurse would initially as- ner is the best choice: It is high in iron and is appropriate for a toddler. sess for which problem? A) allergies Question23 B) scabies The nurse admitting a 5 month-old who vomited C) regression 9 times in the past 6 hours should observe for D) pinworms signs of which overall imbalance? A) Metabolic acidosis Review Information: The correct answer is B) Metabolic alkalosis D: pinworms. Signs of pinworm infection include C) Some increase in the serum hemoglobin intense perianal itching, poor sleep patterns, D) A little decrease in the serum potassium general irritability, restlessness, bed-wetting, distractibility and short attention span. Scabies Review Information: The correct answer is B: is an itchy skin condition caused by a tiny, eight- Metabolic alkalosis. Vomiting causes loss of acid legged burrowing mite called Sarcoptes scabiei . from the stomach. Prolonged vomiting can reThe presence of the mite leads to intense itching sult in excess loss of acid and lead to metabolic in the area of its burrows. alkalosis. Findings include irritability, increased activity, hyperactive reflexes, muscle twitching Question21 The nurse is caring for a newborn with tra- and elevated pulse. Options C and D are correct cheoesophageal fistula. Which nursing diagno- answers but not the best answers since they are too general. sis is a priority? A) Risk for dehydration Question24 B) Ineffective airway clearance A two year-old child is brought to the provider>s C) Altered nutrition office with a chief complaint of mild diarrhea for D) Risk for injury two days. Nutritional counseling by the nurse should include which statement? Review Information: The correct answer is B: A) Place the child on clear liquids and gelatin for Ineffective airway clearance. The most common 24 hours form of TEF is one in which the proximal esopha- B) Continue with the regular diet and include oral geal segment terminates in a blind pouch and rehydration fluids the distal segment is connected to the trachea C) Give bananas, apples, rice and toast as toleror primary bronchus by a short fistula at or near ated the bifurcation. Thus, a priority is maintaining an D) Place NPO for 24 hours, then rehydrate with open airway, preventing aspiration. Other nurs- milk and water ing diagnoses are then addressed. Review Information: The correct answer is B: Question22 The nurse is developing a meal plan that would Continue with the regular diet and include oral provide the maximum possible amount of iron for rehydration fluids. Current recommendations for a child with anemia. Which dinner menu would mild to moderate diarrhea are to maintain a normal diet with fluids to rehydrate. be best? A) Fish sticks, french fries, banana, cookies, Question25 milk The nurse is teaching parents about the approB) Ground beef patty, lima beans, wheat roll, rai- priate diet for a 4 month-old infant with gastrosins, milk enteritis and mild dehydration. In addition to oral C) Chicken nuggets, macaroni, peas, canta- rehydration fluids, the diet should include loupe, milk D) Peanut butter and jelly sandwich, apple slic- A) formula or breast milk Collected by :DeepaRajesh [ 27 ] [email protected] Kuwait

B) broth and tea C) rice cereal and apple juice D) gelatin and ginger ale

ginning with cereal B) Finely ground meat should be started early to provide iron C) Egg white is added early to increase protein Review Information: The correct answer is A: intake formula or breast milk. The usual diet for a young D) Solid foods should be mixed with formula in infant should be followed. a bottle

Question26

A child is injured on the school playground and Review Information: The correct answer is A: appears to have a fractured leg. The first action Solid foods are introduced one at a time beginning with cereal. Solid foods should be added the school nurse should take is one at a time between 4-6 months. If the infant is able to tolerate the food, another may be added A) call for emergency transport to the hospital B) immobilize the limb and joints above and be- in a week. Iron fortified cereal is the recommended first food. low the injury Question29 C) assess the child and the extent of the injury The nurse planning care for a 12 year-old child D) apply cold compresses to the injured area with sickle cell disease in a vaso-occlusive crisis Review Information: The correct answer is of the elbow should include which one of the folC: assess the child and the extent of the injury. lowing as a priority? When applying the nursing process, assessment is the first step in providing care. The «5 Ps» A) Limit fluids of vascular impairment can be used as a guide B) Client controlled analgesia C) Cold compresses to elbow (pain, pulse, pallor, paresthesia, paralysis). D) Passive range of motion exercise Question27 The mother of a 3 month-old infant tells the nurse that she wants to change from formula to whole Review Information: The correct answer is B: milk and add cereal and meats to the diet. What Client controlled analgesia. Management of a should be emphasized as the nurse teaches sickle cell crisis is directed towards supportive and symptomatic treatment. The priority of care about infant nutrition? is pain relief. In a 12 year-old child, client controlA) Solid foods should be introduced at 3-4 led analgesia promotes maximum comfort. months Question30 B) Whole milk is difficult for a young infant to di- The nurse is performing a physical assessment gest on a toddler. Which of the following actions C) Fluoridated tap water should be used to dilute should be the first? milk D) Supplemental apple juice can be used be- A) Perform traumatic procedures tween feedings B) Use minimal physical contact C) Proceed from head to toe Review Information: The correct answer is B: D) Explain the exam in detail Whole milk is difficult for a young infant to digest. Cow>>s milk is not given to infants younger than Review Information: The correct answer 1 year because the tough, hard curd is difficult to is B: Use minimal physical contact. The nurse digest. In addition, it contains little iron and cre- should approach the toddler slowly and use minates a high renal solute load. imal physical contact initially so as to gain the toddler>>s cooperation. Be flexible in the seQuestion28 The nurse is preparing a handout on infant feed- quence of the exam, and give only brief simple ing to be distributed to families visiting the clinic. explanations just prior to the action. Which notation should be included in the teach- Question31 ing materials? What finding signifies that children have attained the stage of concrete operations (Piaget)? A) Solid foods are introduced one at a time beCollected by :DeepaRajesh [ 28 ] [email protected] Kuwait

A) Explores the environment with the use of sight and movement B) Thinks in mental images or word pictures C) Makes the moral judgment that «stealing is wrong» D) Reasons that homework is time-consuming yet necessary

Question34

While teaching the family of a child who will take phenytoin (Dilantin) regularly for seizure control, it is most important for the nurse to teach them about which of the following actions?

Review Information: The correct answer is C: Makes the moral judgment that «stealing is wrong». The stage of concrete operations is depicted by logical thinking and moral judgments.

A) Maintain good oral hygiene and dental care B) Omit medication if the child is seizure free C) Administer acetaminophen to promote sleep D) Serve a diet that is high in iron

The mother of a child with a neural tube defect asks the nurse what she can do to decrease the chances of having another baby with a neural tube defect. What is the best response by the nurse?

Review Information: The correct answer is A: Maintain good oral hygiene and dental care. Swollen and tender gums occur often with use of phenytoin. Good oral hygiene and regular visits to the dentist should be emphasized.

Question32

A) «Folic acid should be taken before and after conception.» B) «Multivitamin supplements are recommended during pregnancy.» C) «A well balanced diet promotes normal fetal development.» D) «Increased dietary iron improves the health of mother and fetus.» Review Information: The correct answer is A: «Folic acid should be taken before and after conception.». The American Academy of Pediatrics recommends that all childbearing women increase folic acid from dietary sources and/or supplements. There is evidence that increased amounts of folic acid prevents neural tube defects.

Question33

The provider orders Lanoxin (digoxin) 0.125 mg PO and furosemide 40 mg every day. Which of these foods would the nurse reinforce for the client to eat at least daily? A) Spaghetti B) Watermelon C) Chicken D) Tomatoes Review Information: The correct answer is B: Watermelon. Watermelon is high in potassium and will replace potassium lost by the diuretic. The other foods are not high in potassium.

Question35

The nurse is offering safety instructions to a parent with a four month-old infant and a four yearold child. Which statement by the parent indicates understanding of appropriate precautions to take with the children? A) «I strap the infant car seat on the front seat to face backwards.» B) «I place my infant in the middle of the living room floor on a blanket to play with my four yearold while I make supper in the kitchen.» C) «My sleeping baby lies so cute in the crib with the little buttocks stuck up in the air while the four year-old naps on the sofa.» D) «I have the four year-old hold and help feed the four month-old a bottle in the kitchen while I make supper.» Review Information: The correct answer is D: «I have the four year-old hold and help feed the four month-old a bottle in the kitchen while I make supper.». The infant seat is to be placed on the rear seat. Small children and infants are not to be left unsupervised. Infants are

Question36

The nurse admits a 7 year-old to the emergency room after a leg injury. The x-rays show a femur fracture near the epiphysis. The parents ask what will be the outcome of this injury. The appropriate response by the nurse should be which of these Collected by :DeepaRajesh [ 29 ] [email protected] Kuwait

statements? A) «The injury is expected to heal quickly because of thin periosteum.» B) «In some instances the result is a retarded bone growth.» C) «Bone growth is stimulated in the affected leg.» D) «This type of injury shows more rapid union than that of younger children.»

Question39

A nurse is providing a parenting class to individuals living in a community of older homes. In discussing formula preparation, which of the following is most important to prevent lead poisoning?

A) Use ready-to-feed commercial infant formula B) Boil the tap water for 10 minutes prior to preparing the formula Review Information: The correct answer is B: C) Let tap water run for 2 minutes before adding «In some instances the result is a retarded bone to concentrate growth.». An epiphyseal (growth) plate fracture D) Buy bottled water labeled «lead free» to mix in a 7 year-old often results in retarded bone the formula growth. The leg often will be different in length than the uninjured leg. Review Information: The correct answer is C: Let tap water run for 2 minutes before adding to concentrate. Use of lead-contaminated water to prepare formula is a major source of poisoning Question37 The parents of a 4 year-old hospitalized child tell in infants. Drinking water may be contaminated the nurse, “We are leaving now and will be back by lead from old lead pipes or lead solder used at 6 PM.” A few hours later the child asks the in sealing water pipes. Letting tap water run for nurse when the parents will come again. What is several minutes will diminish the lead contamination. the best response by the nurse? A) «They will be back right after supper.» B) «In about 2 hours, you will see them.» C) «After you play awhile, they will be here.» D) «When the clock hands are on 6 and 12.»

Question40

Review Information: The correct answer is A: «They will be back right after supper.». Time is not completely understood by a 4 year-old. Preschoolers interpret time with their own frame of reference. Thus, it is best to explain time in relationship to a known, common event.

A) Scratching the head more than usual B) Flakes evident on a student>s shoulders C) Oval pattern occipital hair loss D) Whitish oval specks sticking to the hair

Question38

The nurse is giving instructions to the parents of a child with cystic fibrosis. The nurse would emphasize that pancreatic enzymes should be taken A) once each day B) 3 times daily after meals C) with each meal or snack D) each time carbohydrates are eaten Review Information: The correct answer is C: with each meal or snack. Pancreatic enzymes should be taken with each meal and every snack to allow for digestion of all foods that are eaten.

Which of the following manifestations observed by the school nurse confirms the presence of pediculosis capitis in students?

Review Information: The correct answer is D: Whitish oval specks sticking to the hair. Diagnosis of pediculosis capitis is made by observation of the white eggs (nits) firmly attached to the hair shafts. Treatment can include application of a medicated shampoo with lindane for children over 2 years of age, and meticulous combing and removal of all nits.

Question41

When interviewing the parents of a child with asthma, it is most important to assess the child>s environment for what factor? A) Household pets Collected by :DeepaRajesh [ 30 ] [email protected] Kuwait

B) New furniture C) Lead based paint D) Plants such as cactus

«Keep in mind that for the age this is a normal response to being in the hospital.». The protest phase of separation anxiety is a normal response for a child this age. In toddlers, ages 1 to 3, sepaReview Information: The correct answer is A: ration anxiety is at its peak Household pets. Animal dander is a very common allergen affecting persons with asthma. Other triggers may include pollens, carpeting Question44 and household dust. A couple experienced the loss of a 7 month-old fetus. In planning for discharge, what should the nurse emphasize?

Question42

The mother of a 2 month-old baby calls the nurse 2 days after the first DTaP, IPV, Hepatitis B and HIB immunizations. She reports that the baby feels very warm, cries inconsolably for as long as 3 hours, and has had several shaking spells. In addition to referring her to the emergency room, the nurse should document the reaction on the baby>s record and expect which immunization to be most associated with the findings the infant is displaying? A) DTaP B) Hepatitis B C) Polio D) H. Influenza Review Information: The correct answer is A: DTaP. The majority of reactions occur with the administration of the DTaP vaccination. Contradictions to giving repeat DTaP immunizations include the occurrence of severe side effects after a previous dose as well as signs of encephalopathy within 7 days of the immunization.

Question43

A) To discuss feelings with each other and use support persons B) To focus on the other healthy children and move through the loss C) To seek causes for the fetal death and come to some safe conclusion D) To plan for another pregnancy within 2 years and maintain physical health Review Information: The correct answer is A: To discuss feelings with each other and use support persons. To communicate in a therapeutic manner, the nurse>>s goal is to help the couple begin the grief process by suggesting they talk to each other, seek family, friends and support groups to listen to their feelings.

Question45

The nurse is performing a pre-kindergarten physical on a 5 year-old. The last series of vaccines will be administered. What is the preferred site for injection by the nurse? A) vastus intermedius B) gluteus maximus C) vastus lateralis D) dorsogluteaI

The mother of a 2 year-old hospitalized child asks the nurse>s advice about the child>s screaming every time the mother gets ready to leave the hospital room. What is the best response by the Review Information: The correct answer is C: nurse? vastus lateralis. Vastus lateralis, a large and well developed muscle, is the preferred site, since it is A) «I think you or your partner needs to stay with removed from major nerves and blood vessels. the child while in the hospital.» B) «Oh, that behavior will stop in a few days.» C) «Keep in mind that for the age this is a normal Question46 response to being in the hospital.» A 7 month pregnant woman is admitted with comD) «You might want to «sneak out» of the room plaints of painless vaginal bleeding over several once the child falls asleep.» hours. The nurse should prepare the client for an immediate Review Information: The correct answer is C: Collected by :DeepaRajesh [ 31 ] [email protected] Kuwait

A) Non stress test B) Abdominal ultrasound C) Pelvic exam D) X-ray of abdomen Review Information: The correct answer is B: Abdominal ultrasound. The standard for diagnosis of placenta previa, which is suggested in the client>>s history of painless bleeding, is abdominal ultrasound.

Question47

A nurse entering the room of a postpartum mother observes the baby lying at the edge of the bed while the woman sits in a chair. The mother states «This is not my baby, and I do not want it.» After repositioning the child safely, the nurse>s best response is A) «This is a common occurrence after birth, but you will come to accept the baby.» B) «Many women have postpartum blues and need some time to love the baby.» C) «What a beautiful baby! Her eyes are just like yours.» D) «You seem upset; tell me what the pregnancy and birth were like for you.» Review Information: The correct answer is D: «You seem upset; tell me what the pregnancy and birth were like for you.». A non-judgmental, open ended response facilitates dialogue between the client and nurse.

Question48

The nurse notes that a 2 year-old child recovering from a tonsillectomy has an temperature of 98.2 degrees Fahrenheit at 8:00 AM. At 10:00 AM the child>s parent reports that the child «feels very warm» to touch. The first action by the nurse should be to A) reassure the parent that this is normal B) offer the child cold oral fluids C) reassess the child>s temperature D) administer the prescribed acetaminophen Review Information: The correct answer is C: reassess the child>>s temperature. A child>>s temperature may have rapid fluctuations. The nurse should listen to and show respect for what

parents say. Parental caretakers are often quite sensitive to variations in their children>>s condition that may not be immediately evident to others.

Question49

The nurse is caring for a client who was successfully resuscitated from a pulseless dysrhythmia. Which of the following assessments is critical for the nurse to include in the plan of care? A) hourly urine output B) white blood count C) blood glucose every 4 hours D) temperature every 2 hours Review Information: The correct answer is A: hourly urine output. Clients who have had an episode of decreased glomerular perfusion are at risk for pre-renal failure. This is caused by any abnormal decline in kidney perfusion that reduces glomerular perfusion. Pre-renal failure occurs when the effective arterial blood volume falls. Examples of this phenomena include a drop in circulating blood volume as in a cardiac arrest state or in low cardiac perfusion states such as congestive heart failure associated with a cardiomyopathy. Close observation of hourly urinary output is necessary for early detection of this condition.

Question50

A client is admitted to the rehabilitation unit following a cerebral vascular accident (CVA) and mild dysphagia. The most appropriate intervention for this client is to A) position client in upright position while eating B) place client on a clear liquid diet C) tilt head back to facilitate swallowing reflex D) offer finger foods such as crackers or pretzels Review Information: The correct answer is A: position client in upright position while eating. An upright position facilitates proper chewing and swallowing.

Question51

A 72 year-old client with osteomyelitis requires a Collected by :DeepaRajesh [ 32 ] [email protected] Kuwait

6 week course of intravenous antibiotics. In planning for home care, what is the most important action by the nurse? A) Investigating the client>s insurance coverage for home IV antibiotic therapy B) Determining if there are adequate hand washing facilities in the home C) Assessing the client>s ability to participate in self care and/or the reliability of a caregiver D) Selecting the appropriate venous access device Review Information: The correct answer is C: Assessing the client>>s ability to participate in self care and/or the reliability of a caregiver. The cognitive ability of the client as well as the availability and reliability of a caregiver must be assessed to determine if home care is a feasible option.

Question52

A nurse administers the influenza vaccine to a client in a clinic. Within 15 minutes after the immunization was given, the client complains of itchy and watery eyes, increased anxiety, and difficulty breathing. The nurse expects that the first action in the sequence of care for this client will be to A) Maintain the airway B) Administer epinephrine 1:1000 as ordered C) Monitor for hypotension with shock D) Administer diphenhydramine as ordered Review Information: The correct answer is B: Administer epinephrine 1:1000 as ordered. All the answers are correct given the circumstances, but the priority is to administer the epinephrine, then maintain the airway. In the early stages of anaphylaxis, when the patient has not lost consciousness and is normotensive, administering the epinephrine is first, and applying the oxygen, and watching for hypotension and shock, are later responses. The prevention of a severe crisis is maintained by using diphenhydramine.

Question53

medication A) retards pepsin production B) stimulates hydrochloric acid production C) slows stomach emptying time D) decreases production of hydrochloric acid Review Information: The correct answer is B: stimulates hydrochloric acid production. Decadron increases the production of hydrochloric acid, which may cause gastrointestinal ulcers.

Question54

A client receiving chlorpromazine HCL (Thorazine) is in psychiatric home care. During a home visit the nurse observes the client smacking her lips alternately with grinding her teeth. The nurse recognizes this assessment finding as what? A) Dystonia B) Akathisia C) Brady dyskinesia D) Tardive dyskinesia Review Information: The correct answer is D: Tardive dyskinesia. Signs of tardive dyskinesia include smacking lips, grinding of teeth and «fly catching» tongue movements. These findings are often described as Parkinsonian.

Question55

Which of the following findings contraindicate the use of haloperidol (Haldol) and warrant withholding the dose? A) Drowsiness, lethargy, and inactivity B) Dry mouth, nasal congestion, and blurred vision C) Rash, blood dyscrasias, severe depression D) Hyperglycemia, weight gain, and edema Review Information: The correct answer is C: Rash, blood dyscrasias, severe depression. Rash and blood dyscrasias are side effects of anti-psychotic drugs. A history of severe depression is a contraindication to the use of neuroleptics.

Question56

The nurse is reinforcing teaching to a 24 year-old woman receiving acyclovir (Zovirax) for a Herpes Simplex Virus type 2 infection. Which of these instructions should the nurse give the client?

The nurse instructs the client taking dexamethasone (Decadron) to take it with food or milk. The physiological basis for this instruction is that the A) Complete the entire course of the medication Collected by :DeepaRajesh [ 33 ] [email protected] Kuwait

for an effective cure B) Begin treatment with acyclovir at the onset of symptoms of recurrence C) Stop treatment if she thinks she may be pregnant to prevent birth defects D) Continue to take prophylactic doses for at least 5 years after the diagnosis Review Information: The correct answer is B: Begin treatment with acyclovir at the onset of symptoms of recurrence. When the client is aware of early symptoms, such as pain, itching or tingling, treatment is very effective. Medications for herpes simplex do not cure the disease; they simply decrease the level of symptoms.

Question57

Question59

A 42 year-old male client refuses to take propranolol hydrochloride (Inderal) as prescribed. Which client statement from the assessment data is likely to explain his noncompliance? A) «I have problems with diarrhea.» B) «I have difficulty falling asleep.» C) «I have diminished sexual function.» D) «I often feel jittery.» Review Information: The correct answer is C: «I have diminished sexual function.». Inderal, a beta-blocking agent used in hypertension, prohibits the release of epinephrine into the cells; this may result in hypotension which results in decreased libido and impotence.

A 14 month-old child ingested half a bottle of aspirin tablets. Which of the following would the Question60 nurse expect to see in the child? The nurse caring for a 9 year-old child with a fractured femur is told that a medication error A) Hypothermia occurred. The child received twice the ordered B) Edema dose of morphine an hour ago. Which nursing C) Dyspnea diagnosis is a priority at this time? D) Epistaxis A) Risk for fluid volume deficit related to morReview Information: The correct answer is phine overdose D: Epistaxis. A large dose of aspirin inhibits pro- B) Decreased gastrointestinal mobility related to thrombin formation and lowers platelet levels. mucosal irritation With an overdose, clotting time is prolonged. C) Ineffective breathing patterns related to central nervous system depression D) Altered nutrition related to inability to control nausea and vomiting Question58 An 80 year-old client on digitalis (Lanoxin) reports nausea, vomiting, abdominal cramps and Review Information: The correct answer is C: halo vision. Which of the following laboratory re- Ineffective breathing patterns related to central nervous system depression. Respiratory depressults should the nurse analyze first? sion is a life-threatening risk in this overdose. A) Potassium levels B) Blood pH C) Magnesium levels Question61 D) Blood urea nitrogen Lactulose (Chronulac) has been prescribed for a client with advanced liver disease. Which of the Review Information: The correct answer is A: following assessments would the nurse use to Potassium levels. The most common cause of evaluate the effectiveness of this treatment? digitalis toxicity is a low potassium level. Clients must be taught that it is important to have ad- A) An increase in appetite equate potassium intake especially if taking diu- B) A decrease in fluid retention retics that enhance the loss of potassium while C) A decrease in lethargy they are taking digitalis. D) A reduction in jaundice Review Information: The correct answer is Collected by :DeepaRajesh [ 34 ] [email protected] Kuwait

C: A decrease in lethargy. Lactulose produces an acid environment in the bowel and traps ammonia in the gut; the laxative effect then aids in removing the ammonia from the body. This decreases the effects of hepatic encephalopathy, including lethargy and confusion.

Question62

The nurse is teaching a class on HIV prevention. Which of the following should be emphasized as increasing risk? A) Donating blood B) Using public bathrooms C) Unprotected sex D) Touching a person with AIDS Review Information: The correct answer is C: Unprotected sex. Because HIV is spread through exposure to bodily fluids, unprotected intercourse and shared drug paraphernalia remain the highest risks for infection.

Question63

While interviewing a new admission, the nurse notices that the client is shifting positions, wringing her hands, and avoiding eye contact. It is important for the nurse to A) ask the client what she is feeling B) assess the client for auditory hallucinations C) recognize the behavior as a side effect of medication D) re-focus the discussion on a less anxiety provoking topic Review Information: The correct answer is A: ask the client what she is feeling. The initial step in anxiety intervention is observing, identifying, and assessing anxiety. The nurse should seek client validation of the accuracy of nursing assessments and avoid drawing conclusions based on limited data. In the situation above, the client may simply need to use the restroom but be reluctant to communicate her need!

Question64

A young adult seeks treatment in an outpatient mental health center. The client tells the nurse he is a government official being followed by spies. On further questioning, he reveals that

his warnings must be heeded to prevent nuclear war. What is the most therapeutic approach by the nurse? A) Listen quietly without comment B) Ask for further information on the spies C) Confront the client’s delusion D) Contact the government agency Review Information: The correct answer is A: Listen quietly without comment. The client>>s comments demonstrate grandiose ideas. The most therapeutic response is to listen but avoid being incorporated into the client’s delusional system.

Question65

The nurse is assessing a 17 year-old female client with bulimia. Which of the following laboratory reports would the nurse anticipate? A) Increased serum glucose B) Decreased albumin C) Decreased potassium D) Increased sodium retention Review Information: The correct answer is C: Decreased potassium. In bulimia, loss of electrolytes can occur in addition to other findings of starvation and dehydration.

Question66

A client, recovering from alcoholism, asks the nurse, «What can I do when I start recognizing relapse triggers within myself?» How might the nurse best respond? A) «When you have the impulse to stop in a bar, contact a sober friend and talk with him.» B) «Go to an AA meeting when you feel the urge to drink.» C) «It is important to exercise daily and get involved in activities that will cause you not to think about drug use.» D) «Let’s talk about possible options you have when you recognize relapse triggers in yourself.» Review Information: The correct answer is D: «Let’s talk about possible options you have when you recognize relapse triggers in yourself.». This option encourages the process of self evaluaCollected by :DeepaRajesh [ 35 ] [email protected] Kuwait

tion and problem solving, while avoiding telling the client what to do. Encouraging the client to brainstorm about response options validates the nurse’s belief in the client’s personal competency and reinforces a coping strategy that will be needed when the nurse may not be available to offer solutions.

Question67

Therapeutic nurse-client interaction occurs when the nurse A) assists the client to clarify the meaning of what the client has said B) interprets the client’s covert communication C) praises the client for appropriate feelings and behavior D) advises the client on ways to resolve problems Review Information: The correct answer is A: assists the client to clarify the meaning of what the client has said. Clarification is a facilitating/ therapeutic communication strategy. Interpretation, changing the focus/subject, giving approval, and advising are non-therapeutic/barriers to communication.

A) provide a businesslike atmosphere where clients can work on individual goals B) provide a group forum in which clients decide on unit rules, regulations, and policies C) provide a testing ground for new patterns of behavior while the client takes responsibility for his or her own actions D) discourage expressions of anger because they can be disruptive to other clients Review Information: The correct answer is C: provide a testing ground for new patterns of behavior while the client takes responsibility for his or her own actions. A therapeutic milieu is purposeful and planned to provide safety and a testing ground for new patterns of behavior.

Question70

A client with paranoid delusions stares at the nurse over a period of several days. The client suddenly walks up to the nurse and shouts «You think you’re so perfect and pure and good.» An appropriate response for the nurse is

A) «Is that why you’ve been staring at me?» B) «You seem to be in a really bad mood.» C) «Perfect? I don’t quite understand.» Question68 Which nursing intervention will be most effective D) «You seem angry right now.» in helping a withdrawn client to develop relationReview Information: The correct answer is D: ship skills? A) Offer the client frequent opportunities to inter- «You seem angry right now.». The nurse recognizes the underlying emotion with a matter of fact act with 1 person B) Provide the client with frequent opportunities attitude, but avoids telling the clients how they feel. to interact with other clients C) Assist the client to analyze the meaning of the withdrawn behavior D) Discuss with the client the focus that other Question71 clients have similar problems A client who is a former actress enters the day room wearing a sheer nightgown, high heels, nuReview Information: The correct answer is A: merous bracelets, bright red lipstick and heavily Offer the client frequent opportunities to interact rouged cheeks. Which nursing action is the best with 1 person. The withdrawn client is uncomfort- in response to the client’s attire? able in social interaction. The nurse-client relationship is a corrective relationship in which the A) Gently remind her that she is no longer on client learns both tolerance and skills for relation- stage ships. B) Directly assist client to her room for appropriate apparel C) Quietly point out to her the dress of other clients on the unit Question69 An important goal in the development of a thera- D) Tactfully explain appropriate clothing for the hospital peutic inpatient milieu is to Collected by :DeepaRajesh [ 36 ] [email protected] Kuwait

Review Information: The correct answer is B: Directly assist client to her room for appropriate apparel. It assists the client to maintain self-esteem while modifying behavior.

Question72

When teaching suicide prevention to the parents of a 15 year-old who recently attempted suicide, the nurse describes the following behavioral cue as indicating a need for intervention. A) Angry outbursts at significant others B) Fear of being left alone C) Giving away valued personal items D) Experiencing the loss of a boyfriend Review Information: The correct answer is C: Giving away valued personal items. Eighty percent of all potential suicide victims give some type of indication that self-destructiveness should be addressed. These clues might lead one to suspect that a client is having suicidal thoughts or is developing a plan.

Question73

Which statement made by a client indicates to the nurse that the client may have a thought disorder? A) «I>m so angry about this. Wait until my partner hears about this.» B) «I>m a little confused. What time is it?» C) «I can>t find my <mesmer> shoes. Have you seen them?» D) «I>m fine. It>s my daughter who has the problem.» Review Information: The correct answer is C: «I can>>t find my <>mesmer>> shoes. Have you seen them?». A neologism is a new word self invented by a person and not readily understood by another. Using neologisms is often associated with a thought disorder.

B) handshaking keeps the gesture on a professional level C) refusal to touch a client denotes lack of concern D) inappropriate touch often results in charges of assault and battery Review Information: The correct answer is A: some clients misconstrue hugs as an invitation to sexual advances. Touch denotes positive feelings for another person. The client may interpret hugging and holding hands as sexual advances.

Question75

A client with anorexia is hospitalized on a medical unit due to electrolyte imbalance and cardiac dysrhythmias. Additional assessment findings that the nurse would expect to observe are A) brittle hair, lanugo, amenorrhea B) diarrhea, nausea, vomiting, dental erosion C) hyperthermia, tachycardia, increased metabolic rate D) excessive anxiety about symptoms Review Information: The correct answer is A: brittle hair, lanugo, amenorrhea. Physical findings associated with anorexia also include reduced metabolic rate and lower vital signs.

NCLEX Study Tips Jul31, If you are going to prepare for taking the NCLEX exam and still don>t know what to do i would like to share some effective advices for you.

Picking review courses:

the best choice for review courses is Kaplan or NCSBN (National Council State Boards of Nursing). Kaplan teaches effective techniques on Question74 how to answer exam questions with ease and In a psychiatric setting, the nurse limits touch teaches you to land with The correct answer. or contact used with clients to handshaking because NCSBN sure is another top choice for review course because the contents are very close to A) some clients misconstrue hugs as an invita- the actual exam itself. tion to sexual advances Collected by :DeepaRajesh [ 37 ] [email protected] Kuwait

Never Cram

The current fee to register with NCLEX is $200 and you must indicate at the time of application Cramming is never effective in preparing for the which Board you>ve chosen.Processing times NCLEX. Give yourself at least 3 months to study vary from state to state from (4-16 weeks) for the exam. After you have met the requirements, been approved by the Board and applied for NCLEX you References will be issued an ATT (Authorization to Test) and Lipincott is known to be the best review book for can schedule your NCLEX exam at your convinience. preparing yourself for the exam. Some naysayers say that the NCLEX structured questions are based on lipincott.

You must have an ATT before you can to take the exam.

Do alot of practice testing and never sleep late before the exam day. Just relax you will do fine. The NCLEX exam can be scheduled anywhere NCLEX could be re-taken after 91 days from tak- in the US or it>s territories, and other countries like the Philippines and Hong Kong and is ofing the first exam. fered year-round. 0 comments Labels: nclex review, nclex study tips

You do not have to take the NCLEX exam in the State where you applied.

How To Apply For NCLEX Testing

At some testing centers the appointments go Here are the guidelines on how to apply for very quickly so plan to schedule your appointment early. NCLEX Testing in the US: In order to receive a US nursing license, you must pick a state, complete the application and meet their requirements and if they find you eligible, you will have to undergo testing also commonly known as NCLEX, before you can work as an RN in the USA.

0 comments Labels: how to apply for nclex-rn, nclex testing guidelines US Boards Of Nursing For NCLEX Application

Here are the list of US Nursing Boards state by Each State Nursing Board has its own fee sched- state. Click on them and it will link you to each ule and specific requirements (e.g. CGFNS, state board webpage and check the guidelines on how you could apply for examination (NCLEX) CES, TOEFL, TSE) The time it takes to approve your credentials or reciprocity to practice as a registered nurse in and process your application varies from state your choice of state. to state. Alabama Board of Nursing Alaska Board of Nursing Arizona State Board of Nursing All nurses must pass NCLEX . Arkansas State Board of Nursing California Board of Registered Nursing *California Board of Vocational Nursing and Psychiatric Technicians The Board will supply you with an NCLEX appliColorado Board of Nursing cation which could be downlowded in their webConnecticut Board of Examiners for Nursing pages. Delaware Board of Nursing Collected by :DeepaRajesh [ 38 ] [email protected] Kuwait

District of Columbia Board of Nursing Florida Board of Nursing Georgia Board of Nursing *Georgia State Board of Licensed Practical Nurses Hawaii Board of Nursing Idaho Board of Nursing Illinois Division of Professional Regulation Indiana State Board of Nursing Iowa Board of Nursing Kansas State Board of Nursing Kentucky Board of Nursing Louisiana State Board of Nursing *Louisiana State Board of Practical Nurse Examiners Maine State Board of Nursing Maryland Board of Nursing Massachusetts Board of Registration in Nursing Michigan CIS/Bureau of Health Professions Minnesota Board of Nursing Mississippi Board of Nursing Missouri Division of Professional Registration Montana State Board of Nursing Nebraska Department of Health and Human Services Regulation and Licensure, Nursing and Nursing Support Nevada State Board of Nursing New Hampshire Board of Nursing New Jersey Board of Nursing New Mexico Board of Nursing New York State Board of Nursing North Carolina Board of Nursing North Dakota Board of Nursing Ohio Board of Nursing Oklahoma Board of Nursing Oregon State Board of Nursing Pennsylvania State Board of Nursing Rhode Island Board of Nurse Registration and Nursing Education South Carolina Board of Nursing South Dakota Board of Nursing Tennessee State Board of Nursing Texas Board of Nurse Examiners Utah State Board of Nursing Vermont State Board of Nursing Virginia Board of Nursing Washington State Nursing Care Quality Assurance Commission West Virginia Board of Examiners for Registered Professional Nurses *West Virginia State Board of Examiners for Licensed Practical Nurses Wisconsin Department of Regulation and Licens-

ing Wyoming State Board of Nursing *Some states have separate Web sites for boards of licensed practical or vocational nursing (LPN/ LVN)

Free NCLEX-RN Sample Test Questions For Nursing Review (Pharmacology Set 2) Jul31, A nurse is assigned to perform well-child assessments at a day care center. A staff member interrupts the examinations to ask for assistance. They find a crying 3 year-old child on the floor with mouth wide open and gums bleeding. Two unlabeled open bottles lie nearby. The nurse>s first action should be A) call the poison control center, then 911 B) administer syrup of Ipecac to induce vomiting C) give the child milk to coat her stomach D) ask the staff about the contents of the bottles Review Information: The correct answer is D: ask the staff about the contents of the bottles The nurse needs to assess what the child ingested before determining the next action. Once the substance is identified, the poison control center and emergency response team should be called.

Question2

A client with atrial fibrillation is receiving digoxin (Lanoxin). Which of these assessments is most important for the nurse to perform? A) Monitor blood pressure every 4 hours B) Measure apical pulse prior to administration C) Maintain accurate intake and output records D) Record an EKG strip after administration Review Information: The correct answer is B: Measure apical pulse prior to administration Digitoxin decreases conduction velocity through the AV node and prolongs the refractory period. If the apical heart rate is less than 60 beats/minute, withhold the drug. The apical pulse should be taken with a stethoscope so that there will be no mistake about what the heart rate actually is. Collected by :DeepaRajesh [ 39 ] [email protected] Kuwait

Question3

The nurse is administering an intravenous vesicant chemotherapeutic agent to a client. Which assessment would require the nurse>s immediate action? A) Stomatitis lesion in the mouth B) Severe nausea and vomiting C) Complaints of pain at site of infusion D) A rash on the client>s extremities

Review Information: The correct answer is A: Buffalo hump With high doses of glucocorticoid, iatrogenic Cushing>>s syndrome develops. The exaggerated physiological action causes abnormal fat distribution which results in a moon-shaped face, a intrascapular pad on the neck (buffalo hump) and truncal obesity with slender limbs.

Review Information: The correct answer is C: Complaints of pain at site of infusion A vesicant is a chemotherapeutic agent capable of causing blistering of tissues and possible tissue necrosis if there is extravasation. These agents are irritants which cause pain along the vein wall, with or without inflammation.

Question6

Question4

The nurse practicing in a long term care facility recognizes that elderly clients are at greater risk for drug toxicity than younger adults because of which of the following physiological changes of advancing age? A) Drugs are absorbed more readily from the GI tract B) Elders have less body water and more fat C) The elderly have more rapid hepatic metabolism D) Older people are often malnourished and anemic Review Information: The correct answer is B: Elders have less body water and more fat Because elderly persons have decreased lean body tissue/water in which to distribute medications, more drug remains in the circulatory system with potential for drug toxicity. Increased body fat results in greater amounts of fat-soluble drugs being absorbed, leaving less in circulation, thus increasing the duration of action of the drug

Question5

The nurse is assessing a client who is on long term glucocorticoid therapy. Which of the following findings would the nurse expect? A) Buffalo hump B) Increased muscle mass C) Peripheral edema D) Jaundice

The health care provider has written «Morphine sulfate 2 mgs IV every 3-4 hours prn for pain» on the chart of a child weighing 22 lb. (10 kg). What is the nurse>s initial action? A) Check with the pharmacist B) Hold the medication and contact the provider C) Administer the prescribed dose as ordered D) Give the dose every 6-8 hours Review Information: The correct answer is B: Hold the medication and contact the provider The usual pediatric dose of morphine is 0.1 mg/ kg every 3 to 4 hours. At 10 kg, this child typically should receive 1.0 mg every 3 to 4 hours.

Question7

A client is ordered atropine to be administered preoperatively. Which physiological effect should the nurse monitor for? A) Elevate blood pressure B) Drying up of secretions C) Reduce heart rate D) Enhance sedation Review Information: The correct answer is B: Drying up of secretions Atropine dries secretions which may get in the way during the operative procedure.

Question8

A client is receiving digitalis. The nurse should instruct the client to report which of the following side effects? A) Nausea, vomiting, fatigue B) Rash, dyspnea, edema C) Polyuria, thirst, dry skin D) Hunger, dizziness, diaphoresis Collected by :DeepaRajesh [ 40 ] [email protected] Kuwait

Review Information: The correct answer is A: Nausea, vomiting, fatigue Side effects of digitalis toxicity include fatigue, nausea, vomiting, anorexia, and bradycardia. Digitalis inhibits the sodium potassium ATPase, which makes more calcium available for contractile proteins, resulting in increased cardiac output.

Question9

A client is receiving dexamethasone (Decadron) therapy. What should the nurse plan to monitor in this client? A) Urine output every 4 hours B) Blood glucose levels every 12 hours C) Neurological signs every 2 hours D) Oxygen saturation every 8 hours Review Information: The correct answer is B: Blood glucose levels every 12 hours The drug Decadron increases glycogenesis. This may lead to hyperglycemia. Therefore the blood sugar level and acetone production must be monitored.

Question10

The nurse is caring for a client with schizophrenia who has been treated with quetiapine (Seroquel) for 1 month. Today the client is increasingly agitated and complains of muscle stiffness. Which of these findings should be reported to the health care provider? A) Elevated temperature and sweating. B) Decreased pulse and blood pressure. C) Mental confusion and general weakness. D) Muscle spasms and seizures. Review Information: The correct answer is A: Elevated temperature and sweating. Neuroleptic malignant syndrome (NMS) is a rare disorder that can occur as a side effect of antipsychotic medications. It is characterized by muscular rigidity, tachycardia, hyperthermia, sweating, altered consciousness, autonomic dysfunction, and increase in CPK. This is a life-threatening complication.

Question11

A child presents to the Emergency Department with documented acetaminophen poisoning. In order to provide counseling and education for

the parents, which principle must the nurse understand? A) The problem occurs in stages with recovery within 12-24 hours B) Hepatic problems may occur and may be lifethreatening C) Full and rapid recovery can be expected in most children D) This poisoning is usually fatal, as no antidote is available Review Information: The correct answer is B: Hepatic problems may occur and may be lifethreatening Clinical manifestations associated with acetaminophen poisoning occurs in 4 stages. The third stage is hepatic involvement which may last up to 7 days and be permanent. Clients who do not die in the hepatic stage gradually recover.

Question12

A client has been receiving dexamethasone (Decadron) for control of cerebral edema. Which of the following assessments would indicate that the treatment is effective? A) A positive Babinski>s reflex B) Increased response to motor stimuli C) A widening pulse pressure D) Temperature of 37 degrees Celsius Review Information: The correct answer is B: Increased response to motor stimuli Decadron is a corticosteroid that acts on the cell membrane to decrease inflammatory responses as well as stabilize the blood-brain barrier. Once Decadron reaches a therapeutic level, there should be a decrease in symptomology with improvement in motor skills.

Question13

The provider has ordered transdermal nitroglycerin patches for a client. Which of these instructions should be included when teaching a client about how to use the patches? A) Remove the patch when swimming or bathing B) Apply the patch to any non-hairy area of the body C) Apply a second patch with chest pain D) Remove the patch if ankle edema occurs

Collected by :DeepaRajesh [ 41 ] [email protected] Kuwait

Review Information: The correct answer is B: Apply the patch to any non-hairy area of the body The patch application sites should be rotated.

Question14

with which substance? A) Acetaminophen B) Orange juice C) Low fat milk D) An antacid

A newly admitted client has a diagnosis of de- Review Information: The correct answer is B: pression. She complains of “twitching muscles” Orange juice and a “racing heart”, and states she stopped Ascorbic acid enhances the absorption of iron. taking Zoloft a few days ago because it was not helping her depression. Instead, she began to Question17 take her partner>s Parnate. The nurse should A client with an aplastic sickle cell crisis is receivimmediately assess for which of these adverse ing a blood transfusion and begins to complain of «feeling hot.» Almost immediately, the client reactions? A) Pulmonary edema begins to wheeze. What is the nurse>s first acB) Atrial fibrillation tion? C) Mental status changes A) Stop the blood infusion D) Muscle weakness B) Notify the health care provider C) Take/record vital signs Review Information: The correct answer is C: D) Send blood samples to lab Mental status changes Use of serotonergic agents may result in Se- Review Information: The correct answer is A: rotonin Syndrome with confusion, nausea, pal- Stop the blood infusion pitations, increased muscle tone with twitching If a reaction of any type is suspected during admuscles, and agitation. Serotonin syndrome is ministration of blood products, stop the infusion most often reported in patients taking 2 or more immediately, keep the line open with saline, nomedications that increase CNS serotonin levels tify the health care provider, monitor vital signs by different mechanisms. The most common and other changes, and then send a blood samdrug combinations associated with serotonin ple to the lab. syndrome involve the MAOIs, SSRIs, and the tricyclic antidepressants. Question18 A client confides in the RN that a friend has told Question15 her the medication she takes for depression, A client with bi-polar disorder is taking lithium Wellbutrin, was taken off the market because (Lithane). What should the nurse emphasize it caused seizures. What is an appropriate rewhen teaching about this medication? sponse by the nurse? A) Take the medication before meals A) «Ask your friend about the source of this inB) Maintain adequate daily salt intake formation.» C) Reduce fluid intake to minimize diuresis B) «Omit the next doses until you talk with the D) Use antacids to prevent heartburn doctor.» C) «There were problems, but the recommended dose is changed.» Review Information: The correct answer is B: D) «Your health care provider knows the best Maintain adequate daily salt intake drug for your condition.» Salt intake affects fluid volume, which can affect lithium (Lithane) levels; therefore, maintaining adequate salt intake is advised. Review Information: The correct answer is C: «There were problems, but the recommended dose is changed.» Question16 A client with anemia has a new prescription for Wellbutrin was introduced in the U.S. in 1985 ferrous sulfate. In teaching the client about diet and then withdrawn because of the occurrence and iron supplements, the nurse should empha- of seizures in some patients taking the drug. The size that absorption of iron is enhanced if taken drug was reintroduced in 1989 with specific recCollected by :DeepaRajesh [ 42 ] [email protected] Kuwait

ommendations regarding dose ranges to limit tests.» the occurrence of seizures. The risk of seizure appears to be strongly associated with dose. Review Information: The correct answer is C: «Continue to take your medications even when Question19 When providing discharge teaching to a client you are feeling fine.» with asthma, the nurse will warn against the use The most important piece of information the tuof which of the following over-the-counter medi- berculosis client needs is to understand the importance of medication compliance, even if no cations? longer experiencing symptoms. Clients are most A) Cortisone ointments for skin rashes infectious early in the course of therapy. The B) Aspirin products for pain relief numbers of acid-fast bacilli are greatly reduced C) Cough medications containing guaifenesin as early as 2 weeks after therapy begins. D) Histamine blockers for gastric distress Review Information: The correct answer is B: Aspirin products for pain relief Aspirin is known to induce asthma attacks. Aspirin can also cause nasal polyps and rhinitis. Warn individuals with asthma about signs and symptoms resulting from complications due to aspirin ingestion.

Question20

The nurse is caring for a client who is receiving procainamide (Pronestyl) intravenously. It is important for the nurse to monitor which of the following parameters? A) Hourly urinary output B) Serum potassium levels * C) Continuous EKG readings D) Neurological signs Review Information: The correct answer is C: Continuous EKG readings Procainamide (Pronestyl) is used to suppress cardiac arrhythmias. When administered intravenously, it must be accompanied by continuous cardiac monitoring by ECG.

Question22

The nurse is applying silver sulfadiazine (Silvadene) to a child with severe burns to arms and legs. Which side effect should the nurse be monitoring for? A) Skin discoloration B) Hardened eschar C) Increased neutrophils D) Urine sulfa crystals Review Information: The correct answer is D: Urine sulfa crystals Silver sulfadiazine is a broad spectrum antimicrobial, especially effective against pseudomonas. When applied to extensive areas, however, it may cause a transient neutropenia, as well as renal function changes with sulfa crystals production and kernicterus.

Question23

The nurse is monitoring a client receiving a thrombolytic agent, alteplase (Activase tissue plasminogen activator), for treatment of a myocardial infarction. What outcome indicates the client is receiving adequate therapy within the first hours of treatment? A) Absence of a dysrhythmia (or arrhythmia) Question21 The nurse is providing education for a client with B) Blood pressure reduction newly diagnosed tuberculosis. Which statement C) Cardiac enzymes are within normal limits should be included in the information that is giv- D) Return of ST segment to baseline on ECG en to the client? A) «Isolate yourself from others until you are finReview Information: The correct answer is D: ished taking your medication.» B) «Follow up with your primary care provider in Return of ST segment to baseline on ECG Improved perfusion should result from this medi3 months.» C) «Continue to take your medications even cation, along with the reduction of ST segment elevation. when you are feeling fine.» D) «Continue to get yearly tuberculin skin Collected by :DeepaRajesh [ 43 ] [email protected] Kuwait

Question24

The provider has ordered daily high doses of aspirin for a client with rheumatoid arthritis. The nurse instructs the client to discontinue the medication and contact the provider if which of the following symptoms occur? A) Infection of the gums B) Diarrhea for more than one day C) Numbness in the lower extremities D) Ringing in the ears Review Information: The correct answer is D: Ringing in the ears Aspirin stimulates the central nervous system which may result in ringing in the ears. Deglin, J.D. and Vallerand, A.H. (2001). Davis’ drug guide for nurses. (7th edition). Philadelphia: F.A. Davis Company. Key, J.L. and Hayes, E.R. (2003). Pharmacology, a nursing process approach. (4th edition). Philadelphia: Saunders.

Question25

A nurse is caring for a client who is receiving methyldopa hydrochloride (Aldomet) intravenously. Which of the following assessment findings would indicate to the nurse that the client may be having an adverse reaction to the medication? A) Headache B) Mood changes C) Hyperkalemia D) Palpitations Review Information: The correct answer is B: Mood changes The nurse should assess the client for alterations in mental status such as mood changes. These symptoms should be reported promptly. Deglin, J.D. and Vallerand, A.H. (2001). Davis’ drug guide for nurses. (7th edition). Philadelphia: F.A. Davis Company. Wilson, B.A., Shannon, M.T., and Stang, C.L. (2004). Nurse’s drug guide. Upper Saddle River, New Jersey: Pearson Prentice Hall.

Question26

The nurse is teaching a child and the family about the medication phenytoin (Dilantin) prescribed for seizure control. Which of the following side effects is most likely to occur?

A) Vertigo B) Drowsiness C) Gingival hyperplasia D) Vomiting

Review Information: The correct answer is C: Gingival hyperplasia Swollen and tender gums occur often with use of phenytoin. Good oral hygiene and regular visits to the dentist should be emphasized.

Question27

The use of atropine for treatment of symptomatic bradycardia is contraindicated for a client with which of the following conditions? A) Urinary incontinence B) Glaucoma C) Increased intracranial pressure D) Right sided heart failure Review Information: The correct answer is B: Glaucoma Atropine is contraindicated in clients with angleclosure glaucoma because it can cause pupillary dilation with an increase in aqueous humor, leading to a resultant increase in optic pressure.

Question28

A pregnant woman is hospitalized for treatment of pregnancy induced hypertension (PIH) in the third trimester. She is receiving magnesium sulfate intravenously. The nurse understands that this medication is used mainly for what purpose? A) Maintain normal blood pressure B) Prevent convulsive seizures C) Decrease the respiratory rate D) Increase uterine blood flow Review Information: The correct answer is B: Prevent convulsive seizures Magnesium sulfate is a central nervous system depressant. While it has many systemic effects, it is used in the client with pregnancy induced hypertension (PIH) to prevent seizures.

Question29

The nurse is teaching a group of women in a comCollected by :DeepaRajesh [ 44 ] [email protected] Kuwait

munity clinic about prevention of osteoporosis. Which of the following over-the-counter medications should the nurse recognize as having the most elemental calcium per tablet? A) Calcium chloride B) Calcium citrate C) Calcium gluconate D) Calcium carbonate Review Information: The correct answer is D: Calcium carbonate Calcium carbonate contains 400mg of elemental calcium in 1 gram of calcium carbonate.

bination? A) Enhanced pain relief B) Minimized side effects C) Prevention of drug tolerance D) Increased onset of action Review Information: The correct answer is A: Enhanced pain relief Combination of analgesics with different mechanisms of action can afford greater pain relief.

Question33

A client is receiving erythromycin 500mg IV every 6 hours to treat a pneumonia. Which of the The nurse is administering diltiazem (Cardizem) following is the most common side effect of the to a client. Prior to administration, it is important medication? for the nurse to assess which parameter? A) Blurred vision A) Temperature B) Nausea and vomiting B) Blood pressure C) Severe headache C) Vision D) Insomnia D) Bowel sounds

Question30

Review Information: The correct answer is B: Blood pressure Diltiazem (Cardizem) is a calcium channel blocker that causes systemic vasodilation resulting in decreased blood pressure.

Question31

The nurse is instructing a client with moderate persistent asthma on the proper method for using MDIs (multi-dose inhalers). Which medication should be administered first? A) Steroid B) Anticholinergic C) Mast cell stabilizer D) Beta agonist Review Information: The correct answer is D: Beta agonist The beta-agonist drugs help to relieve bronchospasm by relaxing the smooth muscle of the airway. These drugs should be taken first so that other medications can reach the lungs.

Question32

A post-operative client has a prescription for acetaminophen with codeine. What should the nurse recognizes as a primary effect of this com-

Review Information: The correct answer is B: Nausea and vomiting Nausea is a common side-effect of erythromycin in both oral and intravenous forms.

Question34

The health care provider orders an IV aminophylline infusion at 30 mg/hr. The pharmacy sends a 1,000 ml bag of D5W containing 500 mg of aminophylline. In order to administer 30 mg per hour, the RN will set the infusion rate at: A) 20 ml per hour B) 30 ml per hour C) 50 ml per hour D) 60 ml per hour Review Information: The correct answer is D: 60 ml per hour Using the ratio method to calculate infusion rate: mg to be given (30) : ml to be infused (X) :: mg available (500) : ml of solution (1,000). Solve for X by cross-multiplying: 30 x 1,000 = 500 x X (or cancel), 30,000 = 500 X, X = 30,000/500, X = 60ml per hour.

Question35

The nurse is assessing a 7 year-old after several days of treatment for a documented strep throat. Collected by :DeepaRajesh [ 45 ] [email protected] Kuwait

Which of the following statements suggests that further teaching is needed? A) «Sometimes I take my medicine with fruit juice.» B) «My mother makes me take my medicine right after school.» C) «Sometimes I take the pills in the morning and other times at night.» D) «I am feeling much better than I did last week.» Review Information: The correct answer is C: «Sometimes I take the pills in the morning and other times at night.» Inconsistency in taking the prescribed medication indicates more teaching is needed.

Question36

The nurse is caring for a 10 year-old client who will be placed on heparin therapy. Which assessment is critical for the nurse to make before initiating therapy A) Vital signs B) Weight C) Lung sounds D) Skin turgor Review Information: The correct answer is B: Weight Check the client>>s weight because dosage is calculated on the basis of weight.

Question37

In providing care for a client with pain from a sickle cell crisis, which one of the following medication orders for pain control should be questioned by the nurse? A) Demerol B) Morphine C) Methadone D) Codeine

induced seizures.

Question38

A 5 year-old has been rushed to the emergency room several hours after acetaminophen poisoning. Which laboratory result should receive attention by the nurse? A) Sedimentation rate B) Profile 2 C) Bilirubin D) Neutrophils Review Information: The correct answer is C: Bilirubin Bilirubin, along with liver enzymes ALT and AST, may rise in the second stage (1-3 days) after a significant overdose, indicating cellular necrosis and liver dysfunction. A prolonged prothrombin time may also be found.

Question39

An elderly client is on an anticholinergic metered dose inhaler (MDI) for chronic obstructive pulmonary disease. The nurse would suggest a spacer to A) enhance the administration of the medication B) increase client compliance C) improve aerosol delivery in clients who are not able to coordinate the MDI D) prevent exacerbation of COPD Review Information: The correct answer is C: improve aerosol delivery in clients who are not able to coordinate the MDI Spacers improve the medication delivery in clients who are unable to coordinate the movements of administering a dose with an MDI.

Question40

The nurse is teaching a parent how to administer oral iron supplements to a 2 year-old child. Review Information: The correct answer is A: Which of the following interventions should be Demerol included in the teaching? Meperidine is not recommended in clients with A) Stop the medication if the stools become tarry sickle cell disease. Normeperidine, a metabolite green of meperidine, is a central nervous system stim- B) Give the medicine with orange juice and ulant that produces anxiety, tremors, myoclonus, through a straw and generalized seizures when it accumulates C) Add the medicine to a bottle of formula with repetitive dosing. Clients with sickle cell dis- D) Administer the iron with your child>s meals ease are particularly at risk for normeperidineCollected by :DeepaRajesh [ 46 ] [email protected] Kuwait

Review Information: The correct answer is B: Give the medicine with orange juice and through a straw Absorption of iron is facilitated in an environment rich in Vitamin C. Since liquid iron preparation will stain teeth, a straw is preferred.

Free NCLEX-RN Sample Test Questions For Nursing Review (Pharmacology Set 1) Jul31, Question1

A client has an order for antibiotic therapy after hospital treatment of a staph infection. Which of the following should the nurse emphasize? A) Scheduling follow-up blood cultures B) Completing the full course of medications C) Visiting the provider in a few weeks D) Monitoring for signs of recurrent infection Review Information: The correct answer is B: Completing the full course of medications In order for antibiotic therapy to be effective in eradicating an infection, the client must compete the entire course of prescribed therapy. When findings subside, stopping the medication early may lead to recurrence or subsequent drug resistance.

Question2

A 72 year-old client is admitted for possible dehydration. The nurse knows that older adults are particularly at risk for dehydration because they have A) an increased need for extravascular fluid B) a decreased sensation of thirst C) an increase in diaphoresis D) higher metabolic demands Review Information: The correct answer is B: a decreased sensation of thirst The elderly have a reduction in thirst sensation causing them to consume less fluid. Other risk factors may include fear of incontinence, inability to drink fluids independently and lack of motivation.

Question3

A male client is admitted with a spinal cord injury at level C4. The client asks the nurse how the injury is going to affect his sexual function. The nurse would respond A) «Normal sexual function is not possible.» B) «Sexual functioning will not be impaired at all.» C) «Erections will be possible.» D) «Ejaculation will be normal.» Review Information: The correct answer is C: «Erections will be possible.» Because they are a reflex reaction, erections can be stimulated by stroking the genitalia.

Question4

An 82 year-old client complains of chronic constipation. To improve bowel function, the nurse should first suggest A) Increasing fiber intake to 20-30 grams daily B) Daily use of laxatives C) Avoidance of binding foods such as cheese and chocolate D) Monitoring a balance between activity and rest Review Information: The correct answer is A: Increasing fiber intake to 20-30 grams daily The incorporation of high fiber into the diet is an effective way to promote bowel elimination in the elderly.

Question5

A 4 year-old child is admitted with burns on his legs and lower abdomen. When assessing the child’s hydration status, which of the following indicates a less than adequate fluid replacement? A) Decreasing hematocrit and increasing urine volume B) Rising hematocrit and decreasing urine volume C) Falling hematocrit and decreasing urine volume D) Stable hematocrit and increasing urine volume

Collected by :DeepaRajesh [ 47 ] [email protected] Kuwait

Review Information: The correct answer is B: Rising hematocrit and decreasing urine volume A rising hematocrit indicates a decreased total blood volume, a finding consistent with dehydration.

Question6

A client receiving chemotherapy has developed sores in his mouth. He asks the nurse why this happened. What is the nurse’s best response? A) «It is a sign that the medication is working.» B) «You need to have better oral hygiene.» C) «The cells in the mouth are sensitive to the chemotherapy.» D) «This always happens with chemotherapy.»

D) 20-50 beats/minute Review Information: The correct answer is C: 40-70 beats/minute The intrinsic rate of the AV node is within the range of 40-70 beats per minute.

Question9

A client is to receive 3 doses of potassium chloride 10 mEq in 100cc normal saline to infuse over 30 minutes each. Which of the following is a priority assessment to perform before giving this medication? A) Oral fluid intake B) Bowel sounds C) Grip strength D) Urine output

Review Information: The correct answer is C: «The cells in the mouth are sensitive to the chemotherapy.» The epithelial cells in the mouth are very sensi- Review Information: The correct answer is D: tive to chemotherapy due to their high rate of cell Urine output Potassium chloride should only be administered turnover. after adequate urine output (>20cc/hour for 2 consecutive hours) has been established. Impaired ability to excrete potassium via the kidQuestion7 You are caring for a client with deep vein throm- neys can result in hyperkalemia. bosis who is on Heparin IV. The latest APTT is 50 seconds. If the laboratory normal range is 16-24 Question10 seconds, you would anticipate The unlicensed assistive personnel (UAP) reA) maintaining the current heparin dose B) increasing the heparin as it does not appear ports to the nurse that a client with cirrhosis who had a paracentesis yesterday has become more therapeutic. lethargic and has musty smelling breath. A critiC) giving protamine sulfate as an antidote. D) repeating the blood test 1 hour after giving cal assessment for increasing encephalopathy is heparin. A) monitor the client>s clotting status B) assess upper abdomen for bruits Review Information: The correct answer is A: C) assess for flap-like tremors of the hands D) measure abdominal girth changes maintaining the current heparin dose The range for a therapeutic APTT is 1.5-2 times the control. Therefore the client is receiving a Review Information: The correct answer is C: therapeutic dose of Heparin. assess for flap-like tremors of the hands A client with cirrhosis of the liver who develops subtle changes in mental status and has a musty Question8 A client is admitted with a diagnosis of nodal odor to the breath is at risk for developing more bigeminy. The nurse knows that the atrioven- advanced signs of encephalopathy. tricular (AV) node has an intrinsic rate of A) 60-100 beats/minute Question11 B) 10-30 beats/minute A client is scheduled for an intravenous pyeloC) 40-70 beats/minute Collected by :DeepaRajesh [ 48 ] [email protected] Kuwait

gram (IVP). After the contrast material is injected, which of the following client reactions should be reported immediately? A) Feeling warm B) Face flushing C) Salty taste D) Hives Review Information: The correct answer is D: Hives This is a sign of anaphylaxis and should be reported immediately. The other reactions are considered normal and the client should be informed that they may occur. .

Question12

A client is prescribed an inhaler. How should the nurse instruct the client to breathe in the medication? A) As quickly as possible B) As slowly as possible C) Deeply for 3-4 seconds D) Until hearing whistling by the spacer Review Information: The correct answer is C: Deeply for 3-4 seconds The client should be instructed to breath in the medication for 3-4 seconds in order to receive the correct dosage of medication.

Question13

The nurse is caring for clients over the age of 70. The nurse knows that due to age-related changes, the elderly clients tolerate diets that are A) high protein B) high carbohydrates C) low fat D) high calories Review Information: The correct answer is C: low fat Due to age related changes, the diet of the elderly should include a lower quantity and higher quality of food. Fewer carbohydrates and fats are required in their diets.

Question14

A woman with a 28 week pregnancy is on the way to the emergency department by ambulance with a tentative diagnosis of abruptio placenta. Which should the nurse do first when the woman arrives? A) administer oxygen by mask at 100% B) start a second IV with an 18 gauge cannula C) check fetal heart rate every 15 minutes D) insert urethral catheter with hourly urine outputs Review Information: The correct answer is A: administer oxygen by mask at 100% Administering oxygen in this situation would increase the circulating oxygen in the mother’s circulation to the fetus’s circulation. This action will minimize complications.

Question15

A client in respiratory distress is admitted with arterial blood gas results of: PH 7.30; PO2 58, PCO2 34; and HCO3 19. The nurse determines that the client is in A) metabolic acidosis B) metabolic alkalosis C) respiratory acidosis D) respiratory alkalosis Review Information: The correct answer is A: metabolic acidosis These lab values indicate metabolic acidosis: the PH is low, PCO2 is normal, and bicarbonate level is low.

Question16

A client is diagnosed with gastroesophageal reflux disease (GERD). The nurse>s instruction to the client regarding diet should be to A) avoid all raw fruits and vegetables B) increase intake of milk products C) decrease intake of fatty foods D) focus on 3 average size meals a day Review Information: The correct answer is C: decrease intake of fatty foods GERD may be aggravated by a fatty diet. A diet low in fat would decrease the symptoms of GERD. Other agents which should also be decreased or avoided are: cigarette smoking, caffeine, alcoCollected by :DeepaRajesh [ 49 ] [email protected] Kuwait

hol, chocolate, and meperidine (Demerol).

Question17

months D) The client>s risk for cardiac complications

After surgery, a client with a nasogastric tube complains of nausea. What action would the nurse take? A) Call the health care provider B) Administer an antiemetic C) Put the bed in Fowler’s position D) Check the patency of the tube

Review Information: The correct answer is C: The average blood glucose for the past 2-3 months By testing the portion of the hemoglobin that absorbs glucose, it is possible to determine the average blood glucose over the life span of the red cell, 120 days.

Review Information: The correct answer is D: Check the patency of the tube An indication that the nasogastric tube is obstructed is a client’s complaint of nausea. Nasogastric tubes may become obstructed with mucus or sediment.

Question20

Question18

A client with testicular cancer has had an orchiectomy. Prior to discharge the client expresses his fears related to his prognosis. Which principle should the nurse base the response on? A) Testicular cancer has a cure rate of 90% with early diagnosis B) Testicular cancer has a cure rate of 50% with early diagnosis C) Intensive chemotherapy is the treatment of choice D) Testicular cancer is usually fatal Review Information: The correct answer is A: Testicular cancer has a cure rate of 90% with early diagnosis With aggressive treatment and early detection/ diagnosis the cure rate is 90%.

Question19

A client newly diagnosed with Type I Diabetes Mellitus asks the purpose of the test measuring glycosylated hemoglobin. The nurse should explain that the purpose of this test is to determine: A) The presence of anemia often associated with Diabetes B) The oxygen carrying capacity of the client>s red cells C) The average blood glucose for the past 2-3

A client is admitted for a possible pacemaker insertion. What is the intrinsic rate of the heart>s own pacemaker? A) 30-50 beats/minute B) 60-100 beats/minute C) 20-60 beats/minute D) 90-100 beats/minute Review Information: The correct answer is B: 60-100 beats/minute This is the intrinsic rate of the SA node.

Question21

The nurse discusses nutrition with a pregnant woman who is iron deficient and follows a vegetarian diet. The selection of which foods indicates the woman has learned sources of iron? A) Cereal and dried fruits B) Whole grains and yellow vegetables C) Leafy green vegetables and oranges D) Fish and dairy products Review Information: The correct answer is A: Cereal and dried fruits Both of these foods would be a good source of iron.

Question22

Prior to administering Alteplase (TPA) to a client admitted for a cerebral vascular accident (CVA), it is critical that the nurse assess: A) Neuro signs B) Mental status C) Blood pressure D) PT/PTT Review Information: The correct answer is D: Collected by :DeepaRajesh [ 50 ] [email protected] Kuwait

PT/PTT TPA is a potent thrombolytic enzyme. Because bleeding is the most common side effect, it is most essential to evaluate clotting studies including PT, PTT, APTT, platelets, and hematocrit before beginning therapy.

of your decision.»

Review Information: The correct answer is C: «You have the right to change your mind. You seem anxious. Can we talk about it?» This response indicates acknowledgment of the client’s rights and the opportunity for the client Question23 The nurse enters the room of a client diagnosed to clarify and ventilate concerns. After this, if the with COPD. The client’s skin is pink, and respi- client continues to refuse, the provider should be rations are 8 per minute. The client’s oxygen is notified. running at 6 liters per minute. What should be the nurse’s first action? Question26 A) Call the health care provider A nurse who has been named in a lawsuit can B) Put the client in Fowler’s position use which of these factors for the best protection C) Lower the oxygen rate in a court of law? D) Take the vital signs A) Clinical specialty certification in the associated area of practice Review Information: The correct answer is C: B) Documentation on the specific client record Lower the oxygen rate with a focus on the nursing process In client’s diagnosed with COPD, the drive to C) Yearly evaluations and proficiency reports breathe is hypoxia. If oxygen is delivered at too prepared by nurse’s manager high of a concentration, this drive will be elimi- D) Verification of provider>s orders for the plan nated and the client’s depth and rate of respi- of care with identification of outcomes rations will decrease. Therefore the first action should be to lower the oxygen rate. Review Information: The correct answer is B: Documentation on the specific client record with Question24 The client with goiter is treated with potassium a focus on the nursing process iodide preoperatively. What should the nurse Documentation is the key to protect nurses when a lawsuit is filed. The thorough documentation recognize as the purpose of this medication? should include all steps of the nursing process – A) Reduce vascularity of the thyroid assessment, analysis, plan, intervention, evaluB) Correct chronic hyperthyroidism ation. In addition, it should include pertinent data C) Destroy the thyroid gland function such as times, dosages and sites of actions, D) Balance enzymes and electrolytes assessment data, the nurse’s response to a Review Information: The correct answer is A: change in the client’s condition, specific actions taken, if and when the notification occurred to Reduce vascularity of the thyroid Potassium iodide solution, or Lugol>>s solution the provider or other health care team members, may be used preoperatively to reduce the size and what was prescribed along with the client’s outcomes. and vascularity of the thyroid gland.

Question27

The nurse is caring for clients over the age of 70. One hour before the first treatment is scheduled, The nurse is aware that when giving medications the client becomes anxious and states he does to older clients, it is best to not wish to go through with electroconvulsive A) start low, go slow therapy. Which response by the nurse is most B) avoid stopping a medication entirely C) avoid drugs with side effects that impact cogappropriate? A) «I’ll go with you and will be there with you dur- nition D) review the drug regimen yearly ing the treatment.» B) «You’ll be asleep and won’t remember anyReview Information: The correct answer is A: thing.» C) «You have the right to change your mind. You start low, go slow Due to physiological changes in the elderly, as seem anxious. Can we talk about it?» D) «I’ll call the health care provider to notify them well as conditions such as dehydration, hyper-

Question25

Collected by :DeepaRajesh [ 51 ] [email protected] Kuwait

thermia, immobility and liver disease, the effective metabolism of drugs may decrease. As a result, drugs can accumulate to toxic levels and cause serious adverse reactions.

Question28

You are caring for a hypertensive client with a new order for captopril (Capoten). Which information should the nurse include in client teaching? A) Avoid green leafy vegetables B) Restrict fluids to 1000cc/day C) Avoid the use of salt substitutes D) Take the medication with meals Review Information: The correct answer is C: Avoid the use of salt substitutes Captopril can cause an accumulation of potassium or hyperkalemia. Clients should avoid the use of salt substitutes, which are generally potassium-based.

Question29

A client has bilateral knee pain from osteoarthritis. In addition to taking the prescribed non-steroidal anti-inflammatory drug (NSAID), the nurse should instruct the client to A) start a regular exercise program B) rest the knees as much as possible to decrease inflammation C) avoid foods high in citric acid D) keep the legs elevated when sitting

Question31

Which of these clients should the charge nurse assign to the registered nurse (RN)? A) A 56 year-old with atrial fibrillation receiving digoxin B) A 60 year-old client with COPD on oxygen at 2 L/min C) A 24 year-old post-op client with type 1 diabetes in the process of discharge D) An 80 year-old client recovering 24 hours post right hip replacement Review Information: The correct answer is C: A 24 year-old post-op client with type 1 diabetes in the process of discharge Discharge teaching must be done by an RN. Practical nurses (PNs) or unlicensed assistive personnel (UAPs) can reinforce education after the RN does the initial teaching.

Question32

A hypertensive client is started on atenolol (Tenormin). The nurse instructs the client to immediately report which of these findings? A) Rapid breathing B) Slow, bounding pulse C) Jaundiced sclera D) Weight gain

Review Information: The correct answer is B: Slow, bounding pulse Atenolol (Tenormin) is a beta-blocker that can Review Information: The correct answer is A: cause side effects including bradycardia and hypotension. start a regular exercise program A regular exercise program is beneficial in treat- Question33 ing osteoarthritis. It can restore self-esteem and An 80 year-old client is admitted with a diagnosis improve physical functioning. of malnutrition. In addition to physical assessments, which of the following lab tests should be Question30 An arterial blood gases test (ABG) is ordered closely monitored? for a confused client. The respiratory therapist A) Urine protein draws the blood and then asks the nurse to apply B) Urine creatinine pressure to the area so the therapist can take the C) Serum calcium specimen to the lab. How long should the nurse D) Serum albumin apply pressure to the area? Review Information: The correct answer is D: A) 3 minutes Serum albumin B) 5 minutes Serum albumin is a valuable indicator of protein C) 8 minutes deficiency and, later, nutritional status in adults. D) 10 minutes A normal reading for an elder’s serum albumin is Review Information: The correct answer is B: between 3.0-5.0 g/dl. 5 minutes Question34 It is necessary to apply pressure to the area for Upon admission to an intensive care unit, a client 5 minutes to prevent bleeding and the formation diagnosed with an acute myocardial infarction is of hematomas. ordered oxygen. The nurse knows that the major Collected by :DeepaRajesh [ 52 ] [email protected] Kuwait

reason that oxygen is administered in this situation is to A) saturate the red blood cells B) relieve dyspnea C) decrease cyanosis D) increase oxygen level in the myocardium Review Information: The correct answer is D: increase oxygen level in the myocardium Anoxia of the myocardium occurs in myocardial infarction. Oxygen administration will help relieve dyspnea and cyanosis associated with the condition but the major purpose is to increase the oxygen concentration in the damaged myocardial tissue.

Question35

The nurse is teaching a client with chronic renal failure (CRF) about medications. The client questions the purpose of aluminum hydroxide (Amphojel) in her medication regimen. What is the best explanation for the nurse to give the client about the therapeutic effects of this medication? A) It decreases serum phosphate B) It will reduce serum calcium C) Amphojel increases urine output D) The drug is taken to control gastric acid secretion Review Information: The correct answer is A: It decreases serum phosphate Aluminum binds phosphates that tend to accumulate in the patient with chronic renal failure due to decreased filtration capacity of the kidney. Antacids such as Amphojel are commonly used to accomplish this.

Question36

A 66 year-old client is admitted for mitral valve replacement surgery. The client has a history of mitral valve regurgitation and mitral stenosis since her teenage years. During the admission assessment, the nurse should ask the client if as a child she had A) measles B) rheumatic fever C) hay fever D) encephalitis

During nursing rounds which of these assessments would require immediate corrective action and further instruction to the practical nurse (PN) about proper care? A) The weights of the skin traction of a client are hanging about 2 inches from the floor B) A client with a hip prosthesis 1 day post operatively is lying in bed with internal rotation and adduction of the affected leg C) The nurse observes that the PN moves the extremity of a client with an external fixation device by picking up the frame D) A client with skeletal traction states «The other nurse said that the clear, yellow and crusty drainage around the pin site is a good sign» Review Information: The correct answer is B: A client with a hip prosthesis 1 day post operatively is lying in bed with internal rotation and adduction of the affected leg This position should be prevented in order to prevent dislodgment of the hip prosthesis, especially in the first 48 to 72 hours post-op. The other assessments are not of concern.

Question38

A client diagnosed with gouty arthritis is admitted with severe pain and edema in the right foot. When the nurse develops a plan of care, which intervention should be included? A) high protein diet B) salicylates C) hot compresses to affected joints D) intake of at least 3000cc/day Review Information: The correct answer is D: intake of at least 3000cc/day Fluid intake should be increased to prevent precipitation of urate in the kidneys.

Question39

A 55 year-old woman is taking Prednisone and aspirin (ASA) as part of her treatment for rheumatoid arthritis. Which of the following would be an appropriate intervention for the nurse? A) Assess the pulse rate q 4 hours B) Monitor her level of consciousness q shift C) Test her stools for occult blood D) Discuss fiber in the diet to prevent constipation

Review Information: The correct answer is B: rheumatic fever Clients that present with mitral stenosis often Review Information: The correct answer is C: have a history of rheumatic fever or bacterial en- Test her stools for occult blood docarditis. Both Prednisone and ASA can lead to GI bleeding, therefore monitoring for occult blood would Question37 Collected by :DeepaRajesh [ 53 ] [email protected] Kuwait

be appropriate.

getting out of bed. In the event that this restricted movement could cause more harm, such as asQuestion40 A client with testicular cancer is scheduled for a piration, then a sitter should be requested. These right orchiectomy. The nurse knows that an or- are to be provided by the facility in the event the family cannot do so. This client needs to cough chiectomy is the and be watched rather than restricted. SuctionA) surgical removal of the entire scrotum ing will not prevent aspiration in this situation. B) surgical removal of a testicle Cough suppressants should be avoided for this C) dissection of related lymph nodes client. D) partial surgical removal of the penis Review Information: The correct answer is B: surgical removal of a testicle The affected testicle is surgically removed along with its tunica and spermatic cord.

Question3

A couple trying to conceive asks the nurse when ovulation occurs. The woman reports a regular 32 day cycle. Which response by the nurse is correct? A) Days 7-10 Free NCLEX-RN Sample Test Ques- B) Days 10-13 C) Days 14-16 tions For Nursing Review (Part 5) D) Days 17-19 Jul31, Review Information: The correct answer is D: Days 17-19 Question1 Ovulation occurs 14 days prior to menses. ConA client complains of some discomfort after a sidering that the woman>>s cycle is 32 days, below the knee amputation. Which action by the subtracting 14 from 32 suggests ovulation is at nurse is most appropriate initially? about the 18th day. A) Conduct guided imagery or distraction B) Ensure that the stump is elevated the first day Question4 A newborn is having difficulty maintaining a tempost-op C) Wrap the stump snugly in an elastic bandage perature above 98 degrees Fahrenheit and has been placed in an incubator. Which action is a D) Administer opioid narcotics as ordered nursing priority? Review Information: The correct answer is B: A) Protect the eyes of the neonate from the heat Ensure that the stump is elevated the first day lamp B) Monitor the neonate’s temperature post-op This priority intervention prevents pressure C) Warm all medications and liquids before givcaused by pooling of blood, thus minimizing the ing pain. Without this measure, a firm elastic band- D) Avoid touching the neonate with cold hands age, opioid narcotics, or guided imagery will have little effect. Opioid narcotics are given for severe pain.

Question2

A 78 year-old client with pneumonia has a productive cough, but is confused. Safety protective devices (restraints) have been ordered for this client. How can the nurse prevent aspiration? A) Suction the client frequently while restrained B) Secure all 4 restraints to 1 side of bed C) Obtain a sitter for the client while restrained D) Request an order for a cough suppressant Review Information: The correct answer is C: Obtain a sitter for the client while restrained The plan to use safety devices (restraints) should be rethought. Restraints are used to protect the client from harm caused by removing tubes or

Review Information: The correct answer is B: Monitor the neonate’s temperature When using a warming device the neonate’s temperature should be continuously monitored for undesired elevations. The use of heat lamps is not safe as there is no way to regulate their temperature. Warming medications and fluids is not indicated. While touching with cold hands can startle the infant it does not pose a safety risk.

Question5

Which oxygen delivery system would the nurse apply that would provide the highest concentrations of oxygen to the client? A) Venturi mask B) Partial rebreather mask C) Non-rebreather mask Collected by :DeepaRajesh [ 54 ] [email protected] Kuwait

D) Simple face mask Review Information: The correct answer is C: Non-rebreather mask The non-rebreather mask has a one-way valve that prevents exhales air from entering the reservoir bag and one or more valves covering the air holes on the face mask itself to prevent inhalation of room air but to allow exhalation of air. When a tight seal is achieved around the mask up to 100% of the oxygen is available.

Question6

At a senior citizens meeting a nurse talks with a client who has Type 1 diabetes mellitus. Which statement by the client during the conversation is most predictive of a potential for impaired skin integrity? A) «I give my insulin to myself in my thighs.» B) «Sometimes when I put my shoes on I don>t know where my toes are.» C) «Here are my up and down glucose readings that I wrote on my calendar.» D) «If I bathe more than once a week my skin feels too dry.»

expected with this type of surgery. The bleeding should also be documented in the nurse’s notes.

Question8

The nurse is caring for a 1 year-old child who has 6 teeth. What is the best way for the nurse to give mouth care to this child? A) Using a moist soft brush or cloth to clean teeth and gums B) Swabbing teeth and gums with flavored mouthwash C) Offering a bottle of water for the child to drink D) Brushing with toothpaste and flossing each tooth Review Information: The correct answer is A: Using a moist soft brush or cloth to clean teeth and gums The nurse should use a soft cloth or soft brush to do mouth care so that the child can adjust to the routine of cleaning the mouth and teeth.

Question9

In addition to standard precautions, a nurse should implement contact precautions for which Review Information: The correct answer is B: client? «Sometimes when I put my shoes on I don>>t A) 60 year-old with herpes simplex know where my toes are.» B) 6 year-old with mononucleosis Peripheral neuropathy can lead to lack of sensa- C) 45 year-old with pneumonia tion in the lower extremities. Clients who do not D) 3 year-old with scarlet fever feel pressure and/or pain are at high risk for skin impairment. Review Information: The correct answer is A: 60 year-old with herpes simplex Clients who have herpes simplex infections must Question7 A client returns from surgery after an open reduc- have contact precautions in addition to standard tion of a femur fracture. There is a small blood- precautions because of the associated, potentialstain on the cast. Four hours later, the nurse ob- ly weeping, skin lesions. Contact precautions are serves that the stain has doubled in size. What is used for clients who are infected by microorganisms that are transmitted by direct contact with the best action for the nurse to take? the client, including hand or skin-to-skin contact. A) Call the health care provider B) Access the site by cutting a window in the cast Question10 C) Simply record the findings in the nurse>s Which of the following situations is most likely to notes only produce sepsis in the neonate? D) Outline the spot with a pencil and note the A) Maternal diabetes time and date on the cast B) Prolonged rupture of membranes C) Cesarean delivery Review Information: The correct answer is D: D) Precipitous vaginal birth Outline the spot with a pencil and note the time and date on the cast Review Information: The correct answer is B: This is a good way to assess the amount of bleed- Prolonged rupture of membranes ing over a period of time. The bleeding does not Premature rupture of the membranes (PROM) is appear to be excessive and some bleeding is a leading cause of newborn sepsis. After 12-24 Collected by :DeepaRajesh [ 55 ] [email protected] Kuwait

hours of leaking fluid, measures are taken to reduce the risk to mother and the fetus/newborn.

Question11

The nurse is teaching a parent about side effects of routine immunizations. Which of the following must be reported immediately? A) Irritability B) Slight edema at site C) Local tenderness D) Seizure activity Review Information: The correct answer is D: Seizure activity Other reactions that should be reported include crying for >3 hours, temperature over 104.8 degrees Fahrenheit following DPT immunization, and tender, swollen, reddened areas.

Question12

The nurse is at the community center speaking with retired people about glaucoma. Which comment by one of the retirees would the nurse support to reinforce correct information? A) «I usually avoid driving at night since lights sometimes seem to make things blur.» B) «I take half of the usual dose for my sinuses to maintain my blood pressure.» C) «I have to sit at the side of the pool with the grandchildren since I can>t swim with this eye problem.» D) «I take extra fiber and drink lots of water to avoid getting constipated.» Review Information: The correct answer is D: «I take extra fiber and drink lots of water to avoid getting constipated.» Any activity that involves straining should be avoided in clients with glaucoma. Such activities would increase intraocular pressure.

Provide water feedings at least every 2 hours Protecting the eyes of the neonates is very important to prevent damage when under the ultraviolet lights, but since the blanket is used, extra protection of the eyes is unnecessary. It is recommended that the neonate remain under the lights for extended periods. The neonate’s skin is exposed to the light and the temperature is monitored, but a heater may not be necessary. There is no reason to withhold feedings. Frequent water or feedings are given to help with the excretion of the bilirubin in the stool.

Question14

A nurse is performing the routine daily cleaning of a tracheostomy. During the procedure, the client coughs and displaces the tracheostomy tube. This negative outcome could have avoided by A) placing an obturator at the client’s bedside B) having another nurse assist with the procedure C) fastening clean tracheostomy ties before removing old ties D) placing the client in a flat, supine position Review Information: The correct answer is C: fastening clean tracheostomy ties before removing old ties Fastening clean tracheostomy ties before removing old ones will ensure that the tracheostomy is secured during the entire cleaning procedure. The obturator is useful to keep the airway open only after the tracheostomy outer tube is coughed out. A second nurse is not needed. Changing the position may not prevent a dislodged tracheostomy.

Question15

A 4 year-old hospitalized child begins to have a seizure while playing with hard plastic toys in the hallway. Of the following nursing actions, which one should the nurse do first? Question13 A newborn has hyperbilirubinemia and is un- A) Place the child in the nearest bed dergoing phototherapy with a fiberoptic blanket. B) Administer IV medication to slow down the Which safety measure is most important during seizure C) Place a padded tongue blade in the child>s this process? A) Regulate the neonate’s temperature using a mouth D) Remove the child>s toys from the immediate radiant heater B) Withhold feedings while under the photother- area apy C) Provide water feedings at least every 2 hours Review Information: The correct answer is D: D) Protect the eyes of neonate from the photo- Remove the child>>s toys from the immediate area therapy lights Nursing care for a child having a seizure includes, Review Information: The correct answer is C: maintaining airway patency, ensuring safety, adCollected by :DeepaRajesh [ 56 ] [email protected] Kuwait

ministering medications, and providing emotional support. Since the seizure has already started, nothing should be forced into the child>>s mouth and the child should not be moved. Of the choices given, the first priority would be to provide a safe environment.

D) Peripheral glucose stick

Review Information: The correct answer is C: Pulse oximetry A sudden change in mental status in any postop client should trigger a nursing intervention directed toward respiratory evaluation. Pulse Question16 The nurse is teaching home care to the parents oximetry would be the initial assessment. If availof a child with acute spasmodic croup. The most able, arterial blood gases would be better. Acute respiratory failure is the sudden inability of the important aspects of this care is/are respiratory system to maintain adequate gas A) sedation as needed to prevent exhaustion exchange which may result in hypercapnia and/ B) antibiotic therapy for 10 to 14 days or hypoxemia. Clinical findings of hypoxemia inC) humidified air and increased oral fluids D) antihistamines to decrease allergic response clude these finding which are listed in order of initial to later findings: restlessness, irritability, Review Information: The correct answer is C: agitation, dyspnea, disorientation, confusion, delirium, hallucinations, and loss of consciousness. humidified air and increased oral fluids The most important aspects of home care for a While there may be other factors influencing the child with acute spasmodic croup are humidified client>>s behavior, the first nursing action should air and increased oral fluids. Moisture soothes be directed toward maintaining oxygenation. inflamed membranes. Adequate systemic hydra- Once respiratory or oxygenation issues are ruled tion aids is mucociliary clearance and keeps se- out then significant changes in glucose would be cretions thin, white, watery, and easily removed evaluated. with minimal coughing. Question19 A newborn delivered at home without a birth atQuestion17 The nurse is assigned to care for a client who tendant is admitted to the hospital for observahas a leaking intracranial aneurysm. To minimize tion. The initial temperature is 95 degrees Fahrenheit (35 degrees Celsius) axillary. The nurse the risk of rebleeding, the nurse should plan to recognizes that cold stress may lead to what A) restrict visitors to immediate family B) avoid arousal of the client except for family complication? A) Lowered BMR visits C) keep client>s hips flexed at no less than 90 B) Reduced PaO2 C) Lethargy degrees D) apply a warming blanket for temperatures of D) Metabolic alkalosis 98 degrees Fahrenheit or less Review Information: The correct answer is B: Review Information: The correct answer is A: Reduced PaO2 Cold stress causes increased risk for respirarestrict visitors to immediate family Maintaining a quiet environment will assist in tory distress. The baby delivered in such circumminimizing cerebral rebleeding. When family vis- stances needs careful monitoring. In this situait, the client should not be disturbed. If the client tion, the newborn must be warmed immediately is awake, topics of a general nature are better to increase its temperature to at least 97 degrees choices for discussion than topics that result in Fahrenheit (36 degrees Celsius). emotional or physiological stimulation. Question20 Which contraindication should the nurse assess Question18 A client who is 12 hour post-op becomes con- for prior to giving a child immunizations? fused and says: “Giant sharks are swimming A) Mild cold symptoms across the ceiling.” Which assessment is nec- B) Chronic asthma essary to adequately identify the source of this C) Depressed immune system D) Allergy to eggs client>s behavior? A) Cardiac rhythm strip Review Information: The correct answer is C: B) Pupillary response Depressed immune system C) Pulse oximetry Collected by :DeepaRajesh [ 57 ] [email protected] Kuwait

Children who have a depressed immune system tis. This side effect is age-related and can be derelated to HIV or chemotherapy should not be tected with regular assessment of liver enzymes, given routine immunizations. which are released into the blood from damaged liver cells.

Question21

The nurse is caring for a client with a myocardial infarction. Which finding requires the nurse>s immediate action? A) Periorbital edema B) Dizzy spells C) Lethargy D) Shortness of breath Review Information: The correct answer is B: Dizzy spells Cardiac dysrhythmias may cause a transient drop in cardiac output and decreased blood flow to the brain. Near syncope refers to lightheartedness, dizziness, temporary confusion. Such «spells» may indicate runs of ventricular tachycardia or periods of asystole and should be reported immediately.

Question22

Decentralized scheduling is used on a nursing unit. A chief advantage of this management strategy is that it: A) considers client and staff needs B) conserves time spent on planning C) frees the nurse manager to handle other priorities D) allows requests for special privileges

Question24

A woman in her third trimester complains of severe heartburn. What is appropriate teaching by the nurse to help the woman alleviate these symptoms? A) Drink small amounts of liquids frequently B) Eat the evening meal just before retiring C) Take sodium bicarbonate after each meal D) Sleep with head propped on several pillows Review Information: The correct answer is D: Sleep with head propped on several pillows Heartburn is a burning sensation caused by regurgitation of gastric contents. It is best relieved by sleeping position, eating small meals, and not eating before bedtime.

Question25

A 16 year-old boy is admitted for Ewing>s sarcoma of the tibia. In discussing his care with the parents, the nurse understands that the initial treatment most often includes A) amputation just above the tumor B) surgical excision of the mass C) bone marrow graft in the affected leg D) radiation and chemotherapy

Review Information: The correct answer is D: radiation and chemotherapy The initial treatment of choice for Ewing>>s sarReview Information: The correct answer is A: coma is a combination of radiation and chemoconsiders client and staff needs therapy. Decentralized staffing takes into consideration specific client needs and staff interests and abili- Question26 ties. A new nurse manager is responsible for interviewing applicants for a staff nurse position. Question23 Which interview strategy would be the best apIncluded in teaching the client with tuberculo- proach? sis taking isoniazid (INH) about follow-up home A) Vary the interview style for each candidate to care, the nurse should emphasize that a labora- learn different techniques tory appointment for which of the following lab B) Use simple questions requiring «yes» and tests is critical? «no» answers to gain definitive information A) Liver function C) Obtain an interview guide from human resources for consistency in interviewing each B) Kidney function candidate C) Blood sugar D) Cardiac enzymes D) Ask personal information of each applicant to assure he/she can meet job demands Review Information: The correct answer is A: Liver function Review Information: The correct answer is C: INH can cause hepatocellular injury and hepati- Obtain an interview guide from human resources Collected by :DeepaRajesh [ 58 ] [email protected] Kuwait

for consistency in interviewing each candidate An interview guide used for each candidate enables the nurse manager to be more objective in the decision making. The nurse should use resources available in the agency before attempts to develop one from scratch. Certain personal questions are prohibited, and HR can identify these for novice managers.

Question27

C) oliguria D) neck veins are distended Review Information: The correct answer is C: oliguria Kidneys maintain fluid volume through adjustments in urine volume.

Question30

A 70 year-old woman is evaluated in the emerWhat is the best way that parents of pre-school- gency department for a wrist fracture of unknown ers can begin teaching their child about injury causes. During the process of taking client hisprevention? tory, which of these items should the nurse idenA) Set good examples themselves tify as related to the client’s greatest risk factors B) Protect their child from outside influences for osteoporosis? C) Make sure their child understands all the A) History of menopause at age 50 safety rules B) Taking high doses of steroids for arthritis for D) Discuss the consequences of not wearing many years protective devices C) Maintaining an inactive lifestyle for the past 10 years Review Information: The correct answer is A: D) Drinking 2 glasses of red wine each day for Set good examples themselves the past 30 years The preschool years are the time for parents to begin emphasizing safety principles as well as Review Information: The correct answer is providing protection. Setting a good example B: Taking high doses of steroids for arthritis for themselves is crucial because of the imitative many years behaviors of pre-schoolers; they are quick to no- The use of steroids, especially at high doses over tice discrepancies between what they see and time, increases the risk for osteoporosis. The what they are told. other options also predispose to osteoporosis, as do low bone mass, poor calcium absorption and moderate to high alcohol ingestion. LongQuestion28 A nurse assessing the newborn of a mother with term steroid treatment is the most significant risk diabetes understands that hypoglycemia is re- factor, however. lated to what pathophysiological process? Question31 A) Disruption of fetal glucose supply The nurse is caring for a 2 year-old who is being B) Pancreatic insufficiency treated with chelation therapy, calcium disodium C) Maternal insulin dependency edetate, for lead poisoning. The nurse should be D) Reduced glycogen reserves alert for which of the following side effects? A) Neurotoxicity Review Information: The correct answer is A: B) Hepatomegaly Disruption of fetal glucose supply C) Nephrotoxicity After delivery, the high glucose levels which D) Ototoxicity crossed the placenta to the fetus are suddenly stopped. The newborn continues to secrete insu- Review Information: The correct answer is C: lin in anticipation of glucose. When oral feedings Nephrotoxicity begin, the newborn will adjust insulin production Nephrotoxicity is a common side effect of calciwithin a day or two. um disodium edetate, in addition to lead poisoning in general.

Question29

The nurse is caring for a client with extracellular fluid volume deficit. Which of the following assessments would the nurse anticipate finding? A) bounding pulse B) rapid respirations

Question32

The parents of a toddler ask the nurse how long their child will have to sit in a car seat while in the automobile. What is the nurse’s best response to the parents? A) «Your child must use a care seat until he Collected by :DeepaRajesh [ 59 ] [email protected] Kuwait

weighs at least 40 pounds.» B) «The child must be 5 years of age to use a regular seat belt.» C) «Your child must reach a height of 50 inches to sit in a seat belt.» D) «The child can use a regular seat belt when he can sit still.» Review Information: The correct answer is A: «Your child must use a care seat until he weighs at least 40 pounds.» Children should use car seats until they weigh 40 pounds.

Question33

A client asks the nurse to explain the basic ideas of homeopathic medicine. The response that best explains this approach is that such remedies A) destroy organisms causing disease B) maintain fluid balance C) boost the immune system D) increase bodily energy Review Information: The correct answer is C: boost the immune system The practitioner treats with minute doses of plant, mineral or animal substances which provide a gentle stimulus to the body>>s own defenses.

Review Information: The correct answer is D: remove a mucus plug While no longer recommended for routine suctioning, saline may thin and loosen viscous secretions that are very difficult to move, perhaps making them easier to suction.

Question36

The nurse is performing a gestational age assessment on a newborn delivered 2 hours ago. When coming to a conclusion using the Ballard scale, which of these factors may affect the score? A) Birth weight B) Racial differences C) Fetal distress in labor D) Birth trauma Review Information: The correct answer is C: Fetal distress in labor The effects of earlier distress may alter the findings of reflex responses as measured on the Ballard tool. Other physical characteristics that estimate gestational age, such as amount of lanugo, sole creases and ear cartilage are unaffected by the other factors.

Question37

A nurse is caring for a client who had a closed Question34 reduction of a fractured right wrist followed by A client with a fractured femur has been in Rus- the application of a fiberglass cast 12 hours ago. sell’s traction for 24 hours. Which nursing action Which finding requires the nurse’s immediate atis associated with this therapy? tention? A) Check the skin on the sacrum for breakdown A) Capillary refill of fingers on right hand is 3 secB) Inspect the pin site for signs of infection onds C) Auscultate the lungs for atelectasis B) Skin warm to touch and normally colored D) Perform a neurovascular check for circula- C) Client reports prickling sensation in the right tion hand D) Slight swelling of fingers of right hand Information: The correct answer is D: Perform a neurovascular check for circulation Review Information: The correct answer is While each of these is an important assessment, C: Client reports prickling sensation in the right the neurovascular integrity check is most associ- hand ated with this type of traction. Russell’s traction A prickling sensation is an indication of compartis Buck’s traction with a sling under the knee. ment syndrome and requires immediate action by the nurse. The other findings are normal for a client in this situation. Question35 When suctioning a client>s tracheostomy, the Question38 nurse should instill saline in order to A client is admitted with the diagnosis of pulmoA) decrease the client>s discomfort nary embolism. While taking a history, the client B) reduce viscosity of secretions tells the nurse he was admitted for the same thing C) prevent client aspiration twice before, the last time just 3 months ago. The D) remove a mucus plug nurse would anticipate the provider ordering A) pulmonary embolectomy B) vena caval interruption Collected by :DeepaRajesh [ 60 ] [email protected] Kuwait

C) increasing the Coumadin therapy to an INR of 3-4 D) thrombolytic therapy Review Information: The correct answer is B: vena caval interruption Clients with contraindications to Heparin, recurrent PE or those with complications related to the medical therapy may require vena caval interruption by the placement of a filter device in the inferior vena cava. A filter can be placed transvenously to trap clots before they travel to the pulmonary circulation.

Question39

Which client is at highest risk for developing a pressure ulcer? A) 23 year-old in traction for fractured femur B) 72 year-old with peripheral vascular disease, who is unable to walk without assistance C) 75 year-old with left sided paresthesia who is incontinent of urine and stool D) 30 year-old who is comatose following a ruptured aneurysm Review Information: The correct answer is C: 75 year-old with left sided paresthesia who is incontinent of urine and stool Risk factors for pressure ulcers include: immobility, absence of sensation, decreased LOC, poor nutrition and hydration, skin moisture, incontinence, increased age, decreased immune response. This client has the greatest number of risk factors.

Question40

The nurse is teaching the mother of a 5 monthold about nutrition for her baby. Which statement by the mother indicates the need for further teaching? A) «I>m going to try feeding my baby some rice cereal.» B) «When he wakes at night for a bottle, I feed him.» C) «I dip his pacifier in honey so he>ll take it.» D) «I keep formula in the refrigerator for 24 hours.» Review Information: The correct answer is C: «I dip his pacifier in honey so he>>ll take it.» Honey has been associated with infant botulism and should be avoided. Older children and adults have digestive enzymes that kill the botulism spores.

Free NCLEX-RN Sample Test Questions For Nursing Review (Part 4) Jul31, Question1

The clinic nurse is counseling a substance-abusing post partum client on the risks of continued cocaine use. In order to provide continuity of care, which nursing diagnosis is a priority? A) Social isolation B) Ineffective coping C) Altered parenting D) Sexual dysfunction Review Information: The correct answer is C: Altered parenting The cocaine abusing mother puts her newborn and other children at risk for neglect and abuse. Continuing to use drugs has the potential to impact parenting behaviors. Social service referrals are indicated.

Question2

The nurse is teaching about nonsteroidal antiinflammatory drugs (NSAIDs) to a group of arthritic clients. To minimize the side effects, the nurse should emphasize which of the following actions? A) Reporting joint stiffness in the morning B) Taking the medication 1 hour before or 2 hours after meals C) Using alcohol in moderation unless driving D) Continuing to take aspirin for short term relief Review Information: The correct answer is B: Taking the medication 1 hour before or 2 hours after meals Taking the medication 1 hour before or 2 hours after meals will result in a more rapid effect.

Question3

The nurse is preparing to administer a tube feeding to a postoperative client. To accurately assess for a gastrostomy tube placement, the priority is to A) auscultate the abdomen while instilling 10 cc of air into the tube B) place the end of the tube in water to check for air bubbles C) retract the tube several inches to check for resistance D) measure the length of tubing from nose to epiCollected by :DeepaRajesh [ 61 ] [email protected] Kuwait

gastrium

known to cross the placenta and is therefore reported to be teratogenic.

Review Information: The correct answer is A: auscultate the abdomen while instilling 10 cc of air into the tube If a swoosh of air is heard over the abdominal cavity while instilling air into the gastric tube, this indicates that it is accurately placed in the stomach. The feeding can begin after further assessing the client for bowel sounds.

Question7

Question4

While assessing the vital signs in children, the nurse should know that the apical heart rate is preferred until the radial pulse can be accurately assessed at about what age? A) 1 year of age B) 2 years of age C) 3 years of age D) 4 years of age Review Information: The correct answer is B: 2 years of age A child should be at least 2 years of age to use the radial pulse to assess heart rate.

Question5

A client is receiving Total Parenteral Nutrition (TPN) via a Hickman catheter. The catheter accidentally becomes dislodged from the site. Which action by the nurse should take priority? A) Check that the catheter tip is intact B) Apply a pressure dressing to the site C) Monitor respiratory status D) Assess for mental status changes

Question6

A pregnant client who is at 34 weeks gestation is diagnosed with a pulmonary embolism (PE). Which of these medications would the nurse anticipate the provider ordering? A) Oral Coumadin therapy B) Heparin 5000 units subcutaneously B.I.D. C) Heparin infusion to maintain the PTT at 1.52.5 times the control value D) Heparin by subcutaneous injection to maintain the PTT at 1.5 times the control value

The nurse is caring for a client with Hodgkin>s disease who will be receiving radiation therapy. The nurse recognizes that, as a result of the radiation therapy, the client is most likely to experience A) high fever B) nausea C) face and neck edema D) night sweats Review Information: The correct answer is B: nausea Because the client with Hodgkin>>s disease is usually healthy when therapy begins, the nausea is especially troubling.

Question8

A client is brought to the emergency room following a motor vehicle accident. When assessing the client one-half hour after admission, the nurse notes several physical changes. Which finding would require the nurse>s immediate attention? A) increased restlessness B) tachycardia C) tracheal deviation D) tachypnea Review Information: The correct answer is C: tracheal deviation The deviated trachea is a sign that a mediastinal shift has occurred. This is a medical emergency.

Question9

An 18 month-old child is on peritoneal dialysis in preparation for a renal transplant in the near future. When the nurse obtains the child>s health history, the mother indicates that the child has not had the first measles, mumps, rubella (MMR) immunization. The nurse understands that which of the following is true in regards to giving immunizations to this child? A) Live vaccines are withheld in children with renal chronic illness Review Information: The correct answer is D: B) The MMR vaccine should be given now, prior Heparin by subcutaneous injection to maintain to the transplant C) An inactivated form of the vaccine can be givthe PTT at 1.5 times the control value Several studies have been conducted in preg- en at any time nant women where oral anticoagulation agents D) The risk of vaccine side effects precludes givare contraindicated. Warfarin (Coumadin) is ing the vaccine Collected by :DeepaRajesh [ 62 ] [email protected] Kuwait

Review Information: The correct answer is B: The MMR vaccine should be given now, prior to the transplant MMR is a live virus vaccine, and should be given at this time. Post-transplant, immunosuppressive drugs will be given and the administration of the live vaccine at that time would be contraindicated because of the compromised immune system.

Question10

are to receive electroconvulsive therapy? A) Benzodiazepines B) Chlorpromazine (Thorazine) C) Succinylcholine (Anectine) D) Thiopental sodium (Pentothal Sodium) Review Information: The correct answer is C: Succinylcholine (Anectine) Succinylcholine is given intravenously to promote skeletal muscle relaxation.

The nurse is preparing to take a toddler>s blood pressure for the first time. Which of the following actions should the nurse perform first? A) Explain that the procedure will help him to get well B) Show a cartoon character with a blood pressure cuff C) Explain that the blood pressure checks the heart pump D) Permit handling the equipment before putting the cuff in place

Question13

Review Information: The correct answer is D: Permit handling the equipment before putting the cuff in place The best way to gain the toddler>>s cooperation is to encourage handling the equipment. Detailed explanations are not helpful.

Review Information: The correct answer is D: Recognize personal attitudes about cultural differences and real or expected biases The nurse must discover personal attitudes, prejudices and biases specific to different cultures. Awareness of these will prevent negative consequences for interactions with clients and families across cultures.

Question11

Which statement made by a nurse about the goal of total quality management or continuous quality improvement in a health care setting is correct? A) It is to observe reactive service and product problem solving B) Improvement of the processes in a proactive, preventive mode is paramount C) A chart audits to finds common errors in practice and outcomes associated with goals D) A flow chart to organize daily tasks is critical to the initial stages Review Information: The correct answer is B: Improvement of the processes in a proactive, preventive mode is paramount Total quality management and continuous quality improvement have a major goal of identifying ways to do the right thing at the right time in the right way by proactive problem-solving.

Question12

Which of the following drugs should the nurse anticipate administering to a client before they

Which approach is a priority for the nurse who works with clients from many different cultures? A) Speak at least 2 other languages of clients in the neighborhood B) Learn about the cultures of clients who are most often encountered C) Have a list of persons for referral when interaction with these clients occur D) Recognize personal attitudes about cultural differences and real or expected biases

Question14

A client with chronic obstructive pulmonary disease (COPD) and a history of coronary artery disease is receiving aminophylline, 25mg/hour. Which one of the following findings by the nurse would require immediate intervention? A) Decreased blood pressure and respirations B) Flushing and headache C) Restlessness and palpitations D) Increased heart rate and blood pressure Review Information: The correct answer is C: Restlessness and palpitations Side effects of Aminophylline include restlessness and palpitations.

Question15

A client has gastroesophageal reflux. Which recommendation made by the nurse would be most helpful to the client? A) Avoid liquids unless a thickening agent is used B) Sit upright for at least 1 hour after eating C) Maintain a diet of soft foods and cooked vegCollected by :DeepaRajesh [ 63 ] [email protected] Kuwait

etables D) Avoid eating 2 hours before going to sleep

descent is diagnosed with ovarian cancer. The client states, “I refuse both radiation and chemotherapy because they are ” The next acReview Information: The correct answer is D: tion for the nurse to take is to Avoid eating 2 hours before going to sleep A) document the situation in the notes Eating before sleeping enhances the regurgita- B) report the situation to the health care providtion of stomach contents, which have increased er acidity, into the esophagus. An upright posture C) talk with the client>s family about the situashould be maintained for about 2 hours after eat- tion ing to allow for the stomach emptying. Options A D) ask the client to talk about concerns regarding and C are interventions for clients with swallow- «hot» treatments ing difficulties. Review Information: The correct answer is D: Question16 A client with a panic disorder has a new prescrip- ask the client to talk about concerns regarding tion for Xanax (alprazolam). In teaching the client «hot» treatments about the drug>s actions and side effects, which The «hot-cold» system is found among MexicanAmericans, Puerto Ricans, and other Hispanicof the following should the nurse emphasize? Latinos. Most foods, beverages, herbs, and A) Short-term relief can be expected medicines are categorized as hot or cold, which B) The medication acts as a stimulant are symbolic designations and do not necessarC) Dosage will be increased as tolerated ily indicate temperature or spiciness. Care and D) Initial side effects often continue treatment regimens can be negotiated with cliReview Information: The correct answer is A: ents within this framework. Short-term relief can be expected Question19 Xanax is a short-acting benzodiazepine useful in A 72 year-old client is scheduled to have a cardicontrolling panic symptoms quickly. oversion. A nurse reviews the client’s medication administration record. The nurse should notify the Question17 A client being discharged from the cardiac step- health care provider if the client received which down unit following a myocardial infarction (MI), medication during the preceding 24 hours? is given a prescription for a beta-blocking drug. A A) Digoxin (Lanoxin) nursing student asks the charge nurse why this B) Diltiazem (Cardizem) drug would be used by a client who is not hyper- C) Nitroglycerine ointment tensive. What is an appropriate response by the D) Metoprolol (Toprol XL) charge nurse? A) «Most people develop hypertension following Review Information: The correct answer is A: Digoxin (Lanoxin) an MI.» B) «A beta-Blocker will prevent orthostatic hypo- Digoxin increases ventricular irritability and increases the risk of ventricular fibrillation followtension.» C) «This drug will decrease the workload on his ing cardioversion. The other medications do not increase ventricular irritability. heart.» D) «Beta-blockers increase the strength of heart Question20 contractions.» Which of these clients, all of whom have the findings of a board-like abdomen, would the nurse Review Information: The correct answer is suggest that the provider examine first? C: «This drug will decrease the workload on his A) An elderly client who stated, «My awful pain heart.» in my right side suddenly stopped about 3 hours One action of beta-blockers is to decrease sys- ago.» temic vascular resistance by dilating arterioles. B) A pregnant woman of 8 weeks newly diagThis is useful for the client with coronary artery nosed with an ectopic pregnancy disease, and will reduce the risk of another MI or C) A middle-aged client admitted with diverticusudden death. litis who has taken only clear liquids for the past week Question18 A 35-year-old client of Puerto Rican-American D) A teenager with a history of falling off a bicycle Collected by :DeepaRajesh [ 64 ] [email protected] Kuwait

without hitting the handle bars

The nurse is assessing a comatose client receiving gastric tube feedings. Which of the following Review Information: The correct answer is A: assessments requires an immediate response An elderly client who stated, «My awful pain in from the nurse? my right side suddenly stopped about 3 hours A) Decreased breath sounds in right lower lobe ago.» B) Aspiration of a residual of 100cc of formula This client has the highest risk for hypovolemic C) Decrease in bowel sounds and septic shock since the appendix has most D) Urine output of 250 cc in past 8 hours likely ruptured, based on the history of the pain suddenly stopping over three hours ago. Elderly Review Information: The correct answer is A: clients have less functional reserve for the body Decreased breath sounds in right lower lobe to cope with shock and infection over long peri- The most common problem associated with enods. The others are at risk for shock also, how- teral feedings is atelectasis. Maintain client at 30 ever given that they fall in younger age groups, degrees of head elevation during feedings and they would more likely be able to tolerate an im- monitor for signs of aspiration. Check for tube balance in circulation. A common complication of placement prior to each feeding or every 4 to 8 falling off a bicycle is hitting the handle bars in hours if the client is receiving continuous feedthe upper abdomen often on the left, resulting in ing. a ruptured spleen. Question24 Question21 A client is prescribed warfarin sodium (CoumaThe nurse is teaching parents of a 7 month-old din) to be continued at home. Which focus is about adding table foods. Which of the following critical to be included in the nurse’s discharge is an appropriate finger food? instruction? A) Hot dog pieces A) Maintain a consistent intake of green leafy B) Sliced bananas foods C) Whole grapes B) Report any nose or gum bleeds D) Popcorn C) Take Tylenol for minor pains D) Use a soft toothbrush Review Information: The correct answer is B: Sliced bananas Review Information: The correct answer is B: Finger foods should be bite-size pieces of soft Report any nose or gum bleeds food such as bananas. Hot dogs and grapes can The client should notify the health care provider accidentally be swallowed whole and can occlude if blood is noted in stools or urine, or any other the airway. Popcorn is too difficult to chew at this signs of bleeding occur. age and can irritate the airway if swallowed.

Question22

To prevent drug resistance from developing, the nurse is aware that which of the following is a characteristic of the typical treatment plan to eliminate the tuberculosis bacilli? A) An anti-inflammatory agent B) High doses of B complex vitamins C) Aminoglycoside antibiotics D) Administering two anti-tuberculosis drugs Review Information: The correct answer is D: Administering two anti-tuberculosis drugs Resistance of the tubercle bacilli often occurs to a single antimicrobial agent. Therefore, therapy with multiple drugs over a long period of time helps to ensure eradication of the organism.

Question23

Question25

When teaching a client about the side effects of fluoxetine (Prozac), which of the following will the nurse include? A) Tachycardia blurred vision, hypotension, anorexia B) Orthostatic hypotension, vertigo, reactions to tyramine-rich foods C) Diarrhea, dry mouth, weight loss, reduced libido D) Photosensitivity, seizures, edema, hyperglycemia Review Information: The correct answer is C: Diarrhea, dry mouth, weight loss, reduced libido Commonly reported side effects for fluoxetine (Prozac) are diarrhea, dry mouth, weight loss and reduced libido. Collected by :DeepaRajesh [ 65 ] [email protected] Kuwait

Question26

A newborn weighed 7 pounds 2 ounces at birth. The nurse assesses the newborn at home 2 days later and finds the weight to be 6 pounds 7 ounces. What should the nurse tell the parents about this weight loss? A) The newborn needs additional assessments B) The mother should breast feed more often C) A change to formula is indicated D) The loss is within normal limits Review Information: The correct answer is D: The loss is within normal limits A newborn is expected to lose 5-10% of the birth weight in the first few days post-partum because of changes in elimination and feeding.

Question27

The nurse manager informs the nursing staff at morning report that the clinical nurse specialist will be conducting a research study on staff attitudes toward client care. All staff are invited to participate in the study if they wish. This affirms the ethical principle of A) Anonymity B) Beneficence C) Justice D) Autonomy Review Information: The correct answer is D: Autonomy Individuals must be free to make independent decisions about participation in research without coercion from others.

Question28

The nurse is talking with the family of an 18 months-old newly diagnosed with retinoblastoma. A priority in communicating with the parents is A) Discuss the need for genetic counseling B) Inform them that combined therapy is seldom effective C) Prepare for the child>s permanent disfigurement D) Suggest that total blindness may follow surgery

Question29

The nurse is planning care for an 8 year-old child. Which of the following should be included in the plan of care? A) Encourage child to engage in activities in the playroom B) Promote independence in activities of daily living C) Talk with the child and allow him to express his opinions D) Provide frequent reassurance and cuddling Review Information: The correct answer is A: Encourage child to engage in activities in the playroom According to Erikson, the school age child is in the stage of industry versus inferiority. To help them achieve industry, the nurse should encourage them to carry out tasks and activities in their room or in the playroom.

Question30

The nurse is assigned to care for 4 clients. Which of the following should be assessed immediately after hearing the report? A) The client with asthma who is now ready for discharge B) The client with a peptic ulcer who has been vomiting all night C) The client with chronic renal failure returning from dialysis D) The client with pancreatitis who was admitted yesterday Review Information: The correct answer is B: The client with a peptic ulcer who has been vomiting all night A perforated peptic ulcer could cause nausea, vomiting and abdominal distention, and may be a life threatening situation. The client should be assessed immediately and findings reported to the provider.

Question31

During a routine check-up, an insulin-dependent diabetic has his glycosylated hemoglobin checked. The results indicate a level of 11%. Based on this result, what teaching should the nurse emphasize? Review Information: The correct answer is A: A) Rotation of injection sties B) Insulin mixing and preparation Discuss the need for genetic counseling The hereditary aspects of this disease are well C) Daily blood sugar monitoring documented. While the parents focus on the D) Regular high protein diet needs of this child, they should be aware that Review Information: The correct answer is C: the risk is high for future offspring. Collected by :DeepaRajesh [ 66 ] [email protected] Kuwait

Daily blood sugar monitoring Normal hemoglobin A1C (glycosylated hemoglobin) level is 7 to 9%. Elevation indicates elevated glucose levels over time.

Question32

A client taking isoniazid (INH) for tuberculosis asks the nurse about side effects of the medication. The client should be instructed to immediately report which of these? A) Double vision and visual halos B) Extremity tingling and numbness C) Confusion and lightheadedness D) Sensitivity of sunlight Review Information: The correct answer is B: Extremity tingling and numbness Peripheral neuropathy is the most common side effect of INH and should be reported to the provider. It can be reversed.

gency room (ER). The client, diagnosed with a myocardial infarction, is complaining of substernal chest pain, diaphoresis and nausea. The first action by the nurse should be to A) order an EKG B) administer morphine sulfate C) start an IV D) measure vital signs Review Information: The correct answer is B: administer morphine sulfate Decreasing the clients pain is the most important priority at this time. As long as pain is present there is danger in extending the infarcted area. Morphine will decrease the oxygen demands of the heart and act as a mild diuretic as well. It is probable that an EKG and IV insertion were performed in the ER.

Question36

The nurse admits a 2 year-old child who has had Which of these questions is priority when as- a seizure. Which of the following statement by the child>s parent would be important in detersessing a client with hypertension? A) «What over-the-counter medications do you mining the etiology of the seizure? A) «He has been taking long naps for a week.» take?» B) «Describe your usual exercise and activity B) «He has had an ear infection for the past 2 days.» patterns.» C) «He has been eating more red meat lately.» C) «Tell me about your usual diet.» D) «Describe your family>s cardiovascular his- D) «He seems to be going to the bathroom more frequently.» tory.» Review Information: The correct answer is B: «He has had an ear infection for the past 2 Review Information: The correct answer is days.» A: «What over-the-counter medications do you Contributing factors to seizures in children include those such as age (more common in first take?» Over-the-counter medications, especially those 2 years), infections (late infancy and early childthat contain cold preparations can increase the hood), fatigue, not eating properly and excessive fluid intake or fluid retention. blood pressure to the point of hypertension.

Question33

Question34

The nurse is performing an assessment of the motor function in a client with a head injury. The best technique is A) touching the trapezius muscle or arm firmly B) pinching any body part C) shaking a limb vigorously D) rubbing the sternum Review Information: The correct answer is D: rubbing the sternum The purpose is to assess the non-responsive client’s reaction to a painful stimulus after less noxious methods have been tried.

Question37

Which of these clients would the nurse monitor for the complication of C. difficile diarrhea? A) An adolescent taking medications for acne B) An elderly client living in a retirement center taking prednisone C) A young adult at home taking a prescribed aminoglycoside D) A hospitalized middle aged client receiving clindamycin

Review Information: The correct answer is D: A hospitalized middle aged client receiving clindamycin Hospitalized patients, especially those receiving Question35 A nurse admits a client transferred from the emer- antibiotic therapy, are primary targets for C. diffiCollected by :DeepaRajesh [ 67 ] [email protected] Kuwait

cile. Of clients receiving antibiotics, 5-38% experience antibiotic-associated diarrhea; C. difficile causes 15 to 20% of the cases. Several antibiotic agents have been associated with C. difficile. Broad-spectrum agents, such as clindamycin, ampicillin, amoxicillin, and cephalosporins, are the most frequent sources of C. difficile. Also, C. difficile infection has been caused by the administration of agents containing beta-lactamase inhibitors (i.e., clavulanic acid, sulbactam, tazobactam) and intravenous agents that achieve substantial colonic intraluminal concentrations (i.e., ceftriaxone, nafcillin, oxacillin). Fluoroquinolones, aminoglycosides, vancomycin, and trimethoprim are seldom associated with C. difficile infection or pseudomembranous colitis.

Pain is a complex phenomenon that is perceived differently by each individual. Pain is whatever the client says it is. The other statements are correct but not the most important considerations.

Question40

As a part of a 9 pound full-term newborn>s assessment, the nurse performs a dextro-stick at 1 hour post birth. The serum glucose reading is 45 mg/dl. What action by the nurse is appropriate at this time? A) Give oral glucose water B) Notify the pediatrician C) Repeat the test in 2 hours D) Check the pulse oximetry reading

Review Information: The correct answer is C: Repeat the test in 2 hours Question38 The nurse is performing an assessment on a cli- This blood sugar is within the normal range for a ent who is cachectic and has developed an en- full-term newborn. Normal values are: Premature terocutaneous fistula following surgery to relieve infant: 20-60 mg/dl or 1.1-3.3 mmol/L, Neonate: a small bowel obstruction. The client>s total pro- 30-60 mg/dl or 1.7-3.3 mmol/L, Infant: 40-90 mg/ tein level is reported as 4.5 g/dl. Which of the dl or 2.2-5.0 mmol/L. Critical values are: Infant: <40 mg/dl and in a Newborn: <30 and >300 mg/ following would the nurse anticipate? dl. Because of the increased birth weight which A) Additional potassium will be given IV can be associated with diabetes mellitus, repeatB) Blood for coagulation studies will be drawn C) Total parenteral nutrition (TPN) will be start- ed blood sugars will be drawn ed D) Serum lipase levels will be evaluated Review Information: The correct answer is C: Total parenteral nutrition (TPN) will be started The client is not absorbing nutrients adequately as evidenced by the cachexia and low protein levels. (A normal total serum protein level is 6.08.0 g/dl.) TPN will promote a positive nitrogen balance in this client who is unable to digest and absorb nutrients adequately.

Question39

During a situation of pain management, which statement is a priority to consider for the ethical guidelines of the nurse? A) The client>s self-report is the most important consideration B) Cultural sensitivity is fundamental to pain management C) Clients have the right to have their pain relieved D) Nurses should not prejudge a client>s pain using their own values

Free NCLEX-RN Sample Test Questions For Nursing Review (Part 3) Jul31,

Question1

A client diagnosed with chronic depression is maintained on tranylcypromine (Parnate). An important nursing intervention is to teach the client to avoid which of the following foods? A) Wine, beer, cheese, liver and chocolate B) Wine, citrus fruits, yogurt and broccoli C) Beer, cheese, beef and carrots D) Wine, apples, sour cream and beef steak Review Information: The correct answer is A: Wine, beer, cheese, liver and chocolate These foods are tyramine rich and ingestion of these foods while taking monoamine oxidase inhibitors (MAOIs) can precipitate a life-threatening hypertensive crisis.

Question2

The nurse is working in a high risk antepartum clinic. A 40 year-old woman in the first trimesReview Information: The correct answer is A: ter gives a thorough health history. Which inforThe client>>s self-report is the most important mation should receive priority attention by the consideration nurse? Collected by :DeepaRajesh [ 68 ] [email protected] Kuwait

A) Her father and brother are insulin dependent diabetics B) She has taken 800 mcg of folic acid daily for the past year C) Her husband was treated for tuberculosis as a child D) She reports recent use of over-the counter sinus remedies Review Information: The correct answer is D: She reports recent use of over-the counter sinus remedies Over-the-counter drugs are a possible danger in early pregnancy. A report by the client that she has taken medications should be followed up immediately.

Question3

What must be the priority consideration for nurses when communicating with children? A) Present environment B) Physical condition C) Nonverbal cues D) Developmental level Review Information: The correct answer is D: Developmental level While each of the factors affect communication, the nurse recognizes that developmental differences have implications for processing and understanding information. Consequently, a child’s developmental level must be considered when selecting communication approaches.

Question4

The nurse is assessing a client>s home in preparation for discharge. Which of the following should be given priority consideration? A) Family understanding of client needs B) Financial status C) Location of bathrooms D) Proximity to emergency services

B) The blood alcohol level of the client C) The blood pressure level of the client D) The blood glucose level of the client Review Information: The correct answer is B: The blood alcohol level of the client Blood alcohol levels are generally obtained to determine the level of intoxication. The amount of alcohol consumed determines how much medication the client needs for detoxification and treatment. Reports of alcohol consumption are notoriously inaccurate.

Question6

Which clinical finding would the nurse expect to assess first in a newborn with spastic cerebral palsy? A) cognitive impairment B) hypotonic muscular activity C) seizures D) criss-crossing leg movement Review Information: The correct answer is D: criss-crossing leg movement Cerebral palsy is a neuromuscular impairment resulting in muscular and reflexive hypertonicity and the criss-crossing, or scissoring leg movements.

Question7

Which medication is more helpful in treating bulimia than anorexia? A) Amphetamines B) Sedatives C) Anticholinergics D) Narcotics

Review Information: The correct answer is C: Anticholinergics In contrast to anorexics, individuals with bulimia are troubled by their behavioral characteristics and become depressed. The person feels compelled to binge, purge and fast. Feeling helpless Review Information: The correct answer is A: to stop the behavior, feelings of self-disgust ocFamily understanding of client needs cur. Functional communication patterns between Question8 family members are fundamental to meeting the The nurse is assessing a woman in early labor. needs of the client and family. While positioning for a vaginal exam, she comQuestion5 plains of dizziness and nausea and appears As a general guide for emergency management pale. Her blood pressure has dropped slightly. of acute alcohol intoxication, it is important for What should be the initial nursing action? the nurse initially to obtain data regarding which A) Call the health care provider of the following? B) Encourage deep breathing A) What and how much the client drinks, accord- C) Elevate the foot of the bed ing to family and friends D) Turn her to her left side Collected by :DeepaRajesh [ 69 ] [email protected] Kuwait

Review Information: The correct answer is D: Turn her to her left side The weight of the uterus can put pressure on the vena cava and aorta when a pregnant woman is flat on her back causing supine hypotension. Action is needed to relieve the pressure on the vena cava and aorta. Turning the woman to the side reduces this pressure and relieves postural hypotension.

Question9

A client has been started on a long term corticosteroid therapy. Which of the following comments by the client indicate the need for further teaching? A) «I will keep a weekly weight record.» B) «I will take medication with food.» C) «I will stop taking the medication for 1 week every month.» D) «I will eat foods high in potassium.» Review Information: The correct answer is C: «I will stop taking the medication for 1week every month.» Emphatically warn against discontinuing steroid dosage abruptly because that may produce a fatal adrenal crisis.

Question10

A male client calls for a nurse because of chest pain. Which statement by the client would require the most immediate action by the nurse? A) «When I take in a deep breath, it stabs like a knife.» B) «The pain came on after dinner. That soup seemed very spicy.» C) «When I turn in bed to reach the remote for the TV, my chest hurts.» D) «I feel pressure in the middle of my chest, like an elephant is sitting on my chest.» Review Information: The correct answer is D: «I feel pressure in the middle of my chest, like an elephant is sitting on my chest.» This is a classic description of chest pain in men caused by myocardial ischemia. Women experience vague feelings of fatigue and back and jaw pain.

Question11

A nurse is caring for a client who has just been admitted with an overdose of aspirin. The following lab data is available: PaO2 95, PaCO2 30, pH 7.5, K 3.2 mEq/l. Which should be the nurse>s first action?

A) Monitor respiratory rate B) Monitor intake and output every hour C) Assist the client to breathe into a paper bag D) Prepare to administer oxygen by mask Review Information: The correct answer is C: Assist the client to breathe into a paper bag Side effects of aspirin toxicity include hyperventilation, which can result in respiratory alkalosis in the initial stages. Breathing into a paper bag will prevent further reduction in PaCO2.

Question12

After assessing a 70 year-old male client>s laboratory results during a routine clinic visit, which one of the following findings would indicate an area in which teaching is needed: A) Serum albumin 2.5 g/dl B) LDL Cholesterol 140 mg/dl C) Serum glucose 90 mg/dl D) RBC 5.0 million/mm3 Review Information: The correct answer is A: Serum albumin 2.5 g/dl Serum albumin level is low (normal 3.0 – 5.0 g/dl in elders), indicating nutritional counseling to increase dietary protein is needed. Socioeconomic factors may need to be addressed to help the client comply with the recommendation.

Question13

When teaching a client with a new prescription for lithium (Lithane) for treatment of a bi-polar disorder which of these should the nurse emphasize? A) Maintaining a salt restricted diet B) Reporting vomiting or diarrhea C) Taking other medication as usual D) Substituting generic form if desired Review Information: The correct answer is B: Reporting vomiting or diarrhea If dehydration results from vomiting, diarrhea or excessive perspiration, tolerance to the drug may be altered and symptoms may return.

Question14

A client is discharged on warfarin sulfate (Coumadin). Which statement by the client indicated a need for further teaching? A) «I know I must avoid crowds.» B) «I will keep all laboratory appointments.» Collected by :DeepaRajesh [ 70 ] [email protected] Kuwait

C) «I plan to use an electric razor for shaving.» D) «I will report any bruises for bleeding.» Review Information: The correct answer is A: «I know I must avoid crowds.» There are no specific reasons for the client on Coumadin to avoid crowds. General instructions for any cardiac surgical client include limiting exposure to infection.

Question15

A client is taking tranylcypromine (Parnate) and has received dietary instruction. Which of the following food selections would be contraindicated for this client? A) Fresh juice, carrots, vanilla pudding B) Apple juice, ham salad, fresh pineapple C) Hamburger, fries, strawberry shake D) Red wine, fava beans, aged cheese Review Information: The correct answer is D: Red wine, fava beans, aged cheese Red wine and cheese contain tyramine (as do chicken liver and ripe bananas) and so are contraindicated when taking MAOIs. Fava beans contain other vasopressors that can interact with MAOIs also causing malignant hypertension.

Question16

A client is admitted with severe injuries from an auto accident. The client>s vital signs are BP 120/50, pulse rate 110, and respiratory rate of 28. The initial nursing intervention would be to A) begin intravenous therapy B) initiate continuous blood pressure monitoring C) administer oxygen therapy D) institute cardiac monitoring Review Information: The correct answer is C: administer oxygen therapy Early findings of shock reveal hypoxia with rapid heart rate and rapid respirations, and oxygen is the most critical initial intervention. The other interventions are secondary to oxygen therapy.

B) heart rate C) peripheral pulses D) lung sounds Review Information: The correct answer is D: lung sounds Lung sounds are critical assessments at this point. The nurse should be alert to crackles or a pleural friction rub, highly suggestive of a pulmonary embolism.

Question18

The nurse is administering lidocaine (Xylocaine) to a client with a myocardial infarction. Which of the following assessment findings requires the nurse>s immediate action? A) Central venous pressure reading of 11 B) Respiratory rate of 22 C) Pulse rate of 48 BPM D) Blood pressure of 144/92 Review Information: The correct answer is C: Pulse rate of 48 BPM One of the side effects of lidocaine is bradycardia, heart block, cardiovascular collapse and cardiac arrest (this drug should never be administered without continuous EKG monitoring).

Question19

The nurse is teaching a group of college students about breast self-examination. A woman asks for the best time to perform the monthly exam. What is the best reply by the nurse? A) «The first of every month, because it is easiest to remember» B) «Right after the period, when your breasts are less tender» C) «Do the exam at the same time every month» D) «Ovulation, or mid-cycle is the best time to detect changes»

Review Information: The correct answer is B: «Right after the period, when your breasts are Question17 A client is admitted to the hospital with a diag- less tender» nosis of deep vein thrombosis. During the ini- The best time for a breast self exam (BSE) is a tial assessment, the client complains of sudden week after a menstrual cycle, when the breasts shortness of breath. The SaO2 is 87. The priority are no longer swollen and tender due to hormone elevation. nursing assessment at this time is A) bowel sounds Collected by :DeepaRajesh [ 71 ] [email protected] Kuwait

Question20

T.I.D. to treat bipolar disorder. Which of these inThe nurse is caring for a post-operative client dicate early signs of toxicity? who develops a wound evisceration. The first A) Ataxia and course hand tremors B) Vomiting, diarrhea and lethargy nursing intervention should be to C) Pruritus, rash and photosensitivity A) medicate the client for pain D) Electrolyte imbalance and cardiac arrhythB) call the provider mias C) cover the wound with sterile saline dressing D) place the bed in a flat position Review Information: The correct answer is B: Vomiting, diarrhea and lethargy Review Information: The correct answer is C: These are early signs of lithium toxicity. cover the wound with sterile saline dressing When evisceration occurs, the wound should first Question24 be quickly covered by sterile dressings soaked in The nurse can best ensure the safety of a client sterile saline. This prevents tissue damage until suffering from dementia who wanders from the a repair can be effected. room by which action? A) Repeatedly remind the client of the time and location Question21 The spouse of a client with Alzheimer>s disease B) Explain the risks of walking with no purpose expresses concern about the burden of caregiv- C) Use protective devices to keep the client in ing. Which of the following actions by the nurse the bed or chair in the room D) Attach a wander-guard sensor band to the should be a priority? client>s wrist A) Link the caregiver with a support group B) Ask friends to visit regularly C) Schedule a home visit each week Review Information: The correct answer is D) Request anti-anxiety prescriptions D: Attach a wander-guard sensor band to the Review Information: The correct answer is A: client>>s wrist This type of identification band easily tracks the Link the caregiver with a support group Assisting caregivers to locate and join support client>>s movements and ensures safety while groups is most helpful. Families share feelings the client wanders on the unit. Restriction of acand learn about services such as respite care. tivity is inappropriate for any client unless they Health education is also available through local are potentially harmful to themselves or others. and national Alzheimer>>s chapters.

Question22

Clients taking lithium must be particularly sure to maintain adequate intake of which of these elements? A) Potassium B) Sodium C) Chloride D) Calcium Review Information: The correct answer is B: Sodium Clients taking lithium need to maintain an adequate intake of sodium. Serum lithium concentrations may increase in the presence of conditions that cause sodium loss.

Question23

A client is receiving lithium carbonate 600 mg

Question25

The nurse is teaching a client about the difference between tardive dyskinesia (TD) and neuroleptic malignant syndrome (NMS). Which statement is true with regards to tardive dyskinesia? A) TD develops within hours or years of continued antipsychotic drug use in people under 20 and over 30 B) It can occur in clients taking antipsychotic drugs longer than 2 years C) Tardive dyskinesia occurs within minutes of the first dose of antipsychotic drugs and is reversible D) TD can easily be treated with anticholinergic drugs Review Information: The correct answer is B: It can occur in clients taking antipsychotic drugs Collected by :DeepaRajesh [ 72 ] [email protected] Kuwait

Review Information: The correct answer is A: Provide small feedings every 3 hours Infants with congenital heart defects are at increased risk for developing congestive heart failure. Infants with congestive heart failure have an increased metabolic rate and require additional calories to grow. At the same time, however, rest and conservation of energy for eating is imporQuestion26 The nurse is aware that the effect of antihyper- tant. Feedings should be smaller and every 3 tensive drug therapy may be affected by a 75 hours rather than the usual 4 hour schedule. year-old client>s A) poor nutritional status Question29 B) decreased gastrointestinal motility The nurse is caring for a client receiving intraveC) increased splanchnic blood flow nous nitroglycerin for acute angina. What is the D) altered peripheral resistance most important assessment during treatment? A) Heart rate Review Information: The correct answer is B: B) Neurologic status decreased gastrointestinal motility C) Urine output Together with shrinkage of the gastric mucosa, D) Blood pressure and changes in the levels of hydrochloric acid, this will decrease absorption of medications and Review Information: The correct answer is D: interfere with their actions. Blood pressure The vasodilatation that occurs as a result of this medication can cause profound hypotension. Question27 In response to a call for assistance by a client in The client>>s blood pressure must be evaluated labor, the nurse notes that a loop on the umbili- every 15 minutes until stable and then every 30 cal cord protrudes from the vagina. What is the minutes to every hour. priority nursing action? Question30 A) call the health care provider B) check fetal heart beat A client telephones the clinic to ask about a home C) put the client in knee-chest position pregnancy test she used this morning. The nurse D) turn the client to the side understands that the presence of which hormone strongly suggests a woman is pregnant? Review Information: The correct answer is C: A) Estrogen put the client in knee-chest position B) HCG Immediate action is needed to relieve pressure C) Alpha-fetoprotein on the cord, which puts the fetus at risk due to D) Progesterone hypoxia. The Trendelenburg position accomplishes this. The exposed cord is covered with Review Information: The correct answer is B: saline soaked gauze, not reinserted. The fe- HCG tal heart rate also should be checked, and the Human chorionic gonadotropin (HCG) is the provider called. A prolapsed umbilical cord is a biologic marker on which pregnancy tests are medical emergency. based. Reliability is about 98%, but the test does not conclusively confirm pregnancy. longer than 2 years Tardive dyskinesia is a extrapyramidal side effect that appears after prolonged treatment with antipsychotic medication. Early symptoms of tardive dyskinesia are fasciculations of the tongue or constant smacking of the lips.

Question28

The nurse is caring for a 2 month-old infant with a congenital heart defect. Which of the following is a priority nursing action? A) Provide small feedings every 3 hours B) Maintain intravenous fluids C) Add strained cereal to the diet D) Change to reduced calorie formula

Question31

A client, admitted to the unit because of severe depression and suicidal threats, is placed on suicidal precautions. The nurse should be aware that the danger of the client committing suicide is greatest A) during the night shift when staffing is limited B) when the client’s mood improves with an inCollected by :DeepaRajesh [ 73 ] [email protected] Kuwait

crease in energy level C) at the time of the client>s greatest despair D) after a visit from the client>s estranged partner

exchange.

Question34

The nurse is assessing a client with chronic obstructive pulmonary disease receiving oxygen for Review Information: The correct answer is low PaO2 levels. Which assessment is a nursing B: when the client’s mood improves with an in- priority? crease in energy level A) Evaluating SaO2 levels frequently Suicide potential is often increased when there B) Observing skin color changes is an improvement in mood and energy level. At C) Assessing for clubbing fingers this time ambivalence is often decreased and a D) Identifying tactile fremitus decision is made to commit suicide. Review Information: The correct answer is A: Question32 Evaluating SaO2 levels frequently After 4 electroconvulsive treatments over 2 The best method to evaluate a client>>s oxygenweeks, a client is very upset and states “I am so ation is to evaluate the SaO2. This is just as efconfused. I lose my money. I just can’t remem- fective as an arterial blood gas reading to evaluber telephone numbers.” The most therapeutic ate oxygenation status, and is less traumatic and expensive. response for the nurse to make is A) «You were seriously ill and needed the treatments.» Question35 B) «Don>t get upset. The confusion will clear up The visiting nurse makes a postpartum visit to in a day or two.» a married female client. Upon arrival, the nurse C) «It is to be expected since most clients have observes that the client has a black eye and nuthe same results.» merous bruises on her arms and legs. The initial D) «I can hear your concern and that your confu- nursing intervention would be to sion is upsetting to you.» A) call the police to report indications of domestic violence Review Information: The correct answer is D: B) confront the husband about abusing his wife «I can hear your concern and that your confusion C) leave the home because of the unsafe enviis upsetting to you.» ronment Communicating caring and empathy with the ac- D) interview the client alone to determine the oriknowledgement of feelings is the initial response. gin of the injuries Afterwards, teaching about the expected short term effects would be discussed. Review Information: The correct answer is D: Question33 interview the client alone to determine the origin A woman in labor calls the nurse to assist her in of the injuries the bathroom. The nurse notices a large amount It would be wrong to assume domestic violence of clear fluid on the bed linens. The nurse knows without further assessment. Separate the susthat fetal monitoring must now assess for what pected victim from the partner until battering has complication? been ruled out. A) Early decelerations B) Late accelerations C) Variable decelerations Question36 D) Periodic accelerations When teaching a client about an oral hypoglycemic medication, the nurse should place primary Review Information: The correct answer is C: emphasis on Variable decelerations A) recognizing findings of toxicity When the membranes rupture, there is increased B) taking the medication at specified times risk initially of cord prolapse. Fetal heart rate pat- C) increasing the dosage based on blood gluterns may show variable decelerations, which cose require immediate nursing action to promote gas D) distinguishing hypoglycemia from hyperglycCollected by :DeepaRajesh [ 74 ] [email protected] Kuwait

emia Review Information: The correct answer is B: taking the medication at specified times A regular interval between doses should be maintained since oral hypoglycemics stimulate the islets of Langerhans to produce insulin.

D) Assess the client>s knowledge about his health problems

Review Information: The correct answer is D: Assess the client>>s knowledge about his health problems The nursing process is continuous and cyclical in nature. When a client expresses a specific conQuestion37 Initial postoperative nursing care for an infant cern, the nurse performs a focused assessment who has had a pyloromyotomy would initially in- to gather additional data prior to planning and implementing nursing interventions. clude A) bland diet appropriate for age B) intravenous fluids for 3-4 days Question40 C) NPO then glucose and electrolyte solutions The client asks the nurse how the health care D) formula or breast milk as tolerated provider could tell she was pregnant “just by looking inside.” What is the best explanation by Review Information: The correct answer is C: the nurse? NPO then glucose and electrolyte solutions A) Bluish coloration of the cervix and vaginal Post-operatively, the initial feedings are clear liq- walls uids in small quantities to provide calories and B) Pronounced softening of the cervix electrolytes. C) Clot of very thick mucous that obstructs the cervical canal D) Slight rotation of the uterus to the right Question38 A client is treated in the emergency room for diabetic ketoacidosis and a glucose level of 650mg.D/L. In assessing the client, the nurse>s Review Information: The correct answer is A: review of which of the following tests suggests Bluish coloration of the cervix and vaginal walls Chadwick>>s sign is a bluish-purple coloration an understanding of this health problem? of the cervix and vaginal walls, occurring at 4 A) Serum calcium weeks of pregnancy, that is caused by vasoconB) Serum magnesium gestion. C) Serum creatinine D) Serum potassium Review Information: The correct answer is D: Serum potassium Potassium is lost in diabetic ketoacidosis during rehydration and insulin administration. Review of this lab finding suggests the nurse has knowledge of this problem.

Free NCLEX-RN Sample Test Questions For Nursing Review (Part 2) Jul31,

Question1

The feeling of trust can best be established by the nurse during the process of the development of a nurse-client relationship by which of these Question39 characteristics? A male client is preparing for discharge follow- A) Reliability and kindness ing an acute myocardial infarction. He asks the B) Demeanor and sincerity nurse about his sexual activity once he is home. C) Honesty and consistency What would be the nurse>s initial response? D) Sympathy and appreciativeness A) Give him written material from the American Heart Association about sexual activity with heart Review Information: The correct answer is C: disease Honesty and consistency B) Answer his questions accurately in a private Characteristics of a trusting relationship include environment respect, honesty, consistency, faith and caring. C) Schedule a private, uninterrupted teaching session with both the client and his wife Question2 Collected by :DeepaRajesh [ 75 ] [email protected] Kuwait

A nurse has administered several blood transfusions over 3 days to a 12 year-old client with Thalassemia. What lab value should the nurse monitor closely during this therapy? A) Hemoglobin B) Red Blood Cell Indices C) Platelet count D) Neutrophil percent

of clozapine (Clozaril) therapy? A) Dry mouth B) Rhinitis C) Dry skin D) Extreme salivation

Review Information: The correct answer is D: Extreme salivation A significant number of clients receiving ClozapReview Information: The correct answer is A: ine (Clozaril) therapy experience extreme salivaHemoglobin tion. Hemoglobin should be in a therapeutic range of approximately 10 g/dl (100gL). «This level is low Question6 enough to foster the patient>>s own erythropoi- A client has had a positive reaction to purified esis without enlarging the spleen.» protein derivative (PPD). The client asks the nurse what this means. The nurse should indiQuestion3 cate that the client has The nurse is providing care to a newly a hospital- A) active tuberculosis ized adolescent. What is the major threat experi- B) been exposed to mycobacterium tuberculoenced by the hospitalized adolescent? sis A) Pain management C) never had tuberculosis D) never been infected with mycobacterium tuB) Restricted physical activity C) Altered body image berculosis D) Separation from family Review Information: The correct answer is B: Review Information: The correct answer is C: been exposed to mycobacterium tuberculosis The PPD skin test is used to determine the presAltered body image The hospitalized adolescent may see each of ence of tuberculosis antibodies and a positive rethese as a threat, but the major threat that they sult indicates that the person has been exposed feel when hospitalized is the fear of altered body to mycobacterium tuberculosis. Additional tests image, because of the emphasis on physical ap- are needed to determine if active tuberculosis is present. pearance during this developmental stage.

Question4

A 12 year-old child is admitted with a broken arm and is told surgery is required. The nurse finds him crying and unwilling to talk. What is the most appropriate response by the nurse? A) Give him privacy B) Tell him he will get through the surgery with no problem C) Try to distract him D) Make arrangements for his friends to visit

Question7

A client is receiving and IV antibiotic infusion and is scheduled to have blood drawn at 1:00 pm for a «peak» antibiotic level measurement. The nurse notes that the IV infusion is running behind schedule and will not be competed by 1:00. The nurse should: A) Notify the client>s health care provider B) Stop the infusion at 1:00 pm C) Reschedule the laboratory test D) Increase the infusion rate

Review Information: The correct answer is A: Give him privacy A 12 year-old child needs the opportunity to ex- Review Information: The correct answer is C: press his emotions privately. Reschedule the laboratory test If the antibiotic infusion will not be completed at the time the peak blood level is due to be drawn, Question5 In discharge teaching, the nurse should empha- the nurse should ask that the blood sampling size that which of these is a common side effect time be adjusted Collected by :DeepaRajesh [ 76 ] [email protected] Kuwait

Question8

Review Information: The correct answer is C: A genetic predisposition Malignant hyperthermia is a rare, potentially fatal adverse reaction to inhaled anesthetics. There is a genetic predisposition to this disorder.

B) Questionthis medication dose

Question11

The nurse is caring for a client with a new order for bupropion (Wellbutrin) for treatment of depression. The order reads “Wellbutrin 175 mg. BID x 4 days.” What is the appropriate action? A) Give the medication as ordered

A 9 year-old is taken to the emergency room with right lower quadrant pain and vomiting. When preparing the child for an emergency appenReview Information: The correct an- dectomy, what must the nurse expect to be the child>s greatest fear? swer is B: Questionthis medication A) Change in body image dose B) An unfamiliar environment Bupropion (Wellbutrin) should be started at C) Perceived loss of control 100mg BID for three days then increased to D) Guilt over being hospitalized 150mg BID. When used for depression, it may take up to four weeks for results. Common side Review Information: The correct answer is C: effects are dry mouth, headache, and agitation. Perceived loss of control Doses should be administered in equally spaced For school age children, major fears are loss of time increments throughout the day to minimize control and separation from friends/peers. the risk of seizures. C) Observe the client for mood swings D) Monitor neuro signs frequently

Question9

The clinic nurse is discussing health promotion with a group of parents. A mother is concerned about Reye>s Syndrome, and asks about prevention. Which of these demonstrates appropriate teaching? A) «Immunize your child against this disease.» B) «Seek medical attention for serious injuries.» C) «Report exposure to this illness.» D) «Avoid use of aspirin for viral infections.» Review Information: The correct answer is D: «Avoid use of aspirin for viral infections.» The link between aspirin use and Reye>>s Syndrome has not been confirmed, but evidence suggests that the risk is sufficiently grave to include the warning on aspirin products.

Question10

A post-operative client is admitted to the post-anesthesia recovery room (PACU). The anesthetist reports that malignant hyperthermia occurred during surgery. The nurse recognizes that this complication is related to what factor? A) Allergy to general anesthesia B) Pre-existing bacterial infection C) A genetic predisposition D) Selected surgical procedures

Question12

A client is to begin taking Fosamax. The nurse must emphasize which of these instructions to the client when taking this medication? «Take Fosamax A) on an empty stomach.» B) after meals.» C) with calcium.» D) with milk 2 hours after meals.» Review Information: The correct answer is A: on an empty stomach.» Fosamax should be taken first thing in the morning with 6-8 ounces of plain water at least 30 minutes before other medication or food. Food and fluids (other than water) greatly decrease the absorption of Fosamax. The client must also be instructed to remain in the upright position for 30 minutes following the dose to facilitate passage into the stomach and minimize irritation of the esophagus.

Question13

An older adult client is to receive and antibiotic, gentamicin. What diagnostic finding indicates the client may have difficult excreting the medication? A) High gastric pH B) High serum creatinine Collected by :DeepaRajesh [ 77 ] [email protected] Kuwait

C) Low serum albumin D) Low serum blood urea nitrogen

D) Review the specific procedures unique to the assignment

Review Information: The correct answer is C: Review Information: The correct answer is B: Ask about prior experience with similar clients. High serum creatinine The first step in delegation is to determine the An elevated serum creatinine indicates reduced qualifications of the person to whom one is delrenal function. Reduced renal function will delay egating. By asking about the PN>>s prior exthe excretion of many mediations. perience with similar clients/tasks, the RN can determine whether the PN has the requisite experience to care for the assigned clients. Question14 A nurse is assigned to care for a comatose diabetic on IV insulin therapy. Which task would be Question17 most appropriate to delegate to an unlicensed The mother of a 4 month-old infant asks the assistive personnel (UAP)? nurse about the dangers of sunburn while they A) Check the client>s level of consciousness are on vacation at the beach. Which of the folB) Obtain the regular blood glucose readings lowing is the best advice about sun protection for C) Determine if special skin care is needed this child? D) Answer questions from the client>s spouse A) «Use a sunscreen with a minimum sun proabout the plan of care tective factor of 15.» B) «Applications of sunscreen should be repeatReview Information: The correct answer is B: ed every few hours.» Obtain the regular blood glucose readings C) «An infant should be protected by the maxiThe UAP can safely obtain blood glucose read- mum strength sunscreen.» ings, which are routine tasks. D) «Sunscreens are not recommended in children younger than 6 months.»

Question15

Which of the following laboratory results would suggest to the emergency room nurse that a client admitted after a severe motor vehicle crash is in acidosis? A) Hemoglobin 15 gm/dl B) Chloride 100 mEq/L C) Sodium 130 mEq/L D) Carbon dioxide 20 mEq/L Review Information: The correct answer is D: Carbon dioxide 20 mEq/L Serum carbon dioxide is an indicator of acid-base status. This finding would indicate acidosis.

Review Information: The correct answer is D: «Sunscreens are not recommended in children younger than 6 months.» Infants under 6 months of age should be kept out of the sun or shielded from it. Even on a cloudy day, the infant can be sunburned while near water. A hat and light protective clothing should be worn.

Question18

The nurse administers cimetidine (Tagamet) to a 79 year-old male with a gastric ulcer. Which parameter may be affected by this drug, and should be closely monitored by the nurse? Question16 The nurse has just received report on a group A) Blood pressure of clients and plans to delegate care of several B) Liver function of the clients to a practical nurse (PN). The first C) Mental status thing the RN should do before the delegation of D) Hemoglobin care is A) Provide a time-frame for the completion of the Review Information: The correct answer is C: Mental status client care B) Assure the PN that the RN will be available for The elderly are at risk for developing confusion when taking cimetidine, a drug that interacts with assistance C) Ask about prior experience with similar cli- many other medications. ents Collected by :DeepaRajesh [ 78 ] [email protected] Kuwait

Question19

Review Information: The correct answer is B: The nurse assesses the use of coping mecha- High risk for infection nisms by an adolescent 1 week after the client Cyclosporin (Neoral) inhibits normal immune rehad a motor vehicle accident resulting in multiple sponses. Clients receiving cyclosporin are at risk serious injuries. Which of these characteristics for infection. are most likely to be displayed? A) Ambivalence, dependence, demanding Question22 B) Denial, projection, regression A client on telemetry begins having premature C) Intellectualization, rationalization, repression ventricular beats (PVBs) at 12 per minute. In reD) Identification, assimilation, withdrawal viewing the most recent laboratory results, which would require immediate action by the nurse? Review Information: The correct answer is B: A) Calcium 9 mg/dl Denial, projection, regression B) Magnesium 2.5 mg/dl Helplessness and hopelessness may contribute C) Potassium 2.5 mEq/L to regressive, dependent behavior which often D) PTT 70 seconds occurs at any age with hospitalization. Denying or minimizing the seriousness of the illness Review Information: The correct answer is C: is used to avoid facing the worst situation. Re- Potassium 2.5 mEq/L call that denial is the initial step in the process of The patient is at risk for ventricular dysrhythmias working through any loss. when the potassium level is low. Daniels, R. (2003).

Question20

A 52 year-old post menopausal woman asks the nurse how frequently she should have a mammogram. What is the nurse>s best response? A) «Your doctor will advise you about your risks.» B) «Unless you had previous problems, every 2 years is best.» C) «Once a woman reaches 50, she should have a mammogram yearly.» D) «Yearly mammograms are advised for all women over 35.» Review Information: The correct answer is C: «Once a woman reaches 50, she should have a mammogram yearly.» The American Cancer Society recommends a screening mammogram by age 40, every 1 - 2 years for women 40-49, and every year from age 50. If there are family or personal health risks, other assessments may be recommended.

Question21

The nurse is planning care for a client who is taking cyclosporin (Neoral). What would be an appropriate nursing diagnosis for this client? A) Alteration in body image B) High risk for infection C) Altered growth and development D) Impaired physical mobility

Question23

The nurse is caring for a client who is 4 days post-op for a transverse colostomy. The client is ready for discharge and asks the nurse to empty his colostomy pouch. What is the best response by the nurse? A) «You should be emptying the pouch yourself.» B) «Let me demonstrate to you how to empty the pouch.» C) «What have you learned about emptying your pouch?» D) «Show me what you have learned about emptying your pouch.» Review Information: The correct answer is D: «Show me what you have learned about emptying your pouch.» Most adult learners obtain skills by participating in the activities. Anxiety about discharge can be causing the client to forget that they have mastered the skill of emptying the pouch. The client should show the nurse how the pouch is emptied.

Question24

A 3 year-old child has tympanostomy tubes in place. The child>s parent asks the nurse if he can swim in the family pool. The best response from the nurse is Collected by :DeepaRajesh [ 79 ] [email protected] Kuwait

A) «Your child should not swim at all while the tubes are in place.» B) «Your child may swim in your own pool but not in a lake or ocean.» C) «Your child may swim if he wears ear plugs.» D) «Your child may swim anywhere.»

B) The family must observe the child for dehydration C) Parents should administer the daily intramuscular injections D) The client needs to take daily injections in the short-term

Review Information: The correct answer is C: «Your child may swim if he wears ear plugs.»

Review Information: The correct answer is A: The child should carry a nasal spray for emergency use Water should not enter the ears. Children should Diabetes insipidus results from reduced secreuse ear plugs when bathing or swimming and tion of the antidiuretic hormone, vasopressin. should not put their heads under the water. The child will need to administer daily injections of vasopressin, and should have the nasal spray form of the medication readily available. A mediQuestion25 The nurse is caring for a client with asthma who cal alert tag should be worn. has developed gastroesophageal reflux disease (GERD). Which of the following medications pre- Question28 scribed for the client may aggravate GERD? A client diagnosed with cirrhosis is started on A) Anticholinergics lactulose (Cephulac). The main purpose of the B) Corticosteroids drug for this client is to C) Histamine blocker A) add dietary fiber D) Antibiotics B) reduce ammonia levels C) stimulate peristalsis Review Information: The correct answer is A: D) control portal hypertension Anticholinergics An anticholinergic medication will decrease gas- Review Information: The correct answer is B: tric emptying and the pressure on the lower es- reduce ammonia levels ophageal sphincter. Lactulose blocks the absorption of ammonia from the GI tract and secondarily stimulates bowel elimination. Question26 A client is receiving a nitroglycerin infusion for Question29 unstable angina. What assessment would be a The nurse is explaining the effects of cocaine priority when monitoring the effects of this medi- abuse to a pregnant client. Which of the followcation? ing must the nurse understand as a basis for A) Blood pressure teaching? A) Cocaine use can cause fetal growth retardaB) Cardiac enzymes tion C) ECG analysis D) Respiratory rate B) The drug has been linked to neural tube defects Review Information: The correct answer is A: C) Newborn withdrawal generally occurs immediately after birth Blood pressure Since an effect of this drug is vasodilation, the D) Breast feeding promotes positive parenting client must be monitored for hypotension. behaviors

Question27

The nurse is caring for a 10 year-old child who has just been diagnosed with diabetes insipidus. The parents ask about the treatment prescribed, vasopressin. A What is priority in teaching the child and family about this drug? A) The child should carry a nasal spray for emergency use

Review Information: The correct answer is A: Cocaine use can cause fetal growth retardation Cocaine is vasoconstrictive, and this effect in the placental vessels causes fetal hypoxia and diminished growth. Other risks of continued cocaine use during pregnancy include preterm labor, congenital abnormalities, altered brain deCollected by :DeepaRajesh [ 80 ] [email protected] Kuwait

velopment and subsequent behavioral problems in the infant.

Question30

C) «Let>s see if your partner could bring food from home.» D) «If you don>t eat, I will have to suggest for you to be tube fed.»

A client has just been diagnosed with breast cancer. The nurse enters the room and the client tells the nurse that she is stupid. What is the most therapeutic response by the nurse? A) Explore what is going on with the client B) Accept the client’s statement without comment C) Tell the client that the comment is inappropriate D) Leave the client>s room

Review Information: The correct answer is C: «Let>>s see if your partner could bring food from home.» Reassurance is ineffective when a client is actively delusional. This option avoids both arguing with the client and agreeing with the delusional premise. Option D offers a logical response to a primarily affective concern. When the client’s condition has improved, gentle negation of the Review Information: The correct answer is A: delusional premise can be employed. Explore what is going on with the client Exploring feelings with the verbally aggressive Question33 client helps to put angry feelings into words and A client with tuberculosis is started on Rifampin. then to engage in problem solving. Which one of the following statements by the nurse would be appropriate to include in teaching? «You may notice: Question31 A client has many delusions. As the nurse helps A) an orange-red color to your urine.» the client prepare for breakfast the client com- B) your appetite may increase for the first ments «Don’t waste good food on me. I’m dying week.” from this disease I have.» The appropriate re- C) it is common to experience occasional sleep disturbances.» sponse would be A) «You need some nutritious food to help you D) if you take the medication with food, you may have nausea.» regain your weight.» B) «None of the laboratory reports show that you have any physical disease.» C) «Try to eat a little bit, breakfast is the most Review Information: The correct answer is A: an orange-red color to your urine.» important meal of the day.» D) «I know you believe that you have an incur- Discoloration of the urine and other body fluids may occur. It is a harmless response to the drug, able disease.» but the patient needs to be aware it may hapReview Information: The correct answer is D: pen. «I know you believe that you have an incurable disease.» Question34 This response does not challenge the client’s A client tells the RN she has decided to stop takdelusional system and thus forms an alliance by ing sertraline (Zoloft) because she doesn’t like providing reassurance of desire to help the cli- the nightmares, sex dreams, and obsessions ent. she’s experiencing since starting on the medication. What is an appropriate response by the nurse? Question32 A client with paranoid thoughts refuses to eat be- A) «It is unsafe to abruptly stop taking any precause of the belief that the food is poisoned. The scribed medication.» appropriate statement at this time for the nurse B) «Side effects and benefits should be discussed with your health care provider.» to say is A) «Here, I will pour a little of the juice in a medi- C) «This medication should be continued despite unpleasant symptoms.» cine cup to drink it to show you that it is OK.» B) «The food has been prepared in our kitchen D) «Many medications have potential side effects.» and is not poisoned.» Collected by :DeepaRajesh [ 81 ] [email protected] Kuwait

Review Information: The correct answer is A: «It is unsafe to abruptly stop taking any prescribed medication.» Abrupt withdrawal may occasionally cause serotonin syndrome, consisting of lethargy, nausea, headache, fever, sweating and chills. A slow withdrawal may be prescribed with sertraline to avoid dizziness, nausea, vomiting, and diarrhea.

Question35

A client is admitted to the hospital with findings of liver failure with ascites. The health care provider orders spironolactone (Aldactone). What is the pharmacological effect of this medication? A) Promotes sodium and chloride excretion B) Increases aldosterone levels C) Depletes potassium reserves D) Combines safely with antihypertensives

Question37

The nurse is teaching a school-aged child and family about the use of inhalers prescribed for asthma. What is the best way to evaluate effectiveness of the treatments? A) Rely on child>s self-report B) Use a peak-flow meter C) Note skin color changes D) Monitor pulse rate Review Information: The correct answer is B: Use a peak-flow meter The peak flowmeter, if used correctly, shows effectiveness of inhalants.

Question38

The nurse is teaching a client about the toxicity of digoxin. Which one of the following statements Review Information: The correct answer is A: made by the client to the nurse indicates more Promotes sodium and chloride excretion teaching is needed? Spironolactone promotes sodium and chloride A) «I may experience a loss of appetite.» excretion while sparing potassium and decreas- B) «I can expect occasional double vision.» ing aldosterone levels. It had no effect on am- C) «Nausea and vomiting may last a few days.» monia levels. D) «I must report a bounding pulse of 62 immediately.»

Question36

A client was admitted to the psychiatric unit for severe depression. After several days, the client continues to withdraw from the other clients. Which of these statements by the nurse would be the most appropriate to promote interaction with other clients? A) «Your team here thinks it>s good for you to spend time with others.» B) «It is important for you to participate in group activities.» C) «Come with me so you can paint a picture to help you feel better.» D) «Come play Chinese Checkers with Gloria and me.» Review Information: The correct answer is D: «Come play Chinese Checkers with Gloria and me.» This gradually engages the client in interactions with others in small groups rather than large groups. In addition, focusing on an activity is less anxiety-provoking than unstructured discussion. The statement is an example of a positive behavioral expectation.

Review Information: The correct answer is D: «I must report a bounding pulse of 62 immediately.» Slow heart rate is related to increased cardiac output and an intended effect of digoxin. The ideal heart rate is above 60 BPM with digoxin. The client needs further teaching.

Question39

Which of the following assessments by the nurse would indicate that the client is having a possible adverse response to the isoniazid (INH)? A) Severe headache B) Appearance of jaundice C) Tachycardia D) Decreased hearing Review Information: The correct answer is B: Appearance of jaundice Clients receiving INH therapy are at risk for developing drug induced hepatitis. The appearance of jaundice may indicate that the client has liver damage.

Question40 Collected by :DeepaRajesh [ 82 ] [email protected] Kuwait

The nurse is beginning nutritional counseling/ B) Void a little, clean the meatus, then collect teaching with a pregnant woman. What is the ini- specimen tial step in this interaction? C) Clean the meatus, then urinate into container D) Void continuously and catch some of the A) Teach her how to meet the needs of self and urine her family B) Explain the changes in diet necessary for Review Information: The correct answer is pregnant women A: Clean the meatus, begin voiding, then catch C) Questionher understanding and use urine stream. A clean catch urine is difficult to obtain and requires clear directions. Instructing of the food pyramid D) Conduct a diet history to determine her nor- the client to carefully clean the meatus, then void naturally with a steady stream prevents surface mal eating routines bacteria from contaminating the urine specimen. Review Information: The correct answer is D: As starting and stopping flow can be difficult, Conduct a diet history to determine her normal once the client begins voiding it>>s best to just slip the container into the stream. Other responseating routines. es do not reflect correct technique. Assessment is always the first step in planning teaching for any client. A thorough and accurate history is essential for gathering the needed information.

Free NCLEX-RN Sample Test Questions For Nursing Review (Part 1) Jul31,

These are sample nursing review questions and not actual test questions made for educational and practice test purposes only. 75 questions have been posted here with answer keys.

Question1

A client has been hospitalized after an automobile accident. A full leg cast was applied in the emergency room. The most important reason for the nurse to elevate the casted leg is to A) Promote the client>s comfort B) Reduce the drying time C) Decrease irritation to the skin D) Improve venous return Review Information: The correct answer is D: Improve venous return. Elevating the leg both improves venous return and reduces swelling. Client comfort will be improved as well.

Question2

The nurse is reviewing with a client how to collect a clean catch urine specimen. What is the appropriate sequence to teach the client? A) Clean the meatus, begin voiding, then catch urine stream

Question3

Following change-of-shift report on an orthopedic unit, which client should the nurse see first? A) 16 year-old who had an open reduction of a fractured wrist 10 hours ago B) 20 year-old in skeletal traction for 2 weeks since a motor cycle accident C) 72 year-old recovering from surgery after a hip replacement 2 hours ago D) 75 year-old who is in skin traction prior to planned hip pinning surgery. Review Information: The correct answer is C: 72 year-old recovering from surgery after a hip replacement 2 hours ago. Look for the client who has the most imminent risks and acute vulnerability. The client who returned from surgery 2 hours ago is at risk for life threatening hemorrhage and should be seen first. The 16 year-old should be seen next because it is still the first post-op day. The 75 year-old is potentially vulnerable to agerelated physical and cognitive consequences in skin traction should be seen next. The client who can safely be seen last is the 20 year-old who is 2 weeks post-injury.

Question4

A client with Guillain Barre is in a nonresponsive state, yet vital signs are stable and breathing is independent. What should the nurse document to most accurately describe the client>s condition? A) Comatose, breathing unlabored B) Glascow Coma Scale 8, respirations regular C) Appears to be sleeping, vital signs stable Collected by :DeepaRajesh [ 83 ] [email protected] Kuwait

D) Glascow Coma Scale 13, no ventilator required Review Information: The correct answer is B: Glascow Coma Scale 8, respirations regular. The Glascow Coma Scale provides a standard reference for assessing or monitoring level of consciousness. Any score less than 13 indicates a neurological impairment. Using the term comatose provides too much room for interpretation and is not very precise.

Question5

When caring for a client receiving warfarin sodium (Coumadin), which lab test would the nurse monitor to determine therapeutic response to the drug? A) Bleeding time B) Coagulation time C) Prothrombin time D) Partial thromboplastin time

Question7

A client had 20 mg of Lasix (furosemide) PO at 10 AM. Which would be essential for the nurse to include at the change of shift report? A) The client lost 2 pounds in 24 hours B) The client’s potassium level is 4 mEq/liter. C) The client’s urine output was 1500 cc in 5 hours D) The client is to receive another dose of Lasix at 10 PM Review Information: The correct answer is C: The client’s urine output was 1500 cc in 5 hours. Although all of these may be correct information to include in report, the essential piece would be the urine output.

Question8

A client has been tentatively diagnosed with Graves> disease (hyperthyroidism). Which of these findings noted on the initial nursing assessReview Information: The correct answer is ment requires quick intervention by the nurse? C: Prothrombin time. Coumadin is ordered daily, A) a report of 10 pounds weight loss in the last based on the client>>s prothrombin time (PT). month This test evaluates the adequacy of the extrinsic B) a comment by the client «I just can>t sit system and common pathway in the clotting cas- still.» cade; Coumadin affects the Vitamin K depend- C) the appearance of eyeballs that appear to ent clotting factors. «pop» out of the client>s eye sockets D) a report of the sudden onset of irritability in the past 2 weeks Question6 A client with moderate persistent asthma is admitted for a minor surgical procedure. On ad- Review Information: The correct answer is C: mission the peak flow meter is measured at 480 the appearance of eyeballs that appear to «pop» liters/minute. Post-operatively the client is com- out of the client>>s eye sockets. Exophthalmos plaining of chest tightness. The peak flow has or protruding eyeballs is a distinctive characterisdropped to 200 liters/minute. What should the tic of Graves>> Disease. It can result in corneal abrasions with severe eye pain or damage when nurse do first? the eyelid is unable to blink down over the proA) Notify both the surgeon and provider truding eyeball. Eye drops or ointment may be B) Administer the prn dose of albuterol needed. C) Apply oxygen at 2 liters per nasal cannula D) Repeat the peak flow reading in 30 minutes Review Information: The correct answer is B: Administer the prn dose of albuterol. Peak flow monitoring during exacerbations of asthma is recommended for clients with moderate-tosevere persistent asthma to determine the severity of the exacerbation and to guide the treatment. A peak flow reading of less than 50% of the client>>s baseline reading is a medical alert condition and a short-acting beta-agonist must be taken immediately.

Question9

The nurse has performed the initial assessments of 4 clients admitted with an acute episode of asthma. Which assessment finding would cause the nurse to call the provider immediately? A) prolonged inspiration with each breath B) expiratory wheezes that are suddenly absent in 1 lobe C) expectoration of large amounts of purulent mucous D) appearance of the use of abdominal muscles Collected by :DeepaRajesh [ 84 ] [email protected] Kuwait

for breathing Review Information: The correct answer is B: expiratory wheezes that are suddenly absent in 1 lobe. Acute asthma is characterized by expiratory wheezes caused by obstruction of the airways. Wheezes are a high pitched musical sounds produced by air moving through narrowed airways. Clients often associate wheezes with the feeling of tightness in the chest. However, sudden cessation of wheezing is an ominous or bad sign that indicates an emergency -- the small airways are now collapsed.

Question10

During the initial home visit, a nurse is discussing the care of a client newly diagnosed with Alzheimer>s disease with family members. Which of these interventions would be most helpful at this time? A) leave a book about relaxation techniques B) write out a daily exercise routine for them to assist the client to do C) list actions to improve the client>s daily nutritional intake D) suggest communication strategies

intracranial bleeding or extension of the stroke. Further diagnostic testing may be indicated.

Question12

A school-aged child has had a long leg (hip to ankle) synthetic cast applied 4 hours ago. Which statement from the parent indicates that teaching has been inadequate? A) «I will keep the cast uncovered for the next day to prevent burning of the skin.» B) «I can apply an ice pack over the area to relieve itching inside the cast.» C) «The cast should be propped on at least 2 pillows when my child is lying down.» D) «I think I remember that my child should not stand until after 72 hours.»

Review Information: The correct answer is D: «I think I remember that my child should not stand until after 72 hours.». Synthetic casts will typically set up in 30 minutes and dry in a few hours. Thus, the client may stand within the initial 24 hours. With plaster casts, the set up and drying time, especially in a long leg cast which is thicker than an arm cast, can take up to 72 hours. Both types of casts give off a lot of heat when drying and it is preferable to keep the cast uncovered Review Information: The correct answer is D: for the first 24 hours. Clients may complain of a suggest communication strategies. Alzheimer>>s chill from the wet cast and therefore can simply disease, a progressive chronic illness, greatly be covered lightly with a sheet or blanket. Applychallenges caregivers. The nurse can be of ing ice is a safe method of relieving the itching. greatest assistance in helping the family to use communication strategies to enhance their ability Question13 to relate to the client. By use of select verbal and Which blood serum finding in a client with dianonverbal communication strategies the family betic ketoacidosis alerts the nurse that immedican best support the client’s strengths and cope ate action is required? with any aberrant behavior. A) pH below 7.3 B) Potassium of 5.0 C) HCT of 60 Question11 An 80 year-old client admitted with a diagnosis D) Pa O2 of 79% of possible cerebral vascular accident has had a blood pressure from 160/100 to 180/110 over the Review Information: The correct answer is C: past 2 hours. The nurse has also noted increased HCT of 60. This high hematocrit is indicative of lethargy. Which assessment finding should the severe dehydration which requires priority attention in diabetic ketoacidosis. Without sufficient nurse report immediately to the provider? hydration, all systems of the body are at risk A) Slurred speech for hypoxia from a lack of or sluggish circulaB) Incontinence tion. In the absence of insulin, which facilitates C) Muscle weakness the transport of glucose into the cell, the body D) Rapid pulse breaks down fats and proteins to supply energy Review Information: The correct answer is A: ketones, a by-product of fat metabolism. These Slurred speech. Changes in speech patterns and accumulate causing metabolic acidosis (pH < level of conscious can be indicators of continued 7.3), which would be the second concern for this Collected by :DeepaRajesh [ 85 ] [email protected] Kuwait

client. The potassium and PaO2 levels are near normal.

planted from the client>>s own skin.

Question14

A client is admitted to the emergency room following an acute asthma attack. Which of the following assessments would be expected by the nurse?

The nurse is preparing a client with a deep vein thrombosis (DVT) for a Venous Doppler evaluation. Which of the following would be necessary for preparing the client for this test? A) Client should be NPO after midnight B) Client should receive a sedative medication prior to the test C) Discontinue anti-coagulant therapy prior to the test D) No special preparation is necessary Review Information: The correct answer is D: No special preparation is necessary. This is a non-invasive procedure and does not require preparation other than client education.

Question15

A client is admitted with infective endocarditis (IE). Which finding would alert the nurse to a complication of this condition? A) dyspnea B) heart murmur C) macular rash D) hemorrhage Review Information: The correct answer is B: heart murmur. Large, soft, rapidly developing vegetations attach to the heart valves. They have a tendency to break off, causing emboli and leaving ulcerations on the valve leaflets. These emboli produce findings of cardiac murmur, fever, anorexia, malaise and neurologic sequelae of emboli. Furthermore, the vegetations may travel to various organs such as spleen, kidney, coronary artery, brain and lungs, and obstruct blood flow.

Question16

Question17

A) Diffuse expiratory wheezing B) Loose, productive cough C) No relief from inhalant D) Fever and chills Review Information: The correct answer is A: Diffuse expiratory wheezing. In asthma, the airways are narrowed, creating difficulty getting air in. A wheezing sound results.

Question18

A client has been admitted with a fractured femur and has been placed in skeletal traction. Which of the following nursing interventions should receive priority? A) Maintaining proper body alignment B) Frequent neurovascular assessments of the affected leg C) Inspection of pin sites for evidence of drainage or inflammation D) Applying an over-bed trapeze to assist the client with movement in bed Review Information: The correct answer is B: Frequent neurovascular assessments of the affected leg. The most important activity for the nurse is to assess neurovascular status. Compartment syndrome is a serious complication of fractures. Prompt recognition of this neurovascular problem and early intervention may prevent permanent limb damage.

Question19

The nurse explains an autograft to a client scheduled for excision of a skin tumor. The nurse knows the client understands the procedure when the client says, «I will receive tissue from A) a tissue bank.» B) a pig.» C) my thigh.» D) synthetic skin.»

The nurse is assigned to care for a client who had a myocardial infarction (MI) 2 days ago. The client has many questions about this condition. What area is a priority for the nurse to discuss at this time? A) Daily needs and concerns B) The overview cardiac rehabilitation C) Medication and diet guideline D) Activity and rest guidelines

Review Information: The correct answer is C: my thigh.». Autografts are done with tissue trans-

Review Information: The correct answer is A: Daily needs and concerns. At 2 days post-MI, the Collected by :DeepaRajesh [ 86 ] [email protected] Kuwait

client’s education should be focused on the im- loupe, milk mediate needs and concerns for the day. D) Peanut butter and jelly sandwich, apple slices, milk

Question20

A 3 year-old child is brought to the clinic by his grandmother to be seen for «scratching his bottom and wetting the bed at night.» Based on these complaints, the nurse would initially assess for which problem? A) allergies B) scabies C) regression D) pinworms

Review Information: The correct answer is B: Ground beef patty, lima beans, wheat roll, raisins, milk. Iron rich foods include red meat, fish, egg yolks, green leafy vegetables, legumes, whole grains, and dried fruits such as raisins. This dinner is the best choice: It is high in iron and is appropriate for a toddler.

Question23

The nurse admitting a 5 month-old who vomited Review Information: The correct answer is 9 times in the past 6 hours should observe for D: pinworms. Signs of pinworm infection include signs of which overall imbalance? intense perianal itching, poor sleep patterns, A) Metabolic acidosis general irritability, restlessness, bed-wetting, B) Metabolic alkalosis distractibility and short attention span. Scabies C) Some increase in the serum hemoglobin is an itchy skin condition caused by a tiny, eight- D) A little decrease in the serum potassium legged burrowing mite called Sarcoptes scabiei . The presence of the mite leads to intense itching Review Information: The correct answer is B: in the area of its burrows. Metabolic alkalosis. Vomiting causes loss of acid from the stomach. Prolonged vomiting can result in excess loss of acid and lead to metabolic Question21 The nurse is caring for a newborn with tra- alkalosis. Findings include irritability, increased cheoesophageal fistula. Which nursing diagno- activity, hyperactive reflexes, muscle twitching and elevated pulse. Options C and D are correct sis is a priority? answers but not the best answers since they are A) Risk for dehydration too general. B) Ineffective airway clearance C) Altered nutrition D) Risk for injury Question24 A two year-old child is brought to the provider>s Review Information: The correct answer is B: office with a chief complaint of mild diarrhea for Ineffective airway clearance. The most common two days. Nutritional counseling by the nurse form of TEF is one in which the proximal esopha- should include which statement? geal segment terminates in a blind pouch and A) Place the child on clear liquids and gelatin for the distal segment is connected to the trachea 24 hours or primary bronchus by a short fistula at or near B) Continue with the regular diet and include oral the bifurcation. Thus, a priority is maintaining an rehydration fluids open airway, preventing aspiration. Other nurs- C) Give bananas, apples, rice and toast as tolering diagnoses are then addressed. ated D) Place NPO for 24 hours, then rehydrate with milk and water Question22 The nurse is developing a meal plan that would provide the maximum possible amount of iron for Review Information: The correct answer is B: a child with anemia. Which dinner menu would Continue with the regular diet and include oral rehydration fluids. Current recommendations for be best? A) Fish sticks, french fries, banana, cookies, mild to moderate diarrhea are to maintain a normal diet with fluids to rehydrate. milk B) Ground beef patty, lima beans, wheat roll, raisins, milk C) Chicken nuggets, macaroni, peas, canta- Question25 Collected by :DeepaRajesh [ 87 ] [email protected] Kuwait

The nurse is teaching parents about the appro- ates a high renal solute load. priate diet for a 4 month-old infant with gastroenteritis and mild dehydration. In addition to oral Question28 rehydration fluids, the diet should include The nurse is preparing a handout on infant feeding to be distributed to families visiting the clinic. A) formula or breast milk Which notation should be included in the teachB) broth and tea ing materials? C) rice cereal and apple juice D) gelatin and ginger ale A) Solid foods are introduced one at a time beginning with cereal Review Information: The correct answer is A: B) Finely ground meat should be started early to formula or breast milk. The usual diet for a young provide iron infant should be followed. C) Egg white is added early to increase protein intake D) Solid foods should be mixed with formula in Question26 A child is injured on the school playground and a bottle appears to have a fractured leg. The first action Review Information: The correct answer is A: the school nurse should take is Solid foods are introduced one at a time beginning with cereal. Solid foods should be added A) call for emergency transport to the hospital B) immobilize the limb and joints above and be- one at a time between 4-6 months. If the infant is able to tolerate the food, another may be added low the injury in a week. Iron fortified cereal is the recommendC) assess the child and the extent of the injury ed first food. D) apply cold compresses to the injured area Review Information: The correct answer is C: assess the child and the extent of the injury. When applying the nursing process, assessment is the first step in providing care. The «5 Ps» of vascular impairment can be used as a guide (pain, pulse, pallor, paresthesia, paralysis).

Question29

The nurse planning care for a 12 year-old child with sickle cell disease in a vaso-occlusive crisis of the elbow should include which one of the following as a priority?

A) Limit fluids Question27 B) Client controlled analgesia The mother of a 3 month-old infant tells the nurse C) Cold compresses to elbow that she wants to change from formula to whole D) Passive range of motion exercise milk and add cereal and meats to the diet. What should be emphasized as the nurse teaches Review Information: The correct answer is B: about infant nutrition? Client controlled analgesia. Management of a sickle cell crisis is directed towards supportive A) Solid foods should be introduced at 3-4 and symptomatic treatment. The priority of care months is pain relief. In a 12 year-old child, client controlB) Whole milk is difficult for a young infant to di- led analgesia promotes maximum comfort. gest C) Fluoridated tap water should be used to dilute Question30 milk The nurse is performing a physical assessment D) Supplemental apple juice can be used be- on a toddler. Which of the following actions tween feedings should be the first? Review Information: The correct answer is B: Whole milk is difficult for a young infant to digest. Cow>>s milk is not given to infants younger than 1 year because the tough, hard curd is difficult to digest. In addition, it contains little iron and cre-

A) Perform traumatic procedures B) Use minimal physical contact C) Proceed from head to toe D) Explain the exam in detail Collected by :DeepaRajesh [ 88 ] [email protected] Kuwait

Review Information: The correct answer is B: Use minimal physical contact. The nurse should approach the toddler slowly and use minimal physical contact initially so as to gain the toddler>>s cooperation. Be flexible in the sequence of the exam, and give only brief simple explanations just prior to the action.

these foods would the nurse reinforce for the client to eat at least daily? A) Spaghetti B) Watermelon C) Chicken D) Tomatoes

Review Information: The correct answer is B: What finding signifies that children have attained Watermelon. Watermelon is high in potassium and will replace potassium lost by the diuretic. the stage of concrete operations (Piaget)? The other foods are not high in potassium. A) Explores the environment with the use of sight and movement Question34 B) Thinks in mental images or word pictures While teaching the family of a child who will take C) Makes the moral judgment that «stealing is phenytoin (Dilantin) regularly for seizure control, wrong» it is most important for the nurse to teach them D) Reasons that homework is time-consuming about which of the following actions? yet necessary A) Maintain good oral hygiene and dental care Review Information: The correct answer is B) Omit medication if the child is seizure free C: Makes the moral judgment that «stealing is C) Administer acetaminophen to promote sleep wrong». The stage of concrete operations is de- D) Serve a diet that is high in iron picted by logical thinking and moral judgments. Review Information: The correct answer is A: Maintain good oral hygiene and dental care. Question32 The mother of a child with a neural tube defect Swollen and tender gums occur often with use of asks the nurse what she can do to decrease the phenytoin. Good oral hygiene and regular visits chances of having another baby with a neural to the dentist should be emphasized. tube defect. What is the best response by the nurse? Question35 The nurse is offering safety instructions to a parA) «Folic acid should be taken before and after ent with a four month-old infant and a four yearconception.» old child. Which statement by the parent indiB) «Multivitamin supplements are recommended cates understanding of appropriate precautions during pregnancy.» to take with the children? C) «A well balanced diet promotes normal fetal development.» A) «I strap the infant car seat on the front seat to D) «Increased dietary iron improves the health of face backwards.» mother and fetus.» B) «I place my infant in the middle of the living room floor on a blanket to play with my four yearReview Information: The correct answer is old while I make supper in the kitchen.» A: «Folic acid should be taken before and after C) «My sleeping baby lies so cute in the crib with conception.». The American Academy of Pedi- the little buttocks stuck up in the air while the four atrics recommends that all childbearing women year-old naps on the sofa.» increase folic acid from dietary sources and/or D) «I have the four year-old hold and help feed supplements. There is evidence that increased the four month-old a bottle in the kitchen while I amounts of folic acid prevents neural tube de- make supper.» fects. Review Information: The correct answer is D: «I have the four year-old hold and help feed Question33 The provider orders Lanoxin (digoxin) 0.125 mg the four month-old a bottle in the kitchen while I PO and furosemide 40 mg every day. Which of make supper.». The infant seat is to be placed

Question31

Collected by :DeepaRajesh [ 89 ] [email protected] Kuwait

on the rear seat. Small children and infants are not to be left unsupervised. Infants are

C) with each meal or snack D) each time carbohydrates are eaten

Review Information: The correct answer is C: The nurse admits a 7 year-old to the emergency with each meal or snack. Pancreatic enzymes room after a leg injury. The x-rays show a femur should be taken with each meal and every snack fracture near the epiphysis. The parents ask what to allow for digestion of all foods that are eaten. will be the outcome of this injury. The appropriate response by the nurse should be which of these statements? Question39 A nurse is providing a parenting class to individuA) «The injury is expected to heal quickly be- als living in a community of older homes. In discause of thin periosteum.» cussing formula preparation, which of the followB) «In some instances the result is a retarded ing is most important to prevent lead poisoning? bone growth.» C) «Bone growth is stimulated in the affected A) Use ready-to-feed commercial infant formula leg.» B) Boil the tap water for 10 minutes prior to preD) «This type of injury shows more rapid union paring the formula than that of younger children.» C) Let tap water run for 2 minutes before adding to concentrate Review Information: The correct answer is B: D) Buy bottled water labeled «lead free» to mix «In some instances the result is a retarded bone the formula growth.». An epiphyseal (growth) plate fracture in a 7 year-old often results in retarded bone Review Information: The correct answer is C: growth. The leg often will be different in length Let tap water run for 2 minutes before adding to than the uninjured leg. concentrate. Use of lead-contaminated water to prepare formula is a major source of poisoning in infants. Drinking water may be contaminated Question37 The parents of a 4 year-old hospitalized child tell by lead from old lead pipes or lead solder used the nurse, “We are leaving now and will be back in sealing water pipes. Letting tap water run for at 6 PM.” A few hours later the child asks the several minutes will diminish the lead contaminurse when the parents will come again. What is nation. the best response by the nurse?

Question36

A) «They will be back right after supper.» B) «In about 2 hours, you will see them.» C) «After you play awhile, they will be here.» D) «When the clock hands are on 6 and 12.» Review Information: The correct answer is A: «They will be back right after supper.». Time is not completely understood by a 4 year-old. Preschoolers interpret time with their own frame of reference. Thus, it is best to explain time in relationship to a known, common event.

Question38

The nurse is giving instructions to the parents of a child with cystic fibrosis. The nurse would emphasize that pancreatic enzymes should be taken A) once each day B) 3 times daily after meals

Question40

Which of the following manifestations observed by the school nurse confirms the presence of pediculosis capitis in students? A) Scratching the head more than usual B) Flakes evident on a student>s shoulders C) Oval pattern occipital hair loss D) Whitish oval specks sticking to the hair Review Information: The correct answer is D: Whitish oval specks sticking to the hair. Diagnosis of pediculosis capitis is made by observation of the white eggs (nits) firmly attached to the hair shafts. Treatment can include application of a medicated shampoo with lindane for children over 2 years of age, and meticulous combing and removal of all nits.

Collected by :DeepaRajesh [ 90 ] [email protected] Kuwait

Question41

When interviewing the parents of a child with asthma, it is most important to assess the child>s environment for what factor? A) Household pets B) New furniture C) Lead based paint D) Plants such as cactus Review Information: The correct answer is A: Household pets. Animal dander is a very common allergen affecting persons with asthma. Other triggers may include pollens, carpeting and household dust.

Question42

The mother of a 2 month-old baby calls the nurse 2 days after the first DTaP, IPV, Hepatitis B and HIB immunizations. She reports that the baby feels very warm, cries inconsolably for as long as 3 hours, and has had several shaking spells. In addition to referring her to the emergency room, the nurse should document the reaction on the baby>s record and expect which immunization to be most associated with the findings the infant is displaying? A) DTaP B) Hepatitis B C) Polio D) H. Influenza Review Information: The correct answer is A: DTaP. The majority of reactions occur with the administration of the DTaP vaccination. Contradictions to giving repeat DTaP immunizations include the occurrence of severe side effects after a previous dose as well as signs of encephalopathy within 7 days of the immunization.

Question43

response to being in the hospital.» D) «You might want to «sneak out» of the room once the child falls asleep.» Review Information: The correct answer is C: «Keep in mind that for the age this is a normal response to being in the hospital.». The protest phase of separation anxiety is a normal response for a child this age. In toddlers, ages 1 to 3, separation anxiety is at its peak

Question44

A couple experienced the loss of a 7 month-old fetus. In planning for discharge, what should the nurse emphasize? A) To discuss feelings with each other and use support persons B) To focus on the other healthy children and move through the loss C) To seek causes for the fetal death and come to some safe conclusion D) To plan for another pregnancy within 2 years and maintain physical health Review Information: The correct answer is A: To discuss feelings with each other and use support persons. To communicate in a therapeutic manner, the nurse>>s goal is to help the couple begin the grief process by suggesting they talk to each other, seek family, friends and support groups to listen to their feelings.

Question45

The nurse is performing a pre-kindergarten physical on a 5 year-old. The last series of vaccines will be administered. What is the preferred site for injection by the nurse? A) vastus intermedius B) gluteus maximus C) vastus lateralis D) dorsogluteaI

The mother of a 2 year-old hospitalized child asks the nurse>s advice about the child>s screaming every time the mother gets ready to leave the Review Information: The correct answer is C: hospital room. What is the best response by the vastus lateralis. Vastus lateralis, a large and well developed muscle, is the preferred site, since it is nurse? removed from major nerves and blood vessels. A) «I think you or your partner needs to stay with the child while in the hospital.» B) «Oh, that behavior will stop in a few days.» Question46 C) «Keep in mind that for the age this is a normal A 7 month pregnant woman is admitted with comCollected by :DeepaRajesh [ 91 ] [email protected] Kuwait

plaints of painless vaginal bleeding over several hours. The nurse should prepare the client for an immediate A) Non stress test B) Abdominal ultrasound C) Pelvic exam D) X-ray of abdomen Review Information: The correct answer is B: Abdominal ultrasound. The standard for diagnosis of placenta previa, which is suggested in the client>>s history of painless bleeding, is abdominal ultrasound.

Question47

A nurse entering the room of a postpartum mother observes the baby lying at the edge of the bed while the woman sits in a chair. The mother states «This is not my baby, and I do not want it.» After repositioning the child safely, the nurse>s best response is A) «This is a common occurrence after birth, but you will come to accept the baby.» B) «Many women have postpartum blues and need some time to love the baby.» C) «What a beautiful baby! Her eyes are just like yours.» D) «You seem upset; tell me what the pregnancy and birth were like for you.» Review Information: The correct answer is D: «You seem upset; tell me what the pregnancy and birth were like for you.». A non-judgmental, open ended response facilitates dialogue between the client and nurse.

Question48

The nurse notes that a 2 year-old child recovering from a tonsillectomy has an temperature of 98.2 degrees Fahrenheit at 8:00 AM. At 10:00 AM the child>s parent reports that the child «feels very warm» to touch. The first action by the nurse should be to A) reassure the parent that this is normal B) offer the child cold oral fluids C) reassess the child>s temperature D) administer the prescribed acetaminophen

Review Information: The correct answer is C: reassess the child>>s temperature. A child>>s temperature may have rapid fluctuations. The nurse should listen to and show respect for what parents say. Parental caretakers are often quite sensitive to variations in their children>>s condition that may not be immediately evident to others.

Question49

The nurse is caring for a client who was successfully resuscitated from a pulseless dysrhythmia. Which of the following assessments is critical for the nurse to include in the plan of care? A) hourly urine output B) white blood count C) blood glucose every 4 hours D) temperature every 2 hours Review Information: The correct answer is A: hourly urine output. Clients who have had an episode of decreased glomerular perfusion are at risk for pre-renal failure. This is caused by any abnormal decline in kidney perfusion that reduces glomerular perfusion. Pre-renal failure occurs when the effective arterial blood volume falls. Examples of this phenomena include a drop in circulating blood volume as in a cardiac arrest state or in low cardiac perfusion states such as congestive heart failure associated with a cardiomyopathy. Close observation of hourly urinary output is necessary for early detection of this condition.

Question50

A client is admitted to the rehabilitation unit following a cerebral vascular accident (CVA) and mild dysphagia. The most appropriate intervention for this client is to A) position client in upright position while eating B) place client on a clear liquid diet C) tilt head back to facilitate swallowing reflex D) offer finger foods such as crackers or pretzels Review Information: The correct answer is A: position client in upright position while eating. An upright position facilitates proper chewing and swallowing. Collected by :DeepaRajesh [ 92 ] [email protected] Kuwait

Question53 Question51

A 72 year-old client with osteomyelitis requires a 6 week course of intravenous antibiotics. In planning for home care, what is the most important action by the nurse? A) Investigating the client>s insurance coverage for home IV antibiotic therapy B) Determining if there are adequate hand washing facilities in the home C) Assessing the client>s ability to participate in self care and/or the reliability of a caregiver D) Selecting the appropriate venous access device Review Information: The correct answer is C: Assessing the client>>s ability to participate in self care and/or the reliability of a caregiver. The cognitive ability of the client as well as the availability and reliability of a caregiver must be assessed to determine if home care is a feasible option.

Question52

A nurse administers the influenza vaccine to a client in a clinic. Within 15 minutes after the immunization was given, the client complains of itchy and watery eyes, increased anxiety, and difficulty breathing. The nurse expects that the first action in the sequence of care for this client will be to A) Maintain the airway B) Administer epinephrine 1:1000 as ordered C) Monitor for hypotension with shock D) Administer diphenhydramine as ordered

The nurse instructs the client taking dexamethasone (Decadron) to take it with food or milk. The physiological basis for this instruction is that the medication A) retards pepsin production B) stimulates hydrochloric acid production C) slows stomach emptying time D) decreases production of hydrochloric acid Review Information: The correct answer is B: stimulates hydrochloric acid production. Decadron increases the production of hydrochloric acid, which may cause gastrointestinal ulcers.

Question54

A client receiving chlorpromazine HCL (Thorazine) is in psychiatric home care. During a home visit the nurse observes the client smacking her lips alternately with grinding her teeth. The nurse recognizes this assessment finding as what? A) Dystonia B) Akathisia C) Brady dyskinesia D) Tardive dyskinesia Review Information: The correct answer is D: Tardive dyskinesia. Signs of tardive dyskinesia include smacking lips, grinding of teeth and «fly catching» tongue movements. These findings are often described as Parkinsonian.

Question55

Which of the following findings contraindicate the use of haloperidol (Haldol) and warrant withholding the dose? A) Drowsiness, lethargy, and inactivity B) Dry mouth, nasal congestion, and blurred viReview Information: The correct answer is B: sion Administer epinephrine 1:1000 as ordered. All C) Rash, blood dyscrasias, severe depression the answers are correct given the circumstances, D) Hyperglycemia, weight gain, and edema but the priority is to administer the epinephrine, then maintain the airway. In the early stages of Review Information: The correct answer is anaphylaxis, when the patient has not lost con- C: Rash, blood dyscrasias, severe depression. sciousness and is normotensive, administering Rash and blood dyscrasias are side effects of the epinephrine is first, and applying the oxygen, anti-psychotic drugs. A history of severe depresand watching for hypotension and shock, are lat- sion is a contraindication to the use of neuroleper responses. The prevention of a severe crisis tics. is maintained by using diphenhydramine.

Collected by :DeepaRajesh [ 93 ] [email protected] Kuwait

Question56

digitalis toxicity is a low potassium level. Clients The nurse is reinforcing teaching to a 24 year-old must be taught that it is important to have adwoman receiving acyclovir (Zovirax) for a Herpes equate potassium intake especially if taking diuSimplex Virus type 2 infection. Which of these in- retics that enhance the loss of potassium while they are taking digitalis. structions should the nurse give the client? A) Complete the entire course of the medication for an effective cure B) Begin treatment with acyclovir at the onset of symptoms of recurrence C) Stop treatment if she thinks she may be pregnant to prevent birth defects D) Continue to take prophylactic doses for at least 5 years after the diagnosis Review Information: The correct answer is B: Begin treatment with acyclovir at the onset of symptoms of recurrence. When the client is aware of early symptoms, such as pain, itching or tingling, treatment is very effective. Medications for herpes simplex do not cure the disease; they simply decrease the level of symptoms.

Question57

Question59

A 42 year-old male client refuses to take propranolol hydrochloride (Inderal) as prescribed. Which client statement from the assessment data is likely to explain his noncompliance? A) «I have problems with diarrhea.» B) «I have difficulty falling asleep.» C) «I have diminished sexual function.» D) «I often feel jittery.» Review Information: The correct answer is C: «I have diminished sexual function.». Inderal, a beta-blocking agent used in hypertension, prohibits the release of epinephrine into the cells; this may result in hypotension which results in decreased libido and impotence.

A 14 month-old child ingested half a bottle of aspirin tablets. Which of the following would the Question60 nurse expect to see in the child? The nurse caring for a 9 year-old child with a fractured femur is told that a medication error A) Hypothermia occurred. The child received twice the ordered B) Edema dose of morphine an hour ago. Which nursing C) Dyspnea diagnosis is a priority at this time? D) Epistaxis A) Risk for fluid volume deficit related to morReview Information: The correct answer is phine overdose D: Epistaxis. A large dose of aspirin inhibits pro- B) Decreased gastrointestinal mobility related to thrombin formation and lowers platelet levels. mucosal irritation With an overdose, clotting time is prolonged. C) Ineffective breathing patterns related to central nervous system depression D) Altered nutrition related to inability to control nausea and vomiting Question58 An 80 year-old client on digitalis (Lanoxin) reports nausea, vomiting, abdominal cramps and Review Information: The correct answer is C: halo vision. Which of the following laboratory re- Ineffective breathing patterns related to central nervous system depression. Respiratory depressults should the nurse analyze first? sion is a life-threatening risk in this overdose. A) Potassium levels B) Blood pH C) Magnesium levels Question61 D) Blood urea nitrogen Lactulose (Chronulac) has been prescribed for a client with advanced liver disease. Which of the Review Information: The correct answer is A: following assessments would the nurse use to Potassium levels. The most common cause of evaluate the effectiveness of this treatment? Collected by :DeepaRajesh [ 94 ] [email protected] Kuwait

A) An increase in appetite B) A decrease in fluid retention C) A decrease in lethargy D) A reduction in jaundice Review Information: The correct answer is C: A decrease in lethargy. Lactulose produces an acid environment in the bowel and traps ammonia in the gut; the laxative effect then aids in removing the ammonia from the body. This decreases the effects of hepatic encephalopathy, including lethargy and confusion.

Question62

The nurse is teaching a class on HIV prevention. Which of the following should be emphasized as increasing risk? A) Donating blood B) Using public bathrooms C) Unprotected sex D) Touching a person with AIDS Review Information: The correct answer is C: Unprotected sex. Because HIV is spread through exposure to bodily fluids, unprotected intercourse and shared drug paraphernalia remain the highest risks for infection.

Question63

While interviewing a new admission, the nurse notices that the client is shifting positions, wringing her hands, and avoiding eye contact. It is important for the nurse to A) ask the client what she is feeling B) assess the client for auditory hallucinations C) recognize the behavior as a side effect of medication D) re-focus the discussion on a less anxiety provoking topic Review Information: The correct answer is A: ask the client what she is feeling. The initial step in anxiety intervention is observing, identifying, and assessing anxiety. The nurse should seek client validation of the accuracy of nursing assessments and avoid drawing conclusions based on limited data. In the situation above, the client may simply need to use the restroom but be reluctant to communicate her need!

Question64

A young adult seeks treatment in an outpatient mental health center. The client tells the nurse he is a government official being followed by spies. On further questioning, he reveals that his warnings must be heeded to prevent nuclear war. What is the most therapeutic approach by the nurse? A) Listen quietly without comment B) Ask for further information on the spies C) Confront the client’s delusion D) Contact the government agency Review Information: The correct answer is A: Listen quietly without comment. The client>>s comments demonstrate grandiose ideas. The most therapeutic response is to listen but avoid being incorporated into the client’s delusional system.

Question65

The nurse is assessing a 17 year-old female client with bulimia. Which of the following laboratory reports would the nurse anticipate? A) Increased serum glucose B) Decreased albumin C) Decreased potassium D) Increased sodium retention Review Information: The correct answer is C: Decreased potassium. In bulimia, loss of electrolytes can occur in addition to other findings of starvation and dehydration.

Question66

A client, recovering from alcoholism, asks the nurse, «What can I do when I start recognizing relapse triggers within myself?» How might the nurse best respond? A) «When you have the impulse to stop in a bar, contact a sober friend and talk with him.» B) «Go to an AA meeting when you feel the urge to drink.» C) «It is important to exercise daily and get involved in activities that will cause you not to think about drug use.» D) «Let’s talk about possible options you have Collected by :DeepaRajesh [ 95 ] [email protected] Kuwait

when you recognize relapse triggers in yourself.» Review Information: The correct answer is D: «Let’s talk about possible options you have when you recognize relapse triggers in yourself.». This option encourages the process of self evaluation and problem solving, while avoiding telling the client what to do. Encouraging the client to brainstorm about response options validates the nurse’s belief in the client’s personal competency and reinforces a coping strategy that will be needed when the nurse may not be available to offer solutions.

Question67

Therapeutic nurse-client interaction occurs when the nurse A) assists the client to clarify the meaning of what the client has said B) interprets the client’s covert communication C) praises the client for appropriate feelings and behavior D) advises the client on ways to resolve problems Review Information: The correct answer is A: assists the client to clarify the meaning of what the client has said. Clarification is a facilitating/ therapeutic communication strategy. Interpretation, changing the focus/subject, giving approval, and advising are non-therapeutic/barriers to communication.

client learns both tolerance and skills for relationships.

Question69

An important goal in the development of a therapeutic inpatient milieu is to A) provide a businesslike atmosphere where clients can work on individual goals B) provide a group forum in which clients decide on unit rules, regulations, and policies C) provide a testing ground for new patterns of behavior while the client takes responsibility for his or her own actions D) discourage expressions of anger because they can be disruptive to other clients Review Information: The correct answer is C: provide a testing ground for new patterns of behavior while the client takes responsibility for his or her own actions. A therapeutic milieu is purposeful and planned to provide safety and a testing ground for new patterns of behavior.

Question70

A client with paranoid delusions stares at the nurse over a period of several days. The client suddenly walks up to the nurse and shouts «You think you’re so perfect and pure and good.» An appropriate response for the nurse is

A) «Is that why you’ve been staring at me?» B) «You seem to be in a really bad mood.» C) «Perfect? I don’t quite understand.» Question68 Which nursing intervention will be most effective D) «You seem angry right now.» in helping a withdrawn client to develop relationReview Information: The correct answer is D: ship skills? A) Offer the client frequent opportunities to inter- «You seem angry right now.». The nurse recognizes the underlying emotion with a matter of fact act with 1 person B) Provide the client with frequent opportunities attitude, but avoids telling the clients how they feel. to interact with other clients C) Assist the client to analyze the meaning of the withdrawn behavior D) Discuss with the client the focus that other Question71 clients have similar problems A client who is a former actress enters the day room wearing a sheer nightgown, high heels, nuReview Information: The correct answer is A: merous bracelets, bright red lipstick and heavily Offer the client frequent opportunities to interact rouged cheeks. Which nursing action is the best with 1 person. The withdrawn client is uncomfort- in response to the client’s attire? able in social interaction. The nurse-client relationship is a corrective relationship in which the A) Gently remind her that she is no longer on Collected by :DeepaRajesh [ 96 ] [email protected] Kuwait

stage B) Directly assist client to her room for appropriate apparel C) Quietly point out to her the dress of other clients on the unit D) Tactfully explain appropriate clothing for the hospital Review Information: The correct answer is B: Directly assist client to her room for appropriate apparel. It assists the client to maintain self-esteem while modifying behavior.

Question72

When teaching suicide prevention to the parents of a 15 year-old who recently attempted suicide, the nurse describes the following behavioral cue as indicating a need for intervention. A) Angry outbursts at significant others B) Fear of being left alone C) Giving away valued personal items D) Experiencing the loss of a boyfriend Review Information: The correct answer is C: Giving away valued personal items. Eighty percent of all potential suicide victims give some type of indication that self-destructiveness should be addressed. These clues might lead one to suspect that a client is having suicidal thoughts or is developing a plan.

Question73

Which statement made by a client indicates to the nurse that the client may have a thought disorder? A) «I>m so angry about this. Wait until my partner hears about this.» B) «I>m a little confused. What time is it?» C) «I can>t find my <mesmer> shoes. Have you seen them?» D) «I>m fine. It>s my daughter who has the problem.» Review Information: The correct answer is C: «I can>>t find my <>mesmer>> shoes. Have you seen them?». A neologism is a new word self invented by a person and not readily understood by another. Using neologisms is often associated with a thought disorder.

Question74

In a psychiatric setting, the nurse limits touch or contact used with clients to handshaking because A) some clients misconstrue hugs as an invitation to sexual advances B) handshaking keeps the gesture on a professional level C) refusal to touch a client denotes lack of concern D) inappropriate touch often results in charges of assault and battery Review Information: The correct answer is A: some clients misconstrue hugs as an invitation to sexual advances. Touch denotes positive feelings for another person. The client may interpret hugging and holding hands as sexual advances.

Question75

A client with anorexia is hospitalized on a medical unit due to electrolyte imbalance and cardiac dysrhythmias. Additional assessment findings that the nurse would expect to observe are A) brittle hair, lanugo, amenorrhea B) diarrhea, nausea, vomiting, dental erosion C) hyperthermia, tachycardia, increased metabolic rate D) excessive anxiety about symptoms Review Information: The correct answer is A: brittle hair, lanugo, amenorrhea. Physical findings associated with anorexia also include reduced metabolic rate and lower vital signs.

Free NCLEX-RN Sample Test Questions For Nursing Review (Pharmacology Set 2) Jul31,

A nurse is assigned to perform well-child assessments at a day care center. A staff member interrupts the examinations to ask for assistance. They find a crying 3 year-old child on the floor with mouth wide open and gums bleeding. Two unlabeled open bottles lie nearby. The nurse’s first action should be Collected by :DeepaRajesh [ 97 ] [email protected] Kuwait

A) call the poison control center, then 911 B) administer syrup of Ipecac to induce vomiting C) give the child milk to coat her stomach D) ask the staff about the contents of the bottles

The nurse practicing in a long term care facility recognizes that elderly clients are at greater risk for drug toxicity than younger adults because of which of the following physiological changes of advancing age? A) Drugs are absorbed more readily from the GI Review Information: The correct answer is D: tract ask the staff about the contents of the bottles B) Elders have less body water and more fat The nurse needs to assess what the child in- C) The elderly have more rapid hepatic metabogested before determining the next action. Once lism the substance is identified, the poison control D) Older people are often malnourished and center and emergency response team should be anemic called. Review Information: The correct answer is B: A client with atrial fibrillation is receiving digoxin Elders have less body water and more fat (Lanoxin). Which of these assessments is most Because elderly persons have decreased lean body tissue/water in which to distribute medicaimportant for the nurse to perform? tions, more drug remains in the circulatory system A) Monitor blood pressure every 4 hours with potential for drug toxicity. Increased body fat B) Measure apical pulse prior to administration results in greater amounts of fat-soluble drugs C) Maintain accurate intake and output records being absorbed, leaving less in circulation, thus D) Record an EKG strip after administration increasing the duration of action of the drug

Question2

Review Information: The correct answer is B: Measure apical pulse prior to administration Digitoxin decreases conduction velocity through the AV node and prolongs the refractory period. If the apical heart rate is less than 60 beats/minute, withhold the drug. The apical pulse should be taken with a stethoscope so that there will be no mistake about what the heart rate actually is.

Question3

Question5

The nurse is assessing a client who is on long term glucocorticoid therapy. Which of the following findings would the nurse expect? A) Buffalo hump B) Increased muscle mass C) Peripheral edema D) Jaundice

The nurse is administering an intravenous vesicant chemotherapeutic agent to a client. Which assessment would require the nurse’s immediate action? A) Stomatitis lesion in the mouth B) Severe nausea and vomiting C) Complaints of pain at site of infusion D) A rash on the client’s extremities

Review Information: The correct answer is A: Buffalo hump With high doses of glucocorticoid, iatrogenic Cushing’’s syndrome develops. The exaggerated physiological action causes abnormal fat distribution which results in a moon-shaped face, a intrascapular pad on the neck (buffalo hump) and truncal obesity with slender limbs.

Review Information: The correct answer is C: Complaints of pain at site of infusion A vesicant is a chemotherapeutic agent capable of causing blistering of tissues and possible tissue necrosis if there is extravasation. These agents are irritants which cause pain along the vein wall, with or without inflammation.

Question6

Question4

The health care provider has written “Morphine sulfate 2 mgs IV every 3-4 hours prn for pain” on the chart of a child weighing 22 lb. (10 kg). What is the nurse’s initial action? A) Check with the pharmacist B) Hold the medication and contact the provider C) Administer the prescribed dose as ordered D) Give the dose every 6-8 hours Collected by :DeepaRajesh [ 98 ] [email protected] Kuwait

Review Information: The correct answer is B: Hold the medication and contact the provider The usual pediatric dose of morphine is 0.1 mg/ kg every 3 to 4 hours. At 10 kg, this child typically should receive 1.0 mg every 3 to 4 hours.

Question7

A client is ordered atropine to be administered preoperatively. Which physiological effect should the nurse monitor for? A) Elevate blood pressure B) Drying up of secretions C) Reduce heart rate D) Enhance sedation Review Information: The correct answer is B: Drying up of secretions Atropine dries secretions which may get in the way during the operative procedure.

Question8

A client is receiving digitalis. The nurse should instruct the client to report which of the following side effects? A) Nausea, vomiting, fatigue B) Rash, dyspnea, edema C) Polyuria, thirst, dry skin D) Hunger, dizziness, diaphoresis

The drug Decadron increases glycogenesis. This may lead to hyperglycemia. Therefore the blood sugar level and acetone production must be monitored.

Question10

The nurse is caring for a client with schizophrenia who has been treated with quetiapine (Seroquel) for 1 month. Today the client is increasingly agitated and complains of muscle stiffness. Which of these findings should be reported to the health care provider? A) Elevated temperature and sweating. B) Decreased pulse and blood pressure. C) Mental confusion and general weakness. D) Muscle spasms and seizures. Review Information: The correct answer is A: Elevated temperature and sweating. Neuroleptic malignant syndrome (NMS) is a rare disorder that can occur as a side effect of antipsychotic medications. It is characterized by muscular rigidity, tachycardia, hyperthermia, sweating, altered consciousness, autonomic dysfunction, and increase in CPK. This is a life-threatening complication.

Question11

A child presents to the Emergency Department with documented acetaminophen poisoning. In order to provide counseling and education for the parents, which principle must the nurse unReview Information: The correct answer is A: derstand? Nausea, vomiting, fatigue A) The problem occurs in stages with recovery Side effects of digitalis toxicity include fatigue, within 12-24 hours nausea, vomiting, anorexia, and bradycardia. B) Hepatic problems may occur and may be lifeDigitalis inhibits the sodium potassium ATPase, threatening which makes more calcium available for contrac- C) Full and rapid recovery can be expected in tile proteins, resulting in increased cardiac out- most children put. D) This poisoning is usually fatal, as no antidote is available

Question9

A client is receiving dexamethasone (Decadron) therapy. What should the nurse plan to monitor in this client? A) Urine output every 4 hours B) Blood glucose levels every 12 hours C) Neurological signs every 2 hours D) Oxygen saturation every 8 hours Review Information: The correct answer is B: Blood glucose levels every 12 hours

Review Information: The correct answer is B: Hepatic problems may occur and may be lifethreatening Clinical manifestations associated with acetaminophen poisoning occurs in 4 stages. The third stage is hepatic involvement which may last up to 7 days and be permanent. Clients who do not die in the hepatic stage gradually recover. Collected by :DeepaRajesh [ 99 ] [email protected] Kuwait

Question12

Review Information: The correct answer is C: A client has been receiving dexamethasone Mental status changes (Decadron) for control of cerebral edema. Which Use of serotonergic agents may result in Seof the following assessments would indicate that rotonin Syndrome with confusion, nausea, palpitations, increased muscle tone with twitching the treatment is effective? muscles, and agitation. Serotonin syndrome is A) A positive Babinski’s reflex most often reported in patients taking 2 or more B) Increased response to motor stimuli medications that increase CNS serotonin levels C) A widening pulse pressure by different mechanisms. The most common D) Temperature of 37 degrees Celsius drug combinations associated with serotonin syndrome involve the MAOIs, SSRIs, and the tricyclic antidepressants. Review Information: The correct answer is B: Increased response to motor stimuli Question15 Decadron is a corticosteroid that acts on the cell A client with bi-polar disorder is taking lithium membrane to decrease inflammatory responses (Lithane). What should the nurse emphasize as well as stabilize the blood-brain barrier. Once when teaching about this medication? Decadron reaches a therapeutic level, there A) Take the medication before meals should be a decrease in symptomology with im- B) Maintain adequate daily salt intake provement in motor skills. C) Reduce fluid intake to minimize diuresis D) Use antacids to prevent heartburn

Question13

The provider has ordered transdermal nitroglycerin patches for a client. Which of these instructions should be included when teaching a client about how to use the patches? A) Remove the patch when swimming or bathing B) Apply the patch to any non-hairy area of the body C) Apply a second patch with chest pain D) Remove the patch if ankle edema occurs Review Information: The correct answer is B: Apply the patch to any non-hairy area of the body The patch application sites should be rotated.

Question14

A newly admitted client has a diagnosis of depression. She complains of “twitching muscles” and a “racing heart”, and states she stopped taking Zoloft a few days ago because it was not helping her depression. Instead, she began to take her partner’s Parnate. The nurse should immediately assess for which of these adverse reactions? A) Pulmonary edema B) Atrial fibrillation C) Mental status changes D) Muscle weakness

Review Information: The correct answer is B: Maintain adequate daily salt intake Salt intake affects fluid volume, which can affect lithium (Lithane) levels; therefore, maintaining adequate salt intake is advised.

Question16

A client with anemia has a new prescription for ferrous sulfate. In teaching the client about diet and iron supplements, the nurse should emphasize that absorption of iron is enhanced if taken with which substance? A) Acetaminophen B) Orange juice C) Low fat milk D) An antacid Review Information: The correct answer is B: Orange juice Ascorbic acid enhances the absorption of iron.

Question17

A client with an aplastic sickle cell crisis is receiving a blood transfusion and begins to complain of “feeling hot.” Almost immediately, the client begins to wheeze. What is the nurse’s first action? A) Stop the blood infusion B) Notify the health care provider C) Take/record vital signs D) Send blood samples to lab Collected by :DeepaRajesh [ 100 ] [email protected] Kuwait

Review Information: The correct answer is A: Stop the blood infusion If a reaction of any type is suspected during administration of blood products, stop the infusion immediately, keep the line open with saline, notify the health care provider, monitor vital signs and other changes, and then send a blood sample to the lab.

Question18

A client confides in the RN that a friend has told her the medication she takes for depression, Wellbutrin, was taken off the market because it caused seizures. What is an appropriate response by the nurse? A) “Ask your friend about the source of this information.” B) “Omit the next doses until you talk with the doctor.” C) “There were problems, but the recommended dose is changed.” D) “Your health care provider knows the best drug for your condition.” Review Information: The correct answer is C: “There were problems, but the recommended dose is changed.” Wellbutrin was introduced in the U.S. in 1985 and then withdrawn because of the occurrence of seizures in some patients taking the drug. The drug was reintroduced in 1989 with specific recommendations regarding dose ranges to limit the occurrence of seizures. The risk of seizure appears to be strongly associated with dose.

symptoms resulting from complications due to aspirin ingestion.

Question20

The nurse is caring for a client who is receiving procainamide (Pronestyl) intravenously. It is important for the nurse to monitor which of the following parameters? A) Hourly urinary output B) Serum potassium levels * C) Continuous EKG readings D) Neurological signs Review Information: The correct answer is C: Continuous EKG readings Procainamide (Pronestyl) is used to suppress cardiac arrhythmias. When administered intravenously, it must be accompanied by continuous cardiac monitoring by ECG.

Question21

The nurse is providing education for a client with newly diagnosed tuberculosis. Which statement should be included in the information that is given to the client? A) “Isolate yourself from others until you are finished taking your medication.” B) “Follow up with your primary care provider in 3 months.” C) “Continue to take your medications even when you are feeling fine.” D) “Continue to get yearly tuberculin skin tests.”

Review Information: The correct answer is C: “Continue to take your medications even when you are feeling fine.” Question19 When providing discharge teaching to a client The most important piece of information the tuwith asthma, the nurse will warn against the use berculosis client needs is to understand the imof which of the following over-the-counter medi- portance of medication compliance, even if no longer experiencing symptoms. Clients are most cations? infectious early in the course of therapy. The A) Cortisone ointments for skin rashes numbers of acid-fast bacilli are greatly reduced B) Aspirin products for pain relief as early as 2 weeks after therapy begins. C) Cough medications containing guaifenesin D) Histamine blockers for gastric distress

Question22

Review Information: The correct answer is B: Aspirin products for pain relief Aspirin is known to induce asthma attacks. Aspirin can also cause nasal polyps and rhinitis. Warn individuals with asthma about signs and

The nurse is applying silver sulfadiazine (Silvadene) to a child with severe burns to arms and legs. Which side effect should the nurse be monitoring for? A) Skin discoloration B) Hardened eschar Collected by :DeepaRajesh [ 101 ] [email protected] Kuwait

C) Increased neutrophils D) Urine sulfa crystals Review Information: The correct answer is D: Urine sulfa crystals Silver sulfadiazine is a broad spectrum antimicrobial, especially effective against pseudomonas. When applied to extensive areas, however, it may cause a transient neutropenia, as well as renal function changes with sulfa crystals production and kernicterus.

Question23

The nurse is monitoring a client receiving a thrombolytic agent, alteplase (Activase tissue plasminogen activator), for treatment of a myocardial infarction. What outcome indicates the client is receiving adequate therapy within the first hours of treatment? A) Absence of a dysrhythmia (or arrhythmia) B) Blood pressure reduction C) Cardiac enzymes are within normal limits D) Return of ST segment to baseline on ECG Review Information: The correct answer is D: Return of ST segment to baseline on ECG Improved perfusion should result from this medication, along with the reduction of ST segment elevation.

Question24

The provider has ordered daily high doses of aspirin for a client with rheumatoid arthritis. The nurse instructs the client to discontinue the medication and contact the provider if which of the following symptoms occur? A) Infection of the gums B) Diarrhea for more than one day C) Numbness in the lower extremities D) Ringing in the ears Review Information: The correct answer is D: Ringing in the ears Aspirin stimulates the central nervous system which may result in ringing in the ears. Deglin, J.D. and Vallerand, A.H. (2001). Davis’ drug guide for nurses. (7th edition). Philadelphia: F.A. Davis Company. Key, J.L. and Hayes, E.R. (2003). Pharmacology, a nursing process approach. (4th edition).

Philadelphia: Saunders.

Question25

A nurse is caring for a client who is receiving methyldopa hydrochloride (Aldomet) intravenously. Which of the following assessment findings would indicate to the nurse that the client may be having an adverse reaction to the medication? A) Headache B) Mood changes C) Hyperkalemia D) Palpitations Review Information: The correct answer is B: Mood changes The nurse should assess the client for alterations in mental status such as mood changes. These symptoms should be reported promptly. Deglin, J.D. and Vallerand, A.H. (2001). Davis’ drug guide for nurses. (7th edition). Philadelphia: F.A. Davis Company. Wilson, B.A., Shannon, M.T., and Stang, C.L. (2004). Nurse’s drug guide. Upper Saddle River, New Jersey: Pearson Prentice Hall.

Question26

The nurse is teaching a child and the family about the medication phenytoin (Dilantin) prescribed for seizure control. Which of the following side effects is most likely to occur? A) Vertigo B) Drowsiness C) Gingival hyperplasia D) Vomiting Review Information: The correct answer is C: Gingival hyperplasia Swollen and tender gums occur often with use of phenytoin. Good oral hygiene and regular visits to the dentist should be emphasized.

Question27

The use of atropine for treatment of symptomatic bradycardia is contraindicated for a client with which of the following conditions? A) Urinary incontinence B) Glaucoma C) Increased intracranial pressure D) Right sided heart failure

Collected by :DeepaRajesh [ 102 ] [email protected] Kuwait

Review Information: The correct answer is B: Glaucoma Atropine is contraindicated in clients with angleclosure glaucoma because it can cause pupillary dilation with an increase in aqueous humor, leading to a resultant increase in optic pressure.

Review Information: The correct answer is B: Blood pressure Diltiazem (Cardizem) is a calcium channel blocker that causes systemic vasodilation resulting in decreased blood pressure.

Question28

The nurse is instructing a client with moderate persistent asthma on the proper method for using MDIs (multi-dose inhalers). Which medication should be administered first? A) Steroid B) Anticholinergic C) Mast cell stabilizer D) Beta agonist

A pregnant woman is hospitalized for treatment of pregnancy induced hypertension (PIH) in the third trimester. She is receiving magnesium sulfate intravenously. The nurse understands that this medication is used mainly for what purpose? A) Maintain normal blood pressure B) Prevent convulsive seizures C) Decrease the respiratory rate D) Increase uterine blood flow Review Information: The correct answer is B: Prevent convulsive seizures Magnesium sulfate is a central nervous system depressant. While it has many systemic effects, it is used in the client with pregnancy induced hypertension (PIH) to prevent seizures.

Question29

The nurse is teaching a group of women in a community clinic about prevention of osteoporosis. Which of the following over-the-counter medications should the nurse recognize as having the most elemental calcium per tablet? A) Calcium chloride B) Calcium citrate C) Calcium gluconate D) Calcium carbonate Review Information: The correct answer is D: Calcium carbonate Calcium carbonate contains 400mg of elemental calcium in 1 gram of calcium carbonate.

Question31

Review Information: The correct answer is D: Beta agonist The beta-agonist drugs help to relieve bronchospasm by relaxing the smooth muscle of the airway. These drugs should be taken first so that other medications can reach the lungs.

Question32

A post-operative client has a prescription for acetaminophen with codeine. What should the nurse recognizes as a primary effect of this combination? A) Enhanced pain relief B) Minimized side effects C) Prevention of drug tolerance D) Increased onset of action Review Information: The correct answer is A: Enhanced pain relief Combination of analgesics with different mechanisms of action can afford greater pain relief.

Question33

A client is receiving erythromycin 500mg IV every 6 hours to treat a pneumonia. Which of the following is the most common side effect of the Question30 medication? The nurse is administering diltiazem (Cardizem) A) Blurred vision to a client. Prior to administration, it is important B) Nausea and vomiting for the nurse to assess which parameter? C) Severe headache A) Temperature D) Insomnia B) Blood pressure C) Vision D) Bowel sounds Review Information: The correct answer is B: Collected by :DeepaRajesh [ 103 ] [email protected] Kuwait

Nausea and vomiting Nausea is a common side-effect of erythromycin in both oral and intravenous forms.

C) Lung sounds D) Skin turgor

Review Information: The correct answer is B: The health care provider orders an IV aminophyl- Weight line infusion at 30 mg/hr. The pharmacy sends a Check the client’’s weight because dosage is 1,000 ml bag of D5W containing 500 mg of ami- calculated on the basis of weight. nophylline. In order to administer 30 mg per hour, the RN will set the infusion rate at: Question37 A) 20 ml per hour In providing care for a client with pain from a sickB) 30 ml per hour le cell crisis, which one of the following medicaC) 50 ml per hour tion orders for pain control should be questioned D) 60 ml per hour by the nurse? A) Demerol B) Morphine Review Information: The correct answer is D: C) Methadone 60 ml per hour D) Codeine Using the ratio method to calculate infusion rate: mg to be given (30) : ml to be infused (X) :: mg Review Information: The correct answer is A: available (500) : ml of solution (1,000). Solve for Demerol X by cross-multiplying: 30 x 1,000 = 500 x X (or Meperidine is not recommended in clients with cancel), 30,000 = 500 X, X = 30,000/500, X = sickle cell disease. Normeperidine, a metabolite 60ml per hour. of meperidine, is a central nervous system stimulant that produces anxiety, tremors, myoclonus, and generalized seizures when it accumulates Question35 The nurse is assessing a 7 year-old after several with repetitive dosing. Clients with sickle cell disdays of treatment for a documented strep throat. ease are particularly at risk for normeperidineWhich of the following statements suggests that induced seizures. further teaching is needed? A) “Sometimes I take my medicine with fruit Question38 juice.” A 5 year-old has been rushed to the emergency B) “My mother makes me take my medicine right room several hours after acetaminophen poisonafter school.” ing. Which laboratory result should receive attenC) “Sometimes I take the pills in the morning and tion by the nurse? other times at night.” A) Sedimentation rate D) “I am feeling much better than I did last B) Profile 2 week.” C) Bilirubin D) Neutrophils

Question34

Review Information: The correct answer is C: “Sometimes I take the pills in the morning and other times at night.” Inconsistency in taking the prescribed medication indicates more teaching is needed.

Question36

The nurse is caring for a 10 year-old client who will be placed on heparin therapy. Which assessment is critical for the nurse to make before initiating therapy A) Vital signs B) Weight

Review Information: The correct answer is C: Bilirubin Bilirubin, along with liver enzymes ALT and AST, may rise in the second stage (1-3 days) after a significant overdose, indicating cellular necrosis and liver dysfunction. A prolonged prothrombin time may also be found.

Question39

An elderly client is on an anticholinergic metered dose inhaler (MDI) for chronic obstructive pulmonary disease. The nurse would suggest a spacer Collected by :DeepaRajesh [ 104 ] [email protected] Kuwait

to A) enhance the administration of the medication B) increase client compliance C) improve aerosol delivery in clients who are not able to coordinate the MDI D) prevent exacerbation of COPD Review Information: The correct answer is C: improve aerosol delivery in clients who are not able to coordinate the MDI Spacers improve the medication delivery in clients who are unable to coordinate the movements of administering a dose with an MDI.

Review Information: The correct answer is B: Completing the full course of medications In order for antibiotic therapy to be effective in eradicating an infection, the client must compete the entire course of prescribed therapy. When findings subside, stopping the medication early may lead to recurrence or subsequent drug resistance.

Question2

A 72 year-old client is admitted for possible dehydration. The nurse knows that older adults are particularly at risk for dehydration because they Question40 have The nurse is teaching a parent how to adminis- A) an increased need for extravascular fluid ter oral iron supplements to a 2 year-old child. B) a decreased sensation of thirst Which of the following interventions should be C) an increase in diaphoresis included in the teaching? D) higher metabolic demands A) Stop the medication if the stools become tarry green B) Give the medicine with orange juice and Review Information: The correct answer is B: a decreased sensation of thirst through a straw The elderly have a reduction in thirst sensation C) Add the medicine to a bottle of formula causing them to consume less fluid. Other risk D) Administer the iron with your child’s meals factors may include fear of incontinence, inability to drink fluids independently and lack of motivaReview Information: The correct answer is B: tion. Give the medicine with orange juice and through a straw Absorption of iron is facilitated in an environment Question3 rich in Vitamin C. Since liquid iron preparation A male client is admitted with a spinal cord injury will stain teeth, a straw is preferred. at level C4. The client asks the nurse how the injury is going to affect his sexual function. The 0 comments nurse would respond A) “Normal sexual function is not possible.” Labels: free nclex-rn sample review questions, B) “Sexual functioning will not be impaired at nclex-rn practice test questions, nursing review all.” C) “Erections will be possible.” Free NCLEX-RN Sample Test Ques- D) “Ejaculation will be normal.”

tions For Nursing Review (Pharmacology Set 1) Question1

A client has an order for antibiotic therapy after hospital treatment of a staph infection. Which of the following should the nurse emphasize? A) Scheduling follow-up blood cultures B) Completing the full course of medications C) Visiting the provider in a few weeks D) Monitoring for signs of recurrent infection

Review Information: The correct answer is C: “Erections will be possible.” Because they are a reflex reaction, erections can be stimulated by stroking the genitalia.

Question4 Collected by :DeepaRajesh [ 105 ] [email protected] Kuwait

An 82 year-old client complains of chronic constipation. To improve bowel function, the nurse should first suggest A) Increasing fiber intake to 20-30 grams daily B) Daily use of laxatives C) Avoidance of binding foods such as cheese and chocolate D) Monitoring a balance between activity and rest

turnover.

Question7

You are caring for a client with deep vein thrombosis who is on Heparin IV. The latest APTT is 50 seconds. If the laboratory normal range is 16-24 seconds, you would anticipate A) maintaining the current heparin dose B) increasing the heparin as it does not appear therapeutic. Review Information: The correct answer is A: C) giving protamine sulfate as an antidote. Increasing fiber intake to 20-30 grams daily D) repeating the blood test 1 hour after giving The incorporation of high fiber into the diet is an heparin. effective way to promote bowel elimination in the elderly. Review Information: The correct answer is A: maintaining the current heparin dose Question5 A 4 year-old child is admitted with burns on his The range for a therapeutic APTT is 1.5-2 times legs and lower abdomen. When assessing the the control. Therefore the client is receiving a child’s hydration status, which of the following in- therapeutic dose of Heparin. dicates a less than adequate fluid replacement? A) Decreasing hematocrit and increasing urine volume Question8 B) Rising hematocrit and decreasing urine vol- A client is admitted with a diagnosis of nodal ume bigeminy. The nurse knows that the atriovenC) Falling hematocrit and decreasing urine vol- tricular (AV) node has an intrinsic rate of ume A) 60-100 beats/minute D) Stable hematocrit and increasing urine vol- B) 10-30 beats/minute ume C) 40-70 beats/minute D) 20-50 beats/minute Review Information: The correct answer is B: Rising hematocrit and decreasing urine volume A rising hematocrit indicates a decreased total blood volume, a finding consistent with dehydration.

Review Information: The correct answer is C: 40-70 beats/minute The intrinsic rate of the AV node is within the range of 40-70 beats per minute.

Question6

Question9

Review Information: The correct answer is C: “The cells in the mouth are sensitive to the chemotherapy.” The epithelial cells in the mouth are very sensitive to chemotherapy due to their high rate of cell

Review Information: The correct answer is D: Urine output Potassium chloride should only be administered after adequate urine output (>20cc/hour for 2

A client receiving chemotherapy has developed sores in his mouth. He asks the nurse why this happened. What is the nurse’s best response? A) “It is a sign that the medication is working.” B) “You need to have better oral hygiene.” C) “The cells in the mouth are sensitive to the chemotherapy.” D) “This always happens with chemotherapy.”

A client is to receive 3 doses of potassium chloride 10 mEq in 100cc normal saline to infuse over 30 minutes each. Which of the following is a priority assessment to perform before giving this medication? A) Oral fluid intake B) Bowel sounds C) Grip strength D) Urine output

Collected by :DeepaRajesh [ 106 ] [email protected] Kuwait

consecutive hours) has been established. Impaired ability to excrete potassium via the kidneys can result in hyperkalemia. Review Information: The correct answer is C: Deeply for 3-4 seconds The client should be instructed to breath in the medication for 3-4 seconds in order to receive Question10 The unlicensed assistive personnel (UAP) re- the correct dosage of medication. ports to the nurse that a client with cirrhosis who had a paracentesis yesterday has become more lethargic and has musty smelling breath. A criti- Question13 cal assessment for increasing encephalopathy The nurse is caring for clients over the age of 70. is The nurse knows that due to age-related changA) monitor the client’s clotting status es, the elderly clients tolerate diets that are B) assess upper abdomen for bruits A) high protein C) assess for flap-like tremors of the hands B) high carbohydrates D) measure abdominal girth changes C) low fat D) high calories Review Information: The correct answer is C: assess for flap-like tremors of the hands A client with cirrhosis of the liver who develops subtle changes in mental status and has a musty odor to the breath is at risk for developing more advanced signs of encephalopathy.

Question11

A client is scheduled for an intravenous pyelogram (IVP). After the contrast material is injected, which of the following client reactions should be reported immediately? A) Feeling warm B) Face flushing C) Salty taste D) Hives Review Information: The correct answer is D: Hives This is a sign of anaphylaxis and should be reported immediately. The other reactions are considered normal and the client should be informed that they may occur. .

Question12

A client is prescribed an inhaler. How should the nurse instruct the client to breathe in the medication? A) As quickly as possible B) As slowly as possible C) Deeply for 3-4 seconds D) Until hearing whistling by the spacer

Review Information: The correct answer is C: low fat Due to age related changes, the diet of the elderly should include a lower quantity and higher quality of food. Fewer carbohydrates and fats are required in their diets.

Question14

A woman with a 28 week pregnancy is on the way to the emergency department by ambulance with a tentative diagnosis of abruptio placenta. Which should the nurse do first when the woman arrives? A) administer oxygen by mask at 100% B) start a second IV with an 18 gauge cannula C) check fetal heart rate every 15 minutes D) insert urethral catheter with hourly urine outputs Review Information: The correct answer is A: administer oxygen by mask at 100% Administering oxygen in this situation would increase the circulating oxygen in the mother’s circulation to the fetus’s circulation. This action will minimize complications.

Question15

A client in respiratory distress is admitted with arterial blood gas results of: PH 7.30; PO2 58, PCO2 34; and HCO3 19. The nurse determines that the client is in Collected by :DeepaRajesh [ 107 ] [email protected] Kuwait

A) metabolic acidosis B) metabolic alkalosis C) respiratory acidosis D) respiratory alkalosis

should the nurse base the response on? A) Testicular cancer has a cure rate of 90% with early diagnosis B) Testicular cancer has a cure rate of 50% with early diagnosis C) Intensive chemotherapy is the treatment of Review Information: The correct answer is A: choice metabolic acidosis D) Testicular cancer is usually fatal These lab values indicate metabolic acidosis: the PH is low, PCO2 is normal, and bicarbonate level is low. Review Information: The correct answer is A: Testicular cancer has a cure rate of 90% with early diagnosis Question16 A client is diagnosed with gastroesophageal re- With aggressive treatment and early detection/ flux disease (GERD). The nurse’s instruction to diagnosis the cure rate is 90%. the client regarding diet should be to A) avoid all raw fruits and vegetables Question19 B) increase intake of milk products A client newly diagnosed with Type I Diabetes C) decrease intake of fatty foods Mellitus asks the purpose of the test measurD) focus on 3 average size meals a day ing glycosylated hemoglobin. The nurse should explain that the purpose of this test is to determine: Review Information: The correct answer is C: A) The presence of anemia often associated with decrease intake of fatty foods Diabetes GERD may be aggravated by a fatty diet. A diet B) The oxygen carrying capacity of the client’s low in fat would decrease the symptoms of GERD. red cells Other agents which should also be decreased or C) The average blood glucose for the past 2-3 avoided are: cigarette smoking, caffeine, alco- months hol, chocolate, and meperidine (Demerol). D) The client’s risk for cardiac complications

Question17

After surgery, a client with a nasogastric tube complains of nausea. What action would the nurse take? A) Call the health care provider B) Administer an antiemetic C) Put the bed in Fowler’s position D) Check the patency of the tube Review Information: The correct answer is D: Check the patency of the tube An indication that the nasogastric tube is obstructed is a client’s complaint of nausea. Nasogastric tubes may become obstructed with mucus or sediment.

Question18

A client with testicular cancer has had an orchiectomy. Prior to discharge the client expresses his fears related to his prognosis. Which principle

Review Information: The correct answer is C: The average blood glucose for the past 2-3 months By testing the portion of the hemoglobin that absorbs glucose, it is possible to determine the average blood glucose over the life span of the red cell, 120 days.

Question20

A client is admitted for a possible pacemaker insertion. What is the intrinsic rate of the heart’s own pacemaker? A) 30-50 beats/minute B) 60-100 beats/minute C) 20-60 beats/minute D) 90-100 beats/minute Review Information: The correct answer is B: 60-100 beats/minute This is the intrinsic rate of the SA node. Collected by :DeepaRajesh [ 108 ] [email protected] Kuwait

Question21

The nurse discusses nutrition with a pregnant woman who is iron deficient and follows a vegetarian diet. The selection of which foods indicates the woman has learned sources of iron? A) Cereal and dried fruits B) Whole grains and yellow vegetables C) Leafy green vegetables and oranges D) Fish and dairy products

nated and the client’s depth and rate of respirations will decrease. Therefore the first action should be to lower the oxygen rate.

Question24

The client with goiter is treated with potassium iodide preoperatively. What should the nurse recognize as the purpose of this medication? A) Reduce vascularity of the thyroid B) Correct chronic hyperthyroidism Review Information: The correct answer is A: C) Destroy the thyroid gland function Cereal and dried fruits D) Balance enzymes and electrolytes Both of these foods would be a good source of iron. Review Information: The correct answer is A: Reduce vascularity of the thyroid Potassium iodide solution, or Lugol’’s solution may be used preoperatively to reduce the size Question22 Prior to administering Alteplase (TPA) to a client and vascularity of the thyroid gland. admitted for a cerebral vascular accident (CVA), it is critical that the nurse assess: A) Neuro signs Question25 B) Mental status One hour before the first treatment is scheduled, C) Blood pressure the client becomes anxious and states he does D) PT/PTT not wish to go through with electroconvulsive therapy. Which response by the nurse is most appropriate? Review Information: The correct answer is D: A) “I’ll go with you and will be there with you durPT/PTT ing the treatment.” TPA is a potent thrombolytic enzyme. Because B) “You’ll be asleep and won’t remember anybleeding is the most common side effect, it is thing.” most essential to evaluate clotting studies in- C) “You have the right to change your mind. You cluding PT, PTT, APTT, platelets, and hematocrit seem anxious. Can we talk about it?” before beginning therapy. D) “I’ll call the health care provider to notify them of your decision.”

Question23

The nurse enters the room of a client diagnosed with COPD. The client’s skin is pink, and respirations are 8 per minute. The client’s oxygen is running at 6 liters per minute. What should be the nurse’s first action? A) Call the health care provider B) Put the client in Fowler’s position C) Lower the oxygen rate D) Take the vital signs Review Information: The correct answer is C: Lower the oxygen rate In client’s diagnosed with COPD, the drive to breathe is hypoxia. If oxygen is delivered at too high of a concentration, this drive will be elimi-

Review Information: The correct answer is C: “You have the right to change your mind. You seem anxious. Can we talk about it?” This response indicates acknowledgment of the client’s rights and the opportunity for the client to clarify and ventilate concerns. After this, if the client continues to refuse, the provider should be notified.

Question26

A nurse who has been named in a lawsuit can use which of these factors for the best protection in a court of law? A) Clinical specialty certification in the associated area of practice Collected by :DeepaRajesh [ 109 ] [email protected] Kuwait

B) Documentation on the specific client record with a focus on the nursing process C) Yearly evaluations and proficiency reports prepared by nurse’s manager D) Verification of provider’s orders for the plan of care with identification of outcomes Review Information: The correct answer is B: Documentation on the specific client record with a focus on the nursing process Documentation is the key to protect nurses when a lawsuit is filed. The thorough documentation should include all steps of the nursing process – assessment, analysis, plan, intervention, evaluation. In addition, it should include pertinent data such as times, dosages and sites of actions, assessment data, the nurse’s response to a change in the client’s condition, specific actions taken, if and when the notification occurred to the provider or other health care team members, and what was prescribed along with the client’s outcomes.

Question27

The nurse is caring for clients over the age of 70. The nurse is aware that when giving medications to older clients, it is best to A) start low, go slow B) avoid stopping a medication entirely C) avoid drugs with side effects that impact cognition D) review the drug regimen yearly

C) Avoid the use of salt substitutes D) Take the medication with meals Review Information: The correct answer is C: Avoid the use of salt substitutes Captopril can cause an accumulation of potassium or hyperkalemia. Clients should avoid the use of salt substitutes, which are generally potassium-based.

Question29

A client has bilateral knee pain from osteoarthritis. In addition to taking the prescribed non-steroidal anti-inflammatory drug (NSAID), the nurse should instruct the client to A) start a regular exercise program B) rest the knees as much as possible to decrease inflammation C) avoid foods high in citric acid D) keep the legs elevated when sitting Review Information: The correct answer is A: start a regular exercise program A regular exercise program is beneficial in treating osteoarthritis. It can restore self-esteem and improve physical functioning.

Question30

An arterial blood gases test (ABG) is ordered for a confused client. The respiratory therapist draws the blood and then asks the nurse to apply pressure to the area so the therapist can take the Review Information: The correct answer is A: specimen to the lab. How long should the nurse start low, go slow apply pressure to the area? Due to physiological changes in the elderly, as A) 3 minutes well as conditions such as dehydration, hyper- B) 5 minutes thermia, immobility and liver disease, the effec- C) 8 minutes tive metabolism of drugs may decrease. As a D) 10 minutes result, drugs can accumulate to toxic levels and cause serious adverse reactions. Review Information: The correct answer is B: 5 minutes Question28 It is necessary to apply pressure to the area for You are caring for a hypertensive client with a 5 minutes to prevent bleeding and the formation new order for captopril (Capoten). Which infor- of hematomas. mation should the nurse include in client teaching? A) Avoid green leafy vegetables Question31 B) Restrict fluids to 1000cc/day Which of these clients should the charge nurse Collected by :DeepaRajesh [ 110 ] [email protected] Kuwait

assign to the registered nurse (RN)? A) A 56 year-old with atrial fibrillation receiving digoxin B) A 60 year-old client with COPD on oxygen at 2 L/min C) A 24 year-old post-op client with type 1 diabetes in the process of discharge D) An 80 year-old client recovering 24 hours post right hip replacement

ordered oxygen. The nurse knows that the major reason that oxygen is administered in this situation is to A) saturate the red blood cells B) relieve dyspnea C) decrease cyanosis D) increase oxygen level in the myocardium

Question32

Question35

Review Information: The correct answer is D: Review Information: The correct answer is C: increase oxygen level in the myocardium A 24 year-old post-op client with type 1 diabetes Anoxia of the myocardium occurs in myocardial in the process of discharge infarction. Oxygen administration will help reDischarge teaching must be done by an RN. lieve dyspnea and cyanosis associated with the Practical nurses (PNs) or unlicensed assistive condition but the major purpose is to increase personnel (UAPs) can reinforce education after the oxygen concentration in the damaged myothe RN does the initial teaching. cardial tissue. . A hypertensive client is started on atenolol (Tenormin). The nurse instructs the client to immediately report which of these findings? A) Rapid breathing B) Slow, bounding pulse C) Jaundiced sclera D) Weight gain Review Information: The correct answer is B: Slow, bounding pulse Atenolol (Tenormin) is a beta-blocker that can cause side effects including bradycardia and hypotension.

Question33

An 80 year-old client is admitted with a diagnosis of malnutrition. In addition to physical assessments, which of the following lab tests should be closely monitored? A) Urine protein B) Urine creatinine C) Serum calcium D) Serum albumin Review Information: The correct answer is D: Serum albumin Serum albumin is a valuable indicator of protein deficiency and, later, nutritional status in adults. A normal reading for an elder’s serum albumin is between 3.0-5.0 g/dl.

Question34

Upon admission to an intensive care unit, a client diagnosed with an acute myocardial infarction is

The nurse is teaching a client with chronic renal failure (CRF) about medications. The client questions the purpose of aluminum hydroxide (Amphojel) in her medication regimen. What is the best explanation for the nurse to give the client about the therapeutic effects of this medication? A) It decreases serum phosphate B) It will reduce serum calcium C) Amphojel increases urine output D) The drug is taken to control gastric acid secretion Review Information: The correct answer is A: It decreases serum phosphate Aluminum binds phosphates that tend to accumulate in the patient with chronic renal failure due to decreased filtration capacity of the kidney. Antacids such as Amphojel are commonly used to accomplish this.

Question36

A 66 year-old client is admitted for mitral valve replacement surgery. The client has a history of mitral valve regurgitation and mitral stenosis since her teenage years. During the admission assessment, the nurse should ask the client if as a child she had A) measles B) rheumatic fever C) hay fever D) encephalitis Collected by :DeepaRajesh [ 111 ] [email protected] Kuwait

Review Information: The correct answer is B: rheumatic fever Clients that present with mitral stenosis often have a history of rheumatic fever or bacterial endocarditis.

A) Assess the pulse rate q 4 hours B) Monitor her level of consciousness q shift C) Test her stools for occult blood D) Discuss fiber in the diet to prevent constipation

Question37

Review Information: The correct answer is C: Test her stools for occult blood Both Prednisone and ASA can lead to GI bleeding, therefore monitoring for occult blood would be appropriate.

During nursing rounds which of these assessments would require immediate corrective action and further instruction to the practical nurse (PN) about proper care? A) The weights of the skin traction of a client are hanging about 2 inches from the floor B) A client with a hip prosthesis 1 day post operatively is lying in bed with internal rotation and adduction of the affected leg C) The nurse observes that the PN moves the extremity of a client with an external fixation device by picking up the frame D) A client with skeletal traction states “The other nurse said that the clear, yellow and crusty drainage around the pin site is a good sign” Review Information: The correct answer is B: A client with a hip prosthesis 1 day post operatively is lying in bed with internal rotation and adduction of the affected leg This position should be prevented in order to prevent dislodgment of the hip prosthesis, especially in the first 48 to 72 hours post-op. The other assessments are not of concern.

Question40

A client with testicular cancer is scheduled for a right orchiectomy. The nurse knows that an orchiectomy is the A) surgical removal of the entire scrotum B) surgical removal of a testicle C) dissection of related lymph nodes D) partial surgical removal of the penis Review Information: The correct answer is B: surgical removal of a testicle The affected testicle is surgically removed along with its tunica and spermatic cord. 0 comments Labels: free nclex-rn sample review questions, nclex-rn practice test questions, nursing review

Question38

Free NCLEX-RN Sample Test QuesA client diagnosed with gouty arthritis is admit- tions For Nursing Review (Part 5) ted with severe pain and edema in the right foot. When the nurse develops a plan of care, which intervention should be included? A) high protein diet B) salicylates C) hot compresses to affected joints D) intake of at least 3000cc/day

Review Information: The correct answer is D: intake of at least 3000cc/day Fluid intake should be increased to prevent precipitation of urate in the kidneys.

Question39

A 55 year-old woman is taking Prednisone and aspirin (ASA) as part of her treatment for rheumatoid arthritis. Which of the following would be an appropriate intervention for the nurse?

Question1

A client complains of some discomfort after a below the knee amputation. Which action by the nurse is most appropriate initially? A) Conduct guided imagery or distraction B) Ensure that the stump is elevated the first day post-op C) Wrap the stump snugly in an elastic bandage D) Administer opioid narcotics as ordered Review Information: The correct answer is B: Ensure that the stump is elevated the first day post-op This priority intervention prevents pressure caused by pooling of blood, thus minimizing the pain. Without this measure, a firm elastic bandage, opioid narcotics, or guided imagery will have little effect. Opioid narcotics are given for Collected by :DeepaRajesh [ 112 ] [email protected] Kuwait

severe pain.

Question2

A 78 year-old client with pneumonia has a productive cough, but is confused. Safety protective devices (restraints) have been ordered for this client. How can the nurse prevent aspiration? A) Suction the client frequently while restrained B) Secure all 4 restraints to 1 side of bed C) Obtain a sitter for the client while restrained D) Request an order for a cough suppressant Review Information: The correct answer is C: Obtain a sitter for the client while restrained The plan to use safety devices (restraints) should be rethought. Restraints are used to protect the client from harm caused by removing tubes or getting out of bed. In the event that this restricted movement could cause more harm, such as aspiration, then a sitter should be requested. These are to be provided by the facility in the event the family cannot do so. This client needs to cough and be watched rather than restricted. Suctioning will not prevent aspiration in this situation. Cough suppressants should be avoided for this client.

ing D) Avoid touching the neonate with cold hands Review Information: The correct answer is B: Monitor the neonate’s temperature When using a warming device the neonate’s temperature should be continuously monitored for undesired elevations. The use of heat lamps is not safe as there is no way to regulate their temperature. Warming medications and fluids is not indicated. While touching with cold hands can startle the infant it does not pose a safety risk.

Question5

Which oxygen delivery system would the nurse apply that would provide the highest concentrations of oxygen to the client? A) Venturi mask B) Partial rebreather mask C) Non-rebreather mask D) Simple face mask

Review Information: The correct answer is C: Non-rebreather mask The non-rebreather mask has a one-way valve that prevents exhales air from entering the reservoir bag and one or more valves covering the air holes on the face mask itself to prevent inhaQuestion3 A couple trying to conceive asks the nurse when lation of room air but to allow exhalation of air. ovulation occurs. The woman reports a regular When a tight seal is achieved around the mask 32 day cycle. Which response by the nurse is up to 100% of the oxygen is available. correct? A) Days 7-10 Question6 B) Days 10-13 At a senior citizens meeting a nurse talks with a client who has Type 1 diabetes mellitus. Which C) Days 14-16 D) Days 17-19 statement by the client during the conversation is most predictive of a potential for impaired skin Review Information: The correct answer is D: integrity? Days 17-19 A) “I give my insulin to myself in my thighs.” Ovulation occurs 14 days prior to menses. Con- B) “Sometimes when I put my shoes on I don’t sidering that the woman’’s cycle is 32 days, sub- know where my toes are.” tracting 14 from 32 suggests ovulation is at about C) “Here are my up and down glucose readings the 18th day. that I wrote on my calendar.” D) “If I bathe more than once a week my skin feels too dry.” Question4 A newborn is having difficulty maintaining a temperature above 98 degrees Fahrenheit and has Review Information: The correct answer is been placed in an incubator. Which action is a B: “Sometimes when I put my shoes on I don’’t know where my toes are.” nursing priority? A) Protect the eyes of the neonate from the heat Peripheral neuropathy can lead to lack of sensation in the lower extremities. Clients who do not lamp feel pressure and/or pain are at high risk for skin B) Monitor the neonate’s temperature C) Warm all medications and liquids before giv- impairment. Collected by :DeepaRajesh [ 113 ] [email protected] Kuwait

Question7

60 year-old with herpes simplex A client returns from surgery after an open reduc- Clients who have herpes simplex infections must tion of a femur fracture. There is a small blood- have contact precautions in addition to standard stain on the cast. Four hours later, the nurse ob- precautions because of the associated, potentialserves that the stain has doubled in size. What is ly weeping, skin lesions. Contact precautions are used for clients who are infected by microorganthe best action for the nurse to take? isms that are transmitted by direct contact with A) Call the health care provider B) Access the site by cutting a window in the the client, including hand or skin-to-skin contact. cast C) Simply record the findings in the nurse’s notes Question10 only Which of the following situations is most likely to D) Outline the spot with a pencil and note the produce sepsis in the neonate? time and date on the cast A) Maternal diabetes B) Prolonged rupture of membranes C) Cesarean delivery Review Information: The correct answer is D: D) Precipitous vaginal birth Outline the spot with a pencil and note the time and date on the cast Review Information: The correct answer is B: This is a good way to assess the amount of bleed- Prolonged rupture of membranes ing over a period of time. The bleeding does not Premature rupture of the membranes (PROM) is appear to be excessive and some bleeding is a leading cause of newborn sepsis. After 12-24 expected with this type of surgery. The bleed- hours of leaking fluid, measures are taken to reing should also be documented in the nurse’s duce the risk to mother and the fetus/newborn. notes.

Question8

The nurse is caring for a 1 year-old child who has 6 teeth. What is the best way for the nurse to give mouth care to this child? A) Using a moist soft brush or cloth to clean teeth and gums B) Swabbing teeth and gums with flavored mouthwash C) Offering a bottle of water for the child to drink D) Brushing with toothpaste and flossing each tooth

Question11

The nurse is teaching a parent about side effects of routine immunizations. Which of the following must be reported immediately? A) Irritability B) Slight edema at site C) Local tenderness D) Seizure activity

Review Information: The correct answer is D: Seizure activity Other reactions that should be reported include Review Information: The correct answer is A: crying for >3 hours, temperature over 104.8 deUsing a moist soft brush or cloth to clean teeth grees Fahrenheit following DPT immunization, and gums and tender, swollen, reddened areas. The nurse should use a soft cloth or soft brush to do mouth care so that the child can adjust to the Question12 routine of cleaning the mouth and teeth. The nurse is at the community center speaking with retired people about glaucoma. Which comQuestion9 ment by one of the retirees would the nurse supIn addition to standard precautions, a nurse port to reinforce correct information? should implement contact precautions for which A) “I usually avoid driving at night since lights client? sometimes seem to make things blur.” A) 60 year-old with herpes simplex B) “I take half of the usual dose for my sinuses to B) 6 year-old with mononucleosis maintain my blood pressure.” C) 45 year-old with pneumonia C) “I have to sit at the side of the pool with the D) 3 year-old with scarlet fever grandchildren since I can’t swim with this eye problem.” Review Information: The correct answer is A: D) “I take extra fiber and drink lots of water to Collected by :DeepaRajesh [ 114 ] [email protected] Kuwait

avoid getting constipated.”

ing old ones will ensure that the tracheostomy is secured during the entire cleaning procedure. Review Information: The correct answer is D: The obturator is useful to keep the airway open “I take extra fiber and drink lots of water to avoid only after the tracheostomy outer tube is coughed getting constipated.” out. A second nurse is not needed. Changing the Any activity that involves straining should be position may not prevent a dislodged tracheosavoided in clients with glaucoma. Such activities tomy. would increase intraocular pressure.

Question13

A newborn has hyperbilirubinemia and is undergoing phototherapy with a fiberoptic blanket. Which safety measure is most important during this process? A) Regulate the neonate’s temperature using a radiant heater B) Withhold feedings while under the phototherapy C) Provide water feedings at least every 2 hours D) Protect the eyes of neonate from the phototherapy lights Review Information: The correct answer is C: Provide water feedings at least every 2 hours Protecting the eyes of the neonates is very important to prevent damage when under the ultraviolet lights, but since the blanket is used, extra protection of the eyes is unnecessary. It is recommended that the neonate remain under the lights for extended periods. The neonate’s skin is exposed to the light and the temperature is monitored, but a heater may not be necessary. There is no reason to withhold feedings. Frequent water or feedings are given to help with the excretion of the bilirubin in the stool. .

Question14

A nurse is performing the routine daily cleaning of a tracheostomy. During the procedure, the client coughs and displaces the tracheostomy tube. This negative outcome could have avoided by A) placing an obturator at the client’s bedside B) having another nurse assist with the procedure C) fastening clean tracheostomy ties before removing old ties D) placing the client in a flat, supine position Review Information: The correct answer is C: fastening clean tracheostomy ties before removing old ties Fastening clean tracheostomy ties before remov-

Question15

A 4 year-old hospitalized child begins to have a seizure while playing with hard plastic toys in the hallway. Of the following nursing actions, which one should the nurse do first? A) Place the child in the nearest bed B) Administer IV medication to slow down the seizure C) Place a padded tongue blade in the child’s mouth D) Remove the child’s toys from the immediate area Review Information: The correct answer is D: Remove the child’’s toys from the immediate area Nursing care for a child having a seizure includes, maintaining airway patency, ensuring safety, administering medications, and providing emotional support. Since the seizure has already started, nothing should be forced into the child’’s mouth and the child should not be moved. Of the choices given, the first priority would be to provide a safe environment.

Question16

The nurse is teaching home care to the parents of a child with acute spasmodic croup. The most important aspects of this care is/are A) sedation as needed to prevent exhaustion B) antibiotic therapy for 10 to 14 days C) humidified air and increased oral fluids D) antihistamines to decrease allergic response Review Information: The correct answer is C: humidified air and increased oral fluids The most important aspects of home care for a child with acute spasmodic croup are humidified air and increased oral fluids. Moisture soothes inflamed membranes. Adequate systemic hydration aids is mucociliary clearance and keeps secretions thin, white, watery, and easily removed with minimal coughing.

Collected by :DeepaRajesh [ 115 ] [email protected] Kuwait

Question17

The nurse is assigned to care for a client who has a leaking intracranial aneurysm. To minimize the risk of rebleeding, the nurse should plan to A) restrict visitors to immediate family B) avoid arousal of the client except for family visits C) keep client’s hips flexed at no less than 90 degrees D) apply a warming blanket for temperatures of 98 degrees Fahrenheit or less

Question19

A newborn delivered at home without a birth attendant is admitted to the hospital for observation. The initial temperature is 95 degrees Fahrenheit (35 degrees Celsius) axillary. The nurse recognizes that cold stress may lead to what complication? A) Lowered BMR B) Reduced PaO2 C) Lethargy Review Information: The correct answer is A: D) Metabolic alkalosis restrict visitors to immediate family Maintaining a quiet environment will assist in Review Information: The correct answer is B: minimizing cerebral rebleeding. When family vis- Reduced PaO2 it, the client should not be disturbed. If the client Cold stress causes increased risk for respirais awake, topics of a general nature are better tory distress. The baby delivered in such circumchoices for discussion than topics that result in stances needs careful monitoring. In this situaemotional or physiological stimulation. tion, the newborn must be warmed immediately to increase its temperature to at least 97 degrees Fahrenheit (36 degrees Celsius). Question18 A client who is 12 hour post-op becomes confused and says: “Giant sharks are swimming across the ceiling.” Which assessment is necessary to adequately identify the source of this client’s behavior? A) Cardiac rhythm strip B) Pupillary response C) Pulse oximetry D) Peripheral glucose stick

Review Information: The correct answer is C: Pulse oximetry A sudden change in mental status in any postop client should trigger a nursing intervention directed toward respiratory evaluation. Pulse oximetry would be the initial assessment. If available, arterial blood gases would be better. Acute respiratory failure is the sudden inability of the respiratory system to maintain adequate gas exchange which may result in hypercapnia and/ or hypoxemia. Clinical findings of hypoxemia include these finding which are listed in order of initial to later findings: restlessness, irritability, agitation, dyspnea, disorientation, confusion, delirium, hallucinations, and loss of consciousness. While there may be other factors influencing the client’’s behavior, the first nursing action should be directed toward maintaining oxygenation. Once respiratory or oxygenation issues are ruled out then significant changes in glucose would be evaluated.

Question20

Which contraindication should the nurse assess for prior to giving a child immunizations? A) Mild cold symptoms B) Chronic asthma C) Depressed immune system D) Allergy to eggs Review Information: The correct answer is C: Depressed immune system Children who have a depressed immune system related to HIV or chemotherapy should not be given routine immunizations.

Question21

The nurse is caring for a client with a myocardial infarction. Which finding requires the nurse’s immediate action? A) Periorbital edema B) Dizzy spells C) Lethargy D) Shortness of breath Review Information: The correct answer is B: Dizzy spells Cardiac dysrhythmias may cause a transient drop in cardiac output and decreased blood flow to the brain. Near syncope refers to lightheartedness, dizziness, temporary confusion. Such “spells” may indicate runs of ventricular tachyCollected by :DeepaRajesh [ 116 ] [email protected] Kuwait

cardia or periods of asystole and should be reported immediately.

Question22

Decentralized scheduling is used on a nursing unit. A chief advantage of this management strategy is that it: A) considers client and staff needs B) conserves time spent on planning C) frees the nurse manager to handle other priorities D) allows requests for special privileges

Question25

A 16 year-old boy is admitted for Ewing’s sarcoma of the tibia. In discussing his care with the parents, the nurse understands that the initial treatment most often includes A) amputation just above the tumor B) surgical excision of the mass C) bone marrow graft in the affected leg D) radiation and chemotherapy

Review Information: The correct answer is D: radiation and chemotherapy Review Information: The correct answer is A: The initial treatment of choice for Ewing’’s sarconsiders client and staff needs coma is a combination of radiation and chemoDecentralized staffing takes into consideration therapy. specific client needs and staff interests and abilities. Question26 Question23 A new nurse manager is responsible for interIncluded in teaching the client with tuberculo- viewing applicants for a staff nurse position. sis taking isoniazid (INH) about follow-up home Which interview strategy would be the best apcare, the nurse should emphasize that a labora- proach? tory appointment for which of the following lab A) Vary the interview style for each candidate to tests is critical? learn different techniques A) Liver function B) Use simple questions requiring “yes” and “no” B) Kidney function answers to gain definitive information C) Blood sugar C) Obtain an interview guide from human reD) Cardiac enzymes sources for consistency in interviewing each candidate Review Information: The correct answer is A: D) Ask personal information of each applicant to Liver function assure he/she can meet job demands INH can cause hepatocellular injury and hepatitis. This side effect is age-related and can be detected with regular assessment of liver enzymes, Review Information: The correct answer is C: which are released into the blood from damaged Obtain an interview guide from human resources liver cells. for consistency in interviewing each candidate An interview guide used for each candidate enables the nurse manager to be more objective in Question24 A woman in her third trimester complains of se- the decision making. The nurse should use revere heartburn. What is appropriate teaching sources available in the agency before attempts by the nurse to help the woman alleviate these to develop one from scratch. Certain personal questions are prohibited, and HR can identify symptoms? these for novice managers. A) Drink small amounts of liquids frequently B) Eat the evening meal just before retiring C) Take sodium bicarbonate after each meal D) Sleep with head propped on several pillows

Review Information: The correct answer is D: Sleep with head propped on several pillows Heartburn is a burning sensation caused by regurgitation of gastric contents. It is best relieved by sleeping position, eating small meals, and not eating before bedtime.

Question27

What is the best way that parents of pre-schoolers can begin teaching their child about injury prevention? A) Set good examples themselves B) Protect their child from outside influences C) Make sure their child understands all the safety rules D) Discuss the consequences of not wearing Collected by :DeepaRajesh [ 117 ] [email protected] Kuwait

protective devices

causes. During the process of taking client history, which of these items should the nurse identify as related to the client’s greatest risk factors Review Information: The correct answer is A: for osteoporosis? Set good examples themselves A) History of menopause at age 50 The preschool years are the time for parents to B) Taking high doses of steroids for arthritis for begin emphasizing safety principles as well as many years providing protection. Setting a good example C) Maintaining an inactive lifestyle for the past themselves is crucial because of the imitative 10 years behaviors of pre-schoolers; they are quick to no- D) Drinking 2 glasses of red wine each day for tice discrepancies between what they see and the past 30 years what they are told. Review Information: The correct answer is A nurse assessing the newborn of a mother with B: Taking high doses of steroids for arthritis for diabetes understands that hypoglycemia is re- many years The use of steroids, especially at high doses over lated to what pathophysiological process? time, increases the risk for osteoporosis. The A) Disruption of fetal glucose supply other options also predispose to osteoporosis, B) Pancreatic insufficiency as do low bone mass, poor calcium absorption C) Maternal insulin dependency and moderate to high alcohol ingestion. LongD) Reduced glycogen reserves term steroid treatment is the most significant risk factor, however. Review Information: The correct answer is A: Disruption of fetal glucose supply After delivery, the high glucose levels which crossed the placenta to the fetus are suddenly Question31 stopped. The newborn continues to secrete insu- The nurse is caring for a 2 year-old who is being lin in anticipation of glucose. When oral feedings treated with chelation therapy, calcium disodium begin, the newborn will adjust insulin production edetate, for lead poisoning. The nurse should be alert for which of the following side effects? within a day or two. A) Neurotoxicity B) Hepatomegaly C) Nephrotoxicity Question29 D) Ototoxicity The nurse is caring for a client with extracellular fluid volume deficit. Which of the following assessments would the nurse anticipate finding? Review Information: The correct answer is C: A) bounding pulse Nephrotoxicity B) rapid respirations Nephrotoxicity is a common side effect of calciC) oliguria um disodium edetate, in addition to lead poisonD) neck veins are distended ing in general.

Question28

Review Information: The correct answer is C: oliguria Kidneys maintain fluid volume through adjustments in urine volume.

Question30

A 70 year-old woman is evaluated in the emergency department for a wrist fracture of unknown

Question32

The parents of a toddler ask the nurse how long their child will have to sit in a car seat while in the automobile. What is the nurse’s best response to the parents? A) “Your child must use a care seat until he weighs at least 40 pounds.” B) “The child must be 5 years of age to use a regular seat belt.” C) “Your child must reach a height of 50 inches Collected by :DeepaRajesh [ 118 ] [email protected] Kuwait

to sit in a seat belt.” D) “The child can use a regular seat belt when he can sit still.”

A) decrease the client’s discomfort B) reduce viscosity of secretions C) prevent client aspiration D) remove a mucus plug

Review Information: The correct answer is A: “Your child must use a care seat until he weighs at least 40 pounds.” Children should use car seats until they weigh 40 pounds.

Review Information: The correct answer is D: remove a mucus plug While no longer recommended for routine suctioning, saline may thin and loosen viscous secretions that are very difficult to move, perhaps making them easier to suction.

Question33

A client asks the nurse to explain the basic ideas of homeopathic medicine. The response that best explains this approach is that such remedies A) destroy organisms causing disease B) maintain fluid balance C) boost the immune system D) increase bodily energy

Question36

The nurse is performing a gestational age assessment on a newborn delivered 2 hours ago. When coming to a conclusion using the Ballard scale, which of these factors may affect the score? A) Birth weight B) Racial differences C) Fetal distress in labor D) Birth trauma

Review Information: The correct answer is C: boost the immune system The practitioner treats with minute doses of plant, mineral or animal substances which provide a gentle stimulus to the body’’s own defenses.

Review Information: The correct answer is C: Fetal distress in labor The effects of earlier distress may alter the findings of reflex responses as measured on the Ballard tool. Other physical characteristics that estiQuestion34 A client with a fractured femur has been in Rus- mate gestational age, such as amount of lanugo, sell’s traction for 24 hours. Which nursing action sole creases and ear cartilage are unaffected by the other factors. is associated with this therapy? A) Check the skin on the sacrum for breakdown B) Inspect the pin site for signs of infection C) Auscultate the lungs for atelectasis D) Perform a neurovascular check for circulation

Question37

A nurse is caring for a client who had a closed reduction of a fractured right wrist followed by the application of a fiberglass cast 12 hours ago. Which finding requires the nurse’s immediate atReview Information: The correct answer is D: tention? A) Capillary refill of fingers on right hand is 3 secPerform a neurovascular check for circulation While each of these is an important assessment, onds the neurovascular integrity check is most associ- B) Skin warm to touch and normally colored ated with this type of traction. Russell’s traction C) Client reports prickling sensation in the right hand is Buck’s traction with a sling under the knee. D) Slight swelling of fingers of right hand

Question35

When suctioning a client’s tracheostomy, the nurse should instill saline in order to

Review Information: The correct answer is C: Client reports prickling sensation in the right Collected by :DeepaRajesh [ 119 ] [email protected] Kuwait

hand A prickling sensation is an indication of compartment syndrome and requires immediate action by the nurse. The other findings are normal for a client in this situation.

Question38

A client is admitted with the diagnosis of pulmonary embolism. While taking a history, the client tells the nurse he was admitted for the same thing twice before, the last time just 3 months ago. The nurse would anticipate the provider ordering A) pulmonary embolectomy B) vena caval interruption C) increasing the Coumadin therapy to an INR of 3-4 D) thrombolytic therapy Review Information: The correct answer is B: vena caval interruption Clients with contraindications to Heparin, recurrent PE or those with complications related to the medical therapy may require vena caval interruption by the placement of a filter device in the inferior vena cava. A filter can be placed transvenously to trap clots before they travel to the pulmonary circulation.

Question39

Which client is at highest risk for developing a pressure ulcer? A) 23 year-old in traction for fractured femur B) 72 year-old with peripheral vascular disease, who is unable to walk without assistance C) 75 year-old with left sided paresthesia who is incontinent of urine and stool D) 30 year-old who is comatose following a ruptured aneurysm

Question40

The nurse is teaching the mother of a 5 monthold about nutrition for her baby. Which statement by the mother indicates the need for further teaching? A) “I’m going to try feeding my baby some rice cereal.” B) “When he wakes at night for a bottle, I feed him.” C) “I dip his pacifier in honey so he’ll take it.” D) “I keep formula in the refrigerator for 24 hours.” Review Information: The correct answer is C: “I dip his pacifier in honey so he’’ll take it.” Honey has been associated with infant botulism and should be avoided. Older children and adults have digestive enzymes that kill the botulism spores. 0 comments Labels: free nclex-rn sample review questions, nclex-rn practice test questions, nursing review

Free NCLEX-RN Sample Test Questions For Nursing Review (Part 4) Question1

The clinic nurse is counseling a substance-abusing post partum client on the risks of continued cocaine use. In order to provide continuity of care, which nursing diagnosis is a priority? A) Social isolation B) Ineffective coping C) Altered parenting D) Sexual dysfunction Review Information: The correct answer is C: Altered parenting The cocaine abusing mother puts her newborn and other children at risk for neglect and abuse. Continuing to use drugs has the potential to impact parenting behaviors. Social service referrals are indicated.

Review Information: The correct answer is C: 75 year-old with left sided paresthesia who is incontinent of urine and stool Risk factors for pressure ulcers include: immobility, absence of sensation, decreased LOC, poor nutrition and hydration, skin moisture, incontinence, increased age, decreased immune response. This client has the greatest number of Question2 risk factors. The nurse is teaching about nonsteroidal antiinflammatory drugs (NSAIDs) to a group of arCollected by :DeepaRajesh [ 120 ] [email protected] Kuwait

thritic clients. To minimize the side effects, the nurse should emphasize which of the following actions? A) Reporting joint stiffness in the morning B) Taking the medication 1 hour before or 2 hours after meals C) Using alcohol in moderation unless driving D) Continuing to take aspirin for short term relief

A) 1 year of age B) 2 years of age C) 3 years of age D) 4 years of age Review Information: The correct answer is B: 2 years of age A child should be at least 2 years of age to use the radial pulse to assess heart rate.

Question5 Review Information: The correct answer is B: Taking the medication 1 hour before or 2 hours after meals Taking the medication 1 hour before or 2 hours after meals will result in a more rapid effect.

Question3

The nurse is preparing to administer a tube feeding to a postoperative client. To accurately assess for a gastrostomy tube placement, the priority is to A) auscultate the abdomen while instilling 10 cc of air into the tube B) place the end of the tube in water to check for air bubbles C) retract the tube several inches to check for resistance D) measure the length of tubing from nose to epigastrium

A client is receiving Total Parenteral Nutrition (TPN) via a Hickman catheter. The catheter accidentally becomes dislodged from the site. Which action by the nurse should take priority? A) Check that the catheter tip is intact B) Apply a pressure dressing to the site C) Monitor respiratory status D) Assess for mental status changes

Question6

A pregnant client who is at 34 weeks gestation is diagnosed with a pulmonary embolism (PE). Which of these medications would the nurse anticipate the provider ordering? A) Oral Coumadin therapy B) Heparin 5000 units subcutaneously B.I.D. C) Heparin infusion to maintain the PTT at 1.52.5 times the control value D) Heparin by subcutaneous injection to maintain the PTT at 1.5 times the control value

Review Information: The correct answer is A: auscultate the abdomen while instilling 10 cc of air into the tube If a swoosh of air is heard over the abdominal cavity while instilling air into the gastric tube, this indicates that it is accurately placed in the stomach. The feeding can begin after further assessing the client for bowel sounds.

Review Information: The correct answer is D: Heparin by subcutaneous injection to maintain the PTT at 1.5 times the control value Several studies have been conducted in pregnant women where oral anticoagulation agents are contraindicated. Warfarin (Coumadin) is known to cross the placenta and is therefore reported to be teratogenic.

Question4

Question7

While assessing the vital signs in children, the nurse should know that the apical heart rate is preferred until the radial pulse can be accurately assessed at about what age?

The nurse is caring for a client with Hodgkin’s disease who will be receiving radiation therapy. The nurse recognizes that, as a result of the radiation therapy, the client is most likely to experiCollected by :DeepaRajesh [ 121 ] [email protected] Kuwait

ence A) high fever B) nausea C) face and neck edema D) night sweats Review Information: The correct answer is B: nausea Because the client with Hodgkin’’s disease is usually healthy when therapy begins, the nausea is especially troubling. .

Question8

Review Information: The correct answer is B: The MMR vaccine should be given now, prior to the transplant MMR is a live virus vaccine, and should be given at this time. Post-transplant, immunosuppressive drugs will be given and the administration of the live vaccine at that time would be contraindicated because of the compromised immune system.

Question10

A client is brought to the emergency room following a motor vehicle accident. When assessing the client one-half hour after admission, the nurse notes several physical changes. Which finding would require the nurse’s immediate attention? A) increased restlessness B) tachycardia C) tracheal deviation D) tachypnea

The nurse is preparing to take a toddler’s blood pressure for the first time. Which of the following actions should the nurse perform first? A) Explain that the procedure will help him to get well B) Show a cartoon character with a blood pressure cuff C) Explain that the blood pressure checks the heart pump D) Permit handling the equipment before putting the cuff in place

Review Information: The correct answer is C: tracheal deviation The deviated trachea is a sign that a mediastinal shift has occurred. This is a medical emergency.

Review Information: The correct answer is D: Permit handling the equipment before putting the cuff in place The best way to gain the toddler’’s cooperation is to encourage handling the equipment. Detailed explanations are not helpful.

Question9

Question11

An 18 month-old child is on peritoneal dialysis in preparation for a renal transplant in the near future. When the nurse obtains the child’s health history, the mother indicates that the child has not had the first measles, mumps, rubella (MMR) immunization. The nurse understands that which of the following is true in regards to giving immunizations to this child? A) Live vaccines are withheld in children with renal chronic illness B) The MMR vaccine should be given now, prior to the transplant C) An inactivated form of the vaccine can be given at any time D) The risk of vaccine side effects precludes giving the vaccine

Which statement made by a nurse about the goal of total quality management or continuous quality improvement in a health care setting is correct? A) It is to observe reactive service and product problem solving B) Improvement of the processes in a proactive, preventive mode is paramount C) A chart audits to finds common errors in practice and outcomes associated with goals D) A flow chart to organize daily tasks is critical to the initial stages

Review Information: The correct answer is Collected by :DeepaRajesh [ 122 ] [email protected] Kuwait

B: Improvement of the processes in a proactive, preventive mode is paramount Total quality management and continuous quality improvement have a major goal of identifying ways to do the right thing at the right time in the right way by proactive problem-solving.

Question12

Which of the following drugs should the nurse anticipate administering to a client before they are to receive electroconvulsive therapy? A) Benzodiazepines B) Chlorpromazine (Thorazine) C) Succinylcholine (Anectine) D) Thiopental sodium (Pentothal Sodium)

ease (COPD) and a history of coronary artery disease is receiving aminophylline, 25mg/hour. Which one of the following findings by the nurse would require immediate intervention? A) Decreased blood pressure and respirations B) Flushing and headache C) Restlessness and palpitations D) Increased heart rate and blood pressure

Review Information: The correct answer is C: Restlessness and palpitations Side effects of Aminophylline include restlessness and palpitations.

Question15 Review Information: The correct answer is C: Succinylcholine (Anectine) Succinylcholine is given intravenously to promote skeletal muscle relaxation.

Question13

Which approach is a priority for the nurse who works with clients from many different cultures? A) Speak at least 2 other languages of clients in the neighborhood B) Learn about the cultures of clients who are most often encountered C) Have a list of persons for referral when interaction with these clients occur D) Recognize personal attitudes about cultural differences and real or expected biases

Review Information: The correct answer is D: Recognize personal attitudes about cultural differences and real or expected biases The nurse must discover personal attitudes, prejudices and biases specific to different cultures. Awareness of these will prevent negative consequences for interactions with clients and families across cultures.

Question14

A client has gastroesophageal reflux. Which recommendation made by the nurse would be most helpful to the client? A) Avoid liquids unless a thickening agent is used B) Sit upright for at least 1 hour after eating C) Maintain a diet of soft foods and cooked vegetables D) Avoid eating 2 hours before going to sleep

Review Information: The correct answer is D: Avoid eating 2 hours before going to sleep Eating before sleeping enhances the regurgitation of stomach contents, which have increased acidity, into the esophagus. An upright posture should be maintained for about 2 hours after eating to allow for the stomach emptying. Options A and C are interventions for clients with swallowing difficulties. .

Question16

A client with a panic disorder has a new prescription for Xanax (alprazolam). In teaching the client about the drug’s actions and side effects, which of the following should the nurse emphasize? A) Short-term relief can be expected B) The medication acts as a stimulant C) Dosage will be increased as tolerated D) Initial side effects often continue

A client with chronic obstructive pulmonary disCollected by :DeepaRajesh [ 123 ] [email protected] Kuwait

Review Information: The correct answer is D: ask the client to talk about concerns regarding “hot” treatments The “hot-cold” system is found among MexicanAmericans, Puerto Ricans, and other Hispanic-Latinos. Most foods, beverages, herbs, and medicines are categorized as hot or cold, which are symbolic designations and do not necessarQuestion17 A client being discharged from the cardiac step- ily indicate temperature or spiciness. Care and down unit following a myocardial infarction (MI), treatment regimens can be negotiated with cliis given a prescription for a beta-blocking drug. A ents within this framework. nursing student asks the charge nurse why this drug would be used by a client who is not hypertensive. What is an appropriate response by the Question19 A 72 year-old client is scheduled to have a cardicharge nurse? A) “Most people develop hypertension following oversion. A nurse reviews the client’s medication administration record. The nurse should notify the an MI.” B) “A beta-Blocker will prevent orthostatic hypo- health care provider if the client received which medication during the preceding 24 hours? tension.” C) “This drug will decrease the workload on his A) Digoxin (Lanoxin) B) Diltiazem (Cardizem) heart.” D) “Beta-blockers increase the strength of heart C) Nitroglycerine ointment D) Metoprolol (Toprol XL) contractions.” Review Information: The correct answer is A: Short-term relief can be expected Xanax is a short-acting benzodiazepine useful in controlling panic symptoms quickly.

Review Information: The correct answer is C: “This drug will decrease the workload on his heart.” One action of beta-blockers is to decrease systemic vascular resistance by dilating arterioles. This is useful for the client with coronary artery disease, and will reduce the risk of another MI or sudden death.

Question18

A 35-year-old client of Puerto Rican-American descent is diagnosed with ovarian cancer. The client states, “I refuse both radiation and chemotherapy because they are ‘hot.’” The next action for the nurse to take is to A) document the situation in the notes B) report the situation to the health care provider C) talk with the client’s family about the situation D) ask the client to talk about concerns regarding “hot” treatments

Review Information: The correct answer is A: Digoxin (Lanoxin) Digoxin increases ventricular irritability and increases the risk of ventricular fibrillation following cardioversion. The other medications do not increase ventricular irritability.

Question20

Which of these clients, all of whom have the findings of a board-like abdomen, would the nurse suggest that the provider examine first? A) An elderly client who stated, “My awful pain in my right side suddenly stopped about 3 hours ago.” B) A pregnant woman of 8 weeks newly diagnosed with an ectopic pregnancy C) A middle-aged client admitted with diverticulitis who has taken only clear liquids for the past week D) A teenager with a history of falling off a bicycle without hitting the handle bars

Collected by :DeepaRajesh [ 124 ] [email protected] Kuwait

Review Information: The correct answer is A: An elderly client who stated, “My awful pain in my right side suddenly stopped about 3 hours ago.” This client has the highest risk for hypovolemic and septic shock since the appendix has most likely ruptured, based on the history of the pain suddenly stopping over three hours ago. Elderly clients have less functional reserve for the body to cope with shock and infection over long periods. The others are at risk for shock also, however given that they fall in younger age groups, they would more likely be able to tolerate an imbalance in circulation. A common complication of falling off a bicycle is hitting the handle bars in the upper abdomen often on the left, resulting in a ruptured spleen.

Review Information: The correct answer is D: Administering two anti-tuberculosis drugs Resistance of the tubercle bacilli often occurs to a single antimicrobial agent. Therefore, therapy with multiple drugs over a long period of time helps to ensure eradication of the organism.

Question23

The nurse is assessing a comatose client receiving gastric tube feedings. Which of the following assessments requires an immediate response from the nurse? A) Decreased breath sounds in right lower lobe B) Aspiration of a residual of 100cc of formula C) Decrease in bowel sounds D) Urine output of 250 cc in past 8 hours

Question21

The nurse is teaching parents of a 7 month-old about adding table foods. Which of the following is an appropriate finger food? A) Hot dog pieces B) Sliced bananas C) Whole grapes D) Popcorn

Review Information: The correct answer is B: Sliced bananas Finger foods should be bite-size pieces of soft food such as bananas. Hot dogs and grapes can accidentally be swallowed whole and can occlude the airway. Popcorn is too difficult to chew at this age and can irritate the airway if swallowed.

Question22

To prevent drug resistance from developing, the nurse is aware that which of the following is a characteristic of the typical treatment plan to eliminate the tuberculosis bacilli? A) An anti-inflammatory agent B) High doses of B complex vitamins C) Aminoglycoside antibiotics D) Administering two anti-tuberculosis drugs

Review Information: The correct answer is A: Decreased breath sounds in right lower lobe The most common problem associated with enteral feedings is atelectasis. Maintain client at 30 degrees of head elevation during feedings and monitor for signs of aspiration. Check for tube placement prior to each feeding or every 4 to 8 hours if the client is receiving continuous feeding.

Question24

A client is prescribed warfarin sodium (Coumadin) to be continued at home. Which focus is critical to be included in the nurse’s discharge instruction? A) Maintain a consistent intake of green leafy foods B) Report any nose or gum bleeds C) Take Tylenol for minor pains D) Use a soft toothbrush

Review Information: The correct answer is B: Report any nose or gum bleeds The client should notify the health care provider if blood is noted in stools or urine, or any other signs of bleeding occur.

Collected by :DeepaRajesh [ 125 ] [email protected] Kuwait

Question25

When teaching a client about the side effects of fluoxetine (Prozac), which of the following will the nurse include? A) Tachycardia blurred vision, hypotension, anorexia B) Orthostatic hypotension, vertigo, reactions to tyramine-rich foods C) Diarrhea, dry mouth, weight loss, reduced libido D) Photosensitivity, seizures, edema, hyperglycemia

Review Information: The correct answer is D: Autonomy Individuals must be free to make independent decisions about participation in research without coercion from others.

Question28

The nurse is talking with the family of an 18 months-old newly diagnosed with retinoblastoReview Information: The correct answer is C: ma. A priority in communicating with the parents Diarrhea, dry mouth, weight loss, reduced libido is Commonly reported side effects for fluoxetine A) Discuss the need for genetic counseling (Prozac) are diarrhea, dry mouth, weight loss B) Inform them that combined therapy is seldom and reduced libido. effective C) Prepare for the child’s permanent disfigurement D) Suggest that total blindness may follow surQuestion26 A newborn weighed 7 pounds 2 ounces at birth. gery The nurse assesses the newborn at home 2 days later and finds the weight to be 6 pounds 7 ounces. What should the nurse tell the parents Review Information: The correct answer is A: Discuss the need for genetic counseling about this weight loss? The hereditary aspects of this disease are well A) The newborn needs additional assessments documented. While the parents focus on the B) The mother should breast feed more often needs of this child, they should be aware that C) A change to formula is indicated the risk is high for future offspring. D) The loss is within normal limits

Question29 Review Information: The correct answer is D: The loss is within normal limits A newborn is expected to lose 5-10% of the birth weight in the first few days post-partum because of changes in elimination and feeding.

Question27

The nurse manager informs the nursing staff at morning report that the clinical nurse specialist will be conducting a research study on staff attitudes toward client care. All staff are invited to participate in the study if they wish. This affirms the ethical principle of A) Anonymity B) Beneficence C) Justice D) Autonomy

The nurse is planning care for an 8 year-old child. Which of the following should be included in the plan of care? A) Encourage child to engage in activities in the playroom B) Promote independence in activities of daily living C) Talk with the child and allow him to express his opinions D) Provide frequent reassurance and cuddling

Review Information: The correct answer is A: Encourage child to engage in activities in the playroom According to Erikson, the school age child is in the stage of industry versus inferiority. To help Collected by :DeepaRajesh [ 126 ] [email protected] Kuwait

them achieve industry, the nurse should encourage them to carry out tasks and activities in their room or in the playroom.

Question30

The nurse is assigned to care for 4 clients. Which of the following should be assessed immediately after hearing the report? A) The client with asthma who is now ready for discharge B) The client with a peptic ulcer who has been vomiting all night C) The client with chronic renal failure returning from dialysis D) The client with pancreatitis who was admitted yesterday

Review Information: The correct answer is B: The client with a peptic ulcer who has been vomiting all night A perforated peptic ulcer could cause nausea, vomiting and abdominal distention, and may be a life threatening situation. The client should be assessed immediately and findings reported to the provider. .

Question31

During a routine check-up, an insulin-dependent diabetic has his glycosylated hemoglobin checked. The results indicate a level of 11%. Based on this result, what teaching should the nurse emphasize? A) Rotation of injection sties B) Insulin mixing and preparation C) Daily blood sugar monitoring D) Regular high protein diet

Review Information: The correct answer is C: Daily blood sugar monitoring Normal hemoglobin A1C (glycosylated hemoglobin) level is 7 to 9%. Elevation indicates elevated glucose levels over time.

Question32

A client taking isoniazid (INH) for tuberculosis asks the nurse about side effects of the medication. The client should be instructed to immediately report which of these? A) Double vision and visual halos B) Extremity tingling and numbness C) Confusion and lightheadedness D) Sensitivity of sunlight Review Information: The correct answer is B: Extremity tingling and numbness Peripheral neuropathy is the most common side effect of INH and should be reported to the provider. It can be reversed.

Question33

Which of these questions is priority when assessing a client with hypertension? A) “What over-the-counter medications do you take?” B) “Describe your usual exercise and activity patterns.” C) “Tell me about your usual diet.” D) “Describe your family’s cardiovascular history.”

Review Information: The correct answer is A: “What over-the-counter medications do you take?” Over-the-counter medications, especially those that contain cold preparations can increase the blood pressure to the point of hypertension.

Question34

The nurse is performing an assessment of the motor function in a client with a head injury. The best technique is A) touching the trapezius muscle or arm firmly B) pinching any body part C) shaking a limb vigorously D) rubbing the sternum

Review Information: The correct answer is D: rubbing the sternum Collected by :DeepaRajesh [ 127 ] [email protected] Kuwait

The purpose is to assess the non-responsive client’s reaction to a painful stimulus after less noxious methods have been tried.

Question37

Which of these clients would the nurse monitor for the complication of C. difficile diarrhea? Question35 A) An adolescent taking medications for acne A nurse admits a client transferred from the emer- B) An elderly client living in a retirement center gency room (ER). The client, diagnosed with a taking prednisone myocardial infarction, is complaining of subster- C) A young adult at home taking a prescribed nal chest pain, diaphoresis and nausea. The first aminoglycoside action by the nurse should be to D) A hospitalized middle aged client receiving clindamycin A) order an EKG B) administer morphine sulfate C) start an IV D) measure vital signs

Review Information: The correct answer is B: administer morphine sulfate Decreasing the clients pain is the most important priority at this time. As long as pain is present there is danger in extending the infarcted area. Morphine will decrease the oxygen demands of the heart and act as a mild diuretic as well. It is probable that an EKG and IV insertion were performed in the ER.

Question36

The nurse admits a 2 year-old child who has had a seizure. Which of the following statement by the child’s parent would be important in determining the etiology of the seizure? A) “He has been taking long naps for a week.” B) “He has had an ear infection for the past 2 days.” C) “He has been eating more red meat lately.” D) “He seems to be going to the bathroom more frequently.”

Review Information: The correct answer is D: A hospitalized middle aged client receiving clindamycin Hospitalized patients, especially those receiving antibiotic therapy, are primary targets for C. difficile. Of clients receiving antibiotics, 5-38% experience antibiotic-associated diarrhea; C. difficile causes 15 to 20% of the cases. Several antibiotic agents have been associated with C. difficile. Broad-spectrum agents, such as clindamycin, ampicillin, amoxicillin, and cephalosporins, are the most frequent sources of C. difficile. Also, C. difficile infection has been caused by the administration of agents containing beta-lactamase inhibitors (i.e., clavulanic acid, sulbactam, tazobactam) and intravenous agents that achieve substantial colonic intraluminal concentrations (i.e., ceftriaxone, nafcillin, oxacillin). Fluoroquinolones, aminoglycosides, vancomycin, and trimethoprim are seldom associated with C. difficile infection or pseudomembranous colitis.

Question38

The nurse is performing an assessment on a client who is cachectic and has developed an enterocutaneous fistula following surgery to relieve a small bowel obstruction. The client’s total protein level is reported as 4.5 g/dl. Which of the followReview Information: The correct answer is ing would the nurse anticipate? B: “He has had an ear infection for the past 2 A) Additional potassium will be given IV days.” B) Blood for coagulation studies will be drawn Contributing factors to seizures in children in- C) Total parenteral nutrition (TPN) will be startclude those such as age (more common in first ed 2 years), infections (late infancy and early child- D) Serum lipase levels will be evaluated hood), fatigue, not eating properly and excessive fluid intake or fluid retention. Collected by :DeepaRajesh [ 128 ] [email protected] Kuwait

infant: 20-60 mg/dl or 1.1-3.3 mmol/L, Neonate: 30-60 mg/dl or 1.7-3.3 mmol/L, Infant: 40-90 mg/ Review Information: The correct answer is C: dl or 2.2-5.0 mmol/L. Critical values are: Infant: Total parenteral nutrition (TPN) will be started <40 mg/dl and in a Newborn: <30 and >300 mg/ The client is not absorbing nutrients adequately dl. Because of the increased birth weight which as evidenced by the cachexia and low protein can be associated with diabetes mellitus, repeatlevels. (A normal total serum protein level is 6.0- ed blood sugars will be drawn 8.0 g/dl.) TPN will promote a positive nitrogen balance in this client who is unable to digest and 0 comments absorb nutrients adequately. Labels: free nclex-rn sample review questions, nclex-rn practice test questions, nursing review

Question39

During a situation of pain management, which statement is a priority to consider for the ethical guidelines of the nurse? A) The client’s self-report is the most important consideration B) Cultural sensitivity is fundamental to pain management C) Clients have the right to have their pain relieved D) Nurses should not prejudge a client’s pain using their own values

Review Information: The correct answer is A: The client’’s self-report is the most important consideration Pain is a complex phenomenon that is perceived differently by each individual. Pain is whatever the client says it is. The other statements are correct but not the most important considerations.

Free NCLEX-RN Sample Test Questions For Nursing Review (Part 3) Question1

A client diagnosed with chronic depression is maintained on tranylcypromine (Parnate). An important nursing intervention is to teach the client to avoid which of the following foods? A) Wine, beer, cheese, liver and chocolate B) Wine, citrus fruits, yogurt and broccoli C) Beer, cheese, beef and carrots D) Wine, apples, sour cream and beef steak Review Information: The correct answer is A: Wine, beer, cheese, liver and chocolate These foods are tyramine rich and ingestion of these foods while taking monoamine oxidase inhibitors (MAOIs) can precipitate a life-threatening hypertensive crisis.

Question2

The nurse is working in a high risk antepartum clinic. A 40 year-old woman in the first trimesQuestion40 ter gives a thorough health history. Which inforAs a part of a 9 pound full-term newborn’s as- mation should receive priority attention by the sessment, the nurse performs a dextro-stick at 1 nurse? hour post birth. The serum glucose reading is 45 A) Her father and brother are insulin dependent mg/dl. What action by the nurse is appropriate at diabetics this time? B) She has taken 800 mcg of folic acid daily for A) Give oral glucose water the past year B) Notify the pediatrician C) Her husband was treated for tuberculosis as C) Repeat the test in 2 hours a child D) Check the pulse oximetry reading D) She reports recent use of over-the counter sinus remedies Review Information: The correct answer is C: Repeat the test in 2 hours This blood sugar is within the normal range for a full-term newborn. Normal values are: Premature

Review Information: The correct answer is D: She reports recent use of over-the counter sinus remedies Collected by :DeepaRajesh [ 129 ] [email protected] Kuwait

Over-the-counter drugs are a possible danger in early pregnancy. A report by the client that she has taken medications should be followed up immediately.

Question3

What must be the priority consideration for nurses when communicating with children? A) Present environment B) Physical condition C) Nonverbal cues D) Developmental level Review Information: The correct answer is D: Developmental level While each of the factors affect communication, the nurse recognizes that developmental differences have implications for processing and understanding information. Consequently, a child’s developmental level must be considered when selecting communication approaches.

Question4

The nurse is assessing a client’s home in preparation for discharge. Which of the following should be given priority consideration? A) Family understanding of client needs B) Financial status C) Location of bathrooms D) Proximity to emergency services Review Information: The correct answer is A: Family understanding of client needs Functional communication patterns between family members are fundamental to meeting the needs of the client and family.

Question5

As a general guide for emergency management of acute alcohol intoxication, it is important for the nurse initially to obtain data regarding which of the following? A) What and how much the client drinks, according to family and friends B) The blood alcohol level of the client C) The blood pressure level of the client D) The blood glucose level of the client Review Information: The correct answer is B: The blood alcohol level of the client

Blood alcohol levels are generally obtained to determine the level of intoxication. The amount of alcohol consumed determines how much medication the client needs for detoxification and treatment. Reports of alcohol consumption are notoriously inaccurate.

Question6

Which clinical finding would the nurse expect to assess first in a newborn with spastic cerebral palsy? A) cognitive impairment B) hypotonic muscular activity C) seizures D) criss-crossing leg movement Review Information: The correct answer is D: criss-crossing leg movement Cerebral palsy is a neuromuscular impairment resulting in muscular and reflexive hypertonicity and the criss-crossing, or scissoring leg movements.

Question7

Which medication is more helpful in treating bulimia than anorexia? A) Amphetamines B) Sedatives C) Anticholinergics D) Narcotics Review Information: The correct answer is C: Anticholinergics In contrast to anorexics, individuals with bulimia are troubled by their behavioral characteristics and become depressed. The person feels compelled to binge, purge and fast. Feeling helpless to stop the behavior, feelings of self-disgust occur.

Question8

The nurse is assessing a woman in early labor. While positioning for a vaginal exam, she complains of dizziness and nausea and appears pale. Her blood pressure has dropped slightly. What should be the initial nursing action? A) Call the health care provider B) Encourage deep breathing C) Elevate the foot of the bed D) Turn her to her left side Review Information: The correct answer is D: Collected by :DeepaRajesh [ 130 ] [email protected] Kuwait

Turn her to her left side The weight of the uterus can put pressure on the vena cava and aorta when a pregnant woman is flat on her back causing supine hypotension. Action is needed to relieve the pressure on the vena cava and aorta. Turning the woman to the side reduces this pressure and relieves postural hypotension.

ing lab data is available: PaO2 95, PaCO2 30, pH 7.5, K 3.2 mEq/l. Which should be the nurse’s first action? A) Monitor respiratory rate B) Monitor intake and output every hour C) Assist the client to breathe into a paper bag D) Prepare to administer oxygen by mask

Review Information: The correct answer is C: A client has been started on a long term corticos- Assist the client to breathe into a paper bag teroid therapy. Which of the following comments Side effects of aspirin toxicity include hyperventiby the client indicate the need for further teach- lation, which can result in respiratory alkalosis in the initial stages. Breathing into a paper bag will ing? prevent further reduction in PaCO2. A) “I will keep a weekly weight record.” B) “I will take medication with food.” C) “I will stop taking the medication for 1 week Question12 every month.” After assessing a 70 year-old male client’s laboD) “I will eat foods high in potassium.” ratory results during a routine clinic visit, which one of the following findings would indicate an area in which teaching is needed: Review Information: The correct answer is C: A) Serum albumin 2.5 g/dl “I will stop taking the medication for 1week every B) LDL Cholesterol 140 mg/dl month.” C) Serum glucose 90 mg/dl Emphatically warn against discontinuing steroid D) RBC 5.0 million/mm3 dosage abruptly because that may produce a fatal adrenal crisis. Review Information: The correct answer is A: Serum albumin 2.5 g/dl Question10 A male client calls for a nurse because of chest Serum albumin level is low (normal 3.0 – 5.0 g/dl pain. Which statement by the client would require in elders), indicating nutritional counseling to increase dietary protein is needed. Socioeconomic the most immediate action by the nurse? A) “When I take in a deep breath, it stabs like a factors may need to be addressed to help the client comply with the recommendation. knife.” B) “The pain came on after dinner. That soup seemed very spicy.” Question13 C) “When I turn in bed to reach the remote for the When teaching a client with a new prescription TV, my chest hurts.” for lithium (Lithane) for treatment of a bi-polar D) “I feel pressure in the middle of my chest, like disorder which of these should the nurse emphaan elephant is sitting on my chest.” size? A) Maintaining a salt restricted diet B) Reporting vomiting or diarrhea Review Information: The correct answer is D: C) Taking other medication as usual “I feel pressure in the middle of my chest, like an D) Substituting generic form if desired elephant is sitting on my chest.” This is a classic description of chest pain in men caused by myocardial ischemia. Women experi- Review Information: The correct answer is B: ence vague feelings of fatigue and back and jaw Reporting vomiting or diarrhea pain. If dehydration results from vomiting, diarrhea or excessive perspiration, tolerance to the drug may be altered and symptoms may return. Question11 A nurse is caring for a client who has just been admitted with an overdose of aspirin. The follow-

Question9

Collected by :DeepaRajesh [ 131 ] [email protected] Kuwait

Question14

A client is discharged on warfarin sulfate (Coumadin). Which statement by the client indicated a need for further teaching? A) “I know I must avoid crowds.” B) “I will keep all laboratory appointments.” C) “I plan to use an electric razor for shaving.” D) “I will report any bruises for bleeding.”

Review Information: The correct answer is A: “I know I must avoid crowds.” There are no specific reasons for the client on Coumadin to avoid crowds. General instructions for any cardiac surgical client include limiting exposure to infection.

Question15

A client is taking tranylcypromine (Parnate) and has received dietary instruction. Which of the following food selections would be contraindicated for this client? A) Fresh juice, carrots, vanilla pudding B) Apple juice, ham salad, fresh pineapple C) Hamburger, fries, strawberry shake D) Red wine, fava beans, aged cheese Review Information: The correct answer is D: Red wine, fava beans, aged cheese Red wine and cheese contain tyramine (as do chicken liver and ripe bananas) and so are contraindicated when taking MAOIs. Fava beans contain other vasopressors that can interact with MAOIs also causing malignant hypertension.

Question16

A client is admitted with severe injuries from an auto accident. The client’s vital signs are BP 120/50, pulse rate 110, and respiratory rate of 28. The initial nursing intervention would be to A) begin intravenous therapy B) initiate continuous blood pressure monitoring C) administer oxygen therapy D) institute cardiac monitoring Review Information: The correct answer is C: administer oxygen therapy Early findings of shock reveal hypoxia with rapid heart rate and rapid respirations, and oxygen is the most critical initial intervention. The other interventions are secondary to oxygen therapy.

Question17

A client is admitted to the hospital with a diagnosis of deep vein thrombosis. During the initial assessment, the client complains of sudden shortness of breath. The SaO2 is 87. The priority nursing assessment at this time is A) bowel sounds B) heart rate C) peripheral pulses D) lung sounds Review Information: The correct answer is D: lung sounds Lung sounds are critical assessments at this point. The nurse should be alert to crackles or a pleural friction rub, highly suggestive of a pulmonary embolism.

Question18

The nurse is administering lidocaine (Xylocaine) to a client with a myocardial infarction. Which of the following assessment findings requires the nurse’s immediate action? A) Central venous pressure reading of 11 B) Respiratory rate of 22 C) Pulse rate of 48 BPM D) Blood pressure of 144/92 Review Information: The correct answer is C: Pulse rate of 48 BPM One of the side effects of lidocaine is bradycardia, heart block, cardiovascular collapse and cardiac arrest (this drug should never be administered without continuous EKG monitoring).

Question19

The nurse is teaching a group of college students about breast self-examination. A woman asks for the best time to perform the monthly exam. What is the best reply by the nurse? A) “The first of every month, because it is easiest to remember” B) “Right after the period, when your breasts are less tender” C) “Do the exam at the same time every month” D) “Ovulation, or mid-cycle is the best time to detect changes” Review Information: The correct answer is B: Collected by :DeepaRajesh [ 132 ] [email protected] Kuwait

“Right after the period, when your breasts are less tender” The best time for a breast self exam (BSE) is a week after a menstrual cycle, when the breasts are no longer swollen and tender due to hormone elevation.

Review Information: The correct answer is B: Sodium Clients taking lithium need to maintain an adequate intake of sodium. Serum lithium concentrations may increase in the presence of conditions that cause sodium loss.

Question20

Question23

Review Information: The correct answer is C: cover the wound with sterile saline dressing When evisceration occurs, the wound should first be quickly covered by sterile dressings soaked in sterile saline. This prevents tissue damage until a repair can be effected.

Review Information: The correct answer is B: Vomiting, diarrhea and lethargy These are early signs of lithium toxicity.

The nurse is caring for a post-operative client who develops a wound evisceration. The first nursing intervention should be to A) medicate the client for pain B) call the provider C) cover the wound with sterile saline dressing D) place the bed in a flat position

Question21

The spouse of a client with Alzheimer’s disease expresses concern about the burden of caregiving. Which of the following actions by the nurse should be a priority? A) Link the caregiver with a support group B) Ask friends to visit regularly C) Schedule a home visit each week D) Request anti-anxiety prescriptions

A client is receiving lithium carbonate 600 mg T.I.D. to treat bipolar disorder. Which of these indicate early signs of toxicity? A) Ataxia and course hand tremors B) Vomiting, diarrhea and lethargy C) Pruritus, rash and photosensitivity D) Electrolyte imbalance and cardiac arrhythmias

Question24

The nurse can best ensure the safety of a client suffering from dementia who wanders from the room by which action? A) Repeatedly remind the client of the time and location B) Explain the risks of walking with no purpose C) Use protective devices to keep the client in the bed or chair in the room D) Attach a wander-guard sensor band to the client’s wrist

Review Information: The correct answer is D: Review Information: The correct answer is A: Attach a wander-guard sensor band to the cliLink the caregiver with a support group ent’’s wrist Assisting caregivers to locate and join support This type of identification band easily tracks the groups is most helpful. Families share feelings client’’s movements and ensures safety while the and learn about services such as respite care. client wanders on the unit. Restriction of activity Health education is also available through local is inappropriate for any client unless they are poand national Alzheimer’’s chapters. tentially harmful to themselves or others.

Question22

Clients taking lithium must be particularly sure to maintain adequate intake of which of these elements? A) Potassium B) Sodium C) Chloride D) Calcium

Question25

The nurse is teaching a client about the difference between tardive dyskinesia (TD) and neuroleptic malignant syndrome (NMS). Which statement is true with regards to tardive dyskinesia? A) TD develops within hours or years of continued antipsychotic drug use in people under 20 and over 30 B) It can occur in clients taking antipsychotic drugs longer than 2 years Collected by :DeepaRajesh [ 133 ] [email protected] Kuwait

C) Tardive dyskinesia occurs within minutes of the first dose of antipsychotic drugs and is reversible D) TD can easily be treated with anticholinergic drugs Review Information: The correct answer is B: It can occur in clients taking antipsychotic drugs longer than 2 years Tardive dyskinesia is a extrapyramidal side effect that appears after prolonged treatment with antipsychotic medication. Early symptoms of tardive dyskinesia are fasciculations of the tongue or constant smacking of the lips.

medical emergency.

Question28

The nurse is caring for a 2 month-old infant with a congenital heart defect. Which of the following is a priority nursing action? A) Provide small feedings every 3 hours B) Maintain intravenous fluids C) Add strained cereal to the diet D) Change to reduced calorie formula

Review Information: The correct answer is A: Provide small feedings every 3 hours Infants with congenital heart defects are at increased risk for developing congestive heart failure. Infants with congestive heart failure have an increased metabolic rate and require additional Question26 The nurse is aware that the effect of antihyper- calories to grow. At the same time, however, rest tensive drug therapy may be affected by a 75 and conservation of energy for eating is important. Feedings should be smaller and every 3 year-old client’s hours rather than the usual 4 hour schedule. A) poor nutritional status B) decreased gastrointestinal motility C) increased splanchnic blood flow Question29 D) altered peripheral resistance The nurse is caring for a client receiving intravenous nitroglycerin for acute angina. What is the most important assessment during treatment? Review Information: The correct answer is B: A) Heart rate decreased gastrointestinal motility B) Neurologic status Together with shrinkage of the gastric mucosa, C) Urine output and changes in the levels of hydrochloric acid, D) Blood pressure this will decrease absorption of medications and interfere with their actions. Review Information: The correct answer is D: Question27 Blood pressure In response to a call for assistance by a client in The vasodilatation that occurs as a result of this labor, the nurse notes that a loop on the umbili- medication can cause profound hypotension. cal cord protrudes from the vagina. What is the The client’’s blood pressure must be evaluated every 15 minutes until stable and then every 30 priority nursing action? minutes to every hour. A) call the health care provider B) check fetal heart beat C) put the client in knee-chest position Question30 D) turn the client to the side A client telephones the clinic to ask about a home pregnancy test she used this morning. The nurse understands that the presence of which hormone Review Information: The correct answer is C: strongly suggests a woman is pregnant? put the client in knee-chest position A) Estrogen Immediate action is needed to relieve pressure B) HCG on the cord, which puts the fetus at risk due to C) Alpha-fetoprotein hypoxia. The Trendelenburg position accom- D) Progesterone plishes this. The exposed cord is covered with saline soaked gauze, not reinserted. The fe- Review Information: The correct answer is B: tal heart rate also should be checked, and the HCG provider called. A prolapsed umbilical cord is a Human chorionic gonadotropin (HCG) is the Collected by :DeepaRajesh [ 134 ] [email protected] Kuwait

biologic marker on which pregnancy tests are the bathroom. The nurse notices a large amount based. Reliability is about 98%, but the test does of clear fluid on the bed linens. The nurse knows not conclusively confirm pregnancy. that fetal monitoring must now assess for what complication? A) Early decelerations Question31 A client, admitted to the unit because of severe B) Late accelerations depression and suicidal threats, is placed on sui- C) Variable decelerations cidal precautions. The nurse should be aware D) Periodic accelerations that the danger of the client committing suicide is greatest Review Information: The correct answer is C: A) during the night shift when staffing is limited B) when the client’s mood improves with an in- Variable decelerations When the membranes rupture, there is increased crease in energy level risk initially of cord prolapse. Fetal heart rate patC) at the time of the client’s greatest despair D) after a visit from the client’s estranged part- terns may show variable decelerations, which require immediate nursing action to promote gas ner exchange. Review Information: The correct answer is B: when the client’s mood improves with an increase in energy level Suicide potential is often increased when there is an improvement in mood and energy level. At this time ambivalence is often decreased and a decision is made to commit suicide.

Question32

After 4 electroconvulsive treatments over 2 weeks, a client is very upset and states “I am so confused. I lose my money. I just can’t remember telephone numbers.” The most therapeutic response for the nurse to make is A) “You were seriously ill and needed the treatments.” B) “Don’t get upset. The confusion will clear up in a day or two.” C) “It is to be expected since most clients have the same results.” D) “I can hear your concern and that your confusion is upsetting to you.” Review Information: The correct answer is D: “I can hear your concern and that your confusion is upsetting to you.” Communicating caring and empathy with the acknowledgement of feelings is the initial response. Afterwards, teaching about the expected short term effects would be discussed.

Question33

A woman in labor calls the nurse to assist her in

Question34

The nurse is assessing a client with chronic obstructive pulmonary disease receiving oxygen for low PaO2 levels. Which assessment is a nursing priority? A) Evaluating SaO2 levels frequently B) Observing skin color changes C) Assessing for clubbing fingers D) Identifying tactile fremitus Review Information: The correct answer is A: Evaluating SaO2 levels frequently The best method to evaluate a client’’s oxygenation is to evaluate the SaO2. This is just as effective as an arterial blood gas reading to evaluate oxygenation status, and is less traumatic and expensive.

Question35

The visiting nurse makes a postpartum visit to a married female client. Upon arrival, the nurse observes that the client has a black eye and numerous bruises on her arms and legs. The initial nursing intervention would be to A) call the police to report indications of domestic violence B) confront the husband about abusing his wife C) leave the home because of the unsafe environment D) interview the client alone to determine the origin of the injuries Review Information: The correct answer is D: interview the client alone to determine the origin Collected by :DeepaRajesh [ 135 ] [email protected] Kuwait

of the injuries It would be wrong to assume domestic violence without further assessment. Separate the suspected victim from the partner until battering has been ruled out.

Question36

When teaching a client about an oral hypoglycemic medication, the nurse should place primary emphasis on A) recognizing findings of toxicity B) taking the medication at specified times C) increasing the dosage based on blood glucose D) distinguishing hypoglycemia from hyperglycemia Review Information: The correct answer is B: taking the medication at specified times A regular interval between doses should be maintained since oral hypoglycemics stimulate the islets of Langerhans to produce insulin.

Question37

Initial postoperative nursing care for an infant who has had a pyloromyotomy would initially include A) bland diet appropriate for age B) intravenous fluids for 3-4 days C) NPO then glucose and electrolyte solutions D) formula or breast milk as tolerated Review Information: The correct answer is C: NPO then glucose and electrolyte solutions Post-operatively, the initial feedings are clear liquids in small quantities to provide calories and electrolytes.

Question38

A client is treated in the emergency room for diabetic ketoacidosis and a glucose level of 650mg.D/L. In assessing the client, the nurse’s review of which of the following tests suggests an understanding of this health problem? A) Serum calcium B) Serum magnesium C) Serum creatinine D) Serum potassium Review Information: The correct answer is D:

Serum potassium Potassium is lost in diabetic ketoacidosis during rehydration and insulin administration. Review of this lab finding suggests the nurse has knowledge of this problem.

Question39

A male client is preparing for discharge following an acute myocardial infarction. He asks the nurse about his sexual activity once he is home. What would be the nurse’s initial response? A) Give him written material from the American Heart Association about sexual activity with heart disease B) Answer his questions accurately in a private environment C) Schedule a private, uninterrupted teaching session with both the client and his wife D) Assess the client’s knowledge about his health problems Review Information: The correct answer is D: Assess the client’’s knowledge about his health problems The nursing process is continuous and cyclical in nature. When a client expresses a specific concern, the nurse performs a focused assessment to gather additional data prior to planning and implementing nursing interventions.

Question40

The client asks the nurse how the health care provider could tell she was pregnant “just by looking inside.” What is the best explanation by the nurse? A) Bluish coloration of the cervix and vaginal walls B) Pronounced softening of the cervix C) Clot of very thick mucous that obstructs the cervical canal D) Slight rotation of the uterus to the right Review Information: The correct answer is A: Bluish coloration of the cervix and vaginal walls Chadwick’’s sign is a bluish-purple coloration of the cervix and vaginal walls, occurring at 4 weeks of pregnancy, that is caused by vasocongestion. 0 comments Labels: free nclex-rn sample review questions, Collected by :DeepaRajesh [ 136 ] [email protected] Kuwait

nclex-rn practice test questions, nursing review

Question4 Free NCLEX-RN Sample Test Ques- A 12 year-old child is admitted with a broken arm and is told surgery is required. The nurse finds tions For Nursing Review (Part 2) him crying and unwilling to talk. What is the most appropriate response by the nurse? Question1 The feeling of trust can best be established by A) Give him privacy the nurse during the process of the development B) Tell him he will get through the surgery with of a nurse-client relationship by which of these no problem C) Try to distract him characteristics? D) Make arrangements for his friends to visit A) Reliability and kindness B) Demeanor and sincerity C) Honesty and consistency D) Sympathy and appreciativeness

Review Information: The correct answer is C: Honesty and consistency Characteristics of a trusting relationship include respect, honesty, consistency, faith and caring.

Question2

A nurse has administered several blood transfusions over 3 days to a 12 year-old client with Thalassemia. What lab value should the nurse monitor closely during this therapy? A) Hemoglobin B) Red Blood Cell Indices C) Platelet count D) Neutrophil percent

Review Information: The correct answer is A: Give him privacy A 12 year-old child needs the opportunity to express his emotions privately.

Question5

In discharge teaching, the nurse should emphasize that which of these is a common side effect of clozapine (Clozaril) therapy? A) Dry mouth B) Rhinitis C) Dry skin D) Extreme salivation

Review Information: The correct answer is D: Extreme salivation A significant number of clients receiving ClozapReview Information: The correct answer is A: ine (Clozaril) therapy experience extreme salivation. Hemoglobin Hemoglobin should be in a therapeutic range of approximately 10 g/dl (100gL). “This level is low Question6 enough to foster the patient’’s own erythropoiesis A client has had a positive reaction to purified protein derivative (PPD). The client asks the without enlarging the spleen.” nurse what this means. The nurse should indicate that the client has Question3 The nurse is providing care to a newly a hospital- A) active tuberculosis ized adolescent. What is the major threat experi- B) been exposed to mycobacterium tuberculosis enced by the hospitalized adolescent? C) never had tuberculosis A) Pain management D) never been infected with mycobacterium tuB) Restricted physical activity berculosis C) Altered body image D) Separation from family

Review Information: The correct answer is C: Altered body image The hospitalized adolescent may see each of these as a threat, but the major threat that they feel when hospitalized is the fear of altered body image, because of the emphasis on physical appearance during this developmental stage.

Review Information: The correct answer is B: been exposed to mycobacterium tuberculosis The PPD skin test is used to determine the presence of tuberculosis antibodies and a positive result indicates that the person has been exposed to mycobacterium tuberculosis. Additional tests are needed to determine if active tuberculosis is present. Collected by :DeepaRajesh [ 137 ] [email protected] Kuwait

Question7

A client is receiving and IV antibiotic infusion and is scheduled to have blood drawn at 1:00 pm for a “peak” antibiotic level measurement. The nurse notes that the IV infusion is running behind schedule and will not be competed by 1:00. The nurse should: A) Notify the client’s health care provider B) Stop the infusion at 1:00 pm C) Reschedule the laboratory test D) Increase the infusion rate Review Information: The correct answer is C: Reschedule the laboratory test If the antibiotic infusion will not be completed at the time the peak blood level is due to be drawn, the nurse should ask that the blood sampling time be adjusted

Question8

B) “Seek medical attention for serious injuries.” C) “Report exposure to this illness.” D) “Avoid use of aspirin for viral infections.” Review Information: The correct answer is D: “Avoid use of aspirin for viral infections.” The link between aspirin use and Reye’’s Syndrome has not been confirmed, but evidence suggests that the risk is sufficiently grave to include the warning on aspirin products.

Question10

A post-operative client is admitted to the post-anesthesia recovery room (PACU). The anesthetist reports that malignant hyperthermia occurred during surgery. The nurse recognizes that this complication is related to what factor? A) Allergy to general anesthesia B) Pre-existing bacterial infection C) A genetic predisposition D) Selected surgical procedures

The nurse is caring for a client with a new order for bupropion (Wellbutrin) for treatment of depression. The order reads “Wellbutrin 175 mg. BID x 4 days.” What is the appropriate action? A) Give the medication as ordered

Review Information: The correct answer is C: A genetic predisposition Malignant hyperthermia is a rare, potentially fatal adverse reaction to inhaled anesthetics. There is a genetic predisposition to this disorder.

C) Observe the client for mood swings D) Monitor neuro signs frequently

Question11

Question9

Question12

B) Questionthis medication dose

A 9 year-old is taken to the emergency room with right lower quadrant pain and vomiting. When Review Information: The correct an- preparing the child for an emergency appendectomy, what must the nurse expect to be the swer is B: Questionthis medication child’s greatest fear? dose A) Change in body image Bupropion (Wellbutrin) should be started at B) An unfamiliar environment 100mg BID for three days then increased to C) Perceived loss of control 150mg BID. When used for depression, it may D) Guilt over being hospitalized take up to four weeks for results. Common side effects are dry mouth, headache, and agitation. Review Information: The correct answer is C: Doses should be administered in equally spaced Perceived loss of control time increments throughout the day to minimize For school age children, major fears are loss of the risk of seizures. control and separation from friends/peers. The clinic nurse is discussing health promotion with a group of parents. A mother is concerned about Reye’s Syndrome, and asks about prevention. Which of these demonstrates appropriate teaching? A) “Immunize your child against this disease.”

A client is to begin taking Fosamax. The nurse must emphasize which of these instructions to the client when taking this medication? “Take Fosamax A) on an empty stomach.” B) after meals.” Collected by :DeepaRajesh [ 138 ] [email protected] Kuwait

C) with calcium.” D) with milk 2 hours after meals.”

ent admitted after a severe motor vehicle crash is in acidosis? A) Hemoglobin 15 gm/dl B) Chloride 100 mEq/L Review Information: The correct answer is A: C) Sodium 130 mEq/L on an empty stomach.” D) Carbon dioxide 20 mEq/L Fosamax should be taken first thing in the morning with 6-8 ounces of plain water at least 30 minutes before other medication or food. Food Review Information: The correct answer is D: and fluids (other than water) greatly decrease Carbon dioxide 20 mEq/L the absorption of Fosamax. The client must also Serum carbon dioxide is an indicator of acid-base be instructed to remain in the upright position for status. This finding would indicate acidosis. 30 minutes following the dose to facilitate passage into the stomach and minimize irritation of Question16 the esophagus. The nurse has just received report on a group of clients and plans to delegate care of several of the clients to a practical nurse (PN). The first Question13 An older adult client is to receive and antibiotic, thing the RN should do before the delegation of gentamicin. What diagnostic finding indicates care is the client may have difficult excreting the medi- A) Provide a time-frame for the completion of the client care cation? B) Assure the PN that the RN will be available for A) High gastric pH assistance B) High serum creatinine C) Low serum albumin C) Ask about prior experience with similar clients D) Low serum blood urea nitrogen D) Review the specific procedures unique to the assignment Review Information: The correct answer is B: Review Information: The correct answer is C: High serum creatinine An elevated serum creatinine indicates reduced Ask about prior experience with similar clients. renal function. Reduced renal function will delay The first step in delegation is to determine the qualifications of the person to whom one is delethe excretion of many mediations. gating. By asking about the PN’’s prior experience with similar clients/tasks, the RN can determine Question14 A nurse is assigned to care for a comatose dia- whether the PN has the requisite experience to betic on IV insulin therapy. Which task would be care for the assigned clients. most appropriate to delegate to an unlicensed assistive personnel (UAP)? Question17 A) Check the client’s level of consciousness B) Obtain the regular blood glucose readings The mother of a 4 month-old infant asks the C) Determine if special skin care is needed nurse about the dangers of sunburn while they D) Answer questions from the client’s spouse are on vacation at the beach. Which of the following is the best advice about sun protection for about the plan of care this child? A) “Use a sunscreen with a minimum sun protecReview Information: The correct answer is B: tive factor of 15.” Obtain the regular blood glucose readings B) “Applications of sunscreen should be repeatThe UAP can safely obtain blood glucose read- ed every few hours.” ings, which are routine tasks. C) “An infant should be protected by the maximum strength sunscreen.” D) “Sunscreens are not recommended in chilQuestion15 Which of the following laboratory results would dren younger than 6 months.” suggest to the emergency room nurse that a cliCollected by :DeepaRajesh [ 139 ] [email protected] Kuwait

Review Information: The correct answer is D: “Sunscreens are not recommended in children younger than 6 months.” Infants under 6 months of age should be kept out of the sun or shielded from it. Even on a cloudy day, the infant can be sunburned while near water. A hat and light protective clothing should be worn.

Question18

The nurse administers cimetidine (Tagamet) to a 79 year-old male with a gastric ulcer. Which parameter may be affected by this drug, and should be closely monitored by the nurse? A) Blood pressure B) Liver function C) Mental status D) Hemoglobin Review Information: The correct answer is C: Mental status The elderly are at risk for developing confusion when taking cimetidine, a drug that interacts with many other medications.

Question19

The nurse assesses the use of coping mechanisms by an adolescent 1 week after the client had a motor vehicle accident resulting in multiple serious injuries. Which of these characteristics are most likely to be displayed? A) Ambivalence, dependence, demanding B) Denial, projection, regression C) Intellectualization, rationalization, repression D) Identification, assimilation, withdrawal

B) “Unless you had previous problems, every 2 years is best.” C) “Once a woman reaches 50, she should have a mammogram yearly.” D) “Yearly mammograms are advised for all women over 35.” Review Information: The correct answer is C: “Once a woman reaches 50, she should have a mammogram yearly.” The American Cancer Society recommends a screening mammogram by age 40, every 1 - 2 years for women 40-49, and every year from age 50. If there are family or personal health risks, other assessments may be recommended.

Question21

The nurse is planning care for a client who is taking cyclosporin (Neoral). What would be an appropriate nursing diagnosis for this client? A) Alteration in body image B) High risk for infection C) Altered growth and development D) Impaired physical mobility Review Information: The correct answer is B: High risk for infection Cyclosporin (Neoral) inhibits normal immune responses. Clients receiving cyclosporin are at risk for infection.

Question22

A client on telemetry begins having premature ventricular beats (PVBs) at 12 per minute. In reviewing the most recent laboratory results, which Review Information: The correct answer is B: would require immediate action by the nurse? Denial, projection, regression A) Calcium 9 mg/dl Helplessness and hopelessness may contribute B) Magnesium 2.5 mg/dl to regressive, dependent behavior which often C) Potassium 2.5 mEq/L occurs at any age with hospitalization. Deny- D) PTT 70 seconds ing or minimizing the seriousness of the illness is used to avoid facing the worst situation. Recall that denial is the initial step in the process of Review Information: The correct answer is C: working through any loss. Potassium 2.5 mEq/L The patient is at risk for ventricular dysrhythmias when the potassium level is low. Question20 A 52 year-old post menopausal woman asks the Daniels, R. (2003). nurse how frequently she should have a mammogram. What is the nurse’s best response? Question23 A) “Your doctor will advise you about your The nurse is caring for a client who is 4 days risks.” post-op for a transverse colostomy. The client is Collected by :DeepaRajesh [ 140 ] [email protected] Kuwait

ready for discharge and asks the nurse to empty his colostomy pouch. What is the best response by the nurse? A) “You should be emptying the pouch yourself.” B) “Let me demonstrate to you how to empty the pouch.” C) “What have you learned about emptying your pouch?” D) “Show me what you have learned about emptying your pouch.”

Anticholinergics An anticholinergic medication will decrease gastric emptying and the pressure on the lower esophageal sphincter.

Question26

A client is receiving a nitroglycerin infusion for unstable angina. What assessment would be a priority when monitoring the effects of this medication? A) Blood pressure B) Cardiac enzymes Review Information: The correct answer is D: C) ECG analysis “Show me what you have learned about empty- D) Respiratory rate ing your pouch.” Most adult learners obtain skills by participating in the activities. Anxiety about discharge can be Review Information: The correct answer is A: causing the client to forget that they have mas- Blood pressure tered the skill of emptying the pouch. The client Since an effect of this drug is vasodilation, the should show the nurse how the pouch is emp- client must be monitored for hypotension. tied.

Question24

A 3 year-old child has tympanostomy tubes in place. The child’s parent asks the nurse if he can swim in the family pool. The best response from the nurse is A) “Your child should not swim at all while the tubes are in place.” B) “Your child may swim in your own pool but not in a lake or ocean.” C) “Your child may swim if he wears ear plugs.” D) “Your child may swim anywhere.” Review Information: The correct answer is C: “Your child may swim if he wears ear plugs.”

Question27

The nurse is caring for a 10 year-old child who has just been diagnosed with diabetes insipidus. The parents ask about the treatment prescribed, vasopressin. A What is priority in teaching the child and family about this drug? A) The child should carry a nasal spray for emergency use B) The family must observe the child for dehydration C) Parents should administer the daily intramuscular injections D) The client needs to take daily injections in the short-term

Review Information: The correct answer is A: The child should carry a nasal spray for emergency use Diabetes insipidus results from reduced secretion of the antidiuretic hormone, vasopressin. The child will need to administer daily injections Question25 The nurse is caring for a client with asthma who of vasopressin, and should have the nasal spray has developed gastroesophageal reflux disease form of the medication readily available. A medi(GERD). Which of the following medications pre- cal alert tag should be worn. scribed for the client may aggravate GERD? A) Anticholinergics Question28 B) Corticosteroids A client diagnosed with cirrhosis is started on C) Histamine blocker lactulose (Cephulac). The main purpose of the D) Antibiotics drug for this client is to A) add dietary fiber Review Information: The correct answer is A: B) reduce ammonia levels Water should not enter the ears. Children should use ear plugs when bathing or swimming and should not put their heads under the water.

Collected by :DeepaRajesh [ 141 ] [email protected] Kuwait

C) stimulate peristalsis D) control portal hypertension

Question31

A client has many delusions. As the nurse helps the client prepare for breakfast the client comReview Information: The correct answer is B: ments “Don’t waste good food on me. I’m dying reduce ammonia levels from this disease I have.” The appropriate reLactulose blocks the absorption of ammonia from sponse would be the GI tract and secondarily stimulates bowel A) “You need some nutritious food to help you elimination. regain your weight.” B) “None of the laboratory reports show that you have any physical disease.” Question29 The nurse is explaining the effects of cocaine C) “Try to eat a little bit, breakfast is the most abuse to a pregnant client. Which of the follow- important meal of the day.” ing must the nurse understand as a basis for D) “I know you believe that you have an incurable disease.” teaching? A) Cocaine use can cause fetal growth retardation B) The drug has been linked to neural tube de- Review Information: The correct answer is D: “I know you believe that you have an incurable fects C) Newborn withdrawal generally occurs imme- disease.” This response does not challenge the client’s diately after birth D) Breast feeding promotes positive parenting delusional system and thus forms an alliance by providing reassurance of desire to help the clibehaviors ent. Review Information: The correct answer is A: Cocaine use can cause fetal growth retardation Cocaine is vasoconstrictive, and this effect in the placental vessels causes fetal hypoxia and diminished growth. Other risks of continued cocaine use during pregnancy include preterm labor, congenital abnormalities, altered brain development and subsequent behavioral problems in the infant.

Question32

A client with paranoid thoughts refuses to eat because of the belief that the food is poisoned. The appropriate statement at this time for the nurse to say is A) “Here, I will pour a little of the juice in a medicine cup to drink it to show you that it is OK.” B) “The food has been prepared in our kitchen and is not poisoned.” C) “Let’s see if your partner could bring food from Question30 home.” A client has just been diagnosed with breast D) “If you don’t eat, I will have to suggest for you cancer. The nurse enters the room and the cli- to be tube fed.” ent tells the nurse that she is stupid. What is the most therapeutic response by the nurse? A) Explore what is going on with the client Review Information: The correct answer is C: B) Accept the client’s statement without com- “Let’’s see if your partner could bring food from home.” ment C) Tell the client that the comment is inappropri- Reassurance is ineffective when a client is acate tively delusional. This option avoids both arguing with the client and agreeing with the delusional D) Leave the client’s room premise. Option D offers a logical response to a primarily affective concern. When the client’s Review Information: The correct answer is A: condition has improved, gentle negation of the delusional premise can be employed. Explore what is going on with the client Exploring feelings with the verbally aggressive client helps to put angry feelings into words and Question33 then to engage in problem solving. A client with tuberculosis is started on Rifampin. Collected by :DeepaRajesh [ 142 ] [email protected] Kuwait

Which one of the following statements by the nurse would be appropriate to include in teaching? “You may notice: A) an orange-red color to your urine.” B) your appetite may increase for the first week.” C) it is common to experience occasional sleep disturbances.” D) if you take the medication with food, you may have nausea.”

D) Combines safely with antihypertensives Review Information: The correct answer is A: Promotes sodium and chloride excretion Spironolactone promotes sodium and chloride excretion while sparing potassium and decreasing aldosterone levels. It had no effect on ammonia levels.

Question36

A client was admitted to the psychiatric unit for severe depression. After several days, the client continues to withdraw from the other clients. Which of these statements by the nurse would be the most appropriate to promote interaction with other clients? A) “Your team here thinks it’s good for you to spend time with others.” B) “It is important for you to participate in group Question34 A client tells the RN she has decided to stop tak- activities.” ing sertraline (Zoloft) because she doesn’t like C) “Come with me so you can paint a picture to the nightmares, sex dreams, and obsessions help you feel better.” she’s experiencing since starting on the medi- D) “Come play Chinese Checkers with Gloria cation. What is an appropriate response by the and me.” nurse? A) “It is unsafe to abruptly stop taking any pre- Review Information: The correct answer is D: “Come play Chinese Checkers with Gloria and scribed medication.” B) “Side effects and benefits should be discussed me.” This gradually engages the client in interactions with your health care provider.” C) “This medication should be continued despite with others in small groups rather than large groups. In addition, focusing on an activity is less unpleasant symptoms.” D) “Many medications have potential side ef- anxiety-provoking than unstructured discussion. The statement is an example of a positive befects.” havioral expectation. Review Information: The correct answer is A: an orange-red color to your urine.” Discoloration of the urine and other body fluids may occur. It is a harmless response to the drug, but the patient needs to be aware it may happen.

Review Information: The correct answer is A: “It is unsafe to abruptly stop taking any prescribed medication.” Abrupt withdrawal may occasionally cause serotonin syndrome, consisting of lethargy, nausea, headache, fever, sweating and chills. A slow withdrawal may be prescribed with sertraline to avoid dizziness, nausea, vomiting, and diarrhea.

Question35

A client is admitted to the hospital with findings of liver failure with ascites. The health care provider orders spironolactone (Aldactone). What is the pharmacological effect of this medication? A) Promotes sodium and chloride excretion B) Increases aldosterone levels C) Depletes potassium reserves

Question37

The nurse is teaching a school-aged child and family about the use of inhalers prescribed for asthma. What is the best way to evaluate effectiveness of the treatments? A) Rely on child’s self-report B) Use a peak-flow meter C) Note skin color changes D) Monitor pulse rate Review Information: The correct answer is B: Use a peak-flow meter The peak flowmeter, if used correctly, shows effectiveness of inhalants. Collected by :DeepaRajesh [ 143 ] [email protected] Kuwait

Question38

The nurse is teaching a client about the toxicity of digoxin. Which one of the following statements made by the client to the nurse indicates more teaching is needed? A) “I may experience a loss of appetite.” B) “I can expect occasional double vision.” C) “Nausea and vomiting may last a few days.” D) “I must report a bounding pulse of 62 immediately.” Review Information: The correct answer is D: “I must report a bounding pulse of 62 immediately.” Slow heart rate is related to increased cardiac output and an intended effect of digoxin. The ideal heart rate is above 60 BPM with digoxin. The client needs further teaching.

Question39

Which of the following assessments by the nurse would indicate that the client is having a possible adverse response to the isoniazid (INH)? A) Severe headache B) Appearance of jaundice C) Tachycardia D) Decreased hearing Review Information: The correct answer is B: Appearance of jaundice Clients receiving INH therapy are at risk for developing drug induced hepatitis. The appearance of jaundice may indicate that the client has liver damage.

Question40

The nurse is beginning nutritional counseling/ teaching with a pregnant woman. What is the initial step in this interaction?

Conduct a diet history to determine her normal eating routines. Assessment is always the first step in planning teaching for any client. A thorough and accurate history is essential for gathering the needed information. 0 comments Labels: free nclex-rn sample review questions, nclex-rn practice test questions, nursing review

Free NCLEX-RN Sample Test Questions For Nursing Review (Part 1) These are sample nursing review questions and not actual test questions made for educational and practice test purposes only. 75 questions have been posted here with answer keys.

Question1

A client has been hospitalized after an automobile accident. A full leg cast was applied in the emergency room. The most important reason for the nurse to elevate the casted leg is to A) Promote the client’s comfort B) Reduce the drying time C) Decrease irritation to the skin D) Improve venous return Review Information: The correct answer is D: Improve venous return. Elevating the leg both improves venous return and reduces swelling. Client comfort will be improved as well.

Question2

The nurse is reviewing with a client how to collect a clean catch urine specimen. What is the appropriate sequence to teach the client?

A) Teach her how to meet the needs of self and her family B) Explain the changes in diet necessary for pregnant women

A) Clean the meatus, begin voiding, then catch urine stream B) Void a little, clean the meatus, then collect specimen C) Questionher understanding and use C) Clean the meatus, then urinate into container D) Void continuously and catch some of the of the food pyramid D) Conduct a diet history to determine her nor- urine mal eating routines Review Information: The correct answer is A: Clean the meatus, begin voiding, then catch Review Information: The correct answer is D: urine stream. A clean catch urine is difficult to Collected by :DeepaRajesh [ 144 ] [email protected] Kuwait

obtain and requires clear directions. Instructing the client to carefully clean the meatus, then void naturally with a steady stream prevents surface bacteria from contaminating the urine specimen. As starting and stopping flow can be difficult, once the client begins voiding it’’s best to just slip the container into the stream. Other responses do not reflect correct technique.

Question3

Following change-of-shift report on an orthopedic unit, which client should the nurse see first? A) 16 year-old who had an open reduction of a fractured wrist 10 hours ago B) 20 year-old in skeletal traction for 2 weeks since a motor cycle accident C) 72 year-old recovering from surgery after a hip replacement 2 hours ago D) 75 year-old who is in skin traction prior to planned hip pinning surgery. Review Information: The correct answer is C: 72 year-old recovering from surgery after a hip replacement 2 hours ago. Look for the client who has the most imminent risks and acute vulnerability. The client who returned from surgery 2 hours ago is at risk for life threatening hemorrhage and should be seen first. The 16 year-old should be seen next because it is still the first post-op day. The 75 year-old is potentially vulnerable to agerelated physical and cognitive consequences in skin traction should be seen next. The client who can safely be seen last is the 20 year-old who is 2 weeks post-injury.

Question4

A client with Guillain Barre is in a nonresponsive state, yet vital signs are stable and breathing is independent. What should the nurse document to most accurately describe the client’s condition? A) Comatose, breathing unlabored B) Glascow Coma Scale 8, respirations regular C) Appears to be sleeping, vital signs stable D) Glascow Coma Scale 13, no ventilator required

consciousness. Any score less than 13 indicates a neurological impairment. Using the term comatose provides too much room for interpretation and is not very precise.

Question5

When caring for a client receiving warfarin sodium (Coumadin), which lab test would the nurse monitor to determine therapeutic response to the drug? A) Bleeding time B) Coagulation time C) Prothrombin time D) Partial thromboplastin time Review Information: The correct answer is C: Prothrombin time. Coumadin is ordered daily, based on the client’’s prothrombin time (PT). This test evaluates the adequacy of the extrinsic system and common pathway in the clotting cascade; Coumadin affects the Vitamin K dependent clotting factors.

Question6

A client with moderate persistent asthma is admitted for a minor surgical procedure. On admission the peak flow meter is measured at 480 liters/minute. Post-operatively the client is complaining of chest tightness. The peak flow has dropped to 200 liters/minute. What should the nurse do first? A) Notify both the surgeon and provider B) Administer the prn dose of albuterol C) Apply oxygen at 2 liters per nasal cannula D) Repeat the peak flow reading in 30 minutes Review Information: The correct answer is B: Administer the prn dose of albuterol. Peak flow monitoring during exacerbations of asthma is recommended for clients with moderate-to-severe persistent asthma to determine the severity of the exacerbation and to guide the treatment. A peak flow reading of less than 50% of the client’’s baseline reading is a medical alert condition and a short-acting beta-agonist must be taken immediately.

Review Information: The correct answer is B: Glascow Coma Scale 8, respirations regular. Question7 The Glascow Coma Scale provides a standard A client had 20 mg of Lasix (furosemide) PO at reference for assessing or monitoring level of 10 AM. Which would be essential for the nurse to Collected by :DeepaRajesh [ 145 ] [email protected] Kuwait

include at the change of shift report? A) The client lost 2 pounds in 24 hours B) The client’s potassium level is 4 mEq/liter. C) The client’s urine output was 1500 cc in 5 hours D) The client is to receive another dose of Lasix at 10 PM Review Information: The correct answer is C: The client’s urine output was 1500 cc in 5 hours. Although all of these may be correct information to include in report, the essential piece would be the urine output.

Question8

lobe. Acute asthma is characterized by expiratory wheezes caused by obstruction of the airways. Wheezes are a high pitched musical sounds produced by air moving through narrowed airways. Clients often associate wheezes with the feeling of tightness in the chest. However, sudden cessation of wheezing is an ominous or bad sign that indicates an emergency -- the small airways are now collapsed.

Question10

During the initial home visit, a nurse is discussing the care of a client newly diagnosed with Alzheimer’s disease with family members. Which of these interventions would be most helpful at this time? A) leave a book about relaxation techniques B) write out a daily exercise routine for them to assist the client to do C) list actions to improve the client’s daily nutritional intake D) suggest communication strategies

A client has been tentatively diagnosed with Graves’ disease (hyperthyroidism). Which of these findings noted on the initial nursing assessment requires quick intervention by the nurse? A) a report of 10 pounds weight loss in the last month B) a comment by the client “I just can’t sit still.” C) the appearance of eyeballs that appear to “pop” out of the client’s eye sockets Review Information: The correct answer is D: D) a report of the sudden onset of irritability in suggest communication strategies. Alzheimer’’s the past 2 weeks disease, a progressive chronic illness, greatly challenges caregivers. The nurse can be of Review Information: The correct answer is C: greatest assistance in helping the family to use the appearance of eyeballs that appear to “pop” communication strategies to enhance their ability out of the client’’s eye sockets. Exophthalmos or to relate to the client. By use of select verbal and protruding eyeballs is a distinctive characteristic nonverbal communication strategies the family of Graves’’ Disease. It can result in corneal abra- can best support the client’s strengths and cope sions with severe eye pain or damage when the with any aberrant behavior. eyelid is unable to blink down over the protruding eyeball. Eye drops or ointment may be needed.

Question11

Question9

An 80 year-old client admitted with a diagnosis of possible cerebral vascular accident has had a blood pressure from 160/100 to 180/110 over the past 2 hours. The nurse has also noted increased lethargy. Which assessment finding should the nurse report immediately to the provider? A) Slurred speech B) Incontinence C) Muscle weakness D) Rapid pulse

The nurse has performed the initial assessments of 4 clients admitted with an acute episode of asthma. Which assessment finding would cause the nurse to call the provider immediately? A) prolonged inspiration with each breath B) expiratory wheezes that are suddenly absent in 1 lobe C) expectoration of large amounts of purulent mucous D) appearance of the use of abdominal muscles Review Information: The correct answer is A: for breathing Slurred speech. Changes in speech patterns and level of conscious can be indicators of continued Review Information: The correct answer is B: intracranial bleeding or extension of the stroke. expiratory wheezes that are suddenly absent in 1 Further diagnostic testing may be indicated. Collected by :DeepaRajesh [ 146 ] [email protected] Kuwait

Question12

A school-aged child has had a long leg (hip to ankle) synthetic cast applied 4 hours ago. Which statement from the parent indicates that teaching has been inadequate? A) “I will keep the cast uncovered for the next day to prevent burning of the skin.” B) “I can apply an ice pack over the area to relieve itching inside the cast.” C) “The cast should be propped on at least 2 pillows when my child is lying down.” D) “I think I remember that my child should not stand until after 72 hours.” Review Information: The correct answer is D: “I think I remember that my child should not stand until after 72 hours.”. Synthetic casts will typically set up in 30 minutes and dry in a few hours. Thus, the client may stand within the initial 24 hours. With plaster casts, the set up and drying time, especially in a long leg cast which is thicker than an arm cast, can take up to 72 hours. Both types of casts give off a lot of heat when drying and it is preferable to keep the cast uncovered for the first 24 hours. Clients may complain of a chill from the wet cast and therefore can simply be covered lightly with a sheet or blanket. Applying ice is a safe method of relieving the itching.

Question13

Which blood serum finding in a client with diabetic ketoacidosis alerts the nurse that immediate action is required? A) pH below 7.3 B) Potassium of 5.0 C) HCT of 60 D) Pa O2 of 79% Review Information: The correct answer is C: HCT of 60. This high hematocrit is indicative of severe dehydration which requires priority attention in diabetic ketoacidosis. Without sufficient hydration, all systems of the body are at risk for hypoxia from a lack of or sluggish circulation. In the absence of insulin, which facilitates the transport of glucose into the cell, the body breaks down fats and proteins to supply energy ketones, a by-product of fat metabolism. These accumulate causing metabolic acidosis (pH < 7.3), which would be the second concern for this

client. The potassium and PaO2 levels are near normal.

Question14

The nurse is preparing a client with a deep vein thrombosis (DVT) for a Venous Doppler evaluation. Which of the following would be necessary for preparing the client for this test? A) Client should be NPO after midnight B) Client should receive a sedative medication prior to the test C) Discontinue anti-coagulant therapy prior to the test D) No special preparation is necessary Review Information: The correct answer is D: No special preparation is necessary. This is a non-invasive procedure and does not require preparation other than client education.

Question15

A client is admitted with infective endocarditis (IE). Which finding would alert the nurse to a complication of this condition? A) dyspnea B) heart murmur C) macular rash D) hemorrhage Review Information: The correct answer is B: heart murmur. Large, soft, rapidly developing vegetations attach to the heart valves. They have a tendency to break off, causing emboli and leaving ulcerations on the valve leaflets. These emboli produce findings of cardiac murmur, fever, anorexia, malaise and neurologic sequelae of emboli. Furthermore, the vegetations may travel to various organs such as spleen, kidney, coronary artery, brain and lungs, and obstruct blood flow.

Question16

The nurse explains an autograft to a client scheduled for excision of a skin tumor. The nurse knows the client understands the procedure when the client says, “I will receive tissue from A) a tissue bank.” B) a pig.” C) my thigh.” D) synthetic skin.” Collected by :DeepaRajesh [ 147 ] [email protected] Kuwait

Review Information: The correct answer is C: my thigh.”. Autografts are done with tissue transplanted from the client’’s own skin.

Question17

B) The overview cardiac rehabilitation C) Medication and diet guideline D) Activity and rest guidelines Review Information: The correct answer is A: Daily needs and concerns. At 2 days post-MI, the client’s education should be focused on the immediate needs and concerns for the day.

A client is admitted to the emergency room following an acute asthma attack. Which of the following assessments would be expected by the Question20 nurse? A 3 year-old child is brought to the clinic by his A) Diffuse expiratory wheezing grandmother to be seen for “scratching his botB) Loose, productive cough tom and wetting the bed at night.” Based on these C) No relief from inhalant complaints, the nurse would initially assess for D) Fever and chills which problem? A) allergies Review Information: The correct answer is A: B) scabies Diffuse expiratory wheezing. In asthma, the air- C) regression ways are narrowed, creating difficulty getting air D) pinworms in. A wheezing sound results. Review Information: The correct answer is D: pinworms. Signs of pinworm infection include intense perianal itching, poor sleep patterns, Question18 A client has been admitted with a fractured femur general irritability, restlessness, bed-wetting, and has been placed in skeletal traction. Which distractibility and short attention span. Scabies of the following nursing interventions should re- is an itchy skin condition caused by a tiny, eightlegged burrowing mite called Sarcoptes scabiei . ceive priority? The presence of the mite leads to intense itching A) Maintaining proper body alignment B) Frequent neurovascular assessments of the in the area of its burrows. affected leg C) Inspection of pin sites for evidence of drainage or inflammation Question21 D) Applying an over-bed trapeze to assist the cli- The nurse is caring for a newborn with traent with movement in bed cheoesophageal fistula. Which nursing diagnosis is a priority? Review Information: The correct answer is A) Risk for dehydration B: Frequent neurovascular assessments of the B) Ineffective airway clearance affected leg. The most important activity for the C) Altered nutrition nurse is to assess neurovascular status. Com- D) Risk for injury partment syndrome is a serious complication of fractures. Prompt recognition of this neurovascu- Review Information: The correct answer is B: lar problem and early intervention may prevent Ineffective airway clearance. The most common permanent limb damage. form of TEF is one in which the proximal esophageal segment terminates in a blind pouch and the distal segment is connected to the trachea or primary bronchus by a short fistula at or near Question19 The nurse is assigned to care for a client who the bifurcation. Thus, a priority is maintaining an had a myocardial infarction (MI) 2 days ago. The open airway, preventing aspiration. Other nursclient has many questions about this condition. ing diagnoses are then addressed. What area is a priority for the nurse to discuss at this time? A) Daily needs and concerns Question22 Collected by :DeepaRajesh [ 148 ] [email protected] Kuwait

The nurse is developing a meal plan that would provide the maximum possible amount of iron for a child with anemia. Which dinner menu would be best? A) Fish sticks, french fries, banana, cookies, milk B) Ground beef patty, lima beans, wheat roll, raisins, milk C) Chicken nuggets, macaroni, peas, cantaloupe, milk D) Peanut butter and jelly sandwich, apple slices, milk Review Information: The correct answer is B: Ground beef patty, lima beans, wheat roll, raisins, milk. Iron rich foods include red meat, fish, egg yolks, green leafy vegetables, legumes, whole grains, and dried fruits such as raisins. This dinner is the best choice: It is high in iron and is appropriate for a toddler.

Question23

The nurse admitting a 5 month-old who vomited 9 times in the past 6 hours should observe for signs of which overall imbalance? A) Metabolic acidosis B) Metabolic alkalosis C) Some increase in the serum hemoglobin D) A little decrease in the serum potassium Review Information: The correct answer is B: Metabolic alkalosis. Vomiting causes loss of acid from the stomach. Prolonged vomiting can result in excess loss of acid and lead to metabolic alkalosis. Findings include irritability, increased activity, hyperactive reflexes, muscle twitching and elevated pulse. Options C and D are correct answers but not the best answers since they are too general.

Question24

A two year-old child is brought to the provider’s office with a chief complaint of mild diarrhea for two days. Nutritional counseling by the nurse should include which statement? A) Place the child on clear liquids and gelatin for 24 hours B) Continue with the regular diet and include oral rehydration fluids C) Give bananas, apples, rice and toast as tolerated

D) Place NPO for 24 hours, then rehydrate with milk and water Review Information: The correct answer is B: Continue with the regular diet and include oral rehydration fluids. Current recommendations for mild to moderate diarrhea are to maintain a normal diet with fluids to rehydrate.

Question25

The nurse is teaching parents about the appropriate diet for a 4 month-old infant with gastroenteritis and mild dehydration. In addition to oral rehydration fluids, the diet should include A) formula or breast milk B) broth and tea C) rice cereal and apple juice D) gelatin and ginger ale Review Information: The correct answer is A: formula or breast milk. The usual diet for a young infant should be followed.

Question26

A child is injured on the school playground and appears to have a fractured leg. The first action the school nurse should take is A) call for emergency transport to the hospital B) immobilize the limb and joints above and below the injury C) assess the child and the extent of the injury D) apply cold compresses to the injured area Review Information: The correct answer is C: assess the child and the extent of the injury. When applying the nursing process, assessment is the first step in providing care. The “5 Ps” of vascular impairment can be used as a guide (pain, pulse, pallor, paresthesia, paralysis).

Question27

The mother of a 3 month-old infant tells the nurse that she wants to change from formula to whole milk and add cereal and meats to the diet. What should be emphasized as the nurse teaches about infant nutrition? A) Solid foods should be introduced at 3-4 Collected by :DeepaRajesh [ 149 ] [email protected] Kuwait

months B) Whole milk is difficult for a young infant to digest C) Fluoridated tap water should be used to dilute milk D) Supplemental apple juice can be used between feedings Review Information: The correct answer is B: Whole milk is difficult for a young infant to digest. Cow’’s milk is not given to infants younger than 1 year because the tough, hard curd is difficult to digest. In addition, it contains little iron and creates a high renal solute load.

Question28

The nurse is preparing a handout on infant feeding to be distributed to families visiting the clinic. Which notation should be included in the teaching materials? A) Solid foods are introduced one at a time beginning with cereal B) Finely ground meat should be started early to provide iron C) Egg white is added early to increase protein intake D) Solid foods should be mixed with formula in a bottle

and symptomatic treatment. The priority of care is pain relief. In a 12 year-old child, client controlled analgesia promotes maximum comfort.

Question30

The nurse is performing a physical assessment on a toddler. Which of the following actions should be the first? A) Perform traumatic procedures B) Use minimal physical contact C) Proceed from head to toe D) Explain the exam in detail Review Information: The correct answer is B: Use minimal physical contact. The nurse should approach the toddler slowly and use minimal physical contact initially so as to gain the toddler’’s cooperation. Be flexible in the sequence of the exam, and give only brief simple explanations just prior to the action.

Question31

What finding signifies that children have attained the stage of concrete operations (Piaget)?

A) Explores the environment with the use of sight and movement Review Information: The correct answer is A: B) Thinks in mental images or word pictures Solid foods are introduced one at a time begin- C) Makes the moral judgment that “stealing is ning with cereal. Solid foods should be added wrong” one at a time between 4-6 months. If the infant is D) Reasons that homework is time-consuming able to tolerate the food, another may be added yet necessary in a week. Iron fortified cereal is the recommended first food. Review Information: The correct answer is C: Makes the moral judgment that “stealing is wrong”. The stage of concrete operations is depicted by logical thinking and moral judgments. Question29 The nurse planning care for a 12 year-old child with sickle cell disease in a vaso-occlusive crisis of the elbow should include which one of the fol- Question32 lowing as a priority? The mother of a child with a neural tube defect asks the nurse what she can do to decrease the A) Limit fluids chances of having another baby with a neural B) Client controlled analgesia tube defect. What is the best response by the C) Cold compresses to elbow nurse? D) Passive range of motion exercise A) “Folic acid should be taken before and after Review Information: The correct answer is B: conception.” Client controlled analgesia. Management of a B) “Multivitamin supplements are recommended sickle cell crisis is directed towards supportive during pregnancy.” Collected by :DeepaRajesh [ 150 ] [email protected] Kuwait

C) “A well balanced diet promotes normal fetal old child. Which statement by the parent indidevelopment.” cates understanding of appropriate precautions D) “Increased dietary iron improves the health of to take with the children? mother and fetus.” A) “I strap the infant car seat on the front seat to Review Information: The correct answer is face backwards.” A: “Folic acid should be taken before and after B) “I place my infant in the middle of the living conception.”. The American Academy of Pedi- room floor on a blanket to play with my four yearatrics recommends that all childbearing women old while I make supper in the kitchen.” increase folic acid from dietary sources and/or C) “My sleeping baby lies so cute in the crib with supplements. There is evidence that increased the little buttocks stuck up in the air while the four amounts of folic acid prevents neural tube de- year-old naps on the sofa.” fects. D) “I have the four year-old hold and help feed the four month-old a bottle in the kitchen while I make supper.”

Question33

The provider orders Lanoxin (digoxin) 0.125 mg PO and furosemide 40 mg every day. Which of these foods would the nurse reinforce for the client to eat at least daily? A) Spaghetti B) Watermelon C) Chicken D) Tomatoes Review Information: The correct answer is B: Watermelon. Watermelon is high in potassium and will replace potassium lost by the diuretic. The other foods are not high in potassium.

Review Information: The correct answer is D: “I have the four year-old hold and help feed the four month-old a bottle in the kitchen while I make supper.”. The infant seat is to be placed on the rear seat. Small children and infants are not to be left unsupervised. Infants are

Question36

The nurse admits a 7 year-old to the emergency room after a leg injury. The x-rays show a femur fracture near the epiphysis. The parents ask what will be the outcome of this injury. The appropriate response by the nurse should be which of these statements?

A) “The injury is expected to heal quickly beWhile teaching the family of a child who will take cause of thin periosteum.” phenytoin (Dilantin) regularly for seizure control, B) “In some instances the result is a retarded it is most important for the nurse to teach them bone growth.” C) “Bone growth is stimulated in the affected about which of the following actions? leg.” D) “This type of injury shows more rapid union A) Maintain good oral hygiene and dental care than that of younger children.” B) Omit medication if the child is seizure free C) Administer acetaminophen to promote sleep Review Information: The correct answer is B: D) Serve a diet that is high in iron “In some instances the result is a retarded bone Review Information: The correct answer is growth.”. An epiphyseal (growth) plate fracture A: Maintain good oral hygiene and dental care. in a 7 year-old often results in retarded bone Swollen and tender gums occur often with use of growth. The leg often will be different in length phenytoin. Good oral hygiene and regular visits than the uninjured leg. to the dentist should be emphasized.

Question34

Question35

The nurse is offering safety instructions to a parent with a four month-old infant and a four year-

Question37

The parents of a 4 year-old hospitalized child tell the nurse, “We are leaving now and will be back at 6 PM.” A few hours later the child asks the Collected by :DeepaRajesh [ 151 ] [email protected] Kuwait

nurse when the parents will come again. What is the best response by the nurse?

several minutes will diminish the lead contamination.

A) “They will be back right after supper.” B) “In about 2 hours, you will see them.” C) “After you play awhile, they will be here.” D) “When the clock hands are on 6 and 12.”

Question40

Review Information: The correct answer is A: “They will be back right after supper.”. Time is not completely understood by a 4 year-old. Preschoolers interpret time with their own frame of reference. Thus, it is best to explain time in relationship to a known, common event.

A) Scratching the head more than usual B) Flakes evident on a student’s shoulders C) Oval pattern occipital hair loss D) Whitish oval specks sticking to the hair

Question38

The nurse is giving instructions to the parents of a child with cystic fibrosis. The nurse would emphasize that pancreatic enzymes should be taken A) once each day B) 3 times daily after meals C) with each meal or snack D) each time carbohydrates are eaten Review Information: The correct answer is C: with each meal or snack. Pancreatic enzymes should be taken with each meal and every snack to allow for digestion of all foods that are eaten.

Question39

A nurse is providing a parenting class to individuals living in a community of older homes. In discussing formula preparation, which of the following is most important to prevent lead poisoning? A) Use ready-to-feed commercial infant formula B) Boil the tap water for 10 minutes prior to preparing the formula C) Let tap water run for 2 minutes before adding to concentrate D) Buy bottled water labeled “lead free” to mix the formula Review Information: The correct answer is C: Let tap water run for 2 minutes before adding to concentrate. Use of lead-contaminated water to prepare formula is a major source of poisoning in infants. Drinking water may be contaminated by lead from old lead pipes or lead solder used in sealing water pipes. Letting tap water run for

Which of the following manifestations observed by the school nurse confirms the presence of pediculosis capitis in students?

Review Information: The correct answer is D: Whitish oval specks sticking to the hair. Diagnosis of pediculosis capitis is made by observation of the white eggs (nits) firmly attached to the hair shafts. Treatment can include application of a medicated shampoo with lindane for children over 2 years of age, and meticulous combing and removal of all nits.

Question41

When interviewing the parents of a child with asthma, it is most important to assess the child’s environment for what factor? A) Household pets B) New furniture C) Lead based paint D) Plants such as cactus Review Information: The correct answer is A: Household pets. Animal dander is a very common allergen affecting persons with asthma. Other triggers may include pollens, carpeting and household dust.

Question42

The mother of a 2 month-old baby calls the nurse 2 days after the first DTaP, IPV, Hepatitis B and HIB immunizations. She reports that the baby feels very warm, cries inconsolably for as long as 3 hours, and has had several shaking spells. In addition to referring her to the emergency room, the nurse should document the reaction on the baby’s record and expect which immunization to be most associated with the findings the infant is displaying? Collected by :DeepaRajesh [ 152 ] [email protected] Kuwait

A) DTaP B) Hepatitis B C) Polio D) H. Influenza Review Information: The correct answer is A: DTaP. The majority of reactions occur with the administration of the DTaP vaccination. Contradictions to giving repeat DTaP immunizations include the occurrence of severe side effects after a previous dose as well as signs of encephalopathy within 7 days of the immunization.

Question43

port persons. To communicate in a therapeutic manner, the nurse’’s goal is to help the couple begin the grief process by suggesting they talk to each other, seek family, friends and support groups to listen to their feelings.

Question45

The nurse is performing a pre-kindergarten physical on a 5 year-old. The last series of vaccines will be administered. What is the preferred site for injection by the nurse? A) vastus intermedius B) gluteus maximus C) vastus lateralis D) dorsogluteaI

The mother of a 2 year-old hospitalized child asks the nurse’s advice about the child’s screaming every time the mother gets ready to leave the hospital room. What is the best response by the Review Information: The correct answer is C: vastus lateralis. Vastus lateralis, a large and well nurse? developed muscle, is the preferred site, since it is A) “I think you or your partner needs to stay with removed from major nerves and blood vessels. the child while in the hospital.” B) “Oh, that behavior will stop in a few days.” C) “Keep in mind that for the age this is a normal Question46 response to being in the hospital.” A 7 month pregnant woman is admitted with comD) “You might want to “sneak out” of the room plaints of painless vaginal bleeding over several once the child falls asleep.” hours. The nurse should prepare the client for an immediate Review Information: The correct answer is C: “Keep in mind that for the age this is a normal A) Non stress test response to being in the hospital.”. The protest B) Abdominal ultrasound phase of separation anxiety is a normal response C) Pelvic exam for a child this age. In toddlers, ages 1 to 3, sepa- D) X-ray of abdomen ration anxiety is at its peak Review Information: The correct answer is B: Abdominal ultrasound. The standard for diagnosis of placenta previa, which is suggested in the Question44 A couple experienced the loss of a 7 month-old client’’s history of painless bleeding, is abdomifetus. In planning for discharge, what should the nal ultrasound. nurse emphasize? A) To discuss feelings with each other and use support persons B) To focus on the other healthy children and move through the loss C) To seek causes for the fetal death and come to some safe conclusion D) To plan for another pregnancy within 2 years and maintain physical health

Question47

A nurse entering the room of a postpartum mother observes the baby lying at the edge of the bed while the woman sits in a chair. The mother states “This is not my baby, and I do not want it.” After repositioning the child safely, the nurse’s best response is

A) “This is a common occurrence after birth, but Review Information: The correct answer is A: you will come to accept the baby.” To discuss feelings with each other and use sup- B) “Many women have postpartum blues and Collected by :DeepaRajesh [ 153 ] [email protected] Kuwait

need some time to love the baby.” C) “What a beautiful baby! Her eyes are just like yours.” D) “You seem upset; tell me what the pregnancy and birth were like for you.”

when the effective arterial blood volume falls. Examples of this phenomena include a drop in circulating blood volume as in a cardiac arrest state or in low cardiac perfusion states such as congestive heart failure associated with a cardiomyopathy. Close observation of hourly urinary Review Information: The correct answer is D: output is necessary for early detection of this “You seem upset; tell me what the pregnancy and condition. birth were like for you.”. A non-judgmental, open ended response facilitates dialogue between the client and nurse. Question50 A client is admitted to the rehabilitation unit following a cerebral vascular accident (CVA) and Question48 mild dysphagia. The most appropriate intervenThe nurse notes that a 2 year-old child recov- tion for this client is to ering from a tonsillectomy has an temperature of 98.2 degrees Fahrenheit at 8:00 AM. At 10:00 A) position client in upright position while eating AM the child’s parent reports that the child “feels B) place client on a clear liquid diet very warm” to touch. The first action by the nurse C) tilt head back to facilitate swallowing reflex should be to D) offer finger foods such as crackers or pretzels A) reassure the parent that this is normal B) offer the child cold oral fluids Review Information: The correct answer is A: C) reassess the child’s temperature position client in upright position while eating. An D) administer the prescribed acetaminophen upright position facilitates proper chewing and swallowing. Review Information: The correct answer is C: reassess the child’’s temperature. A child’’s temperature may have rapid fluctuations. The Question51 nurse should listen to and show respect for what A 72 year-old client with osteomyelitis requires a parents say. Parental caretakers are often quite 6 week course of intravenous antibiotics. In plansensitive to variations in their children’’s condi- ning for home care, what is the most important tion that may not be immediately evident to oth- action by the nurse? ers. A) Investigating the client’s insurance coverage for home IV antibiotic therapy Question49 B) Determining if there are adequate hand washThe nurse is caring for a client who was success- ing facilities in the home fully resuscitated from a pulseless dysrhythmia. C) Assessing the client’s ability to participate in Which of the following assessments is critical for self care and/or the reliability of a caregiver the nurse to include in the plan of care? D) Selecting the appropriate venous access device A) hourly urine output Review Information: The correct answer is C: B) white blood count C) blood glucose every 4 hours Assessing the client’’s ability to participate in self D) temperature every 2 hours care and/or the reliability of a caregiver. The cognitive ability of the client as well as the availability Review Information: The correct answer is A: and reliability of a caregiver must be assessed to hourly urine output. Clients who have had an epi- determine if home care is a feasible option. sode of decreased glomerular perfusion are at risk for pre-renal failure. This is caused by any abnormal decline in kidney perfusion that reduc- Question52 es glomerular perfusion. Pre-renal failure occurs A nurse administers the influenza vaccine to a Collected by :DeepaRajesh [ 154 ] [email protected] Kuwait

client in a clinic. Within 15 minutes after the immunization was given, the client complains of itchy and watery eyes, increased anxiety, and difficulty breathing. The nurse expects that the first action in the sequence of care for this client will be to A) Maintain the airway B) Administer epinephrine 1:1000 as ordered C) Monitor for hypotension with shock D) Administer diphenhydramine as ordered

Tardive dyskinesia. Signs of tardive dyskinesia include smacking lips, grinding of teeth and “fly catching” tongue movements. These findings are often described as Parkinsonian.

Question55

Which of the following findings contraindicate the use of haloperidol (Haldol) and warrant withholding the dose? A) Drowsiness, lethargy, and inactivity B) Dry mouth, nasal congestion, and blurred viReview Information: The correct answer is B: sion Administer epinephrine 1:1000 as ordered. All C) Rash, blood dyscrasias, severe depression the answers are correct given the circumstances, D) Hyperglycemia, weight gain, and edema but the priority is to administer the epinephrine, then maintain the airway. In the early stages of Review Information: The correct answer is anaphylaxis, when the patient has not lost con- C: Rash, blood dyscrasias, severe depression. sciousness and is normotensive, administering Rash and blood dyscrasias are side effects of the epinephrine is first, and applying the oxygen, anti-psychotic drugs. A history of severe depresand watching for hypotension and shock, are lat- sion is a contraindication to the use of neuroleper responses. The prevention of a severe crisis tics. is maintained by using diphenhydramine.

Question53

The nurse instructs the client taking dexamethasone (Decadron) to take it with food or milk. The physiological basis for this instruction is that the medication A) retards pepsin production B) stimulates hydrochloric acid production C) slows stomach emptying time D) decreases production of hydrochloric acid Review Information: The correct answer is B: stimulates hydrochloric acid production. Decadron increases the production of hydrochloric acid, which may cause gastrointestinal ulcers.

Question54

A client receiving chlorpromazine HCL (Thorazine) is in psychiatric home care. During a home visit the nurse observes the client smacking her lips alternately with grinding her teeth. The nurse recognizes this assessment finding as what? A) Dystonia B) Akathisia C) Brady dyskinesia D) Tardive dyskinesia Review Information: The correct answer is D:

Question56

The nurse is reinforcing teaching to a 24 year-old woman receiving acyclovir (Zovirax) for a Herpes Simplex Virus type 2 infection. Which of these instructions should the nurse give the client? A) Complete the entire course of the medication for an effective cure B) Begin treatment with acyclovir at the onset of symptoms of recurrence C) Stop treatment if she thinks she may be pregnant to prevent birth defects D) Continue to take prophylactic doses for at least 5 years after the diagnosis Review Information: The correct answer is B: Begin treatment with acyclovir at the onset of symptoms of recurrence. When the client is aware of early symptoms, such as pain, itching or tingling, treatment is very effective. Medications for herpes simplex do not cure the disease; they simply decrease the level of symptoms.

Question57

A 14 month-old child ingested half a bottle of aspirin tablets. Which of the following would the nurse expect to see in the child? Collected by :DeepaRajesh [ 155 ] [email protected] Kuwait

A) Hypothermia B) Edema C) Dyspnea D) Epistaxis Review Information: The correct answer is D: Epistaxis. A large dose of aspirin inhibits prothrombin formation and lowers platelet levels. With an overdose, clotting time is prolonged.

Question58

An 80 year-old client on digitalis (Lanoxin) reports nausea, vomiting, abdominal cramps and halo vision. Which of the following laboratory results should the nurse analyze first? A) Potassium levels B) Blood pH C) Magnesium levels D) Blood urea nitrogen

occurred. The child received twice the ordered dose of morphine an hour ago. Which nursing diagnosis is a priority at this time? A) Risk for fluid volume deficit related to morphine overdose B) Decreased gastrointestinal mobility related to mucosal irritation C) Ineffective breathing patterns related to central nervous system depression D) Altered nutrition related to inability to control nausea and vomiting Review Information: The correct answer is C: Ineffective breathing patterns related to central nervous system depression. Respiratory depression is a life-threatening risk in this overdose.

Question61

Lactulose (Chronulac) has been prescribed for a client with advanced liver disease. Which of the Review Information: The correct answer is A: following assessments would the nurse use to Potassium levels. The most common cause of evaluate the effectiveness of this treatment? digitalis toxicity is a low potassium level. Clients must be taught that it is important to have ad- A) An increase in appetite equate potassium intake especially if taking diu- B) A decrease in fluid retention retics that enhance the loss of potassium while C) A decrease in lethargy D) A reduction in jaundice they are taking digitalis.

Question59

A 42 year-old male client refuses to take propranolol hydrochloride (Inderal) as prescribed. Which client statement from the assessment data is likely to explain his noncompliance? A) “I have problems with diarrhea.” B) “I have difficulty falling asleep.” C) “I have diminished sexual function.” D) “I often feel jittery.” Review Information: The correct answer is C: “I have diminished sexual function.”. Inderal, a beta-blocking agent used in hypertension, prohibits the release of epinephrine into the cells; this may result in hypotension which results in decreased libido and impotence.

Review Information: The correct answer is C: A decrease in lethargy. Lactulose produces an acid environment in the bowel and traps ammonia in the gut; the laxative effect then aids in removing the ammonia from the body. This decreases the effects of hepatic encephalopathy, including lethargy and confusion.

Question62

The nurse is teaching a class on HIV prevention. Which of the following should be emphasized as increasing risk? A) Donating blood B) Using public bathrooms C) Unprotected sex D) Touching a person with AIDS

Review Information: The correct answer is C: Unprotected sex. Because HIV is spread through Question60 The nurse caring for a 9 year-old child with a exposure to bodily fluids, unprotected intercourse fractured femur is told that a medication error and shared drug paraphernalia remain the highCollected by :DeepaRajesh [ 156 ] [email protected] Kuwait

est risks for infection.

Question63

While interviewing a new admission, the nurse notices that the client is shifting positions, wringing her hands, and avoiding eye contact. It is important for the nurse to A) ask the client what she is feeling B) assess the client for auditory hallucinations C) recognize the behavior as a side effect of medication D) re-focus the discussion on a less anxiety provoking topic Review Information: The correct answer is A: ask the client what she is feeling. The initial step in anxiety intervention is observing, identifying, and assessing anxiety. The nurse should seek client validation of the accuracy of nursing assessments and avoid drawing conclusions based on limited data. In the situation above, the client may simply need to use the restroom but be reluctant to communicate her need!

Question64

A young adult seeks treatment in an outpatient mental health center. The client tells the nurse he is a government official being followed by spies. On further questioning, he reveals that his warnings must be heeded to prevent nuclear war. What is the most therapeutic approach by the nurse? A) Listen quietly without comment B) Ask for further information on the spies C) Confront the client’s delusion D) Contact the government agency Review Information: The correct answer is A: Listen quietly without comment. The client’’s comments demonstrate grandiose ideas. The most therapeutic response is to listen but avoid being incorporated into the client’s delusional system.

Question65

The nurse is assessing a 17 year-old female client with bulimia. Which of the following laboratory reports would the nurse anticipate? A) Increased serum glucose

B) Decreased albumin C) Decreased potassium D) Increased sodium retention Review Information: The correct answer is C: Decreased potassium. In bulimia, loss of electrolytes can occur in addition to other findings of starvation and dehydration.

Question66

A client, recovering from alcoholism, asks the nurse, “What can I do when I start recognizing relapse triggers within myself?” How might the nurse best respond? A) “When you have the impulse to stop in a bar, contact a sober friend and talk with him.” B) “Go to an AA meeting when you feel the urge to drink.” C) “It is important to exercise daily and get involved in activities that will cause you not to think about drug use.” D) “Let’s talk about possible options you have when you recognize relapse triggers in yourself.” Review Information: The correct answer is D: “Let’s talk about possible options you have when you recognize relapse triggers in yourself.”. This option encourages the process of self evaluation and problem solving, while avoiding telling the client what to do. Encouraging the client to brainstorm about response options validates the nurse’s belief in the client’s personal competency and reinforces a coping strategy that will be needed when the nurse may not be available to offer solutions.

Question67

Therapeutic nurse-client interaction occurs when the nurse A) assists the client to clarify the meaning of what the client has said B) interprets the client’s covert communication C) praises the client for appropriate feelings and behavior D) advises the client on ways to resolve problems Review Information: The correct answer is A: assists the client to clarify the meaning of what the client has said. Clarification is a facilitating/ Collected by :DeepaRajesh [ 157 ] [email protected] Kuwait

therapeutic communication strategy. Interpreta- suddenly walks up to the nurse and shouts “You tion, changing the focus/subject, giving approv- think you’re so perfect and pure and good.” An al, and advising are non-therapeutic/barriers to appropriate response for the nurse is communication. A) “Is that why you’ve been staring at me?” B) “You seem to be in a really bad mood.” C) “Perfect? I don’t quite understand.” Question68 Which nursing intervention will be most effective D) “You seem angry right now.” in helping a withdrawn client to develop relationReview Information: The correct answer is D: ship skills? A) Offer the client frequent opportunities to inter- “You seem angry right now.”. The nurse recognizes the underlying emotion with a matter of fact act with 1 person B) Provide the client with frequent opportunities attitude, but avoids telling the clients how they feel. to interact with other clients C) Assist the client to analyze the meaning of the withdrawn behavior D) Discuss with the client the focus that other Question71 clients have similar problems A client who is a former actress enters the day room wearing a sheer nightgown, high heels, nuReview Information: The correct answer is A: merous bracelets, bright red lipstick and heavily Offer the client frequent opportunities to interact rouged cheeks. Which nursing action is the best with 1 person. The withdrawn client is uncomfort- in response to the client’s attire? able in social interaction. The nurse-client relationship is a corrective relationship in which the A) Gently remind her that she is no longer on client learns both tolerance and skills for relation- stage ships. B) Directly assist client to her room for appropriate apparel C) Quietly point out to her the dress of other clients on the unit Question69 An important goal in the development of a thera- D) Tactfully explain appropriate clothing for the hospital peutic inpatient milieu is to A) provide a businesslike atmosphere where clients can work on individual goals B) provide a group forum in which clients decide on unit rules, regulations, and policies C) provide a testing ground for new patterns of behavior while the client takes responsibility for his or her own actions D) discourage expressions of anger because they can be disruptive to other clients Review Information: The correct answer is C: provide a testing ground for new patterns of behavior while the client takes responsibility for his or her own actions. A therapeutic milieu is purposeful and planned to provide safety and a testing ground for new patterns of behavior.

Question70

A client with paranoid delusions stares at the nurse over a period of several days. The client

Review Information: The correct answer is B: Directly assist client to her room for appropriate apparel. It assists the client to maintain self-esteem while modifying behavior.

Question72

When teaching suicide prevention to the parents of a 15 year-old who recently attempted suicide, the nurse describes the following behavioral cue as indicating a need for intervention. A) Angry outbursts at significant others B) Fear of being left alone C) Giving away valued personal items D) Experiencing the loss of a boyfriend Review Information: The correct answer is C: Giving away valued personal items. Eighty percent of all potential suicide victims give some type of indication that self-destructiveness should be Collected by :DeepaRajesh [ 158 ] [email protected] Kuwait

addressed. These clues might lead one to suspect that a client is having suicidal thoughts or is developing a plan.

Question73

Which statement made by a client indicates to the nurse that the client may have a thought disorder? A) “I’m so angry about this. Wait until my partner hears about this.” B) “I’m a little confused. What time is it?” C) “I can’t find my ‘mesmer’ shoes. Have you seen them?” D) “I’m fine. It’s my daughter who has the problem.”

A) brittle hair, lanugo, amenorrhea B) diarrhea, nausea, vomiting, dental erosion C) hyperthermia, tachycardia, increased metabolic rate D) excessive anxiety about symptoms Review Information: The correct answer is A: brittle hair, lanugo, amenorrhea. Physical findings associated with anorexia also include reduced metabolic rate and lower vital signs.

Free NCLEX-RN Sample Test Questions For Nursing Review (Pharmacology Set 2) Jul31,

A nurse is assigned to perform well-child assessments at a day care center. A staff member interrupts the examinations to ask for assistance. They find a crying 3 year-old child on the floor with mouth wide open and gums bleeding. Two unlabeled open bottles lie nearby. The nurse’s first action should be A) call the poison control center, then 911 B) administer syrup of Ipecac to induce vomiting Question74 C) give the child milk to coat her stomach In a psychiatric setting, the nurse limits touch D) ask the staff about the contents of the bottles or contact used with clients to handshaking because Review Information: The correct answer is D: A) some clients misconstrue hugs as an invita- ask the staff about the contents of the bottles tion to sexual advances The nurse needs to assess what the child inB) handshaking keeps the gesture on a profes- gested before determining the next action. Once sional level the substance is identified, the poison control C) refusal to touch a client denotes lack of con- center and emergency response team should be cern called. D) inappropriate touch often results in charges of assault and battery Question2 Review Information: The correct answer is C: “I can’’t find my ‘’mesmer’’ shoes. Have you seen them?”. A neologism is a new word self invented by a person and not readily understood by another. Using neologisms is often associated with a thought disorder.

Review Information: The correct answer is A: some clients misconstrue hugs as an invitation to sexual advances. Touch denotes positive feelings for another person. The client may interpret hugging and holding hands as sexual advances.

Question75

A client with anorexia is hospitalized on a medical unit due to electrolyte imbalance and cardiac dysrhythmias. Additional assessment findings that the nurse would expect to observe are

A client with atrial fibrillation is receiving digoxin (Lanoxin). Which of these assessments is most important for the nurse to perform? A) Monitor blood pressure every 4 hours B) Measure apical pulse prior to administration C) Maintain accurate intake and output records D) Record an EKG strip after administration

Review Information: The correct answer is B: Measure apical pulse prior to administration Digitoxin decreases conduction velocity through the AV node and prolongs the refractory period. If the apical heart rate is less than 60 beats/minute, withhold the drug. The apical pulse should be Collected by :DeepaRajesh [ 159 ] [email protected] Kuwait

taken with a stethoscope so that there will be no mistake about what the heart rate actually is.

Question3

C) Peripheral edema D) Jaundice

The nurse is administering an intravenous vesicant chemotherapeutic agent to a client. Which assessment would require the nurse’s immediate action? A) Stomatitis lesion in the mouth B) Severe nausea and vomiting C) Complaints of pain at site of infusion D) A rash on the client’s extremities

Review Information: The correct answer is A: Buffalo hump With high doses of glucocorticoid, iatrogenic Cushing>>s syndrome develops. The exaggerated physiological action causes abnormal fat distribution which results in a moon-shaped face, a intrascapular pad on the neck (buffalo hump) and truncal obesity with slender limbs.

Review Information: The correct answer is C: Complaints of pain at site of infusion A vesicant is a chemotherapeutic agent capable of causing blistering of tissues and possible tissue necrosis if there is extravasation. These agents are irritants which cause pain along the vein wall, with or without inflammation.

Question6

Question4

The nurse practicing in a long term care facility recognizes that elderly clients are at greater risk for drug toxicity than younger adults because of which of the following physiological changes of advancing age? A) Drugs are absorbed more readily from the GI tract B) Elders have less body water and more fat C) The elderly have more rapid hepatic metabolism D) Older people are often malnourished and anemic Review Information: The correct answer is B: Elders have less body water and more fat Because elderly persons have decreased lean body tissue/water in which to distribute medications, more drug remains in the circulatory system with potential for drug toxicity. Increased body fat results in greater amounts of fat-soluble drugs being absorbed, leaving less in circulation, thus increasing the duration of action of the drug

Question5

The nurse is assessing a client who is on long term glucocorticoid therapy. Which of the following findings would the nurse expect? A) Buffalo hump B) Increased muscle mass

The health care provider has written «Morphine sulfate 2 mgs IV every 3-4 hours prn for pain» on the chart of a child weighing 22 lb. (10 kg). What is the nurse>s initial action? A) Check with the pharmacist B) Hold the medication and contact the provider C) Administer the prescribed dose as ordered D) Give the dose every 6-8 hours Review Information: The correct answer is B: Hold the medication and contact the provider The usual pediatric dose of morphine is 0.1 mg/ kg every 3 to 4 hours. At 10 kg, this child typically should receive 1.0 mg every 3 to 4 hours.

Question7

A client is ordered atropine to be administered preoperatively. Which physiological effect should the nurse monitor for? A) Elevate blood pressure B) Drying up of secretions C) Reduce heart rate D) Enhance sedation Review Information: The correct answer is B: Drying up of secretions Atropine dries secretions which may get in the way during the operative procedure.

Question8

A client is receiving digitalis. The nurse should instruct the client to report which of the following side effects? A) Nausea, vomiting, fatigue B) Rash, dyspnea, edema C) Polyuria, thirst, dry skin Collected by :DeepaRajesh [ 160 ] [email protected] Kuwait

D) Hunger, dizziness, diaphoresis

with documented acetaminophen poisoning. In order to provide counseling and education for the parents, which principle must the nurse unReview Information: The correct answer is A: derstand? Nausea, vomiting, fatigue A) The problem occurs in stages with recovery Side effects of digitalis toxicity include fatigue, within 12-24 hours nausea, vomiting, anorexia, and bradycardia. B) Hepatic problems may occur and may be lifeDigitalis inhibits the sodium potassium ATPase, threatening which makes more calcium available for contrac- C) Full and rapid recovery can be expected in tile proteins, resulting in increased cardiac out- most children put. D) This poisoning is usually fatal, as no antidote is available

Question9

A client is receiving dexamethasone (Decadron) therapy. What should the nurse plan to monitor in this client? A) Urine output every 4 hours B) Blood glucose levels every 12 hours C) Neurological signs every 2 hours D) Oxygen saturation every 8 hours Review Information: The correct answer is B: Blood glucose levels every 12 hours The drug Decadron increases glycogenesis. This may lead to hyperglycemia. Therefore the blood sugar level and acetone production must be monitored.

Question10

The nurse is caring for a client with schizophrenia who has been treated with quetiapine (Seroquel) for 1 month. Today the client is increasingly agitated and complains of muscle stiffness. Which of these findings should be reported to the health care provider? A) Elevated temperature and sweating. B) Decreased pulse and blood pressure. C) Mental confusion and general weakness. D) Muscle spasms and seizures.

Review Information: The correct answer is B: Hepatic problems may occur and may be lifethreatening Clinical manifestations associated with acetaminophen poisoning occurs in 4 stages. The third stage is hepatic involvement which may last up to 7 days and be permanent. Clients who do not die in the hepatic stage gradually recover.

Question12

A client has been receiving dexamethasone (Decadron) for control of cerebral edema. Which of the following assessments would indicate that the treatment is effective? A) A positive Babinski>s reflex B) Increased response to motor stimuli C) A widening pulse pressure D) Temperature of 37 degrees Celsius

Review Information: The correct answer is B: Increased response to motor stimuli Decadron is a corticosteroid that acts on the cell membrane to decrease inflammatory responses as well as stabilize the blood-brain barrier. Once Decadron reaches a therapeutic level, there Review Information: The correct answer is A: should be a decrease in symptomology with improvement in motor skills. Elevated temperature and sweating. Neuroleptic malignant syndrome (NMS) is a rare disorder that can occur as a side effect of antipsy- Question13 chotic medications. It is characterized by muscu- The provider has ordered transdermal nitroglyclar rigidity, tachycardia, hyperthermia, sweating, erin patches for a client. Which of these instrucaltered consciousness, autonomic dysfunction, tions should be included when teaching a client and increase in CPK. This is a life-threatening about how to use the patches? complication. A) Remove the patch when swimming or bathing B) Apply the patch to any non-hairy area of the Question11 A child presents to the Emergency Department body Collected by :DeepaRajesh [ 161 ] [email protected] Kuwait

C) Apply a second patch with chest pain D) Remove the patch if ankle edema occurs Review Information: The correct answer is B: Apply the patch to any non-hairy area of the body The patch application sites should be rotated.

Question14

A newly admitted client has a diagnosis of depression. She complains of “twitching muscles” and a “racing heart”, and states she stopped taking Zoloft a few days ago because it was not helping her depression. Instead, she began to take her partner>s Parnate. The nurse should immediately assess for which of these adverse reactions? A) Pulmonary edema B) Atrial fibrillation C) Mental status changes D) Muscle weakness Review Information: The correct answer is C: Mental status changes Use of serotonergic agents may result in Serotonin Syndrome with confusion, nausea, palpitations, increased muscle tone with twitching muscles, and agitation. Serotonin syndrome is most often reported in patients taking 2 or more medications that increase CNS serotonin levels by different mechanisms. The most common drug combinations associated with serotonin syndrome involve the MAOIs, SSRIs, and the tricyclic antidepressants.

Question15

A client with bi-polar disorder is taking lithium (Lithane). What should the nurse emphasize when teaching about this medication? A) Take the medication before meals B) Maintain adequate daily salt intake C) Reduce fluid intake to minimize diuresis D) Use antacids to prevent heartburn Review Information: The correct answer is B: Maintain adequate daily salt intake Salt intake affects fluid volume, which can affect lithium (Lithane) levels; therefore, maintaining adequate salt intake is advised.

Question16

A client with anemia has a new prescription for ferrous sulfate. In teaching the client about diet and iron supplements, the nurse should emphasize that absorption of iron is enhanced if taken with which substance? A) Acetaminophen B) Orange juice C) Low fat milk D) An antacid Review Information: The correct answer is B: Orange juice Ascorbic acid enhances the absorption of iron.

Question17

A client with an aplastic sickle cell crisis is receiving a blood transfusion and begins to complain of «feeling hot.» Almost immediately, the client begins to wheeze. What is the nurse>s first action? A) Stop the blood infusion B) Notify the health care provider C) Take/record vital signs D) Send blood samples to lab Review Information: The correct answer is A: Stop the blood infusion If a reaction of any type is suspected during administration of blood products, stop the infusion immediately, keep the line open with saline, notify the health care provider, monitor vital signs and other changes, and then send a blood sample to the lab.

Question18

A client confides in the RN that a friend has told her the medication she takes for depression, Wellbutrin, was taken off the market because it caused seizures. What is an appropriate response by the nurse? A) «Ask your friend about the source of this information.» B) «Omit the next doses until you talk with the doctor.» C) «There were problems, but the recommended dose is changed.» D) «Your health care provider knows the best drug for your condition.» Review Information: The correct answer is C: «There were problems, but the recommended Collected by :DeepaRajesh [ 162 ] [email protected] Kuwait

dose is changed.» Wellbutrin was introduced in the U.S. in 1985 and then withdrawn because of the occurrence of seizures in some patients taking the drug. The drug was reintroduced in 1989 with specific recommendations regarding dose ranges to limit the occurrence of seizures. The risk of seizure appears to be strongly associated with dose.

B) «Follow up with your primary care provider in 3 months.» C) «Continue to take your medications even when you are feeling fine.» D) «Continue to get yearly tuberculin skin tests.»

Review Information: The correct answer is C: «Continue to take your medications even when Question19 When providing discharge teaching to a client you are feeling fine.» with asthma, the nurse will warn against the use The most important piece of information the tuof which of the following over-the-counter medi- berculosis client needs is to understand the importance of medication compliance, even if no cations? longer experiencing symptoms. Clients are most A) Cortisone ointments for skin rashes infectious early in the course of therapy. The B) Aspirin products for pain relief numbers of acid-fast bacilli are greatly reduced C) Cough medications containing guaifenesin as early as 2 weeks after therapy begins. D) Histamine blockers for gastric distress Review Information: The correct answer is B: Aspirin products for pain relief Aspirin is known to induce asthma attacks. Aspirin can also cause nasal polyps and rhinitis. Warn individuals with asthma about signs and symptoms resulting from complications due to aspirin ingestion.

Question20

The nurse is caring for a client who is receiving procainamide (Pronestyl) intravenously. It is important for the nurse to monitor which of the following parameters? A) Hourly urinary output B) Serum potassium levels * C) Continuous EKG readings D) Neurological signs Review Information: The correct answer is C: Continuous EKG readings Procainamide (Pronestyl) is used to suppress cardiac arrhythmias. When administered intravenously, it must be accompanied by continuous cardiac monitoring by ECG.

Question22

The nurse is applying silver sulfadiazine (Silvadene) to a child with severe burns to arms and legs. Which side effect should the nurse be monitoring for? A) Skin discoloration B) Hardened eschar C) Increased neutrophils D) Urine sulfa crystals Review Information: The correct answer is D: Urine sulfa crystals Silver sulfadiazine is a broad spectrum antimicrobial, especially effective against pseudomonas. When applied to extensive areas, however, it may cause a transient neutropenia, as well as renal function changes with sulfa crystals production and kernicterus.

Question23

The nurse is monitoring a client receiving a thrombolytic agent, alteplase (Activase tissue plasminogen activator), for treatment of a myocardial infarction. What outcome indicates the client is receiving adequate therapy within the first hours of treatment? A) Absence of a dysrhythmia (or arrhythmia) Question21 The nurse is providing education for a client with B) Blood pressure reduction newly diagnosed tuberculosis. Which statement C) Cardiac enzymes are within normal limits should be included in the information that is giv- D) Return of ST segment to baseline on ECG en to the client? A) «Isolate yourself from others until you are finReview Information: The correct answer is D: ished taking your medication.» Collected by :DeepaRajesh [ 163 ] [email protected] Kuwait

Return of ST segment to baseline on ECG Improved perfusion should result from this medication, along with the reduction of ST segment elevation.

Question24

The provider has ordered daily high doses of aspirin for a client with rheumatoid arthritis. The nurse instructs the client to discontinue the medication and contact the provider if which of the following symptoms occur? A) Infection of the gums B) Diarrhea for more than one day C) Numbness in the lower extremities D) Ringing in the ears Review Information: The correct answer is D: Ringing in the ears Aspirin stimulates the central nervous system which may result in ringing in the ears. Deglin, J.D. and Vallerand, A.H. (2001). Davis’ drug guide for nurses. (7th edition). Philadelphia: F.A. Davis Company. Key, J.L. and Hayes, E.R. (2003). Pharmacology, a nursing process approach. (4th edition). Philadelphia: Saunders.

Question25

A nurse is caring for a client who is receiving methyldopa hydrochloride (Aldomet) intravenously. Which of the following assessment findings would indicate to the nurse that the client may be having an adverse reaction to the medication? A) Headache B) Mood changes C) Hyperkalemia D) Palpitations Review Information: The correct answer is B: Mood changes The nurse should assess the client for alterations in mental status such as mood changes. These symptoms should be reported promptly. Deglin, J.D. and Vallerand, A.H. (2001). Davis’ drug guide for nurses. (7th edition). Philadelphia: F.A. Davis Company. Wilson, B.A., Shannon, M.T., and Stang, C.L. (2004). Nurse’s drug guide. Upper Saddle River, New Jersey: Pearson Prentice Hall.

Question26

The nurse is teaching a child and the family about the medication phenytoin (Dilantin) prescribed for seizure control. Which of the following side effects is most likely to occur? A) Vertigo B) Drowsiness C) Gingival hyperplasia D) Vomiting

Review Information: The correct answer is C: Gingival hyperplasia Swollen and tender gums occur often with use of phenytoin. Good oral hygiene and regular visits to the dentist should be emphasized.

Question27

The use of atropine for treatment of symptomatic bradycardia is contraindicated for a client with which of the following conditions? A) Urinary incontinence B) Glaucoma C) Increased intracranial pressure D) Right sided heart failure Review Information: The correct answer is B: Glaucoma Atropine is contraindicated in clients with angleclosure glaucoma because it can cause pupillary dilation with an increase in aqueous humor, leading to a resultant increase in optic pressure.

Question28

A pregnant woman is hospitalized for treatment of pregnancy induced hypertension (PIH) in the third trimester. She is receiving magnesium sulfate intravenously. The nurse understands that this medication is used mainly for what purpose? A) Maintain normal blood pressure B) Prevent convulsive seizures C) Decrease the respiratory rate D) Increase uterine blood flow Review Information: The correct answer is B: Prevent convulsive seizures Magnesium sulfate is a central nervous system depressant. While it has many systemic effects, it is used in the client with pregnancy induced Collected by :DeepaRajesh [ 164 ] [email protected] Kuwait

hypertension (PIH) to prevent seizures.

Question32

A post-operative client has a prescription for Question29 acetaminophen with codeine. What should the The nurse is teaching a group of women in a com- nurse recognizes as a primary effect of this community clinic about prevention of osteoporosis. bination? Which of the following over-the-counter medica- A) Enhanced pain relief tions should the nurse recognize as having the B) Minimized side effects most elemental calcium per tablet? C) Prevention of drug tolerance A) Calcium chloride D) Increased onset of action B) Calcium citrate C) Calcium gluconate D) Calcium carbonate Review Information: The correct answer is A: Enhanced pain relief Combination of analgesics with different mechaReview Information: The correct answer is D: nisms of action can afford greater pain relief. Calcium carbonate Calcium carbonate contains 400mg of elemental calcium in 1 gram of calcium carbonate. Question33 A client is receiving erythromycin 500mg IV eveQuestion30 ry 6 hours to treat a pneumonia. Which of the The nurse is administering diltiazem (Cardizem) following is the most common side effect of the to a client. Prior to administration, it is important medication? for the nurse to assess which parameter? A) Blurred vision A) Temperature B) Nausea and vomiting B) Blood pressure C) Severe headache C) Vision D) Insomnia D) Bowel sounds Review Information: The correct answer is B: Blood pressure Diltiazem (Cardizem) is a calcium channel blocker that causes systemic vasodilation resulting in decreased blood pressure.

Question31

The nurse is instructing a client with moderate persistent asthma on the proper method for using MDIs (multi-dose inhalers). Which medication should be administered first? A) Steroid B) Anticholinergic C) Mast cell stabilizer D) Beta agonist Review Information: The correct answer is D: Beta agonist The beta-agonist drugs help to relieve bronchospasm by relaxing the smooth muscle of the airway. These drugs should be taken first so that other medications can reach the lungs.

Review Information: The correct answer is B: Nausea and vomiting Nausea is a common side-effect of erythromycin in both oral and intravenous forms.

Question34

The health care provider orders an IV aminophylline infusion at 30 mg/hr. The pharmacy sends a 1,000 ml bag of D5W containing 500 mg of aminophylline. In order to administer 30 mg per hour, the RN will set the infusion rate at: A) 20 ml per hour B) 30 ml per hour C) 50 ml per hour D) 60 ml per hour Review Information: The correct answer is D: 60 ml per hour Using the ratio method to calculate infusion rate: mg to be given (30) : ml to be infused (X) :: mg available (500) : ml of solution (1,000). Solve for X by cross-multiplying: 30 x 1,000 = 500 x X (or cancel), 30,000 = 500 X, X = 30,000/500, X = Collected by :DeepaRajesh [ 165 ] [email protected] Kuwait

60ml per hour.

of meperidine, is a central nervous system stimulant that produces anxiety, tremors, myoclonus, and generalized seizures when it accumulates Question35 The nurse is assessing a 7 year-old after several with repetitive dosing. Clients with sickle cell disdays of treatment for a documented strep throat. ease are particularly at risk for normeperidineWhich of the following statements suggests that induced seizures. further teaching is needed? A) «Sometimes I take my medicine with fruit Question38 A 5 year-old has been rushed to the emergency juice.» B) «My mother makes me take my medicine right room several hours after acetaminophen poisoning. Which laboratory result should receive attenafter school.» C) «Sometimes I take the pills in the morning tion by the nurse? A) Sedimentation rate and other times at night.» D) «I am feeling much better than I did last B) Profile 2 C) Bilirubin week.» D) Neutrophils Review Information: The correct answer is C: «Sometimes I take the pills in the morning and other times at night.» Inconsistency in taking the prescribed medication indicates more teaching is needed.

Question36

The nurse is caring for a 10 year-old client who will be placed on heparin therapy. Which assessment is critical for the nurse to make before initiating therapy A) Vital signs B) Weight C) Lung sounds D) Skin turgor Review Information: The correct answer is B: Weight Check the client>>s weight because dosage is calculated on the basis of weight.

Question37

In providing care for a client with pain from a sickle cell crisis, which one of the following medication orders for pain control should be questioned by the nurse? A) Demerol B) Morphine C) Methadone D) Codeine

Review Information: The correct answer is C: Bilirubin Bilirubin, along with liver enzymes ALT and AST, may rise in the second stage (1-3 days) after a significant overdose, indicating cellular necrosis and liver dysfunction. A prolonged prothrombin time may also be found.

Question39

An elderly client is on an anticholinergic metered dose inhaler (MDI) for chronic obstructive pulmonary disease. The nurse would suggest a spacer to A) enhance the administration of the medication B) increase client compliance C) improve aerosol delivery in clients who are not able to coordinate the MDI D) prevent exacerbation of COPD Review Information: The correct answer is C: improve aerosol delivery in clients who are not able to coordinate the MDI Spacers improve the medication delivery in clients who are unable to coordinate the movements of administering a dose with an MDI.

Question40

The nurse is teaching a parent how to administer oral iron supplements to a 2 year-old child. Review Information: The correct answer is A: Which of the following interventions should be included in the teaching? Demerol Meperidine is not recommended in clients with A) Stop the medication if the stools become tarry sickle cell disease. Normeperidine, a metabolite green Collected by :DeepaRajesh [ 166 ] [email protected] Kuwait

B) Give the medicine with orange juice and through a straw C) Add the medicine to a bottle of formula D) Administer the iron with your child>s meals Review Information: The correct answer is B: Give the medicine with orange juice and through a straw Absorption of iron is facilitated in an environment rich in Vitamin C. Since liquid iron preparation will stain teeth, a straw is preferred. 0 comments Labels: free nclex-rn sample review questions, nclex-rn practice test questions, nursing review

Free NCLEX-RN Sample Test Questions For Nursing Review (Pharmacology Set 1) Question1

A client has an order for antibiotic therapy after hospital treatment of a staph infection. Which of the following should the nurse emphasize? A) Scheduling follow-up blood cultures B) Completing the full course of medications C) Visiting the provider in a few weeks D) Monitoring for signs of recurrent infection Review Information: The correct answer is B: Completing the full course of medications In order for antibiotic therapy to be effective in eradicating an infection, the client must compete the entire course of prescribed therapy. When findings subside, stopping the medication early may lead to recurrence or subsequent drug resistance.

Question2

Review Information: The correct answer is B: a decreased sensation of thirst The elderly have a reduction in thirst sensation causing them to consume less fluid. Other risk factors may include fear of incontinence, inability to drink fluids independently and lack of motivation.

Question3

A male client is admitted with a spinal cord injury at level C4. The client asks the nurse how the injury is going to affect his sexual function. The nurse would respond A) «Normal sexual function is not possible.» B) «Sexual functioning will not be impaired at all.» C) «Erections will be possible.» D) «Ejaculation will be normal.»

Review Information: The correct answer is C: «Erections will be possible.» Because they are a reflex reaction, erections can be stimulated by stroking the genitalia.

Question4

An 82 year-old client complains of chronic constipation. To improve bowel function, the nurse should first suggest A) Increasing fiber intake to 20-30 grams daily B) Daily use of laxatives C) Avoidance of binding foods such as cheese and chocolate D) Monitoring a balance between activity and rest Review Information: The correct answer is A: Increasing fiber intake to 20-30 grams daily The incorporation of high fiber into the diet is an effective way to promote bowel elimination in the elderly.

A 72 year-old client is admitted for possible dehydration. The nurse knows that older adults are particularly at risk for dehydration because they have A) an increased need for extravascular fluid Question5 B) a decreased sensation of thirst A 4 year-old child is admitted with burns on his C) an increase in diaphoresis legs and lower abdomen. When assessing the D) higher metabolic demands child’s hydration status, which of the following indicates a less than adequate fluid replacement? Collected by :DeepaRajesh [ 167 ] [email protected] Kuwait

A) Decreasing hematocrit and increasing urine The range for a therapeutic APTT is 1.5-2 times volume the control. Therefore the client is receiving a B) Rising hematocrit and decreasing urine vol- therapeutic dose of Heparin. ume C) Falling hematocrit and decreasing urine volume Question8 D) Stable hematocrit and increasing urine vol- A client is admitted with a diagnosis of nodal ume bigeminy. The nurse knows that the atrioventricular (AV) node has an intrinsic rate of A) 60-100 beats/minute Review Information: The correct answer is B: B) 10-30 beats/minute Rising hematocrit and decreasing urine volume C) 40-70 beats/minute A rising hematocrit indicates a decreased total D) 20-50 beats/minute blood volume, a finding consistent with dehydration. Review Information: The correct answer is C: 40-70 beats/minute The intrinsic rate of the AV node is within the Question6 A client receiving chemotherapy has developed range of 40-70 beats per minute. sores in his mouth. He asks the nurse why this happened. What is the nurse’s best response? A) «It is a sign that the medication is working.» Question9 B) «You need to have better oral hygiene.» A client is to receive 3 doses of potassium chloC) «The cells in the mouth are sensitive to the ride 10 mEq in 100cc normal saline to infuse chemotherapy.» over 30 minutes each. Which of the following is a D) «This always happens with chemotherapy.» priority assessment to perform before giving this medication? A) Oral fluid intake B) Bowel sounds C) Grip strength Review Information: The correct answer is D) Urine output C: «The cells in the mouth are sensitive to the chemotherapy.» The epithelial cells in the mouth are very sensi- Review Information: The correct answer is D: tive to chemotherapy due to their high rate of cell Urine output turnover. Potassium chloride should only be administered after adequate urine output (>20cc/hour for 2 consecutive hours) has been established. Impaired ability to excrete potassium via the kidQuestion7 You are caring for a client with deep vein throm- neys can result in hyperkalemia. bosis who is on Heparin IV. The latest APTT is 50 seconds. If the laboratory normal range is 16-24 seconds, you would anticipate Question10 A) maintaining the current heparin dose The unlicensed assistive personnel (UAP) reB) increasing the heparin as it does not appear ports to the nurse that a client with cirrhosis who therapeutic. had a paracentesis yesterday has become more C) giving protamine sulfate as an antidote. lethargic and has musty smelling breath. A critiD) repeating the blood test 1 hour after giving cal assessment for increasing encephalopathy heparin. is A) monitor the client>s clotting status B) assess upper abdomen for bruits Review Information: The correct answer is A: C) assess for flap-like tremors of the hands maintaining the current heparin dose D) measure abdominal girth changes Collected by :DeepaRajesh [ 168 ] [email protected] Kuwait

D) high calories Review Information: The correct answer is C: assess for flap-like tremors of the hands A client with cirrhosis of the liver who develops subtle changes in mental status and has a musty odor to the breath is at risk for developing more advanced signs of encephalopathy.

Question11

A client is scheduled for an intravenous pyelogram (IVP). After the contrast material is injected, which of the following client reactions should be reported immediately? A) Feeling warm B) Face flushing C) Salty taste D) Hives Review Information: The correct answer is D: Hives This is a sign of anaphylaxis and should be reported immediately. The other reactions are considered normal and the client should be informed that they may occur. .

Question12

A client is prescribed an inhaler. How should the nurse instruct the client to breathe in the medication? A) As quickly as possible B) As slowly as possible C) Deeply for 3-4 seconds D) Until hearing whistling by the spacer Review Information: The correct answer is C: Deeply for 3-4 seconds The client should be instructed to breath in the medication for 3-4 seconds in order to receive the correct dosage of medication.

Question13

The nurse is caring for clients over the age of 70. The nurse knows that due to age-related changes, the elderly clients tolerate diets that are A) high protein B) high carbohydrates C) low fat

Review Information: The correct answer is C: low fat Due to age related changes, the diet of the elderly should include a lower quantity and higher quality of food. Fewer carbohydrates and fats are required in their diets.

Question14

A woman with a 28 week pregnancy is on the way to the emergency department by ambulance with a tentative diagnosis of abruptio placenta. Which should the nurse do first when the woman arrives? A) administer oxygen by mask at 100% B) start a second IV with an 18 gauge cannula C) check fetal heart rate every 15 minutes D) insert urethral catheter with hourly urine outputs Review Information: The correct answer is A: administer oxygen by mask at 100% Administering oxygen in this situation would increase the circulating oxygen in the mother’s circulation to the fetus’s circulation. This action will minimize complications.

Question15

A client in respiratory distress is admitted with arterial blood gas results of: PH 7.30; PO2 58, PCO2 34; and HCO3 19. The nurse determines that the client is in A) metabolic acidosis B) metabolic alkalosis C) respiratory acidosis D) respiratory alkalosis

Review Information: The correct answer is A: metabolic acidosis These lab values indicate metabolic acidosis: the PH is low, PCO2 is normal, and bicarbonate level is low.

Question16 Collected by :DeepaRajesh [ 169 ] [email protected] Kuwait

A client is diagnosed with gastroesophageal re- With aggressive treatment and early detection/ flux disease (GERD). The nurse>s instruction to diagnosis the cure rate is 90%. the client regarding diet should be to A) avoid all raw fruits and vegetables B) increase intake of milk products Question19 C) decrease intake of fatty foods A client newly diagnosed with Type I Diabetes D) focus on 3 average size meals a day Mellitus asks the purpose of the test measuring glycosylated hemoglobin. The nurse should explain that the purpose of this test is to deterReview Information: The correct answer is C: mine: decrease intake of fatty foods A) The presence of anemia often associated with GERD may be aggravated by a fatty diet. A diet Diabetes low in fat would decrease the symptoms of GERD. B) The oxygen carrying capacity of the client>s Other agents which should also be decreased or red cells avoided are: cigarette smoking, caffeine, alco- C) The average blood glucose for the past 2-3 hol, chocolate, and meperidine (Demerol). months D) The client>s risk for cardiac complications

Question17

After surgery, a client with a nasogastric tube complains of nausea. What action would the nurse take? A) Call the health care provider B) Administer an antiemetic C) Put the bed in Fowler’s position D) Check the patency of the tube Review Information: The correct answer is D: Check the patency of the tube An indication that the nasogastric tube is obstructed is a client’s complaint of nausea. Nasogastric tubes may become obstructed with mucus or sediment.

Question18

A client with testicular cancer has had an orchiectomy. Prior to discharge the client expresses his fears related to his prognosis. Which principle should the nurse base the response on? A) Testicular cancer has a cure rate of 90% with early diagnosis B) Testicular cancer has a cure rate of 50% with early diagnosis C) Intensive chemotherapy is the treatment of choice D) Testicular cancer is usually fatal Review Information: The correct answer is A: Testicular cancer has a cure rate of 90% with early diagnosis

Review Information: The correct answer is C: The average blood glucose for the past 2-3 months By testing the portion of the hemoglobin that absorbs glucose, it is possible to determine the average blood glucose over the life span of the red cell, 120 days.

Question20

A client is admitted for a possible pacemaker insertion. What is the intrinsic rate of the heart>s own pacemaker? A) 30-50 beats/minute B) 60-100 beats/minute C) 20-60 beats/minute D) 90-100 beats/minute Review Information: The correct answer is B: 60-100 beats/minute This is the intrinsic rate of the SA node.

Question21

The nurse discusses nutrition with a pregnant woman who is iron deficient and follows a vegetarian diet. The selection of which foods indicates the woman has learned sources of iron? A) Cereal and dried fruits B) Whole grains and yellow vegetables C) Leafy green vegetables and oranges Collected by :DeepaRajesh [ 170 ] [email protected] Kuwait

D) Fish and dairy products

recognize as the purpose of this medication? A) Reduce vascularity of the thyroid B) Correct chronic hyperthyroidism Review Information: The correct answer is A: C) Destroy the thyroid gland function Cereal and dried fruits D) Balance enzymes and electrolytes Both of these foods would be a good source of iron. Review Information: The correct answer is A: Prior to administering Alteplase (TPA) to a client Reduce vascularity of the thyroid admitted for a cerebral vascular accident (CVA), Potassium iodide solution, or Lugol>>s solution may be used preoperatively to reduce the size it is critical that the nurse assess: and vascularity of the thyroid gland. A) Neuro signs B) Mental status C) Blood pressure D) PT/PTT Question25 One hour before the first treatment is scheduled, the client becomes anxious and states he does Review Information: The correct answer is D: not wish to go through with electroconvulsive PT/PTT therapy. Which response by the nurse is most TPA is a potent thrombolytic enzyme. Because appropriate? bleeding is the most common side effect, it is A) «I’ll go with you and will be there with you durmost essential to evaluate clotting studies in- ing the treatment.» cluding PT, PTT, APTT, platelets, and hematocrit B) «You’ll be asleep and won’t remember anybefore beginning therapy. thing.» C) «You have the right to change your mind. You seem anxious. Can we talk about it?» D) «I’ll call the health care provider to notify them Question23 The nurse enters the room of a client diagnosed of your decision.» with COPD. The client’s skin is pink, and respirations are 8 per minute. The client’s oxygen is running at 6 liters per minute. What should be Review Information: The correct answer is C: «You have the right to change your mind. You the nurse’s first action? seem anxious. Can we talk about it?» A) Call the health care provider This response indicates acknowledgment of the B) Put the client in Fowler’s position client’s rights and the opportunity for the client C) Lower the oxygen rate to clarify and ventilate concerns. After this, if the D) Take the vital signs client continues to refuse, the provider should be notified. Review Information: The correct answer is C: Lower the oxygen rate In client’s diagnosed with COPD, the drive to Question26 breathe is hypoxia. If oxygen is delivered at too A nurse who has been named in a lawsuit can high of a concentration, this drive will be elimi- use which of these factors for the best protection nated and the client’s depth and rate of respi- in a court of law? rations will decrease. Therefore the first action A) Clinical specialty certification in the associshould be to lower the oxygen rate. ated area of practice B) Documentation on the specific client record with a focus on the nursing process C) Yearly evaluations and proficiency reports Question24 The client with goiter is treated with potassium prepared by nurse’s manager iodide preoperatively. What should the nurse D) Verification of provider>s orders for the plan

Question22

Collected by :DeepaRajesh [ 171 ] [email protected] Kuwait

of care with identification of outcomes Review Information: The correct answer is B: Documentation on the specific client record with a focus on the nursing process Documentation is the key to protect nurses when a lawsuit is filed. The thorough documentation should include all steps of the nursing process – assessment, analysis, plan, intervention, evaluation. In addition, it should include pertinent data such as times, dosages and sites of actions, assessment data, the nurse’s response to a change in the client’s condition, specific actions taken, if and when the notification occurred to the provider or other health care team members, and what was prescribed along with the client’s outcomes.

Question27

The nurse is caring for clients over the age of 70. The nurse is aware that when giving medications to older clients, it is best to A) start low, go slow B) avoid stopping a medication entirely C) avoid drugs with side effects that impact cognition D) review the drug regimen yearly

Avoid the use of salt substitutes Captopril can cause an accumulation of potassium or hyperkalemia. Clients should avoid the use of salt substitutes, which are generally potassium-based.

Question29

A client has bilateral knee pain from osteoarthritis. In addition to taking the prescribed non-steroidal anti-inflammatory drug (NSAID), the nurse should instruct the client to A) start a regular exercise program B) rest the knees as much as possible to decrease inflammation C) avoid foods high in citric acid D) keep the legs elevated when sitting Review Information: The correct answer is A: start a regular exercise program A regular exercise program is beneficial in treating osteoarthritis. It can restore self-esteem and improve physical functioning.

Question30

An arterial blood gases test (ABG) is ordered for a confused client. The respiratory therapist draws the blood and then asks the nurse to apply pressure to the area so the therapist can take the Review Information: The correct answer is A: specimen to the lab. How long should the nurse start low, go slow apply pressure to the area? Due to physiological changes in the elderly, as A) 3 minutes well as conditions such as dehydration, hyper- B) 5 minutes thermia, immobility and liver disease, the effec- C) 8 minutes tive metabolism of drugs may decrease. As a D) 10 minutes result, drugs can accumulate to toxic levels and cause serious adverse reactions. Review Information: The correct answer is B: 5 minutes Question28 It is necessary to apply pressure to the area for You are caring for a hypertensive client with a 5 minutes to prevent bleeding and the formation new order for captopril (Capoten). Which infor- of hematomas. mation should the nurse include in client teaching? A) Avoid green leafy vegetables Question31 B) Restrict fluids to 1000cc/day Which of these clients should the charge nurse C) Avoid the use of salt substitutes assign to the registered nurse (RN)? D) Take the medication with meals A) A 56 year-old with atrial fibrillation receiving digoxin B) A 60 year-old client with COPD on oxygen at Review Information: The correct answer is C: 2 L/min Collected by :DeepaRajesh [ 172 ] [email protected] Kuwait

C) A 24 year-old post-op client with type 1 diabetes in the process of discharge D) An 80 year-old client recovering 24 hours post right hip replacement

Question34

Upon admission to an intensive care unit, a client diagnosed with an acute myocardial infarction is ordered oxygen. The nurse knows that the major reason that oxygen is administered in this situation is to A) saturate the red blood cells B) relieve dyspnea C) decrease cyanosis D) increase oxygen level in the myocardium

Review Information: The correct answer is C: A 24 year-old post-op client with type 1 diabetes in the process of discharge Discharge teaching must be done by an RN. Practical nurses (PNs) or unlicensed assistive Review Information: The correct answer is D: personnel (UAPs) can reinforce education after increase oxygen level in the myocardium the RN does the initial teaching. Anoxia of the myocardium occurs in myocardial infarction. Oxygen administration will help relieve dyspnea and cyanosis associated with the condition but the major purpose is to increase Question32 A hypertensive client is started on atenolol (Ten- the oxygen concentration in the damaged myoormin). The nurse instructs the client to immedi- cardial tissue. . ately report which of these findings? A) Rapid breathing B) Slow, bounding pulse Question35 C) Jaundiced sclera The nurse is teaching a client with chronic reD) Weight gain nal failure (CRF) about medications. The client questions the purpose of aluminum hydroxide (Amphojel) in her medication regimen. What is Review Information: The correct answer is B: the best explanation for the nurse to give the cliSlow, bounding pulse ent about the therapeutic effects of this medicaAtenolol (Tenormin) is a beta-blocker that can tion? cause side effects including bradycardia and hy- A) It decreases serum phosphate potension. B) It will reduce serum calcium C) Amphojel increases urine output D) The drug is taken to control gastric acid secretion Question33 An 80 year-old client is admitted with a diagnosis of malnutrition. In addition to physical assessments, which of the following lab tests should be Review Information: The correct answer is A: It decreases serum phosphate closely monitored? Aluminum binds phosphates that tend to accuA) Urine protein mulate in the patient with chronic renal failure B) Urine creatinine due to decreased filtration capacity of the kidney. C) Serum calcium Antacids such as Amphojel are commonly used D) Serum albumin to accomplish this. Review Information: The correct answer is D: Serum albumin Serum albumin is a valuable indicator of protein deficiency and, later, nutritional status in adults. A normal reading for an elder’s serum albumin is between 3.0-5.0 g/dl.

Question36

A 66 year-old client is admitted for mitral valve replacement surgery. The client has a history of mitral valve regurgitation and mitral stenosis since her teenage years. During the admission assessment, the nurse should ask the client if as a child she had Collected by :DeepaRajesh [ 173 ] [email protected] Kuwait

A) measles B) rheumatic fever C) hay fever D) encephalitis

Review Information: The correct answer is B: rheumatic fever Clients that present with mitral stenosis often have a history of rheumatic fever or bacterial endocarditis.

Question37

During nursing rounds which of these assessments would require immediate corrective action and further instruction to the practical nurse (PN) about proper care? A) The weights of the skin traction of a client are hanging about 2 inches from the floor B) A client with a hip prosthesis 1 day post operatively is lying in bed with internal rotation and adduction of the affected leg C) The nurse observes that the PN moves the extremity of a client with an external fixation device by picking up the frame D) A client with skeletal traction states «The other nurse said that the clear, yellow and crusty drainage around the pin site is a good sign» Review Information: The correct answer is B: A client with a hip prosthesis 1 day post operatively is lying in bed with internal rotation and adduction of the affected leg This position should be prevented in order to prevent dislodgment of the hip prosthesis, especially in the first 48 to 72 hours post-op. The other assessments are not of concern.

Review Information: The correct answer is D: intake of at least 3000cc/day Fluid intake should be increased to prevent precipitation of urate in the kidneys.

Question39

A 55 year-old woman is taking Prednisone and aspirin (ASA) as part of her treatment for rheumatoid arthritis. Which of the following would be an appropriate intervention for the nurse? A) Assess the pulse rate q 4 hours B) Monitor her level of consciousness q shift C) Test her stools for occult blood D) Discuss fiber in the diet to prevent constipation Review Information: The correct answer is C: Test her stools for occult blood Both Prednisone and ASA can lead to GI bleeding, therefore monitoring for occult blood would be appropriate.

Question40

A client with testicular cancer is scheduled for a right orchiectomy. The nurse knows that an orchiectomy is the A) surgical removal of the entire scrotum B) surgical removal of a testicle C) dissection of related lymph nodes D) partial surgical removal of the penis Review Information: The correct answer is B: surgical removal of a testicle The affected testicle is surgically removed along with its tunica and spermatic cord. 0 comments

Question38

A client diagnosed with gouty arthritis is admitted with severe pain and edema in the right foot. When the nurse develops a plan of care, which intervention should be included? A) high protein diet B) salicylates C) hot compresses to affected joints D) intake of at least 3000cc/day

Labels: free nclex-rn sample review questions, nclex-rn practice test questions, nursing review

Free NCLEX-RN Sample Test Questions For Nursing Review (Part 5) Question1

A client complains of some discomfort after a below the knee amputation. Which action by the nurse is most appropriate initially? Collected by :DeepaRajesh [ 174 ] [email protected] Kuwait

A) Conduct guided imagery or distraction B) Days 10-13 B) Ensure that the stump is elevated the first day C) Days 14-16 post-op D) Days 17-19 C) Wrap the stump snugly in an elastic bandage D) Administer opioid narcotics as ordered Review Information: The correct answer is D: Days 17-19 Review Information: The correct answer is B: Ovulation occurs 14 days prior to menses. ConEnsure that the stump is elevated the first day sidering that the woman>>s cycle is 32 days, post-op subtracting 14 from 32 suggests ovulation is at This priority intervention prevents pressure about the 18th day. caused by pooling of blood, thus minimizing the pain. Without this measure, a firm elastic bandage, opioid narcotics, or guided imagery will Question4 have little effect. Opioid narcotics are given for A newborn is having difficulty maintaining a temsevere pain. perature above 98 degrees Fahrenheit and has been placed in an incubator. Which action is a nursing priority? A) Protect the eyes of the neonate from the heat Question2 A 78 year-old client with pneumonia has a pro- lamp ductive cough, but is confused. Safety protective B) Monitor the neonate’s temperature devices (restraints) have been ordered for this C) Warm all medications and liquids before giving client. How can the nurse prevent aspiration? A) Suction the client frequently while restrained D) Avoid touching the neonate with cold hands B) Secure all 4 restraints to 1 side of bed C) Obtain a sitter for the client while restrained D) Request an order for a cough suppressant Review Information: The correct answer is C: Obtain a sitter for the client while restrained The plan to use safety devices (restraints) should be rethought. Restraints are used to protect the client from harm caused by removing tubes or getting out of bed. In the event that this restricted movement could cause more harm, such as aspiration, then a sitter should be requested. These are to be provided by the facility in the event the family cannot do so. This client needs to cough and be watched rather than restricted. Suctioning will not prevent aspiration in this situation. Cough suppressants should be avoided for this client.

Question3

A couple trying to conceive asks the nurse when ovulation occurs. The woman reports a regular 32 day cycle. Which response by the nurse is correct? A) Days 7-10

Review Information: The correct answer is B: Monitor the neonate’s temperature When using a warming device the neonate’s temperature should be continuously monitored for undesired elevations. The use of heat lamps is not safe as there is no way to regulate their temperature. Warming medications and fluids is not indicated. While touching with cold hands can startle the infant it does not pose a safety risk.

Question5

Which oxygen delivery system would the nurse apply that would provide the highest concentrations of oxygen to the client? A) Venturi mask B) Partial rebreather mask C) Non-rebreather mask D) Simple face mask

Collected by :DeepaRajesh [ 175 ] [email protected] Kuwait

Review Information: The correct answer is C: Non-rebreather mask The non-rebreather mask has a one-way valve that prevents exhales air from entering the reservoir bag and one or more valves covering the air holes on the face mask itself to prevent inhalation of room air but to allow exhalation of air. When a tight seal is achieved around the mask up to 100% of the oxygen is available.

Question6

At a senior citizens meeting a nurse talks with a client who has Type 1 diabetes mellitus. Which statement by the client during the conversation is most predictive of a potential for impaired skin integrity? A) «I give my insulin to myself in my thighs.» B) «Sometimes when I put my shoes on I don>t know where my toes are.» C) «Here are my up and down glucose readings that I wrote on my calendar.» D) «If I bathe more than once a week my skin feels too dry.»

Review Information: The correct answer is D: Outline the spot with a pencil and note the time and date on the cast This is a good way to assess the amount of bleeding over a period of time. The bleeding does not appear to be excessive and some bleeding is expected with this type of surgery. The bleeding should also be documented in the nurse’s notes.

Question8

The nurse is caring for a 1 year-old child who has 6 teeth. What is the best way for the nurse to give mouth care to this child? A) Using a moist soft brush or cloth to clean teeth and gums B) Swabbing teeth and gums with flavored mouthwash C) Offering a bottle of water for the child to drink D) Brushing with toothpaste and flossing each tooth

Review Information: The correct answer is A: Using a moist soft brush or cloth to clean teeth and gums The nurse should use a soft cloth or soft brush to do mouth care so that the child can adjust to the routine of cleaning the mouth and teeth.

Review Information: The correct answer is B: «Sometimes when I put my shoes on I don>>t know where my toes are.» Peripheral neuropathy can lead to lack of sensation in the lower extremities. Clients who do not feel pressure and/or pain are at high risk for skin Question9 impairment. In addition to standard precautions, a nurse should implement contact precautions for which client? Question7 A) 60 year-old with herpes simplex A client returns from surgery after an open reduc- B) 6 year-old with mononucleosis tion of a femur fracture. There is a small blood- C) 45 year-old with pneumonia stain on the cast. Four hours later, the nurse ob- D) 3 year-old with scarlet fever serves that the stain has doubled in size. What is the best action for the nurse to take? A) Call the health care provider B) Access the site by cutting a window in the cast Review Information: The correct answer is A: C) Simply record the findings in the nurse>s 60 year-old with herpes simplex notes only Clients who have herpes simplex infections must D) Outline the spot with a pencil and note the have contact precautions in addition to standard time and date on the cast precautions because of the associated, potentially weeping, skin lesions. Contact precautions are Collected by :DeepaRajesh [ 176 ] [email protected] Kuwait

used for clients who are infected by microorgan- B) «I take half of the usual dose for my sinuses isms that are transmitted by direct contact with to maintain my blood pressure.» the client, including hand or skin-to-skin contact. C) «I have to sit at the side of the pool with the grandchildren since I can>t swim with this eye problem.» D) «I take extra fiber and drink lots of water to Question10 Which of the following situations is most likely to avoid getting constipated.» produce sepsis in the neonate? A) Maternal diabetes B) Prolonged rupture of membranes C) Cesarean delivery Review Information: The correct answer is D: D) Precipitous vaginal birth «I take extra fiber and drink lots of water to avoid getting constipated.» Any activity that involves straining should be avoided in clients with glaucoma. Such activities Review Information: The correct answer is B: would increase intraocular pressure. Prolonged rupture of membranes Premature rupture of the membranes (PROM) is a leading cause of newborn sepsis. After 12-24 Question13 hours of leaking fluid, measures are taken to re- A newborn has hyperbilirubinemia and is unduce the risk to mother and the fetus/newborn. dergoing phototherapy with a fiberoptic blanket. Which safety measure is most important during this process? A) Regulate the neonate’s temperature using a radiant heater B) Withhold feedings while under the phototherQuestion11 The nurse is teaching a parent about side effects apy of routine immunizations. Which of the following C) Provide water feedings at least every 2 hours D) Protect the eyes of neonate from the photomust be reported immediately? therapy lights A) Irritability B) Slight edema at site C) Local tenderness Review Information: The correct answer is C: D) Seizure activity Provide water feedings at least every 2 hours Protecting the eyes of the neonates is very important to prevent damage when under the ultraviolet lights, but since the blanket is used, extra Review Information: The correct answer is D: protection of the eyes is unnecessary. It is recommended that the neonate remain under the Seizure activity Other reactions that should be reported include lights for extended periods. The neonate’s skin is crying for >3 hours, temperature over 104.8 de- exposed to the light and the temperature is monigrees Fahrenheit following DPT immunization, tored, but a heater may not be necessary. There is no reason to withhold feedings. Frequent waand tender, swollen, reddened areas. ter or feedings are given to help with the excretion of the bilirubin in the stool. . Question12 The nurse is at the community center speaking with retired people about glaucoma. Which comment by one of the retirees would the nurse support to reinforce correct information? A) «I usually avoid driving at night since lights sometimes seem to make things blur.»

Question14

A nurse is performing the routine daily cleaning of a tracheostomy. During the procedure, the client coughs and displaces the tracheostomy tube. This negative outcome could have avoided by Collected by :DeepaRajesh [ 177 ] [email protected] Kuwait

A) placing an obturator at the client’s bedside B) having another nurse assist with the procedure C) fastening clean tracheostomy ties before removing old ties D) placing the client in a flat, supine position

Review Information: The correct answer is C: fastening clean tracheostomy ties before removing old ties Fastening clean tracheostomy ties before removing old ones will ensure that the tracheostomy is secured during the entire cleaning procedure. The obturator is useful to keep the airway open only after the tracheostomy outer tube is coughed out. A second nurse is not needed. Changing the position may not prevent a dislodged tracheostomy.

Question15

A 4 year-old hospitalized child begins to have a seizure while playing with hard plastic toys in the hallway. Of the following nursing actions, which one should the nurse do first? A) Place the child in the nearest bed B) Administer IV medication to slow down the seizure C) Place a padded tongue blade in the child>s mouth D) Remove the child>s toys from the immediate area

Review Information: The correct answer is D: Remove the child>>s toys from the immediate area Nursing care for a child having a seizure includes, maintaining airway patency, ensuring safety, administering medications, and providing emotional support. Since the seizure has already started, nothing should be forced into the child>>s mouth and the child should not be moved. Of the choices given, the first priority would be to provide a safe environment.

Question16

The nurse is teaching home care to the parents of a child with acute spasmodic croup. The most important aspects of this care is/are A) sedation as needed to prevent exhaustion B) antibiotic therapy for 10 to 14 days C) humidified air and increased oral fluids D) antihistamines to decrease allergic response

Review Information: The correct answer is C: humidified air and increased oral fluids The most important aspects of home care for a child with acute spasmodic croup are humidified air and increased oral fluids. Moisture soothes inflamed membranes. Adequate systemic hydration aids is mucociliary clearance and keeps secretions thin, white, watery, and easily removed with minimal coughing.

Question17

The nurse is assigned to care for a client who has a leaking intracranial aneurysm. To minimize the risk of rebleeding, the nurse should plan to A) restrict visitors to immediate family B) avoid arousal of the client except for family visits C) keep client>s hips flexed at no less than 90 degrees D) apply a warming blanket for temperatures of 98 degrees Fahrenheit or less

Review Information: The correct answer is A: restrict visitors to immediate family Maintaining a quiet environment will assist in minimizing cerebral rebleeding. When family visit, the client should not be disturbed. If the client is awake, topics of a general nature are better choices for discussion than topics that result in emotional or physiological stimulation.

Question18

A client who is 12 hour post-op becomes confused and says: “Giant sharks are swimming across the ceiling.” Which assessment is necessary to adequately identify the source of this client>s behavior? Collected by :DeepaRajesh [ 178 ] [email protected] Kuwait

A) Cardiac rhythm strip B) Pupillary response C) Pulse oximetry D) Peripheral glucose stick

Review Information: The correct answer is C: Pulse oximetry A sudden change in mental status in any postop client should trigger a nursing intervention directed toward respiratory evaluation. Pulse oximetry would be the initial assessment. If available, arterial blood gases would be better. Acute respiratory failure is the sudden inability of the respiratory system to maintain adequate gas exchange which may result in hypercapnia and/ or hypoxemia. Clinical findings of hypoxemia include these finding which are listed in order of initial to later findings: restlessness, irritability, agitation, dyspnea, disorientation, confusion, delirium, hallucinations, and loss of consciousness. While there may be other factors influencing the client>>s behavior, the first nursing action should be directed toward maintaining oxygenation. Once respiratory or oxygenation issues are ruled out then significant changes in glucose would be evaluated.

Fahrenheit (36 degrees Celsius).

Question20

Which contraindication should the nurse assess for prior to giving a child immunizations? A) Mild cold symptoms B) Chronic asthma C) Depressed immune system D) Allergy to eggs

Review Information: The correct answer is C: Depressed immune system Children who have a depressed immune system related to HIV or chemotherapy should not be given routine immunizations.

Question21

The nurse is caring for a client with a myocardial infarction. Which finding requires the nurse>s immediate action? A) Periorbital edema B) Dizzy spells C) Lethargy D) Shortness of breath

Question19

A newborn delivered at home without a birth attendant is admitted to the hospital for observation. The initial temperature is 95 degrees Fahrenheit (35 degrees Celsius) axillary. The nurse recognizes that cold stress may lead to what complication? A) Lowered BMR B) Reduced PaO2 C) Lethargy D) Metabolic alkalosis

Review Information: The correct answer is B: Dizzy spells Cardiac dysrhythmias may cause a transient drop in cardiac output and decreased blood flow to the brain. Near syncope refers to lightheartedness, dizziness, temporary confusion. Such «spells» may indicate runs of ventricular tachycardia or periods of asystole and should be reported immediately.

Question22 Review Information: The correct answer is B: Reduced PaO2 Cold stress causes increased risk for respiratory distress. The baby delivered in such circumstances needs careful monitoring. In this situation, the newborn must be warmed immediately to increase its temperature to at least 97 degrees

Decentralized scheduling is used on a nursing unit. A chief advantage of this management strategy is that it: A) considers client and staff needs B) conserves time spent on planning C) frees the nurse manager to handle other priorities D) allows requests for special privileges Collected by :DeepaRajesh [ 179 ] [email protected] Kuwait

Question25 Review Information: The correct answer is A: considers client and staff needs Decentralized staffing takes into consideration specific client needs and staff interests and abilities.

Question23

Included in teaching the client with tuberculosis taking isoniazid (INH) about follow-up home care, the nurse should emphasize that a laboratory appointment for which of the following lab tests is critical? A) Liver function B) Kidney function C) Blood sugar D) Cardiac enzymes

A 16 year-old boy is admitted for Ewing>s sarcoma of the tibia. In discussing his care with the parents, the nurse understands that the initial treatment most often includes A) amputation just above the tumor B) surgical excision of the mass C) bone marrow graft in the affected leg D) radiation and chemotherapy

Review Information: The correct answer is D: radiation and chemotherapy The initial treatment of choice for Ewing>>s sarcoma is a combination of radiation and chemotherapy.

Question26

A new nurse manager is responsible for interviewing applicants for a staff nurse position. Review Information: The correct answer is A: Which interview strategy would be the best apLiver function proach? INH can cause hepatocellular injury and hepati- A) Vary the interview style for each candidate to tis. This side effect is age-related and can be de- learn different techniques tected with regular assessment of liver enzymes, B) Use simple questions requiring «yes» and which are released into the blood from damaged «no» answers to gain definitive information liver cells. C) Obtain an interview guide from human resources for consistency in interviewing each candidate Question24 D) Ask personal information of each applicant to A woman in her third trimester complains of se- assure he/she can meet job demands vere heartburn. What is appropriate teaching by the nurse to help the woman alleviate these symptoms? Review Information: The correct answer is C: Obtain an interview guide from human resources A) Drink small amounts of liquids frequently for consistency in interviewing each candidate B) Eat the evening meal just before retiring C) Take sodium bicarbonate after each meal An interview guide used for each candidate enables the nurse manager to be more objective in D) Sleep with head propped on several pillows the decision making. The nurse should use resources available in the agency before attempts to develop one from scratch. Certain personal questions are prohibited, and HR can identify Review Information: The correct answer is D: these for novice managers. Sleep with head propped on several pillows Heartburn is a burning sensation caused by regurgitation of gastric contents. It is best relieved Question27 by sleeping position, eating small meals, and not What is the best way that parents of pre-schooleating before bedtime. ers can begin teaching their child about injury Collected by :DeepaRajesh [ 180 ] [email protected] Kuwait

prevention? A) Set good examples themselves B) Protect their child from outside influences C) Make sure their child understands all the safety rules D) Discuss the consequences of not wearing protective devices

oliguria Kidneys maintain fluid volume through adjustments in urine volume.

Question30

A 70 year-old woman is evaluated in the emergency department for a wrist fracture of unknown causes. During the process of taking client hisReview Information: The correct answer is A: tory, which of these items should the nurse idenSet good examples themselves tify as related to the client’s greatest risk factors The preschool years are the time for parents to for osteoporosis? begin emphasizing safety principles as well as A) History of menopause at age 50 providing protection. Setting a good example B) Taking high doses of steroids for arthritis for themselves is crucial because of the imitative many years behaviors of pre-schoolers; they are quick to no- C) Maintaining an inactive lifestyle for the past tice discrepancies between what they see and 10 years what they are told. D) Drinking 2 glasses of red wine each day for the past 30 years

Question28

Review Information: The correct answer is B: Taking high doses of steroids for arthritis for many years The use of steroids, especially at high doses over time, increases the risk for osteoporosis. The other options also predispose to osteoporosis, as do low bone mass, poor calcium absorption and moderate to high alcohol ingestion. Longterm steroid treatment is the most significant risk Review Information: The correct answer is A: factor, however. Disruption of fetal glucose supply After delivery, the high glucose levels which crossed the placenta to the fetus are suddenly Question31 stopped. The newborn continues to secrete insu- The nurse is caring for a 2 year-old who is being lin in anticipation of glucose. When oral feedings treated with chelation therapy, calcium disodium begin, the newborn will adjust insulin production edetate, for lead poisoning. The nurse should be within a day or two. alert for which of the following side effects? A) Neurotoxicity B) Hepatomegaly C) Nephrotoxicity Question29 The nurse is caring for a client with extracellular D) Ototoxicity fluid volume deficit. Which of the following assessments would the nurse anticipate finding? Review Information: The correct answer is C: A) bounding pulse Nephrotoxicity B) rapid respirations Nephrotoxicity is a common side effect of calciC) oliguria um disodium edetate, in addition to lead poisonD) neck veins are distended ing in general. A nurse assessing the newborn of a mother with diabetes understands that hypoglycemia is related to what pathophysiological process? A) Disruption of fetal glucose supply B) Pancreatic insufficiency C) Maternal insulin dependency D) Reduced glycogen reserves

Review Information: The correct answer is C:

Question32

The parents of a toddler ask the nurse how long Collected by :DeepaRajesh [ 181 ] [email protected] Kuwait

their child will have to sit in a car seat while in the automobile. What is the nurse’s best response to the parents? A) «Your child must use a care seat until he weighs at least 40 pounds.» B) «The child must be 5 years of age to use a regular seat belt.» C) «Your child must reach a height of 50 inches to sit in a seat belt.» D) «The child can use a regular seat belt when he can sit still.»

Review Information: The correct answer is A: «Your child must use a care seat until he weighs at least 40 pounds.» Children should use car seats until they weigh 40 pounds.

Question33

A client asks the nurse to explain the basic ideas of homeopathic medicine. The response that best explains this approach is that such remedies A) destroy organisms causing disease B) maintain fluid balance C) boost the immune system D) increase bodily energy

Review Information: The correct answer is C: boost the immune system The practitioner treats with minute doses of plant, mineral or animal substances which provide a gentle stimulus to the body>>s own defenses.

Perform a neurovascular check for circulation While each of these is an important assessment, the neurovascular integrity check is most associated with this type of traction. Russell’s traction is Buck’s traction with a sling under the knee.

Question35

When suctioning a client>s tracheostomy, the nurse should instill saline in order to A) decrease the client>s discomfort B) reduce viscosity of secretions C) prevent client aspiration D) remove a mucus plug Review Information: The correct answer is D: remove a mucus plug While no longer recommended for routine suctioning, saline may thin and loosen viscous secretions that are very difficult to move, perhaps making them easier to suction.

Question36

The nurse is performing a gestational age assessment on a newborn delivered 2 hours ago. When coming to a conclusion using the Ballard scale, which of these factors may affect the score? A) Birth weight B) Racial differences C) Fetal distress in labor D) Birth trauma

Review Information: The correct answer is C: Fetal distress in labor The effects of earlier distress may alter the findQuestion34 ings of reflex responses as measured on the BalA client with a fractured femur has been in Rus- lard tool. Other physical characteristics that estisell’s traction for 24 hours. Which nursing action mate gestational age, such as amount of lanugo, is associated with this therapy? sole creases and ear cartilage are unaffected by A) Check the skin on the sacrum for breakdown the other factors. B) Inspect the pin site for signs of infection C) Auscultate the lungs for atelectasis D) Perform a neurovascular check for circula- Question37 tion A nurse is caring for a client who had a closed reduction of a fractured right wrist followed by the application of a fiberglass cast 12 hours ago. Review Information: The correct answer is D: Which finding requires the nurse’s immediate atCollected by :DeepaRajesh [ 182 ] [email protected] Kuwait

tention? A) Capillary refill of fingers on right hand is 3 seconds B) Skin warm to touch and normally colored C) Client reports prickling sensation in the right hand D) Slight swelling of fingers of right hand Review Information: The correct answer is C: Client reports prickling sensation in the right hand A prickling sensation is an indication of compartment syndrome and requires immediate action by the nurse. The other findings are normal for a client in this situation.

Question38

A client is admitted with the diagnosis of pulmonary embolism. While taking a history, the client tells the nurse he was admitted for the same thing twice before, the last time just 3 months ago. The nurse would anticipate the provider ordering A) pulmonary embolectomy B) vena caval interruption C) increasing the Coumadin therapy to an INR of 3-4 D) thrombolytic therapy Review Information: The correct answer is B: vena caval interruption Clients with contraindications to Heparin, recurrent PE or those with complications related to the medical therapy may require vena caval interruption by the placement of a filter device in the inferior vena cava. A filter can be placed transvenously to trap clots before they travel to the pulmonary circulation.

Question39

Which client is at highest risk for developing a pressure ulcer? A) 23 year-old in traction for fractured femur B) 72 year-old with peripheral vascular disease, who is unable to walk without assistance C) 75 year-old with left sided paresthesia who is incontinent of urine and stool D) 30 year-old who is comatose following a ruptured aneurysm

Review Information: The correct answer is C: 75 year-old with left sided paresthesia who is incontinent of urine and stool Risk factors for pressure ulcers include: immobility, absence of sensation, decreased LOC, poor nutrition and hydration, skin moisture, incontinence, increased age, decreased immune response. This client has the greatest number of risk factors.

Question40

The nurse is teaching the mother of a 5 monthold about nutrition for her baby. Which statement by the mother indicates the need for further teaching? A) «I>m going to try feeding my baby some rice cereal.» B) «When he wakes at night for a bottle, I feed him.» C) «I dip his pacifier in honey so he>ll take it.» D) «I keep formula in the refrigerator for 24 hours.» Review Information: The correct answer is C: «I dip his pacifier in honey so he>>ll take it.» Honey has been associated with infant botulism and should be avoided. Older children and adults have digestive enzymes that kill the botulism spores. 0 comments Labels: free nclex-rn sample review questions, nclex-rn practice test questions, nursing review

Free NCLEX-RN Sample Test Questions For Nursing Review (Part 4) Question1

The clinic nurse is counseling a substance-abusing post partum client on the risks of continued cocaine use. In order to provide continuity of care, which nursing diagnosis is a priority? A) Social isolation B) Ineffective coping C) Altered parenting D) Sexual dysfunction Review Information: The correct answer is C: Collected by :DeepaRajesh [ 183 ] [email protected] Kuwait

Altered parenting The cocaine abusing mother puts her newborn and other children at risk for neglect and abuse. Continuing to use drugs has the potential to impact parenting behaviors. Social service referrals are indicated.

Question2

The nurse is teaching about nonsteroidal antiinflammatory drugs (NSAIDs) to a group of arthritic clients. To minimize the side effects, the nurse should emphasize which of the following actions? A) Reporting joint stiffness in the morning B) Taking the medication 1 hour before or 2 hours after meals C) Using alcohol in moderation unless driving D) Continuing to take aspirin for short term relief

cavity while instilling air into the gastric tube, this indicates that it is accurately placed in the stomach. The feeding can begin after further assessing the client for bowel sounds.

Question4

While assessing the vital signs in children, the nurse should know that the apical heart rate is preferred until the radial pulse can be accurately assessed at about what age? A) 1 year of age B) 2 years of age C) 3 years of age D) 4 years of age Review Information: The correct answer is B: 2 years of age A child should be at least 2 years of age to use the radial pulse to assess heart rate.

Question5 Review Information: The correct answer is B: Taking the medication 1 hour before or 2 hours after meals Taking the medication 1 hour before or 2 hours after meals will result in a more rapid effect.

Question3

The nurse is preparing to administer a tube feeding to a postoperative client. To accurately assess for a gastrostomy tube placement, the priority is to A) auscultate the abdomen while instilling 10 cc of air into the tube B) place the end of the tube in water to check for air bubbles C) retract the tube several inches to check for resistance D) measure the length of tubing from nose to epigastrium

Review Information: The correct answer is A: auscultate the abdomen while instilling 10 cc of air into the tube If a swoosh of air is heard over the abdominal

A client is receiving Total Parenteral Nutrition (TPN) via a Hickman catheter. The catheter accidentally becomes dislodged from the site. Which action by the nurse should take priority? A) Check that the catheter tip is intact B) Apply a pressure dressing to the site C) Monitor respiratory status D) Assess for mental status changes

Question6

A pregnant client who is at 34 weeks gestation is diagnosed with a pulmonary embolism (PE). Which of these medications would the nurse anticipate the provider ordering? A) Oral Coumadin therapy B) Heparin 5000 units subcutaneously B.I.D. C) Heparin infusion to maintain the PTT at 1.52.5 times the control value D) Heparin by subcutaneous injection to maintain the PTT at 1.5 times the control value

Review Information: The correct answer is D: Heparin by subcutaneous injection to maintain the PTT at 1.5 times the control value Several studies have been conducted in pregCollected by :DeepaRajesh [ 184 ] [email protected] Kuwait

nant women where oral anticoagulation agents are contraindicated. Warfarin (Coumadin) is known to cross the placenta and is therefore reported to be teratogenic.

Question7

The nurse is caring for a client with Hodgkin>s disease who will be receiving radiation therapy. The nurse recognizes that, as a result of the radiation therapy, the client is most likely to experience A) high fever B) nausea C) face and neck edema D) night sweats Review Information: The correct answer is B: nausea Because the client with Hodgkin>>s disease is usually healthy when therapy begins, the nausea is especially troubling. .

of the following is true in regards to giving immunizations to this child? A) Live vaccines are withheld in children with renal chronic illness B) The MMR vaccine should be given now, prior to the transplant C) An inactivated form of the vaccine can be given at any time D) The risk of vaccine side effects precludes giving the vaccine

Review Information: The correct answer is B: The MMR vaccine should be given now, prior to the transplant MMR is a live virus vaccine, and should be given at this time. Post-transplant, immunosuppressive drugs will be given and the administration of the live vaccine at that time would be contraindicated because of the compromised immune system.

Question10

The nurse is preparing to take a toddler>s blood A client is brought to the emergency room fol- pressure for the first time. Which of the following lowing a motor vehicle accident. When assess- actions should the nurse perform first? ing the client one-half hour after admission, the A) Explain that the procedure will help him to get nurse notes several physical changes. Which well finding would require the nurse>s immediate at- B) Show a cartoon character with a blood pressure cuff tention? C) Explain that the blood pressure checks the A) increased restlessness heart pump B) tachycardia D) Permit handling the equipment before putting C) tracheal deviation the cuff in place D) tachypnea

Question8

Review Information: The correct answer is C: tracheal deviation The deviated trachea is a sign that a mediastinal shift has occurred. This is a medical emergency.

Review Information: The correct answer is D: Permit handling the equipment before putting the cuff in place The best way to gain the toddler>>s cooperation is to encourage handling the equipment. Detailed explanations are not helpful.

Question9

Question11

An 18 month-old child is on peritoneal dialysis in preparation for a renal transplant in the near future. When the nurse obtains the child>s health history, the mother indicates that the child has not had the first measles, mumps, rubella (MMR) immunization. The nurse understands that which

Which statement made by a nurse about the goal of total quality management or continuous quality improvement in a health care setting is correct? A) It is to observe reactive service and product problem solving Collected by :DeepaRajesh [ 185 ] [email protected] Kuwait

B) Improvement of the processes in a proactive, preventive mode is paramount C) A chart audits to finds common errors in practice and outcomes associated with goals D) A flow chart to organize daily tasks is critical to the initial stages

Review Information: The correct answer is B: Improvement of the processes in a proactive, preventive mode is paramount Total quality management and continuous quality improvement have a major goal of identifying ways to do the right thing at the right time in the right way by proactive problem-solving.

Question12

Which of the following drugs should the nurse anticipate administering to a client before they are to receive electroconvulsive therapy? A) Benzodiazepines B) Chlorpromazine (Thorazine) C) Succinylcholine (Anectine) D) Thiopental sodium (Pentothal Sodium)

Recognize personal attitudes about cultural differences and real or expected biases The nurse must discover personal attitudes, prejudices and biases specific to different cultures. Awareness of these will prevent negative consequences for interactions with clients and families across cultures.

Question14

A client with chronic obstructive pulmonary disease (COPD) and a history of coronary artery disease is receiving aminophylline, 25mg/hour. Which one of the following findings by the nurse would require immediate intervention? A) Decreased blood pressure and respirations B) Flushing and headache C) Restlessness and palpitations D) Increased heart rate and blood pressure

Review Information: The correct answer is C: Restlessness and palpitations Side effects of Aminophylline include restlessness and palpitations.

Question15 Review Information: The correct answer is C: Succinylcholine (Anectine) Succinylcholine is given intravenously to promote skeletal muscle relaxation.

Question13

Which approach is a priority for the nurse who works with clients from many different cultures? A) Speak at least 2 other languages of clients in the neighborhood B) Learn about the cultures of clients who are most often encountered C) Have a list of persons for referral when interaction with these clients occur D) Recognize personal attitudes about cultural differences and real or expected biases

Review Information: The correct answer is D:

A client has gastroesophageal reflux. Which recommendation made by the nurse would be most helpful to the client? A) Avoid liquids unless a thickening agent is used B) Sit upright for at least 1 hour after eating C) Maintain a diet of soft foods and cooked vegetables D) Avoid eating 2 hours before going to sleep

Review Information: The correct answer is D: Avoid eating 2 hours before going to sleep Eating before sleeping enhances the regurgitation of stomach contents, which have increased acidity, into the esophagus. An upright posture should be maintained for about 2 hours after eating to allow for the stomach emptying. Options A and C are interventions for clients with swallowing difficulties. . Collected by :DeepaRajesh [ 186 ] [email protected] Kuwait

Question16

A client with a panic disorder has a new prescription for Xanax (alprazolam). In teaching the client about the drug>s actions and side effects, which of the following should the nurse emphasize? A) Short-term relief can be expected B) The medication acts as a stimulant C) Dosage will be increased as tolerated D) Initial side effects often continue

Review Information: The correct answer is A: Short-term relief can be expected Xanax is a short-acting benzodiazepine useful in controlling panic symptoms quickly.

Question17

A client being discharged from the cardiac stepdown unit following a myocardial infarction (MI), is given a prescription for a beta-blocking drug. A nursing student asks the charge nurse why this drug would be used by a client who is not hypertensive. What is an appropriate response by the charge nurse? A) «Most people develop hypertension following an MI.» B) «A beta-Blocker will prevent orthostatic hypotension.» C) «This drug will decrease the workload on his heart.» D) «Beta-blockers increase the strength of heart contractions.»

Review Information: The correct answer is C: «This drug will decrease the workload on his heart.» One action of beta-blockers is to decrease systemic vascular resistance by dilating arterioles. This is useful for the client with coronary artery disease, and will reduce the risk of another MI or sudden death.

Question18

A 35-year-old client of Puerto Rican-American descent is diagnosed with ovarian cancer. The

client states, “I refuse both radiation and chemotherapy because they are ” The next action for the nurse to take is to A) document the situation in the notes B) report the situation to the health care provider C) talk with the client>s family about the situation D) ask the client to talk about concerns regarding «hot» treatments

Review Information: The correct answer is D: ask the client to talk about concerns regarding «hot» treatments The «hot-cold» system is found among MexicanAmericans, Puerto Ricans, and other HispanicLatinos. Most foods, beverages, herbs, and medicines are categorized as hot or cold, which are symbolic designations and do not necessarily indicate temperature or spiciness. Care and treatment regimens can be negotiated with clients within this framework.

Question19

A 72 year-old client is scheduled to have a cardioversion. A nurse reviews the client’s medication administration record. The nurse should notify the health care provider if the client received which medication during the preceding 24 hours? A) Digoxin (Lanoxin) B) Diltiazem (Cardizem) C) Nitroglycerine ointment D) Metoprolol (Toprol XL)

Review Information: The correct answer is A: Digoxin (Lanoxin) Digoxin increases ventricular irritability and increases the risk of ventricular fibrillation following cardioversion. The other medications do not increase ventricular irritability.

Question20

Which of these clients, all of whom have the findings of a board-like abdomen, would the nurse suggest that the provider examine first? Collected by :DeepaRajesh [ 187 ] [email protected] Kuwait

A) An elderly client who stated, «My awful pain in my right side suddenly stopped about 3 hours ago.» B) A pregnant woman of 8 weeks newly diagnosed with an ectopic pregnancy C) A middle-aged client admitted with diverticulitis who has taken only clear liquids for the past week D) A teenager with a history of falling off a bicycle without hitting the handle bars

To prevent drug resistance from developing, the nurse is aware that which of the following is a characteristic of the typical treatment plan to eliminate the tuberculosis bacilli? A) An anti-inflammatory agent B) High doses of B complex vitamins C) Aminoglycoside antibiotics D) Administering two anti-tuberculosis drugs

Review Information: The correct answer is A: An elderly client who stated, «My awful pain in my right side suddenly stopped about 3 hours ago.» This client has the highest risk for hypovolemic and septic shock since the appendix has most likely ruptured, based on the history of the pain suddenly stopping over three hours ago. Elderly clients have less functional reserve for the body to cope with shock and infection over long periods. The others are at risk for shock also, however given that they fall in younger age groups, they would more likely be able to tolerate an imbalance in circulation. A common complication of falling off a bicycle is hitting the handle bars in the upper abdomen often on the left, resulting in a ruptured spleen.

Review Information: The correct answer is D: Administering two anti-tuberculosis drugs Resistance of the tubercle bacilli often occurs to a single antimicrobial agent. Therefore, therapy with multiple drugs over a long period of time helps to ensure eradication of the organism.

Question23

The nurse is assessing a comatose client receiving gastric tube feedings. Which of the following assessments requires an immediate response from the nurse? A) Decreased breath sounds in right lower lobe B) Aspiration of a residual of 100cc of formula C) Decrease in bowel sounds D) Urine output of 250 cc in past 8 hours

Question21

The nurse is teaching parents of a 7 month-old about adding table foods. Which of the following is an appropriate finger food? A) Hot dog pieces B) Sliced bananas C) Whole grapes D) Popcorn

Review Information: The correct answer is B: Sliced bananas Finger foods should be bite-size pieces of soft food such as bananas. Hot dogs and grapes can accidentally be swallowed whole and can occlude the airway. Popcorn is too difficult to chew at this age and can irritate the airway if swallowed.

Question22

Review Information: The correct answer is A: Decreased breath sounds in right lower lobe The most common problem associated with enteral feedings is atelectasis. Maintain client at 30 degrees of head elevation during feedings and monitor for signs of aspiration. Check for tube placement prior to each feeding or every 4 to 8 hours if the client is receiving continuous feeding.

Question24

A client is prescribed warfarin sodium (Coumadin) to be continued at home. Which focus is critical to be included in the nurse’s discharge instruction? A) Maintain a consistent intake of green leafy foods B) Report any nose or gum bleeds C) Take Tylenol for minor pains Collected by :DeepaRajesh [ 188 ] [email protected] Kuwait

D) Use a soft toothbrush

Review Information: The correct answer is B: Report any nose or gum bleeds The client should notify the health care provider if blood is noted in stools or urine, or any other signs of bleeding occur.

Question25

When teaching a client about the side effects of fluoxetine (Prozac), which of the following will the nurse include? A) Tachycardia blurred vision, hypotension, anorexia B) Orthostatic hypotension, vertigo, reactions to tyramine-rich foods C) Diarrhea, dry mouth, weight loss, reduced libido D) Photosensitivity, seizures, edema, hyperglycemia

Question27

The nurse manager informs the nursing staff at morning report that the clinical nurse specialist will be conducting a research study on staff attitudes toward client care. All staff are invited to participate in the study if they wish. This affirms the ethical principle of A) Anonymity B) Beneficence C) Justice D) Autonomy

Review Information: The correct answer is D: Autonomy Individuals must be free to make independent decisions about participation in research without coercion from others.

Question28

The nurse is talking with the family of an 18 months-old newly diagnosed with retinoblastoReview Information: The correct answer is C: ma. A priority in communicating with the parents Diarrhea, dry mouth, weight loss, reduced libido is Commonly reported side effects for fluoxetine A) Discuss the need for genetic counseling (Prozac) are diarrhea, dry mouth, weight loss B) Inform them that combined therapy is seldom and reduced libido. effective C) Prepare for the child>s permanent disfigurement Question26 D) Suggest that total blindness may follow surA newborn weighed 7 pounds 2 ounces at birth. gery The nurse assesses the newborn at home 2 days later and finds the weight to be 6 pounds 7 ounces. What should the nurse tell the parents Review Information: The correct answer is A: Discuss the need for genetic counseling about this weight loss? A) The newborn needs additional assessments The hereditary aspects of this disease are well B) The mother should breast feed more often documented. While the parents focus on the C) A change to formula is indicated needs of this child, they should be aware that the risk is high for future offspring. D) The loss is within normal limits

Question29 Review Information: The correct answer is D: The loss is within normal limits A newborn is expected to lose 5-10% of the birth weight in the first few days post-partum because of changes in elimination and feeding.

The nurse is planning care for an 8 year-old child. Which of the following should be included in the plan of care? A) Encourage child to engage in activities in the playroom B) Promote independence in activities of daily living Collected by :DeepaRajesh [ 189 ] [email protected] Kuwait

C) Talk with the child and allow him to express his opinions D) Provide frequent reassurance and cuddling

Review Information: The correct answer is A: Encourage child to engage in activities in the playroom According to Erikson, the school age child is in the stage of industry versus inferiority. To help them achieve industry, the nurse should encourage them to carry out tasks and activities in their room or in the playroom.

Question30

The nurse is assigned to care for 4 clients. Which of the following should be assessed immediately after hearing the report? A) The client with asthma who is now ready for discharge B) The client with a peptic ulcer who has been vomiting all night C) The client with chronic renal failure returning from dialysis D) The client with pancreatitis who was admitted yesterday

Review Information: The correct answer is B: The client with a peptic ulcer who has been vomiting all night A perforated peptic ulcer could cause nausea, vomiting and abdominal distention, and may be a life threatening situation. The client should be assessed immediately and findings reported to the provider. .

Question31

During a routine check-up, an insulin-dependent diabetic has his glycosylated hemoglobin checked. The results indicate a level of 11%. Based on this result, what teaching should the nurse emphasize? A) Rotation of injection sties B) Insulin mixing and preparation C) Daily blood sugar monitoring D) Regular high protein diet

Review Information: The correct answer is C: Daily blood sugar monitoring Normal hemoglobin A1C (glycosylated hemoglobin) level is 7 to 9%. Elevation indicates elevated glucose levels over time.

Question32

A client taking isoniazid (INH) for tuberculosis asks the nurse about side effects of the medication. The client should be instructed to immediately report which of these? A) Double vision and visual halos B) Extremity tingling and numbness C) Confusion and lightheadedness D) Sensitivity of sunlight Review Information: The correct answer is B: Extremity tingling and numbness Peripheral neuropathy is the most common side effect of INH and should be reported to the provider. It can be reversed.

Question33

Which of these questions is priority when assessing a client with hypertension? A) «What over-the-counter medications do you take?» B) «Describe your usual exercise and activity patterns.» C) «Tell me about your usual diet.» D) «Describe your family>s cardiovascular history.»

Review Information: The correct answer is A: «What over-the-counter medications do you take?» Over-the-counter medications, especially those that contain cold preparations can increase the blood pressure to the point of hypertension.

Question34

The nurse is performing an assessment of the Collected by :DeepaRajesh [ 190 ] [email protected] Kuwait

motor function in a client with a head injury. The best technique is A) touching the trapezius muscle or arm firmly B) pinching any body part C) shaking a limb vigorously D) rubbing the sternum

Review Information: The correct answer is D: rubbing the sternum The purpose is to assess the non-responsive client’s reaction to a painful stimulus after less noxious methods have been tried.

Review Information: The correct answer is B: «He has had an ear infection for the past 2 days.» Contributing factors to seizures in children include those such as age (more common in first 2 years), infections (late infancy and early childhood), fatigue, not eating properly and excessive fluid intake or fluid retention.

Question37

Which of these clients would the nurse monitor for the complication of C. difficile diarrhea? Question35 A) An adolescent taking medications for acne A nurse admits a client transferred from the emer- B) An elderly client living in a retirement center gency room (ER). The client, diagnosed with a taking prednisone myocardial infarction, is complaining of subster- C) A young adult at home taking a prescribed nal chest pain, diaphoresis and nausea. The first aminoglycoside action by the nurse should be to D) A hospitalized middle aged client receiving A) order an EKG clindamycin B) administer morphine sulfate C) start an IV D) measure vital signs

Review Information: The correct answer is B: administer morphine sulfate Decreasing the clients pain is the most important priority at this time. As long as pain is present there is danger in extending the infarcted area. Morphine will decrease the oxygen demands of the heart and act as a mild diuretic as well. It is probable that an EKG and IV insertion were performed in the ER.

Question36

The nurse admits a 2 year-old child who has had a seizure. Which of the following statement by the child>s parent would be important in determining the etiology of the seizure? A) «He has been taking long naps for a week.» B) «He has had an ear infection for the past 2 days.» C) «He has been eating more red meat lately.» D) «He seems to be going to the bathroom more frequently.»

Review Information: The correct answer is D: A hospitalized middle aged client receiving clindamycin Hospitalized patients, especially those receiving antibiotic therapy, are primary targets for C. difficile. Of clients receiving antibiotics, 5-38% experience antibiotic-associated diarrhea; C. difficile causes 15 to 20% of the cases. Several antibiotic agents have been associated with C. difficile. Broad-spectrum agents, such as clindamycin, ampicillin, amoxicillin, and cephalosporins, are the most frequent sources of C. difficile. Also, C. difficile infection has been caused by the administration of agents containing beta-lactamase inhibitors (i.e., clavulanic acid, sulbactam, tazobactam) and intravenous agents that achieve substantial colonic intraluminal concentrations (i.e., ceftriaxone, nafcillin, oxacillin). Fluoroquinolones, aminoglycosides, vancomycin, and trimethoprim are seldom associated with C. difficile infection or pseudomembranous colitis.

Question38

The nurse is performing an assessment on a cliCollected by :DeepaRajesh [ 191 ] [email protected] Kuwait

ent who is cachectic and has developed an enterocutaneous fistula following surgery to relieve a small bowel obstruction. The client>s total protein level is reported as 4.5 g/dl. Which of the following would the nurse anticipate? A) Additional potassium will be given IV B) Blood for coagulation studies will be drawn C) Total parenteral nutrition (TPN) will be started D) Serum lipase levels will be evaluated

mg/dl. What action by the nurse is appropriate at this time? A) Give oral glucose water B) Notify the pediatrician C) Repeat the test in 2 hours D) Check the pulse oximetry reading

Review Information: The correct answer is C: Repeat the test in 2 hours This blood sugar is within the normal range for a full-term newborn. Normal values are: Premature infant: 20-60 mg/dl or 1.1-3.3 mmol/L, Neonate: 30-60 mg/dl or 1.7-3.3 mmol/L, Infant: 40-90 mg/ Review Information: The correct answer is C: dl or 2.2-5.0 mmol/L. Critical values are: Infant: Total parenteral nutrition (TPN) will be started <40 mg/dl and in a Newborn: <30 and >300 mg/ The client is not absorbing nutrients adequately dl. Because of the increased birth weight which as evidenced by the cachexia and low protein can be associated with diabetes mellitus, repeatlevels. (A normal total serum protein level is 6.0- ed blood sugars will be drawn 8.0 g/dl.) TPN will promote a positive nitrogen balance in this client who is unable to digest and 0 comments absorb nutrients adequately. Labels: free nclex-rn sample review questions, nclex-rn practice test questions, nursing review

Question39

During a situation of pain management, which statement is a priority to consider for the ethical guidelines of the nurse? A) The client>s self-report is the most important consideration B) Cultural sensitivity is fundamental to pain management C) Clients have the right to have their pain relieved D) Nurses should not prejudge a client>s pain using their own values

Review Information: The correct answer is A: The client>>s self-report is the most important consideration Pain is a complex phenomenon that is perceived differently by each individual. Pain is whatever the client says it is. The other statements are correct but not the most important considerations.

Free NCLEX-RN Sample Test Questions For Nursing Review (Part 3) Question1

A client diagnosed with chronic depression is maintained on tranylcypromine (Parnate). An important nursing intervention is to teach the client to avoid which of the following foods? A) Wine, beer, cheese, liver and chocolate B) Wine, citrus fruits, yogurt and broccoli C) Beer, cheese, beef and carrots D) Wine, apples, sour cream and beef steak Review Information: The correct answer is A: Wine, beer, cheese, liver and chocolate These foods are tyramine rich and ingestion of these foods while taking monoamine oxidase inhibitors (MAOIs) can precipitate a life-threatening hypertensive crisis.

Question2

The nurse is working in a high risk antepartum clinic. A 40 year-old woman in the first trimesQuestion40 ter gives a thorough health history. Which inforAs a part of a 9 pound full-term newborn>s as- mation should receive priority attention by the sessment, the nurse performs a dextro-stick at 1 nurse? hour post birth. The serum glucose reading is 45 A) Her father and brother are insulin dependent Collected by :DeepaRajesh [ 192 ] [email protected] Kuwait

diabetics B) She has taken 800 mcg of folic acid daily for the past year C) Her husband was treated for tuberculosis as a child D) She reports recent use of over-the counter sinus remedies Review Information: The correct answer is D: She reports recent use of over-the counter sinus remedies Over-the-counter drugs are a possible danger in early pregnancy. A report by the client that she has taken medications should be followed up immediately.

Question3

What must be the priority consideration for nurses when communicating with children? A) Present environment B) Physical condition C) Nonverbal cues D) Developmental level Review Information: The correct answer is D: Developmental level While each of the factors affect communication, the nurse recognizes that developmental differences have implications for processing and understanding information. Consequently, a child’s developmental level must be considered when selecting communication approaches.

Question4

The nurse is assessing a client>s home in preparation for discharge. Which of the following should be given priority consideration? A) Family understanding of client needs B) Financial status C) Location of bathrooms D) Proximity to emergency services Review Information: The correct answer is A: Family understanding of client needs Functional communication patterns between family members are fundamental to meeting the needs of the client and family.

Question5

of acute alcohol intoxication, it is important for the nurse initially to obtain data regarding which of the following? A) What and how much the client drinks, according to family and friends B) The blood alcohol level of the client C) The blood pressure level of the client D) The blood glucose level of the client Review Information: The correct answer is B: The blood alcohol level of the client Blood alcohol levels are generally obtained to determine the level of intoxication. The amount of alcohol consumed determines how much medication the client needs for detoxification and treatment. Reports of alcohol consumption are notoriously inaccurate.

Question6

Which clinical finding would the nurse expect to assess first in a newborn with spastic cerebral palsy? A) cognitive impairment B) hypotonic muscular activity C) seizures D) criss-crossing leg movement Review Information: The correct answer is D: criss-crossing leg movement Cerebral palsy is a neuromuscular impairment resulting in muscular and reflexive hypertonicity and the criss-crossing, or scissoring leg movements.

Question7

Which medication is more helpful in treating bulimia than anorexia? A) Amphetamines B) Sedatives C) Anticholinergics D) Narcotics Review Information: The correct answer is C: Anticholinergics In contrast to anorexics, individuals with bulimia are troubled by their behavioral characteristics and become depressed. The person feels compelled to binge, purge and fast. Feeling helpless to stop the behavior, feelings of self-disgust occur.

As a general guide for emergency management Collected by :DeepaRajesh [ 193 ] [email protected] Kuwait

Question8

The nurse is assessing a woman in early labor. Review Information: The correct answer is D: While positioning for a vaginal exam, she com- «I feel pressure in the middle of my chest, like an plains of dizziness and nausea and appears elephant is sitting on my chest.» pale. Her blood pressure has dropped slightly. This is a classic description of chest pain in men caused by myocardial ischemia. Women experiWhat should be the initial nursing action? ence vague feelings of fatigue and back and jaw A) Call the health care provider pain. B) Encourage deep breathing C) Elevate the foot of the bed D) Turn her to her left side Question11 A nurse is caring for a client who has just been Review Information: The correct answer is D: admitted with an overdose of aspirin. The folTurn her to her left side lowing lab data is available: PaO2 95, PaCO2 The weight of the uterus can put pressure on the 30, pH 7.5, K 3.2 mEq/l. Which should be the vena cava and aorta when a pregnant woman nurse>s first action? is flat on her back causing supine hypotension. A) Monitor respiratory rate Action is needed to relieve the pressure on the B) Monitor intake and output every hour vena cava and aorta. Turning the woman to the C) Assist the client to breathe into a paper bag side reduces this pressure and relieves postural D) Prepare to administer oxygen by mask hypotension.

Question9

Review Information: The correct answer is C: Assist the client to breathe into a paper bag Side effects of aspirin toxicity include hyperventilation, which can result in respiratory alkalosis in the initial stages. Breathing into a paper bag will prevent further reduction in PaCO2.

A client has been started on a long term corticosteroid therapy. Which of the following comments by the client indicate the need for further teaching? A) «I will keep a weekly weight record.» B) «I will take medication with food.» C) «I will stop taking the medication for 1 week Question12 every month.» After assessing a 70 year-old male client>s laboD) «I will eat foods high in potassium.» ratory results during a routine clinic visit, which one of the following findings would indicate an area in which teaching is needed: Review Information: The correct answer is C: A) Serum albumin 2.5 g/dl «I will stop taking the medication for 1week every B) LDL Cholesterol 140 mg/dl month.» C) Serum glucose 90 mg/dl Emphatically warn against discontinuing steroid D) RBC 5.0 million/mm3 dosage abruptly because that may produce a fatal adrenal crisis. Review Information: The correct answer is A: Serum albumin 2.5 g/dl Question10 A male client calls for a nurse because of chest Serum albumin level is low (normal 3.0 – 5.0 g/dl pain. Which statement by the client would require in elders), indicating nutritional counseling to increase dietary protein is needed. Socioeconomic the most immediate action by the nurse? A) «When I take in a deep breath, it stabs like a factors may need to be addressed to help the client comply with the recommendation. knife.» B) «The pain came on after dinner. That soup seemed very spicy.» Question13 C) «When I turn in bed to reach the remote for When teaching a client with a new prescription the TV, my chest hurts.» for lithium (Lithane) for treatment of a bi-polar D) «I feel pressure in the middle of my chest, like disorder which of these should the nurse emphaan elephant is sitting on my chest.» size? A) Maintaining a salt restricted diet Collected by :DeepaRajesh [ 194 ] [email protected] Kuwait

B) Reporting vomiting or diarrhea C) Taking other medication as usual D) Substituting generic form if desired Review Information: The correct answer is B: Reporting vomiting or diarrhea If dehydration results from vomiting, diarrhea or excessive perspiration, tolerance to the drug may be altered and symptoms may return.

Question14

A client is discharged on warfarin sulfate (Coumadin). Which statement by the client indicated a need for further teaching? A) «I know I must avoid crowds.» B) «I will keep all laboratory appointments.» C) «I plan to use an electric razor for shaving.» D) «I will report any bruises for bleeding.»

Review Information: The correct answer is A: «I know I must avoid crowds.» There are no specific reasons for the client on Coumadin to avoid crowds. General instructions for any cardiac surgical client include limiting exposure to infection.

Question15

A client is taking tranylcypromine (Parnate) and has received dietary instruction. Which of the following food selections would be contraindicated for this client? A) Fresh juice, carrots, vanilla pudding B) Apple juice, ham salad, fresh pineapple C) Hamburger, fries, strawberry shake D) Red wine, fava beans, aged cheese Review Information: The correct answer is D: Red wine, fava beans, aged cheese Red wine and cheese contain tyramine (as do chicken liver and ripe bananas) and so are contraindicated when taking MAOIs. Fava beans contain other vasopressors that can interact with MAOIs also causing malignant hypertension.

Question16

28. The initial nursing intervention would be to A) begin intravenous therapy B) initiate continuous blood pressure monitoring C) administer oxygen therapy D) institute cardiac monitoring Review Information: The correct answer is C: administer oxygen therapy Early findings of shock reveal hypoxia with rapid heart rate and rapid respirations, and oxygen is the most critical initial intervention. The other interventions are secondary to oxygen therapy.

Question17

A client is admitted to the hospital with a diagnosis of deep vein thrombosis. During the initial assessment, the client complains of sudden shortness of breath. The SaO2 is 87. The priority nursing assessment at this time is A) bowel sounds B) heart rate C) peripheral pulses D) lung sounds Review Information: The correct answer is D: lung sounds Lung sounds are critical assessments at this point. The nurse should be alert to crackles or a pleural friction rub, highly suggestive of a pulmonary embolism.

Question18

The nurse is administering lidocaine (Xylocaine) to a client with a myocardial infarction. Which of the following assessment findings requires the nurse>s immediate action? A) Central venous pressure reading of 11 B) Respiratory rate of 22 C) Pulse rate of 48 BPM D) Blood pressure of 144/92 Review Information: The correct answer is C: Pulse rate of 48 BPM One of the side effects of lidocaine is bradycardia, heart block, cardiovascular collapse and cardiac arrest (this drug should never be administered without continuous EKG monitoring).

A client is admitted with severe injuries from an Question19 auto accident. The client>s vital signs are BP The nurse is teaching a group of college students 120/50, pulse rate 110, and respiratory rate of about breast self-examination. A woman asks for Collected by :DeepaRajesh [ 195 ] [email protected] Kuwait

the best time to perform the monthly exam. What Health education is also available through local is the best reply by the nurse? and national Alzheimer>>s chapters. A) «The first of every month, because it is easiest to remember» Question22 B) «Right after the period, when your breasts are Clients taking lithium must be particularly sure to less tender» maintain adequate intake of which of these eleC) «Do the exam at the same time every ments? month» A) Potassium D) «Ovulation, or mid-cycle is the best time to B) Sodium detect changes» C) Chloride D) Calcium Review Information: The correct answer is B: «Right after the period, when your breasts are less tender» The best time for a breast self exam (BSE) is a week after a menstrual cycle, when the breasts are no longer swollen and tender due to hormone elevation.

Review Information: The correct answer is B: Sodium Clients taking lithium need to maintain an adequate intake of sodium. Serum lithium concentrations may increase in the presence of conditions that cause sodium loss.

Question20

Question23

Review Information: The correct answer is C: cover the wound with sterile saline dressing When evisceration occurs, the wound should first be quickly covered by sterile dressings soaked in sterile saline. This prevents tissue damage until a repair can be effected.

Review Information: The correct answer is B: Vomiting, diarrhea and lethargy These are early signs of lithium toxicity.

The nurse is caring for a post-operative client who develops a wound evisceration. The first nursing intervention should be to A) medicate the client for pain B) call the provider C) cover the wound with sterile saline dressing D) place the bed in a flat position

Question21

The spouse of a client with Alzheimer>s disease expresses concern about the burden of caregiving. Which of the following actions by the nurse should be a priority? A) Link the caregiver with a support group B) Ask friends to visit regularly C) Schedule a home visit each week D) Request anti-anxiety prescriptions Review Information: The correct answer is A: Link the caregiver with a support group Assisting caregivers to locate and join support groups is most helpful. Families share feelings and learn about services such as respite care.

A client is receiving lithium carbonate 600 mg T.I.D. to treat bipolar disorder. Which of these indicate early signs of toxicity? A) Ataxia and course hand tremors B) Vomiting, diarrhea and lethargy C) Pruritus, rash and photosensitivity D) Electrolyte imbalance and cardiac arrhythmias

Question24

The nurse can best ensure the safety of a client suffering from dementia who wanders from the room by which action? A) Repeatedly remind the client of the time and location B) Explain the risks of walking with no purpose C) Use protective devices to keep the client in the bed or chair in the room D) Attach a wander-guard sensor band to the client>s wrist Review Information: The correct answer is D: Attach a wander-guard sensor band to the client>>s wrist This type of identification band easily tracks the client>>s movements and ensures safety while the client wanders on the unit. Restriction of acCollected by :DeepaRajesh [ 196 ] [email protected] Kuwait

tivity is inappropriate for any client unless they C) put the client in knee-chest position are potentially harmful to themselves or others. D) turn the client to the side

Question25

The nurse is teaching a client about the difference between tardive dyskinesia (TD) and neuroleptic malignant syndrome (NMS). Which statement is true with regards to tardive dyskinesia? A) TD develops within hours or years of continued antipsychotic drug use in people under 20 and over 30 B) It can occur in clients taking antipsychotic drugs longer than 2 years C) Tardive dyskinesia occurs within minutes of the first dose of antipsychotic drugs and is reversible D) TD can easily be treated with anticholinergic drugs Review Information: The correct answer is B: It can occur in clients taking antipsychotic drugs longer than 2 years Tardive dyskinesia is a extrapyramidal side effect that appears after prolonged treatment with antipsychotic medication. Early symptoms of tardive dyskinesia are fasciculations of the tongue or constant smacking of the lips.

Review Information: The correct answer is C: put the client in knee-chest position Immediate action is needed to relieve pressure on the cord, which puts the fetus at risk due to hypoxia. The Trendelenburg position accomplishes this. The exposed cord is covered with saline soaked gauze, not reinserted. The fetal heart rate also should be checked, and the provider called. A prolapsed umbilical cord is a medical emergency.

Question28

The nurse is caring for a 2 month-old infant with a congenital heart defect. Which of the following is a priority nursing action? A) Provide small feedings every 3 hours B) Maintain intravenous fluids C) Add strained cereal to the diet D) Change to reduced calorie formula

Review Information: The correct answer is A: Provide small feedings every 3 hours Infants with congenital heart defects are at increased risk for developing congestive heart failure. Infants with congestive heart failure have an increased metabolic rate and require additional Question26 The nurse is aware that the effect of antihyper- calories to grow. At the same time, however, rest tensive drug therapy may be affected by a 75 and conservation of energy for eating is important. Feedings should be smaller and every 3 year-old client>s hours rather than the usual 4 hour schedule. A) poor nutritional status B) decreased gastrointestinal motility C) increased splanchnic blood flow Question29 D) altered peripheral resistance The nurse is caring for a client receiving intravenous nitroglycerin for acute angina. What is the most important assessment during treatment? Review Information: The correct answer is B: A) Heart rate decreased gastrointestinal motility B) Neurologic status Together with shrinkage of the gastric mucosa, C) Urine output and changes in the levels of hydrochloric acid, D) Blood pressure this will decrease absorption of medications and interfere with their actions. Review Information: The correct answer is D: Blood pressure Question27 In response to a call for assistance by a client in The vasodilatation that occurs as a result of this labor, the nurse notes that a loop on the umbili- medication can cause profound hypotension. cal cord protrudes from the vagina. What is the The client>>s blood pressure must be evaluated every 15 minutes until stable and then every 30 priority nursing action? minutes to every hour. A) call the health care provider B) check fetal heart beat Collected by :DeepaRajesh [ 197 ] [email protected] Kuwait

Question30

A client telephones the clinic to ask about a home pregnancy test she used this morning. The nurse understands that the presence of which hormone strongly suggests a woman is pregnant? A) Estrogen B) HCG C) Alpha-fetoprotein D) Progesterone Review Information: The correct answer is B: HCG Human chorionic gonadotropin (HCG) is the biologic marker on which pregnancy tests are based. Reliability is about 98%, but the test does not conclusively confirm pregnancy.

Question31

A client, admitted to the unit because of severe depression and suicidal threats, is placed on suicidal precautions. The nurse should be aware that the danger of the client committing suicide is greatest A) during the night shift when staffing is limited B) when the client’s mood improves with an increase in energy level C) at the time of the client>s greatest despair D) after a visit from the client>s estranged partner

Review Information: The correct answer is B: when the client’s mood improves with an increase in energy level Suicide potential is often increased when there is an improvement in mood and energy level. At this time ambivalence is often decreased and a decision is made to commit suicide.

Question32

After 4 electroconvulsive treatments over 2 weeks, a client is very upset and states “I am so confused. I lose my money. I just can’t remember telephone numbers.” The most therapeutic response for the nurse to make is A) «You were seriously ill and needed the treatments.» B) «Don>t get upset. The confusion will clear up in a day or two.» C) «It is to be expected since most clients have the same results.» D) «I can hear your concern and that your confu-

sion is upsetting to you.» Review Information: The correct answer is D: «I can hear your concern and that your confusion is upsetting to you.» Communicating caring and empathy with the acknowledgement of feelings is the initial response. Afterwards, teaching about the expected short term effects would be discussed.

Question33

A woman in labor calls the nurse to assist her in the bathroom. The nurse notices a large amount of clear fluid on the bed linens. The nurse knows that fetal monitoring must now assess for what complication? A) Early decelerations B) Late accelerations C) Variable decelerations D) Periodic accelerations Review Information: The correct answer is C: Variable decelerations When the membranes rupture, there is increased risk initially of cord prolapse. Fetal heart rate patterns may show variable decelerations, which require immediate nursing action to promote gas exchange.

Question34

The nurse is assessing a client with chronic obstructive pulmonary disease receiving oxygen for low PaO2 levels. Which assessment is a nursing priority? A) Evaluating SaO2 levels frequently B) Observing skin color changes C) Assessing for clubbing fingers D) Identifying tactile fremitus Review Information: The correct answer is A: Evaluating SaO2 levels frequently The best method to evaluate a client>>s oxygenation is to evaluate the SaO2. This is just as effective as an arterial blood gas reading to evaluate oxygenation status, and is less traumatic and expensive.

Question35

The visiting nurse makes a postpartum visit to a married female client. Upon arrival, the nurse observes that the client has a black eye and nuCollected by :DeepaRajesh [ 198 ] [email protected] Kuwait

merous bruises on her arms and legs. The initial nursing intervention would be to A) call the police to report indications of domestic violence B) confront the husband about abusing his wife C) leave the home because of the unsafe environment D) interview the client alone to determine the origin of the injuries Review Information: The correct answer is D: interview the client alone to determine the origin of the injuries It would be wrong to assume domestic violence without further assessment. Separate the suspected victim from the partner until battering has been ruled out.

Question36

When teaching a client about an oral hypoglycemic medication, the nurse should place primary emphasis on A) recognizing findings of toxicity B) taking the medication at specified times C) increasing the dosage based on blood glucose D) distinguishing hypoglycemia from hyperglycemia Review Information: The correct answer is B: taking the medication at specified times A regular interval between doses should be maintained since oral hypoglycemics stimulate the islets of Langerhans to produce insulin.

Question37

Initial postoperative nursing care for an infant who has had a pyloromyotomy would initially include A) bland diet appropriate for age B) intravenous fluids for 3-4 days C) NPO then glucose and electrolyte solutions D) formula or breast milk as tolerated Review Information: The correct answer is C: NPO then glucose and electrolyte solutions Post-operatively, the initial feedings are clear liquids in small quantities to provide calories and electrolytes.

Question38

A client is treated in the emergency room for diabetic ketoacidosis and a glucose level of 650mg.D/L. In assessing the client, the nurse>s review of which of the following tests suggests an understanding of this health problem? A) Serum calcium B) Serum magnesium C) Serum creatinine D) Serum potassium Review Information: The correct answer is D: Serum potassium Potassium is lost in diabetic ketoacidosis during rehydration and insulin administration. Review of this lab finding suggests the nurse has knowledge of this problem.

Question39

A male client is preparing for discharge following an acute myocardial infarction. He asks the nurse about his sexual activity once he is home. What would be the nurse>s initial response? A) Give him written material from the American Heart Association about sexual activity with heart disease B) Answer his questions accurately in a private environment C) Schedule a private, uninterrupted teaching session with both the client and his wife D) Assess the client>s knowledge about his health problems Review Information: The correct answer is D: Assess the client>>s knowledge about his health problems The nursing process is continuous and cyclical in nature. When a client expresses a specific concern, the nurse performs a focused assessment to gather additional data prior to planning and implementing nursing interventions.

Question40

The client asks the nurse how the health care provider could tell she was pregnant “just by looking inside.” What is the best explanation by the nurse? A) Bluish coloration of the cervix and vaginal walls B) Pronounced softening of the cervix C) Clot of very thick mucous that obstructs the Collected by :DeepaRajesh [ 199 ] [email protected] Kuwait

cervical canal D) Slight rotation of the uterus to the right

ized adolescent. What is the major threat experienced by the hospitalized adolescent? A) Pain management B) Restricted physical activity Review Information: The correct answer is A: C) Altered body image Bluish coloration of the cervix and vaginal walls D) Separation from family Chadwick>>s sign is a bluish-purple coloration of the cervix and vaginal walls, occurring at 4 Review Information: The correct answer is C: weeks of pregnancy, that is caused by vasocon- Altered body image gestion. The hospitalized adolescent may see each of these as a threat, but the major threat that they 0 comments feel when hospitalized is the fear of altered body image, because of the emphasis on physical apLabels: free nclex-rn sample review questions, pearance during this developmental stage. nclex-rn practice test questions, nursing review

Question4

Free NCLEX-RN Sample Test Ques- A 12 year-old child is admitted with a broken arm and is told surgery is required. The nurse finds tions For Nursing Review (Part 2) him crying and unwilling to talk. What is the most appropriate response by the nurse? Question1 The feeling of trust can best be established by A) Give him privacy the nurse during the process of the development B) Tell him he will get through the surgery with of a nurse-client relationship by which of these no problem C) Try to distract him characteristics? D) Make arrangements for his friends to visit A) Reliability and kindness B) Demeanor and sincerity C) Honesty and consistency D) Sympathy and appreciativeness

Review Information: The correct answer is C: Honesty and consistency Characteristics of a trusting relationship include respect, honesty, consistency, faith and caring.

Question2

A nurse has administered several blood transfusions over 3 days to a 12 year-old client with Thalassemia. What lab value should the nurse monitor closely during this therapy? A) Hemoglobin B) Red Blood Cell Indices C) Platelet count D) Neutrophil percent

Review Information: The correct answer is A: Give him privacy A 12 year-old child needs the opportunity to express his emotions privately.

Question5

In discharge teaching, the nurse should emphasize that which of these is a common side effect of clozapine (Clozaril) therapy? A) Dry mouth B) Rhinitis C) Dry skin D) Extreme salivation

Review Information: The correct answer is D: Extreme salivation A significant number of clients receiving ClozapReview Information: The correct answer is A: ine (Clozaril) therapy experience extreme salivation. Hemoglobin Hemoglobin should be in a therapeutic range of approximately 10 g/dl (100gL). «This level is low Question6 enough to foster the patient>>s own erythropoi- A client has had a positive reaction to purified protein derivative (PPD). The client asks the esis without enlarging the spleen.» nurse what this means. The nurse should indicate that the client has Question3 The nurse is providing care to a newly a hospital- A) active tuberculosis Collected by :DeepaRajesh [ 200 ] [email protected] Kuwait

B) been exposed to mycobacterium tuberculosis C) never had tuberculosis D) never been infected with mycobacterium tuberculosis Review Information: The correct answer is B: been exposed to mycobacterium tuberculosis The PPD skin test is used to determine the presence of tuberculosis antibodies and a positive result indicates that the person has been exposed to mycobacterium tuberculosis. Additional tests are needed to determine if active tuberculosis is present.

Question7

A client is receiving and IV antibiotic infusion and is scheduled to have blood drawn at 1:00 pm for a «peak» antibiotic level measurement. The nurse notes that the IV infusion is running behind schedule and will not be competed by 1:00. The nurse should: A) Notify the client>s health care provider B) Stop the infusion at 1:00 pm C) Reschedule the laboratory test D) Increase the infusion rate Review Information: The correct answer is C: Reschedule the laboratory test If the antibiotic infusion will not be completed at the time the peak blood level is due to be drawn, the nurse should ask that the blood sampling time be adjusted

Question8

150mg BID. When used for depression, it may take up to four weeks for results. Common side effects are dry mouth, headache, and agitation. Doses should be administered in equally spaced time increments throughout the day to minimize the risk of seizures.

Question9

The clinic nurse is discussing health promotion with a group of parents. A mother is concerned about Reye>s Syndrome, and asks about prevention. Which of these demonstrates appropriate teaching? A) «Immunize your child against this disease.» B) «Seek medical attention for serious injuries.» C) «Report exposure to this illness.» D) «Avoid use of aspirin for viral infections.» Review Information: The correct answer is D: «Avoid use of aspirin for viral infections.» The link between aspirin use and Reye>>s Syndrome has not been confirmed, but evidence suggests that the risk is sufficiently grave to include the warning on aspirin products.

Question10

A post-operative client is admitted to the post-anesthesia recovery room (PACU). The anesthetist reports that malignant hyperthermia occurred during surgery. The nurse recognizes that this complication is related to what factor? A) Allergy to general anesthesia B) Pre-existing bacterial infection C) A genetic predisposition D) Selected surgical procedures

The nurse is caring for a client with a new order for bupropion (Wellbutrin) for treatment of depression. The order reads “Wellbutrin 175 mg. BID x 4 days.” What is the appropriate action? A) Give the medication as ordered

Review Information: The correct answer is C: A genetic predisposition Malignant hyperthermia is a rare, potentially fatal adverse reaction to inhaled anesthetics. There is a genetic predisposition to this disorder.

C) Observe the client for mood swings D) Monitor neuro signs frequently

Question11

B) Questionthis medication dose

A 9 year-old is taken to the emergency room with right lower quadrant pain and vomiting. When Review Information: The correct an- preparing the child for an emergency appenswer is B: Questionthis medication dectomy, what must the nurse expect to be the child>s greatest fear? dose A) Change in body image Bupropion (Wellbutrin) should be started at B) An unfamiliar environment 100mg BID for three days then increased to C) Perceived loss of control Collected by :DeepaRajesh [ 201 ] [email protected] Kuwait

D) Guilt over being hospitalized Review Information: The correct answer is C: Perceived loss of control For school age children, major fears are loss of control and separation from friends/peers.

B) Obtain the regular blood glucose readings C) Determine if special skin care is needed D) Answer questions from the client>s spouse about the plan of care

Review Information: The correct answer is B: Obtain the regular blood glucose readings Question12 A client is to begin taking Fosamax. The nurse The UAP can safely obtain blood glucose readmust emphasize which of these instructions to ings, which are routine tasks. the client when taking this medication? «Take Fosamax Question15 A) on an empty stomach.» Which of the following laboratory results would B) after meals.» suggest to the emergency room nurse that a cliC) with calcium.» ent admitted after a severe motor vehicle crash D) with milk 2 hours after meals.» is in acidosis? A) Hemoglobin 15 gm/dl B) Chloride 100 mEq/L Review Information: The correct answer is A: C) Sodium 130 mEq/L on an empty stomach.» D) Carbon dioxide 20 mEq/L Fosamax should be taken first thing in the morning with 6-8 ounces of plain water at least 30 minutes before other medication or food. Food Review Information: The correct answer is D: and fluids (other than water) greatly decrease Carbon dioxide 20 mEq/L the absorption of Fosamax. The client must also Serum carbon dioxide is an indicator of acid-base be instructed to remain in the upright position for status. This finding would indicate acidosis. 30 minutes following the dose to facilitate passage into the stomach and minimize irritation of Question16 the esophagus. The nurse has just received report on a group of clients and plans to delegate care of several Question13 of the clients to a practical nurse (PN). The first An older adult client is to receive and antibiotic, thing the RN should do before the delegation of gentamicin. What diagnostic finding indicates care is the client may have difficult excreting the medi- A) Provide a time-frame for the completion of the cation? client care A) High gastric pH B) Assure the PN that the RN will be available for B) High serum creatinine assistance C) Ask about prior experience with similar cliC) Low serum albumin ents D) Low serum blood urea nitrogen D) Review the specific procedures unique to the assignment Review Information: The correct answer is B: Review Information: The correct answer is C: High serum creatinine An elevated serum creatinine indicates reduced Ask about prior experience with similar clients. renal function. Reduced renal function will delay The first step in delegation is to determine the qualifications of the person to whom one is delthe excretion of many mediations. egating. By asking about the PN>>s prior experience with similar clients/tasks, the RN can Question14 A nurse is assigned to care for a comatose dia- determine whether the PN has the requisite exbetic on IV insulin therapy. Which task would be perience to care for the assigned clients. most appropriate to delegate to an unlicensed assistive personnel (UAP)? A) Check the client>s level of consciousness Question17 Collected by :DeepaRajesh [ 202 ] [email protected] Kuwait

The mother of a 4 month-old infant asks the nurse about the dangers of sunburn while they are on vacation at the beach. Which of the following is the best advice about sun protection for this child? A) «Use a sunscreen with a minimum sun protective factor of 15.» B) «Applications of sunscreen should be repeated every few hours.» C) «An infant should be protected by the maximum strength sunscreen.» D) «Sunscreens are not recommended in children younger than 6 months.»

Helplessness and hopelessness may contribute to regressive, dependent behavior which often occurs at any age with hospitalization. Denying or minimizing the seriousness of the illness is used to avoid facing the worst situation. Recall that denial is the initial step in the process of working through any loss.

Question20

A 52 year-old post menopausal woman asks the nurse how frequently she should have a mammogram. What is the nurse>s best response? A) «Your doctor will advise you about your risks.» B) «Unless you had previous problems, every 2 Review Information: The correct answer is D: years is best.» «Sunscreens are not recommended in children C) «Once a woman reaches 50, she should have younger than 6 months.» a mammogram yearly.» Infants under 6 months of age should be kept out D) «Yearly mammograms are advised for all of the sun or shielded from it. Even on a cloudy women over 35.» day, the infant can be sunburned while near water. A hat and light protective clothing should be worn. Review Information: The correct answer is C: «Once a woman reaches 50, she should have a mammogram yearly.» Question18 The nurse administers cimetidine (Tagamet) to a The American Cancer Society recommends a 79 year-old male with a gastric ulcer. Which pa- screening mammogram by age 40, every 1 - 2 rameter may be affected by this drug, and should years for women 40-49, and every year from age 50. If there are family or personal health risks, be closely monitored by the nurse? other assessments may be recommended. A) Blood pressure B) Liver function C) Mental status Question21 D) Hemoglobin The nurse is planning care for a client who is taking cyclosporin (Neoral). What would be an apReview Information: The correct answer is C: propriate nursing diagnosis for this client? Mental status A) Alteration in body image The elderly are at risk for developing confusion B) High risk for infection when taking cimetidine, a drug that interacts with C) Altered growth and development many other medications. D) Impaired physical mobility

Question19

The nurse assesses the use of coping mechanisms by an adolescent 1 week after the client had a motor vehicle accident resulting in multiple serious injuries. Which of these characteristics are most likely to be displayed? A) Ambivalence, dependence, demanding B) Denial, projection, regression C) Intellectualization, rationalization, repression D) Identification, assimilation, withdrawal Review Information: The correct answer is B: Denial, projection, regression

Review Information: The correct answer is B: High risk for infection Cyclosporin (Neoral) inhibits normal immune responses. Clients receiving cyclosporin are at risk for infection.

Question22

A client on telemetry begins having premature ventricular beats (PVBs) at 12 per minute. In reviewing the most recent laboratory results, which would require immediate action by the nurse? A) Calcium 9 mg/dl Collected by :DeepaRajesh [ 203 ] [email protected] Kuwait

B) Magnesium 2.5 mg/dl C) Potassium 2.5 mEq/L D) PTT 70 seconds Review Information: The correct answer is C: Potassium 2.5 mEq/L The patient is at risk for ventricular dysrhythmias when the potassium level is low. Daniels, R. (2003).

Question23

The nurse is caring for a client who is 4 days post-op for a transverse colostomy. The client is ready for discharge and asks the nurse to empty his colostomy pouch. What is the best response by the nurse? A) «You should be emptying the pouch yourself.» B) «Let me demonstrate to you how to empty the pouch.» C) «What have you learned about emptying your pouch?» D) «Show me what you have learned about emptying your pouch.» Review Information: The correct answer is D: «Show me what you have learned about emptying your pouch.» Most adult learners obtain skills by participating in the activities. Anxiety about discharge can be causing the client to forget that they have mastered the skill of emptying the pouch. The client should show the nurse how the pouch is emptied.

Question24

A 3 year-old child has tympanostomy tubes in place. The child>s parent asks the nurse if he can swim in the family pool. The best response from the nurse is A) «Your child should not swim at all while the tubes are in place.» B) «Your child may swim in your own pool but not in a lake or ocean.» C) «Your child may swim if he wears ear plugs.» D) «Your child may swim anywhere.» Review Information: The correct answer is C: «Your child may swim if he wears ear plugs.»

Water should not enter the ears. Children should use ear plugs when bathing or swimming and should not put their heads under the water.

Question25

The nurse is caring for a client with asthma who has developed gastroesophageal reflux disease (GERD). Which of the following medications prescribed for the client may aggravate GERD? A) Anticholinergics B) Corticosteroids C) Histamine blocker D) Antibiotics Review Information: The correct answer is A: Anticholinergics An anticholinergic medication will decrease gastric emptying and the pressure on the lower esophageal sphincter.

Question26

A client is receiving a nitroglycerin infusion for unstable angina. What assessment would be a priority when monitoring the effects of this medication? A) Blood pressure B) Cardiac enzymes C) ECG analysis D) Respiratory rate Review Information: The correct answer is A: Blood pressure Since an effect of this drug is vasodilation, the client must be monitored for hypotension.

Question27

The nurse is caring for a 10 year-old child who has just been diagnosed with diabetes insipidus. The parents ask about the treatment prescribed, vasopressin. A What is priority in teaching the child and family about this drug? A) The child should carry a nasal spray for emergency use B) The family must observe the child for dehydration C) Parents should administer the daily intramuscular injections D) The client needs to take daily injections in the short-term Review Information: The correct answer is A: Collected by :DeepaRajesh [ 204 ] [email protected] Kuwait

The child should carry a nasal spray for emergency use Diabetes insipidus results from reduced secretion of the antidiuretic hormone, vasopressin. The child will need to administer daily injections of vasopressin, and should have the nasal spray form of the medication readily available. A medical alert tag should be worn.

Question28

ent tells the nurse that she is stupid. What is the most therapeutic response by the nurse? A) Explore what is going on with the client B) Accept the client’s statement without comment C) Tell the client that the comment is inappropriate D) Leave the client>s room

A client diagnosed with cirrhosis is started on lactulose (Cephulac). The main purpose of the drug for this client is to A) add dietary fiber B) reduce ammonia levels C) stimulate peristalsis D) control portal hypertension

Review Information: The correct answer is A: Explore what is going on with the client Exploring feelings with the verbally aggressive client helps to put angry feelings into words and then to engage in problem solving.

Review Information: The correct answer is A: Cocaine use can cause fetal growth retardation Cocaine is vasoconstrictive, and this effect in the placental vessels causes fetal hypoxia and diminished growth. Other risks of continued cocaine use during pregnancy include preterm labor, congenital abnormalities, altered brain development and subsequent behavioral problems in the infant.

Question32

Question31

A client has many delusions. As the nurse helps the client prepare for breakfast the client comReview Information: The correct answer is B: ments «Don’t waste good food on me. I’m dying reduce ammonia levels from this disease I have.» The appropriate reLactulose blocks the absorption of ammonia from sponse would be the GI tract and secondarily stimulates bowel A) «You need some nutritious food to help you elimination. regain your weight.» B) «None of the laboratory reports show that you have any physical disease.» Question29 The nurse is explaining the effects of cocaine C) «Try to eat a little bit, breakfast is the most abuse to a pregnant client. Which of the follow- important meal of the day.» ing must the nurse understand as a basis for D) «I know you believe that you have an incurable disease.» teaching? A) Cocaine use can cause fetal growth retardation B) The drug has been linked to neural tube de- Review Information: The correct answer is D: «I know you believe that you have an incurable fects C) Newborn withdrawal generally occurs imme- disease.» This response does not challenge the client’s diately after birth D) Breast feeding promotes positive parenting delusional system and thus forms an alliance by providing reassurance of desire to help the clibehaviors ent. A client with paranoid thoughts refuses to eat because of the belief that the food is poisoned. The appropriate statement at this time for the nurse to say is A) «Here, I will pour a little of the juice in a medicine cup to drink it to show you that it is OK.» B) «The food has been prepared in our kitchen and is not poisoned.» C) «Let>s see if your partner could bring food from home.» Question30 A client has just been diagnosed with breast D) «If you don>t eat, I will have to suggest for cancer. The nurse enters the room and the cli- you to be tube fed.» Collected by :DeepaRajesh [ 205 ] [email protected] Kuwait

Review Information: The correct answer is C: «Let>>s see if your partner could bring food from home.» Reassurance is ineffective when a client is actively delusional. This option avoids both arguing with the client and agreeing with the delusional premise. Option D offers a logical response to a primarily affective concern. When the client’s condition has improved, gentle negation of the delusional premise can be employed.

Question33

A client with tuberculosis is started on Rifampin. Which one of the following statements by the nurse would be appropriate to include in teaching? «You may notice: A) an orange-red color to your urine.» B) your appetite may increase for the first week.” C) it is common to experience occasional sleep disturbances.» D) if you take the medication with food, you may have nausea.»

scribed medication.» Abrupt withdrawal may occasionally cause serotonin syndrome, consisting of lethargy, nausea, headache, fever, sweating and chills. A slow withdrawal may be prescribed with sertraline to avoid dizziness, nausea, vomiting, and diarrhea.

Question35

A client is admitted to the hospital with findings of liver failure with ascites. The health care provider orders spironolactone (Aldactone). What is the pharmacological effect of this medication? A) Promotes sodium and chloride excretion B) Increases aldosterone levels C) Depletes potassium reserves D) Combines safely with antihypertensives Review Information: The correct answer is A: Promotes sodium and chloride excretion Spironolactone promotes sodium and chloride excretion while sparing potassium and decreasing aldosterone levels. It had no effect on ammonia levels.

Question36

A client was admitted to the psychiatric unit for severe depression. After several days, the client continues to withdraw from the other clients. Which of these statements by the nurse would be the most appropriate to promote interaction with other clients? A) «Your team here thinks it>s good for you to spend time with others.» Question34 B) «It is important for you to participate in group A client tells the RN she has decided to stop tak- activities.» ing sertraline (Zoloft) because she doesn’t like C) «Come with me so you can paint a picture to the nightmares, sex dreams, and obsessions help you feel better.» she’s experiencing since starting on the medi- D) «Come play Chinese Checkers with Gloria cation. What is an appropriate response by the and me.» nurse? A) «It is unsafe to abruptly stop taking any pre- Review Information: The correct answer is D: «Come play Chinese Checkers with Gloria and scribed medication.» B) «Side effects and benefits should be dis- me.» cussed with your health care provider.» This gradually engages the client in interactions C) «This medication should be continued despite with others in small groups rather than large unpleasant symptoms.» groups. In addition, focusing on an activity is less D) «Many medications have potential side ef- anxiety-provoking than unstructured discussion. fects.» The statement is an example of a positive behavioral expectation. Review Information: The correct answer is A: an orange-red color to your urine.» Discoloration of the urine and other body fluids may occur. It is a harmless response to the drug, but the patient needs to be aware it may happen.

Review Information: The correct answer is A: «It is unsafe to abruptly stop taking any pre-

Question37

The nurse is teaching a school-aged child and Collected by :DeepaRajesh [ 206 ] [email protected] Kuwait

family about the use of inhalers prescribed for The nurse is beginning nutritional counseling/ asthma. What is the best way to evaluate effec- teaching with a pregnant woman. What is the initiveness of the treatments? tial step in this interaction? A) Rely on child>s self-report B) Use a peak-flow meter C) Note skin color changes D) Monitor pulse rate

A) Teach her how to meet the needs of self and her family B) Explain the changes in diet necessary for pregnant women

Review Information: The correct answer is B: Use a peak-flow meter The peak flowmeter, if used correctly, shows effectiveness of inhalants.

D) Conduct a diet history to determine her normal eating routines

Question38

The nurse is teaching a client about the toxicity of digoxin. Which one of the following statements made by the client to the nurse indicates more teaching is needed? A) «I may experience a loss of appetite.» B) «I can expect occasional double vision.» C) «Nausea and vomiting may last a few days.» D) «I must report a bounding pulse of 62 immediately.» Review Information: The correct answer is D: «I must report a bounding pulse of 62 immediately.» Slow heart rate is related to increased cardiac output and an intended effect of digoxin. The ideal heart rate is above 60 BPM with digoxin. The client needs further teaching.

Question39

Which of the following assessments by the nurse would indicate that the client is having a possible adverse response to the isoniazid (INH)? A) Severe headache B) Appearance of jaundice C) Tachycardia D) Decreased hearing Review Information: The correct answer is B: Appearance of jaundice Clients receiving INH therapy are at risk for developing drug induced hepatitis. The appearance of jaundice may indicate that the client has liver damage.

C) Questionher understanding and use of the food pyramid

Review Information: The correct answer is D: Conduct a diet history to determine her normal eating routines. Assessment is always the first step in planning teaching for any client. A thorough and accurate history is essential for gathering the needed information. 0 comments Labels: free nclex-rn sample review questions, nclex-rn practice test questions, nursing review

Free NCLEX-RN Sample Test Questions For Nursing Review (Part 1) These are sample nursing review questions and not actual test questions made for educational and practice test purposes only. 75 questions have been posted here with answer keys.

Question1

A client has been hospitalized after an automobile accident. A full leg cast was applied in the emergency room. The most important reason for the nurse to elevate the casted leg is to A) Promote the client>s comfort B) Reduce the drying time C) Decrease irritation to the skin D) Improve venous return Review Information: The correct answer is D: Improve venous return. Elevating the leg both improves venous return and reduces swelling. Client comfort will be improved as well.

Question40 Collected by :DeepaRajesh [ 207 ] [email protected] Kuwait

Question2

The nurse is reviewing with a client how to collect a clean catch urine specimen. What is the appropriate sequence to teach the client? A) Clean the meatus, begin voiding, then catch urine stream B) Void a little, clean the meatus, then collect specimen C) Clean the meatus, then urinate into container D) Void continuously and catch some of the urine

Question4

A client with Guillain Barre is in a nonresponsive state, yet vital signs are stable and breathing is independent. What should the nurse document to most accurately describe the client>s condition? A) Comatose, breathing unlabored B) Glascow Coma Scale 8, respirations regular C) Appears to be sleeping, vital signs stable D) Glascow Coma Scale 13, no ventilator required Review Information: The correct answer is B: Glascow Coma Scale 8, respirations regular. The Glascow Coma Scale provides a standard reference for assessing or monitoring level of consciousness. Any score less than 13 indicates a neurological impairment. Using the term comatose provides too much room for interpretation and is not very precise.

Review Information: The correct answer is A: Clean the meatus, begin voiding, then catch urine stream. A clean catch urine is difficult to obtain and requires clear directions. Instructing the client to carefully clean the meatus, then void naturally with a steady stream prevents surface bacteria from contaminating the urine specimen. As starting and stopping flow can be difficult, once the client begins voiding it>>s best to just slip the container into the stream. Other respons- Question5 es do not reflect correct technique. When caring for a client receiving warfarin sodium (Coumadin), which lab test would the nurse monitor to determine therapeutic response to the Question3 drug? Following change-of-shift report on an orthoped- A) Bleeding time ic unit, which client should the nurse see first? B) Coagulation time A) 16 year-old who had an open reduction of a C) Prothrombin time fractured wrist 10 hours ago D) Partial thromboplastin time B) 20 year-old in skeletal traction for 2 weeks since a motor cycle accident Review Information: The correct answer is C) 72 year-old recovering from surgery after a C: Prothrombin time. Coumadin is ordered daily, hip replacement 2 hours ago based on the client>>s prothrombin time (PT). D) 75 year-old who is in skin traction prior to This test evaluates the adequacy of the extrinsic planned hip pinning surgery. system and common pathway in the clotting cascade; Coumadin affects the Vitamin K dependReview Information: The correct answer is C: ent clotting factors. 72 year-old recovering from surgery after a hip replacement 2 hours ago. Look for the client who has the most imminent risks and acute vulnerabil- Question6 ity. The client who returned from surgery 2 hours A client with moderate persistent asthma is adago is at risk for life threatening hemorrhage and mitted for a minor surgical procedure. On adshould be seen first. The 16 year-old should be mission the peak flow meter is measured at 480 seen next because it is still the first post-op day. liters/minute. Post-operatively the client is comThe 75 year-old is potentially vulnerable to age- plaining of chest tightness. The peak flow has related physical and cognitive consequences in dropped to 200 liters/minute. What should the skin traction should be seen next. The client who nurse do first? can safely be seen last is the 20 year-old who is A) Notify both the surgeon and provider 2 weeks post-injury. B) Administer the prn dose of albuterol C) Apply oxygen at 2 liters per nasal cannula D) Repeat the peak flow reading in 30 minutes Collected by :DeepaRajesh [ 208 ] [email protected] Kuwait

Review Information: The correct answer is B: Administer the prn dose of albuterol. Peak flow monitoring during exacerbations of asthma is recommended for clients with moderate-tosevere persistent asthma to determine the severity of the exacerbation and to guide the treatment. A peak flow reading of less than 50% of the client>>s baseline reading is a medical alert condition and a short-acting beta-agonist must be taken immediately.

Question7

A client had 20 mg of Lasix (furosemide) PO at 10 AM. Which would be essential for the nurse to include at the change of shift report? A) The client lost 2 pounds in 24 hours B) The client’s potassium level is 4 mEq/liter. C) The client’s urine output was 1500 cc in 5 hours D) The client is to receive another dose of Lasix at 10 PM Review Information: The correct answer is C: The client’s urine output was 1500 cc in 5 hours. Although all of these may be correct information to include in report, the essential piece would be the urine output.

needed.

Question9

The nurse has performed the initial assessments of 4 clients admitted with an acute episode of asthma. Which assessment finding would cause the nurse to call the provider immediately? A) prolonged inspiration with each breath B) expiratory wheezes that are suddenly absent in 1 lobe C) expectoration of large amounts of purulent mucous D) appearance of the use of abdominal muscles for breathing Review Information: The correct answer is B: expiratory wheezes that are suddenly absent in 1 lobe. Acute asthma is characterized by expiratory wheezes caused by obstruction of the airways. Wheezes are a high pitched musical sounds produced by air moving through narrowed airways. Clients often associate wheezes with the feeling of tightness in the chest. However, sudden cessation of wheezing is an ominous or bad sign that indicates an emergency -- the small airways are now collapsed.

Question10

During the initial home visit, a nurse is discussing the care of a client newly diagnosed with A client has been tentatively diagnosed with Alzheimer>s disease with family members. Graves> disease (hyperthyroidism). Which of Which of these interventions would be most these findings noted on the initial nursing assess- helpful at this time? A) leave a book about relaxation techniques ment requires quick intervention by the nurse? A) a report of 10 pounds weight loss in the last B) write out a daily exercise routine for them to assist the client to do month B) a comment by the client «I just can>t sit C) list actions to improve the client>s daily nutritional intake still.» C) the appearance of eyeballs that appear to D) suggest communication strategies «pop» out of the client>s eye sockets D) a report of the sudden onset of irritability in Review Information: The correct answer is D: suggest communication strategies. Alzheimer>>s the past 2 weeks disease, a progressive chronic illness, greatly Review Information: The correct answer is C: challenges caregivers. The nurse can be of the appearance of eyeballs that appear to «pop» greatest assistance in helping the family to use out of the client>>s eye sockets. Exophthalmos communication strategies to enhance their ability or protruding eyeballs is a distinctive characteris- to relate to the client. By use of select verbal and tic of Graves>> Disease. It can result in corneal nonverbal communication strategies the family abrasions with severe eye pain or damage when can best support the client’s strengths and cope the eyelid is unable to blink down over the pro- with any aberrant behavior. truding eyeball. Eye drops or ointment may be

Question8

Collected by :DeepaRajesh [ 209 ] [email protected] Kuwait

Question11

An 80 year-old client admitted with a diagnosis of possible cerebral vascular accident has had a blood pressure from 160/100 to 180/110 over the past 2 hours. The nurse has also noted increased lethargy. Which assessment finding should the nurse report immediately to the provider? A) Slurred speech B) Incontinence C) Muscle weakness D) Rapid pulse

ate action is required? A) pH below 7.3 B) Potassium of 5.0 C) HCT of 60 D) Pa O2 of 79%

Review Information: The correct answer is C: HCT of 60. This high hematocrit is indicative of severe dehydration which requires priority attention in diabetic ketoacidosis. Without sufficient hydration, all systems of the body are at risk for hypoxia from a lack of or sluggish circulation. In the absence of insulin, which facilitates Review Information: The correct answer is A: the transport of glucose into the cell, the body Slurred speech. Changes in speech patterns and breaks down fats and proteins to supply energy level of conscious can be indicators of continued ketones, a by-product of fat metabolism. These intracranial bleeding or extension of the stroke. accumulate causing metabolic acidosis (pH < 7.3), which would be the second concern for this Further diagnostic testing may be indicated. client. The potassium and PaO2 levels are near normal.

Question12

A school-aged child has had a long leg (hip to ankle) synthetic cast applied 4 hours ago. Which statement from the parent indicates that teaching has been inadequate? A) «I will keep the cast uncovered for the next day to prevent burning of the skin.» B) «I can apply an ice pack over the area to relieve itching inside the cast.» C) «The cast should be propped on at least 2 pillows when my child is lying down.» D) «I think I remember that my child should not stand until after 72 hours.» Review Information: The correct answer is D: «I think I remember that my child should not stand until after 72 hours.». Synthetic casts will typically set up in 30 minutes and dry in a few hours. Thus, the client may stand within the initial 24 hours. With plaster casts, the set up and drying time, especially in a long leg cast which is thicker than an arm cast, can take up to 72 hours. Both types of casts give off a lot of heat when drying and it is preferable to keep the cast uncovered for the first 24 hours. Clients may complain of a chill from the wet cast and therefore can simply be covered lightly with a sheet or blanket. Applying ice is a safe method of relieving the itching.

Question13

Which blood serum finding in a client with diabetic ketoacidosis alerts the nurse that immedi-

Question14

The nurse is preparing a client with a deep vein thrombosis (DVT) for a Venous Doppler evaluation. Which of the following would be necessary for preparing the client for this test? A) Client should be NPO after midnight B) Client should receive a sedative medication prior to the test C) Discontinue anti-coagulant therapy prior to the test D) No special preparation is necessary Review Information: The correct answer is D: No special preparation is necessary. This is a non-invasive procedure and does not require preparation other than client education.

Question15

A client is admitted with infective endocarditis (IE). Which finding would alert the nurse to a complication of this condition? A) dyspnea B) heart murmur C) macular rash D) hemorrhage Review Information: The correct answer is B: heart murmur. Large, soft, rapidly developing vegetations attach to the heart valves. They have a tendency to break off, causing emboli and leavCollected by :DeepaRajesh [ 210 ] [email protected] Kuwait

ing ulcerations on the valve leaflets. These emboli produce findings of cardiac murmur, fever, anorexia, malaise and neurologic sequelae of emboli. Furthermore, the vegetations may travel to various organs such as spleen, kidney, coronary artery, brain and lungs, and obstruct blood flow.

Question16

The nurse explains an autograft to a client scheduled for excision of a skin tumor. The nurse knows the client understands the procedure when the client says, «I will receive tissue from A) a tissue bank.» B) a pig.» C) my thigh.» D) synthetic skin.» Review Information: The correct answer is C: my thigh.». Autografts are done with tissue transplanted from the client>>s own skin.

Question17

Review Information: The correct answer is B: Frequent neurovascular assessments of the affected leg. The most important activity for the nurse is to assess neurovascular status. Compartment syndrome is a serious complication of fractures. Prompt recognition of this neurovascular problem and early intervention may prevent permanent limb damage.

Question19

The nurse is assigned to care for a client who had a myocardial infarction (MI) 2 days ago. The client has many questions about this condition. What area is a priority for the nurse to discuss at this time? A) Daily needs and concerns B) The overview cardiac rehabilitation C) Medication and diet guideline D) Activity and rest guidelines Review Information: The correct answer is A: Daily needs and concerns. At 2 days post-MI, the client’s education should be focused on the immediate needs and concerns for the day.

A client is admitted to the emergency room following an acute asthma attack. Which of the following assessments would be expected by the nurse? Question20 A 3 year-old child is brought to the clinic by his A) Diffuse expiratory wheezing grandmother to be seen for «scratching his botB) Loose, productive cough tom and wetting the bed at night.» Based on C) No relief from inhalant these complaints, the nurse would initially asD) Fever and chills sess for which problem? A) allergies Review Information: The correct answer is A: B) scabies Diffuse expiratory wheezing. In asthma, the air- C) regression ways are narrowed, creating difficulty getting air D) pinworms in. A wheezing sound results. Review Information: The correct answer is D: pinworms. Signs of pinworm infection include intense perianal itching, poor sleep patterns, Question18 A client has been admitted with a fractured femur general irritability, restlessness, bed-wetting, and has been placed in skeletal traction. Which distractibility and short attention span. Scabies of the following nursing interventions should re- is an itchy skin condition caused by a tiny, eightlegged burrowing mite called Sarcoptes scabiei . ceive priority? The presence of the mite leads to intense itching A) Maintaining proper body alignment B) Frequent neurovascular assessments of the in the area of its burrows. affected leg C) Inspection of pin sites for evidence of drainage or inflammation Question21 D) Applying an over-bed trapeze to assist the cli- The nurse is caring for a newborn with traent with movement in bed cheoesophageal fistula. Which nursing diagnoCollected by :DeepaRajesh [ 211 ] [email protected] Kuwait

sis is a priority? A) Risk for dehydration B) Ineffective airway clearance C) Altered nutrition D) Risk for injury Review Information: The correct answer is B: Ineffective airway clearance. The most common form of TEF is one in which the proximal esophageal segment terminates in a blind pouch and the distal segment is connected to the trachea or primary bronchus by a short fistula at or near the bifurcation. Thus, a priority is maintaining an open airway, preventing aspiration. Other nursing diagnoses are then addressed.

Question22

The nurse is developing a meal plan that would provide the maximum possible amount of iron for a child with anemia. Which dinner menu would be best? A) Fish sticks, french fries, banana, cookies, milk B) Ground beef patty, lima beans, wheat roll, raisins, milk C) Chicken nuggets, macaroni, peas, cantaloupe, milk D) Peanut butter and jelly sandwich, apple slices, milk Review Information: The correct answer is B: Ground beef patty, lima beans, wheat roll, raisins, milk. Iron rich foods include red meat, fish, egg yolks, green leafy vegetables, legumes, whole grains, and dried fruits such as raisins. This dinner is the best choice: It is high in iron and is appropriate for a toddler.

Question23

The nurse admitting a 5 month-old who vomited 9 times in the past 6 hours should observe for signs of which overall imbalance? A) Metabolic acidosis B) Metabolic alkalosis C) Some increase in the serum hemoglobin D) A little decrease in the serum potassium Review Information: The correct answer is B: Metabolic alkalosis. Vomiting causes loss of acid from the stomach. Prolonged vomiting can result in excess loss of acid and lead to metabolic

alkalosis. Findings include irritability, increased activity, hyperactive reflexes, muscle twitching and elevated pulse. Options C and D are correct answers but not the best answers since they are too general.

Question24

A two year-old child is brought to the provider>s office with a chief complaint of mild diarrhea for two days. Nutritional counseling by the nurse should include which statement? A) Place the child on clear liquids and gelatin for 24 hours B) Continue with the regular diet and include oral rehydration fluids C) Give bananas, apples, rice and toast as tolerated D) Place NPO for 24 hours, then rehydrate with milk and water Review Information: The correct answer is B: Continue with the regular diet and include oral rehydration fluids. Current recommendations for mild to moderate diarrhea are to maintain a normal diet with fluids to rehydrate.

Question25

The nurse is teaching parents about the appropriate diet for a 4 month-old infant with gastroenteritis and mild dehydration. In addition to oral rehydration fluids, the diet should include A) formula or breast milk B) broth and tea C) rice cereal and apple juice D) gelatin and ginger ale Review Information: The correct answer is A: formula or breast milk. The usual diet for a young infant should be followed.

Question26

A child is injured on the school playground and appears to have a fractured leg. The first action the school nurse should take is A) call for emergency transport to the hospital B) immobilize the limb and joints above and below the injury C) assess the child and the extent of the injury Collected by :DeepaRajesh [ 212 ] [email protected] Kuwait

D) apply cold compresses to the injured area Review Information: The correct answer is C: assess the child and the extent of the injury. When applying the nursing process, assessment is the first step in providing care. The «5 Ps» of vascular impairment can be used as a guide (pain, pulse, pallor, paresthesia, paralysis).

in a week. Iron fortified cereal is the recommended first food.

Question29

The nurse planning care for a 12 year-old child with sickle cell disease in a vaso-occlusive crisis of the elbow should include which one of the following as a priority?

Question27

A) Limit fluids The mother of a 3 month-old infant tells the nurse B) Client controlled analgesia that she wants to change from formula to whole C) Cold compresses to elbow milk and add cereal and meats to the diet. What D) Passive range of motion exercise should be emphasized as the nurse teaches Review Information: The correct answer is B: about infant nutrition? Client controlled analgesia. Management of a A) Solid foods should be introduced at 3-4 sickle cell crisis is directed towards supportive and symptomatic treatment. The priority of care months B) Whole milk is difficult for a young infant to di- is pain relief. In a 12 year-old child, client controlled analgesia promotes maximum comfort. gest C) Fluoridated tap water should be used to dilute milk D) Supplemental apple juice can be used be- Question30 tween feedings The nurse is performing a physical assessment on a toddler. Which of the following actions Review Information: The correct answer is B: should be the first? Whole milk is difficult for a young infant to digest. Cow>>s milk is not given to infants younger than A) Perform traumatic procedures 1 year because the tough, hard curd is difficult to B) Use minimal physical contact digest. In addition, it contains little iron and cre- C) Proceed from head to toe ates a high renal solute load. D) Explain the exam in detail

Question28

The nurse is preparing a handout on infant feeding to be distributed to families visiting the clinic. Which notation should be included in the teaching materials? A) Solid foods are introduced one at a time beginning with cereal B) Finely ground meat should be started early to provide iron C) Egg white is added early to increase protein intake D) Solid foods should be mixed with formula in a bottle

Review Information: The correct answer is B: Use minimal physical contact. The nurse should approach the toddler slowly and use minimal physical contact initially so as to gain the toddler>>s cooperation. Be flexible in the sequence of the exam, and give only brief simple explanations just prior to the action.

Question31

What finding signifies that children have attained the stage of concrete operations (Piaget)?

A) Explores the environment with the use of sight and movement Review Information: The correct answer is A: B) Thinks in mental images or word pictures Solid foods are introduced one at a time begin- C) Makes the moral judgment that «stealing is ning with cereal. Solid foods should be added wrong» one at a time between 4-6 months. If the infant is D) Reasons that homework is time-consuming able to tolerate the food, another may be added yet necessary Collected by :DeepaRajesh [ 213 ] [email protected] Kuwait

Review Information: The correct answer is C: Makes the moral judgment that «stealing is wrong». The stage of concrete operations is depicted by logical thinking and moral judgments.

Question32

The mother of a child with a neural tube defect asks the nurse what she can do to decrease the chances of having another baby with a neural tube defect. What is the best response by the nurse? A) «Folic acid should be taken before and after conception.» B) «Multivitamin supplements are recommended during pregnancy.» C) «A well balanced diet promotes normal fetal development.» D) «Increased dietary iron improves the health of mother and fetus.» Review Information: The correct answer is A: «Folic acid should be taken before and after conception.». The American Academy of Pediatrics recommends that all childbearing women increase folic acid from dietary sources and/or supplements. There is evidence that increased amounts of folic acid prevents neural tube defects.

Question33

The provider orders Lanoxin (digoxin) 0.125 mg PO and furosemide 40 mg every day. Which of these foods would the nurse reinforce for the client to eat at least daily? A) Spaghetti B) Watermelon C) Chicken D) Tomatoes Review Information: The correct answer is B: Watermelon. Watermelon is high in potassium and will replace potassium lost by the diuretic. The other foods are not high in potassium.

Question34

While teaching the family of a child who will take phenytoin (Dilantin) regularly for seizure control,

it is most important for the nurse to teach them about which of the following actions? A) Maintain good oral hygiene and dental care B) Omit medication if the child is seizure free C) Administer acetaminophen to promote sleep D) Serve a diet that is high in iron Review Information: The correct answer is A: Maintain good oral hygiene and dental care. Swollen and tender gums occur often with use of phenytoin. Good oral hygiene and regular visits to the dentist should be emphasized.

Question35

The nurse is offering safety instructions to a parent with a four month-old infant and a four yearold child. Which statement by the parent indicates understanding of appropriate precautions to take with the children? A) «I strap the infant car seat on the front seat to face backwards.» B) «I place my infant in the middle of the living room floor on a blanket to play with my four yearold while I make supper in the kitchen.» C) «My sleeping baby lies so cute in the crib with the little buttocks stuck up in the air while the four year-old naps on the sofa.» D) «I have the four year-old hold and help feed the four month-old a bottle in the kitchen while I make supper.» Review Information: The correct answer is D: «I have the four year-old hold and help feed the four month-old a bottle in the kitchen while I make supper.». The infant seat is to be placed on the rear seat. Small children and infants are not to be left unsupervised. Infants are

Question36

The nurse admits a 7 year-old to the emergency room after a leg injury. The x-rays show a femur fracture near the epiphysis. The parents ask what will be the outcome of this injury. The appropriate response by the nurse should be which of these statements? A) «The injury is expected to heal quickly because of thin periosteum.» B) «In some instances the result is a retarded Collected by :DeepaRajesh [ 214 ] [email protected] Kuwait

bone growth.» C) «Bone growth is stimulated in the affected leg.» D) «This type of injury shows more rapid union than that of younger children.»

cussing formula preparation, which of the following is most important to prevent lead poisoning?

A) Use ready-to-feed commercial infant formula B) Boil the tap water for 10 minutes prior to preparing the formula Review Information: The correct answer is B: C) Let tap water run for 2 minutes before adding «In some instances the result is a retarded bone to concentrate growth.». An epiphyseal (growth) plate fracture D) Buy bottled water labeled «lead free» to mix in a 7 year-old often results in retarded bone the formula growth. The leg often will be different in length than the uninjured leg. Review Information: The correct answer is C: Let tap water run for 2 minutes before adding to concentrate. Use of lead-contaminated water to prepare formula is a major source of poisoning Question37 The parents of a 4 year-old hospitalized child tell in infants. Drinking water may be contaminated the nurse, “We are leaving now and will be back by lead from old lead pipes or lead solder used at 6 PM.” A few hours later the child asks the in sealing water pipes. Letting tap water run for nurse when the parents will come again. What is several minutes will diminish the lead contamination. the best response by the nurse? A) «They will be back right after supper.» B) «In about 2 hours, you will see them.» C) «After you play awhile, they will be here.» D) «When the clock hands are on 6 and 12.» Review Information: The correct answer is A: «They will be back right after supper.». Time is not completely understood by a 4 year-old. Preschoolers interpret time with their own frame of reference. Thus, it is best to explain time in relationship to a known, common event.

Question38

The nurse is giving instructions to the parents of a child with cystic fibrosis. The nurse would emphasize that pancreatic enzymes should be taken A) once each day B) 3 times daily after meals C) with each meal or snack D) each time carbohydrates are eaten Review Information: The correct answer is C: with each meal or snack. Pancreatic enzymes should be taken with each meal and every snack to allow for digestion of all foods that are eaten.

Question39

A nurse is providing a parenting class to individuals living in a community of older homes. In dis-

Question40

Which of the following manifestations observed by the school nurse confirms the presence of pediculosis capitis in students? A) Scratching the head more than usual B) Flakes evident on a student>s shoulders C) Oval pattern occipital hair loss D) Whitish oval specks sticking to the hair Review Information: The correct answer is D: Whitish oval specks sticking to the hair. Diagnosis of pediculosis capitis is made by observation of the white eggs (nits) firmly attached to the hair shafts. Treatment can include application of a medicated shampoo with lindane for children over 2 years of age, and meticulous combing and removal of all nits.

Question41

When interviewing the parents of a child with asthma, it is most important to assess the child>s environment for what factor? A) Household pets B) New furniture C) Lead based paint D) Plants such as cactus Review Information: The correct answer is A: Collected by :DeepaRajesh [ 215 ] [email protected] Kuwait

Household pets. Animal dander is a very common allergen affecting persons with asthma. Other triggers may include pollens, carpeting and household dust.

Question42

The mother of a 2 month-old baby calls the nurse 2 days after the first DTaP, IPV, Hepatitis B and HIB immunizations. She reports that the baby feels very warm, cries inconsolably for as long as 3 hours, and has had several shaking spells. In addition to referring her to the emergency room, the nurse should document the reaction on the baby>s record and expect which immunization to be most associated with the findings the infant is displaying? A) DTaP B) Hepatitis B C) Polio D) H. Influenza Review Information: The correct answer is A: DTaP. The majority of reactions occur with the administration of the DTaP vaccination. Contradictions to giving repeat DTaP immunizations include the occurrence of severe side effects after a previous dose as well as signs of encephalopathy within 7 days of the immunization.

Question43

Question44

A couple experienced the loss of a 7 month-old fetus. In planning for discharge, what should the nurse emphasize? A) To discuss feelings with each other and use support persons B) To focus on the other healthy children and move through the loss C) To seek causes for the fetal death and come to some safe conclusion D) To plan for another pregnancy within 2 years and maintain physical health Review Information: The correct answer is A: To discuss feelings with each other and use support persons. To communicate in a therapeutic manner, the nurse>>s goal is to help the couple begin the grief process by suggesting they talk to each other, seek family, friends and support groups to listen to their feelings.

Question45

The nurse is performing a pre-kindergarten physical on a 5 year-old. The last series of vaccines will be administered. What is the preferred site for injection by the nurse? A) vastus intermedius B) gluteus maximus C) vastus lateralis D) dorsogluteaI

The mother of a 2 year-old hospitalized child asks the nurse>s advice about the child>s screaming every time the mother gets ready to leave the hospital room. What is the best response by the Review Information: The correct answer is C: nurse? vastus lateralis. Vastus lateralis, a large and well developed muscle, is the preferred site, since it is A) «I think you or your partner needs to stay with removed from major nerves and blood vessels. the child while in the hospital.» B) «Oh, that behavior will stop in a few days.» C) «Keep in mind that for the age this is a normal Question46 response to being in the hospital.» A 7 month pregnant woman is admitted with comD) «You might want to «sneak out» of the room plaints of painless vaginal bleeding over several once the child falls asleep.» hours. The nurse should prepare the client for an immediate Review Information: The correct answer is C: «Keep in mind that for the age this is a normal A) Non stress test response to being in the hospital.». The protest B) Abdominal ultrasound phase of separation anxiety is a normal response C) Pelvic exam for a child this age. In toddlers, ages 1 to 3, sepa- D) X-ray of abdomen ration anxiety is at its peak Collected by :DeepaRajesh [ 216 ] [email protected] Kuwait

Review Information: The correct answer is B: Abdominal ultrasound. The standard for diagnosis of placenta previa, which is suggested in the client>>s history of painless bleeding, is abdominal ultrasound.

Question47

A nurse entering the room of a postpartum mother observes the baby lying at the edge of the bed while the woman sits in a chair. The mother states «This is not my baby, and I do not want it.» After repositioning the child safely, the nurse>s best response is A) «This is a common occurrence after birth, but you will come to accept the baby.» B) «Many women have postpartum blues and need some time to love the baby.» C) «What a beautiful baby! Her eyes are just like yours.» D) «You seem upset; tell me what the pregnancy and birth were like for you.» Review Information: The correct answer is D: «You seem upset; tell me what the pregnancy and birth were like for you.». A non-judgmental, open ended response facilitates dialogue between the client and nurse.

Question48

The nurse notes that a 2 year-old child recovering from a tonsillectomy has an temperature of 98.2 degrees Fahrenheit at 8:00 AM. At 10:00 AM the child>s parent reports that the child «feels very warm» to touch. The first action by the nurse should be to A) reassure the parent that this is normal B) offer the child cold oral fluids C) reassess the child>s temperature D) administer the prescribed acetaminophen Review Information: The correct answer is C: reassess the child>>s temperature. A child>>s temperature may have rapid fluctuations. The nurse should listen to and show respect for what parents say. Parental caretakers are often quite sensitive to variations in their children>>s condition that may not be immediately evident to others.

Question49

The nurse is caring for a client who was successfully resuscitated from a pulseless dysrhythmia. Which of the following assessments is critical for the nurse to include in the plan of care? A) hourly urine output B) white blood count C) blood glucose every 4 hours D) temperature every 2 hours Review Information: The correct answer is A: hourly urine output. Clients who have had an episode of decreased glomerular perfusion are at risk for pre-renal failure. This is caused by any abnormal decline in kidney perfusion that reduces glomerular perfusion. Pre-renal failure occurs when the effective arterial blood volume falls. Examples of this phenomena include a drop in circulating blood volume as in a cardiac arrest state or in low cardiac perfusion states such as congestive heart failure associated with a cardiomyopathy. Close observation of hourly urinary output is necessary for early detection of this condition.

Question50

A client is admitted to the rehabilitation unit following a cerebral vascular accident (CVA) and mild dysphagia. The most appropriate intervention for this client is to A) position client in upright position while eating B) place client on a clear liquid diet C) tilt head back to facilitate swallowing reflex D) offer finger foods such as crackers or pretzels Review Information: The correct answer is A: position client in upright position while eating. An upright position facilitates proper chewing and swallowing.

Question51

A 72 year-old client with osteomyelitis requires a 6 week course of intravenous antibiotics. In planning for home care, what is the most important action by the nurse? A) Investigating the client>s insurance coverage Collected by :DeepaRajesh [ 217 ] [email protected] Kuwait

for home IV antibiotic therapy B) Determining if there are adequate hand washing facilities in the home C) Assessing the client>s ability to participate in self care and/or the reliability of a caregiver D) Selecting the appropriate venous access device Review Information: The correct answer is C: Assessing the client>>s ability to participate in self care and/or the reliability of a caregiver. The cognitive ability of the client as well as the availability and reliability of a caregiver must be assessed to determine if home care is a feasible option.

Question52

A nurse administers the influenza vaccine to a client in a clinic. Within 15 minutes after the immunization was given, the client complains of itchy and watery eyes, increased anxiety, and difficulty breathing. The nurse expects that the first action in the sequence of care for this client will be to A) Maintain the airway B) Administer epinephrine 1:1000 as ordered C) Monitor for hypotension with shock D) Administer diphenhydramine as ordered

Review Information: The correct answer is B: stimulates hydrochloric acid production. Decadron increases the production of hydrochloric acid, which may cause gastrointestinal ulcers.

Question54

A client receiving chlorpromazine HCL (Thorazine) is in psychiatric home care. During a home visit the nurse observes the client smacking her lips alternately with grinding her teeth. The nurse recognizes this assessment finding as what? A) Dystonia B) Akathisia C) Brady dyskinesia D) Tardive dyskinesia Review Information: The correct answer is D: Tardive dyskinesia. Signs of tardive dyskinesia include smacking lips, grinding of teeth and «fly catching» tongue movements. These findings are often described as Parkinsonian.

Question55

Which of the following findings contraindicate the use of haloperidol (Haldol) and warrant withholding the dose? A) Drowsiness, lethargy, and inactivity B) Dry mouth, nasal congestion, and blurred viReview Information: The correct answer is B: sion Administer epinephrine 1:1000 as ordered. All C) Rash, blood dyscrasias, severe depression the answers are correct given the circumstances, D) Hyperglycemia, weight gain, and edema but the priority is to administer the epinephrine, then maintain the airway. In the early stages of Review Information: The correct answer is anaphylaxis, when the patient has not lost con- C: Rash, blood dyscrasias, severe depression. sciousness and is normotensive, administering Rash and blood dyscrasias are side effects of the epinephrine is first, and applying the oxygen, anti-psychotic drugs. A history of severe depresand watching for hypotension and shock, are lat- sion is a contraindication to the use of neuroleper responses. The prevention of a severe crisis tics. is maintained by using diphenhydramine.

Question53

The nurse instructs the client taking dexamethasone (Decadron) to take it with food or milk. The physiological basis for this instruction is that the medication A) retards pepsin production B) stimulates hydrochloric acid production C) slows stomach emptying time D) decreases production of hydrochloric acid

Question56

The nurse is reinforcing teaching to a 24 year-old woman receiving acyclovir (Zovirax) for a Herpes Simplex Virus type 2 infection. Which of these instructions should the nurse give the client? A) Complete the entire course of the medication for an effective cure B) Begin treatment with acyclovir at the onset of symptoms of recurrence Collected by :DeepaRajesh [ 218 ] [email protected] Kuwait

C) Stop treatment if she thinks she may be pregnant to prevent birth defects D) Continue to take prophylactic doses for at least 5 years after the diagnosis

Which client statement from the assessment data is likely to explain his noncompliance?

A) «I have problems with diarrhea.» B) «I have difficulty falling asleep.» Review Information: The correct answer is C) «I have diminished sexual function.» B: Begin treatment with acyclovir at the onset D) «I often feel jittery.» of symptoms of recurrence. When the client is aware of early symptoms, such as pain, itching Review Information: The correct answer is C: or tingling, treatment is very effective. Medica- «I have diminished sexual function.». Inderal, a tions for herpes simplex do not cure the disease; beta-blocking agent used in hypertension, prothey simply decrease the level of symptoms. hibits the release of epinephrine into the cells; this may result in hypotension which results in decreased libido and impotence.

Question57

A 14 month-old child ingested half a bottle of aspirin tablets. Which of the following would the Question60 nurse expect to see in the child? The nurse caring for a 9 year-old child with a fractured femur is told that a medication error A) Hypothermia occurred. The child received twice the ordered B) Edema dose of morphine an hour ago. Which nursing C) Dyspnea diagnosis is a priority at this time? D) Epistaxis A) Risk for fluid volume deficit related to morReview Information: The correct answer is phine overdose D: Epistaxis. A large dose of aspirin inhibits pro- B) Decreased gastrointestinal mobility related to thrombin formation and lowers platelet levels. mucosal irritation With an overdose, clotting time is prolonged. C) Ineffective breathing patterns related to central nervous system depression D) Altered nutrition related to inability to control nausea and vomiting Question58 An 80 year-old client on digitalis (Lanoxin) reports nausea, vomiting, abdominal cramps and Review Information: The correct answer is C: halo vision. Which of the following laboratory re- Ineffective breathing patterns related to central nervous system depression. Respiratory depressults should the nurse analyze first? sion is a life-threatening risk in this overdose. A) Potassium levels B) Blood pH Question61 C) Magnesium levels D) Blood urea nitrogen Lactulose (Chronulac) has been prescribed for a client with advanced liver disease. Which of the Review Information: The correct answer is A: following assessments would the nurse use to Potassium levels. The most common cause of evaluate the effectiveness of this treatment? digitalis toxicity is a low potassium level. Clients must be taught that it is important to have ad- A) An increase in appetite equate potassium intake especially if taking diu- B) A decrease in fluid retention retics that enhance the loss of potassium while C) A decrease in lethargy they are taking digitalis. D) A reduction in jaundice

Question59

A 42 year-old male client refuses to take propranolol hydrochloride (Inderal) as prescribed.

Review Information: The correct answer is C: A decrease in lethargy. Lactulose produces an acid environment in the bowel and traps ammonia in the gut; the laxative effect then aids in Collected by :DeepaRajesh [ 219 ] [email protected] Kuwait

removing the ammonia from the body. This decreases the effects of hepatic encephalopathy, A) Listen quietly without comment including lethargy and confusion. B) Ask for further information on the spies C) Confront the client’s delusion D) Contact the government agency

Question62

The nurse is teaching a class on HIV prevention. Which of the following should be emphasized as increasing risk? A) Donating blood B) Using public bathrooms C) Unprotected sex D) Touching a person with AIDS Review Information: The correct answer is C: Unprotected sex. Because HIV is spread through exposure to bodily fluids, unprotected intercourse and shared drug paraphernalia remain the highest risks for infection.

Question63

While interviewing a new admission, the nurse notices that the client is shifting positions, wringing her hands, and avoiding eye contact. It is important for the nurse to A) ask the client what she is feeling B) assess the client for auditory hallucinations C) recognize the behavior as a side effect of medication D) re-focus the discussion on a less anxiety provoking topic Review Information: The correct answer is A: ask the client what she is feeling. The initial step in anxiety intervention is observing, identifying, and assessing anxiety. The nurse should seek client validation of the accuracy of nursing assessments and avoid drawing conclusions based on limited data. In the situation above, the client may simply need to use the restroom but be reluctant to communicate her need!

Question64

A young adult seeks treatment in an outpatient mental health center. The client tells the nurse he is a government official being followed by spies. On further questioning, he reveals that his warnings must be heeded to prevent nuclear war. What is the most therapeutic approach by the nurse?

Review Information: The correct answer is A: Listen quietly without comment. The client>>s comments demonstrate grandiose ideas. The most therapeutic response is to listen but avoid being incorporated into the client’s delusional system.

Question65

The nurse is assessing a 17 year-old female client with bulimia. Which of the following laboratory reports would the nurse anticipate? A) Increased serum glucose B) Decreased albumin C) Decreased potassium D) Increased sodium retention Review Information: The correct answer is C: Decreased potassium. In bulimia, loss of electrolytes can occur in addition to other findings of starvation and dehydration.

Question66

A client, recovering from alcoholism, asks the nurse, «What can I do when I start recognizing relapse triggers within myself?» How might the nurse best respond? A) «When you have the impulse to stop in a bar, contact a sober friend and talk with him.» B) «Go to an AA meeting when you feel the urge to drink.» C) «It is important to exercise daily and get involved in activities that will cause you not to think about drug use.» D) «Let’s talk about possible options you have when you recognize relapse triggers in yourself.» Review Information: The correct answer is D: «Let’s talk about possible options you have when you recognize relapse triggers in yourself.». This option encourages the process of self evaluation and problem solving, while avoiding telling the client what to do. Encouraging the client to brainstorm about response options validates the Collected by :DeepaRajesh [ 220 ] [email protected] Kuwait

nurse’s belief in the client’s personal competency and reinforces a coping strategy that will be needed when the nurse may not be available to offer solutions.

B) provide a group forum in which clients decide on unit rules, regulations, and policies C) provide a testing ground for new patterns of behavior while the client takes responsibility for his or her own actions D) discourage expressions of anger because Question67 Therapeutic nurse-client interaction occurs when they can be disruptive to other clients the nurse A) assists the client to clarify the meaning of what Review Information: The correct answer is C: provide a testing ground for new patterns of bethe client has said havior while the client takes responsibility for his B) interprets the client’s covert communication C) praises the client for appropriate feelings and or her own actions. A therapeutic milieu is purposeful and planned to provide safety and a testbehavior D) advises the client on ways to resolve prob- ing ground for new patterns of behavior. lems Review Information: The correct answer is A: assists the client to clarify the meaning of what the client has said. Clarification is a facilitating/ therapeutic communication strategy. Interpretation, changing the focus/subject, giving approval, and advising are non-therapeutic/barriers to communication.

Question70

A) provide a businesslike atmosphere where clients can work on individual goals

Review Information: The correct answer is B: Directly assist client to her room for appropriate

A client with paranoid delusions stares at the nurse over a period of several days. The client suddenly walks up to the nurse and shouts «You think you’re so perfect and pure and good.» An appropriate response for the nurse is

A) «Is that why you’ve been staring at me?» B) «You seem to be in a really bad mood.» C) «Perfect? I don’t quite understand.» Question68 Which nursing intervention will be most effective D) «You seem angry right now.» in helping a withdrawn client to develop relationReview Information: The correct answer is D: ship skills? A) Offer the client frequent opportunities to inter- «You seem angry right now.». The nurse recognizes the underlying emotion with a matter of fact act with 1 person B) Provide the client with frequent opportunities attitude, but avoids telling the clients how they feel. to interact with other clients C) Assist the client to analyze the meaning of the withdrawn behavior D) Discuss with the client the focus that other Question71 clients have similar problems A client who is a former actress enters the day room wearing a sheer nightgown, high heels, nuReview Information: The correct answer is A: merous bracelets, bright red lipstick and heavily Offer the client frequent opportunities to interact rouged cheeks. Which nursing action is the best with 1 person. The withdrawn client is uncomfort- in response to the client’s attire? able in social interaction. The nurse-client relationship is a corrective relationship in which the A) Gently remind her that she is no longer on client learns both tolerance and skills for relation- stage ships. B) Directly assist client to her room for appropriate apparel C) Quietly point out to her the dress of other clients on the unit Question69 An important goal in the development of a thera- D) Tactfully explain appropriate clothing for the hospital peutic inpatient milieu is to

Collected by :DeepaRajesh [ 221 ] [email protected] Kuwait

apparel. It assists the client to maintain self-esteem while modifying behavior.

Question72

When teaching suicide prevention to the parents of a 15 year-old who recently attempted suicide, the nurse describes the following behavioral cue as indicating a need for intervention. A) Angry outbursts at significant others B) Fear of being left alone C) Giving away valued personal items D) Experiencing the loss of a boyfriend Review Information: The correct answer is C: Giving away valued personal items. Eighty percent of all potential suicide victims give some type of indication that self-destructiveness should be addressed. These clues might lead one to suspect that a client is having suicidal thoughts or is developing a plan.

Question73

Which statement made by a client indicates to the nurse that the client may have a thought disorder? A) «I>m so angry about this. Wait until my partner hears about this.» B) «I>m a little confused. What time is it?» C) «I can>t find my <mesmer> shoes. Have you seen them?» D) «I>m fine. It>s my daughter who has the problem.»

C) refusal to touch a client denotes lack of concern D) inappropriate touch often results in charges of assault and battery Review Information: The correct answer is A: some clients misconstrue hugs as an invitation to sexual advances. Touch denotes positive feelings for another person. The client may interpret hugging and holding hands as sexual advances.

Question75

A client with anorexia is hospitalized on a medical unit due to electrolyte imbalance and cardiac dysrhythmias. Additional assessment findings that the nurse would expect to observe are A) brittle hair, lanugo, amenorrhea B) diarrhea, nausea, vomiting, dental erosion C) hyperthermia, tachycardia, increased metabolic rate D) excessive anxiety about symptoms Review Information: The correct answer is A: brittle hair, lanugo, amenorrhea. Physical findings associated with anorexia also include reduced metabolic rate and lower vital signs.

Review Information: The correct answer is C: «I can>>t find my <>mesmer>> shoes. Have you seen them?». A neologism is a new word self invented by a person and not readily understood by another. Using neologisms is often associated with a thought disorder.

Question74

In a psychiatric setting, the nurse limits touch or contact used with clients to handshaking because A) some clients misconstrue hugs as an invitation to sexual advances B) handshaking keeps the gesture on a professional level Collected by :DeepaRajesh [ 222 ] [email protected] Kuwait

Related Documents

Nclex Test Review
April 2020 19
Nclex Review
October 2019 26
Nclex-rn Test Plan
May 2020 4
Nclex- Review [6514].docx
November 2019 19
Ob Post Test Nclex
June 2020 14